Ultra Resumenes CTO 8ª Ed-WWW.FREELIBROS.ORG.pdf

Share Embed Donate


Short Description

Download Ultra Resumenes CTO 8ª Ed-WWW.FREELIBROS.ORG.pdf...

Description

No e stá p e rm itid a la re p ro d u c c ió n to ta l o p a rc ia l d e e s te lib ro , su tra ta m ie n to in fo rm á tic o , la tra n s m is ió n d e n in g ú n o tro fo rm a to o p o r c u a lq u ie r m e d io , ya sea e le c tró n ic o , m e c á n ic o , p o r fo to c o p ia , p o r re g istro y o tro s m e d io s, sin el p e rm iso p re v io d e lo s t itu la re s d e l c o p y rig h t. © C T O E D IT O R IA L , S .L . 2011 C / N ú ñ e z d e B a lb o a , 1 15 ; 2 8 0 0 6 M a d rid T fn o .: 91 7 82 4 3 30 - Fa x: 91 7 82 4 3 43 E -m ail: c to e d ito ria l@ c to m e d ic in a .c o m P á g in a W e b : w w w .g ru p o c to .e s IS B N : 9 7 8 -8 4 -1 5 0 6 2 -6 0 -8 D e p ó s ito le g a l: M -15894 -20 1 1 Im p re so en E sp a ñ a - P rin te d in S p a in C A Ñ IZ A R E S A R T E S G R Á F IC A S , S .L. C / P é re z Fle rre ra , 7; 2 8 0 0 2 M a d rid T fn o .: 91 4 1 5 6 8 8 6

2

www.FreeLibros.me

La m e d ic in a e s u n a c ie n c ia s o m e tid a a un c a m b io c o n s ta n te . A m e d id a q u e la in v e s tig a c ió n y la e x p e rie n c ia c lín ic a a m p lía n n u e stro s c o n o c im ie n to s , son n e c e s a rio s c a m b io s en los tra ta m ie n to s y la fa rm a c o te ra p la . Lo s e d ito re s de e sta o b ra h an c o n tra s ta d o su s re s u lta d o s co n fu e n te s c o n s id e ­ ra d a s d e c o n fia n z a , e n un e s fu e rz o p o r p ro p o rc io n a r In fo rm a c ió n c o m p le ta y g e n e ra l, d e a c u e rd o co n los c rite rio s a c e p ta d o s en el m o m e n to d e la p u b lic a c ió n . Sin e m b a rg o , d e b id o a la p o s ib ilid a d d e q u e e x is ta n e rro re s h u m a n o s o se p ro d u z c a n c a m b io s en la s c ie n c ia s m é d ic a s , ni los e d ito re s ni c u a lq u ie r o tra fu e n te Im p lic a d a en la p re p a ra c ió n o la p u b lic a c ió n d e esta o b ra g a ra n tiz a n q u e la In fo rm a c ió n c o n te n id a en la m is m a sea e x a c ta y c o m p le ta en to d o s lo s a s p e c to s , ni so n re s p o n s a ­ b le s de lo s e rro re s u o m is io n e s ni d e lo s re s u lta d o s d e riv a d o s d e l e m p le o de d ic h a In fo rm a c ió n . Por e llo , se re c o m ie n d a a los le c to re s q u e c o n tra s te n d ich a in fo rm a c ió n co n o tra s fu e n te s . Por e je m p lo , y en p a rtic u la r, se a c o n s e ja re v is a r el p r o s p e c to in fo rm a tiv o q u e a c o m p a ñ a a ca d a m e d ic a m e n to q u e d e se e n a d m in is tra r, p a ra a s e g u ra rs e d e q u e la in fo rm a c ió n c o n te n id a en e ste lib ro e s c o rre c ta y d e q u e no se h an p ro d u c id o m o d ific a c io n e s en la d o sis re c o m e n d a d a o en las c o n tra in d ic a c io ­ n e s p a ra la a d m in is tr a c ió n . Esta re c o m e n d a c ió n re su lta d e p a rtic u la r im p o rta n c ia e n re la c ió n con fá rm a c o s n u e v o s o d e u so p o co fre c u e n te . Lo s le c to re s ta m b ié n d e b e n c o n s u lta r a su p ro p io la b o ­ ra to rio p a ra c o n o c e r lo s v a lo re s n o rm a le s.

3

www.FreeLibros.me

P a u ca se d m a tu ra Cari Friedrich G auss

La m a y o ría d e las o p o sic io n e s m é d ica s, e n tre e lla s el e x a m e n M IR, tie n e n un te m a rio m u y a m p lio y, p o r ta n to , po co d e fin id o . A u n q u e el tip o d e p re g u n ta s

p u e d e n e stim a rse to m a n d o co m o base

c o n v o c a to ria s p re via s, re a lm e n te n a d ie c o n o c e su v e rd a d e ra d istrib u ció n h asta el d ía d el e x a m e n . E ste h e c h o p u e d e in c ita r al o p o sito r a e stu d ia rse cu a n ta m ás in fo rm a c ió n , m ejo r. Son m u c h o s los q u e e lig e n e ste e n fo q u e , lo q u e se re fle ja en un m al re su lta d o . A n te u na o p o sició n d e e sta s c a ra c te rísti­ cas, el p la n te a m ie n to m ás in te lig e n te es se le c c io n a r el m a te ria l d e e stu d io co n la m a y o r so b rie d a d p o sib le . Es m u c h o m ás im p o rta n te un e n fo q u e se le c tiv o y c o n c e p tu a l q u e la m e m o riz a c ió n m e cá n ica d e g ra n d e s v o lú m e n e s d e in fo rm a c ió n . Esta filo so fía es la q u e im p re g n a to d o n u e stro m a te ria l de e s tu d io , e m p e z a n d o p o r n u e stro M a n u a l, ya a m p lia m e n te c o n s o lid a d o . Por e ste m o tivo , es la o b ra p rin c ip a l en la lite ra tu ra m é d ic a para la p re p a ra c ió n d e e ste tip o d e o p o s ic io n e s , ta n to en E sp añ a (e x a m e n M IR) c o m o en o tro s p a íse s d e h a b la h isp a n a . A p e sar d e lo d ic h o , en n u e stro d ía a d ía , e sta m o s a c o s tu m b ra d o s a p re g u n ta s c o m o esta s: "V oy a p re ­ p a ra r el e x a m e n M IR, p ero só lo d isp o n g o d e tres m e se s, ¿có m o p u e d o hacerlo ?". "Lle v o e stu d ia n d o to d o el añ o y q u e d a n p o ca s se m a n a s para el e x a m e n , ¿de q u é fo rm a p u e d o re p asar lo m ás im p o rta n ­ te, sin q u e m e lle v e m u c h o tiem p o ?". A u n q u e el M a n u a l CTO es una o b ra ó p tim a para p re p a ra r este tip o d e o p o sic io n e s, h ay p e rso n a s q u e , p o r d iv e rsa s c irc u n sta n c ia s, no p u e d e n re u n ir cad a d ía to d as las h o ras n e ce sa ria s p a ra e stu d ia rlo p o r c o m p le to . Y a q u e lla s q u e sí lo tie n e n , se b en e ficia ría n d e u na ve rsió n re su m id a , p a ra un re p a so rá p id o las se m a n a s p re v ia s al e x a m e n . Los U ltra rresú m en e s n ace n en re sp u e sta a e sta n e c e sid a d . La p rim e ra p a rte d e la o b ra re su m e los c o n c e p to s m ás p re g u n ta d o s d e ca d a a sig n a tu ra m e d ia n te ta b la s, g ráfico s y o tro s re cu rso s d id á c tic o s. En e sta se c c ió n , se sin te tiz a la m a y o r p a rte d e las a s ig n a ­ tu ra s d e l M a n u a l CTO 8 .a ed ició n . H ay a lg u n a s m a te ria s q u e no in c lu im o s, c o m o es el ca so In m u n o ­ lo gía y E p id e m io lo g ía -E s ta d ístic a . El m o tiv o es q u e el M a n u a l CTO ya es, en sí m ism o , m u y s in té tic o en su s c o rre s p o n d ie n te s c a p ítu lo s . Por ello , se ría m u y d ifícil h a c e r un re su m e n d e e sto s c a p ítu lo s sin p e rd e r in fo rm a c ió n im p o rta n te . D ad o q u e son a sig n a tu ra s m u y b re ve s en n u e stro M a n u a l, n u e stra re co m e n d a c ió n es u tiliz a rlo d ire c ta m e n te , ta n to p a ra el e stu d io c o m o en el re p a so . Para el resto d e las a s ig n a tu ra s , so b re to d o en las m ás e x te n sa s, los U ltra rresú m en es h a ce n u na e x tra o rd in a ria sín te sis d e los c o n te n id o s m ás p re g u n ta d o s, p e rm itie n d o un re p aso m u y e fic a z . In c lu so p u e d e n e m p le a rse c o m o ú n ic a fu e n te d e e stu d io , si la lim ita c ió n d e tie m p o es im p o rta n te . Por e sta s ra z o n e s, en cad a a sig n a tu ra , los d ife re n tes cap ítu lo s ap a re ce n ord en a d o se n fu n ció n d e su im p o rta n cia en el e x a m e n MIR, co m e n z a n d o p o r el m ás re le v a n te . La s e g u n d a p a rte d e l lib ro , q u e h e m o s lla m a d o P re g u n ta s y re s p u e sta s m á s fre cu e n te s, es o tra h e ­ rra m ie n ta p a ra a h o rra r aún m ás tie m p o . En la m ay o ría d e las o p o sic io n e s para re sid e n c ia s m é d ica s, las a sig n a tu ra s m ás re le v a n te s son las q u e , c o lo q u ia lm e n te , c o n o c e m o s c o m o "m é d icas" (C ard io lo g ía, N efro lo g ía, E n d o c rin o lo g ía , etc.). O tras m a te ria s, a p e sar d e su c o m p le jid a d , son m en o s p re g u n ta d a s y, por ta n to , d e im p o rt a n c ia s e c u n d a ria p a ra e s ta s p r u e b a s , c o m o s u c e d e con P s iq u ia tría u O fta l­ m o lo g ía . P ara e s t a s a sig n a tu ra s "m e n o re s” h e m o s e la b o ra d o un m a te ria l n u e v o . Está e s tru c tu ra d o en fo rm a d e p re g u n ta s y re sp u e sta s, lo q u e p e r m ite un re p aso d irig id o d e los c o n c e p to s y c a s o s clín ico s q u e m ás ap are ce n . Tal e n fo q u e e x ig e la p a rticip ació n a ctiv a del opositor, ya q u e le o b lig a a e v o c a r la re sp u e sta a cad a c u e stió n , sin q u e p u ed a verla e scrita en tre cinco p o sib le s, lo q u e p e rm ite un co n si d e ra b le a h o rro d e tie m p o , q u e p o d rá d e stin a rse a un e stu d io m ás p ro fu n d o d e o tra s a sig n a tu ra s m ás p re g u n ta d a s. En e sta se c ció n h e m o s in c lu id o p re g u n ta s y re sp u e sta s so b re te m a s n o c o n te n id o s en la p rim e ra p a rte (In m u n o lo g ía , P la n ifica ció n y g e stió n , etc.). In clu so h em o s in tro d u c id o p re g u n ta s so b re m aterias poco habituales, co m o O n co lo g ía y paciente term in al o G eriatría. De cad a una por separado, en las distintas o p o sicio n e s m éd icas aparecen po cas cu estio n es, au nq ue en co n ju n to suponen un núm ero co n sid erab le. Por su p u e sto , n o es re c o m e n d a b le un estu d io p o rm e n o riz a d o d e estos tem as, ya q u e son a m p lia ­ m e n te v a ria b le s y a p e n a s son p re g u n ta d o s. Sin e m b a rg o , p u e d e n ser re v isa d o s m u y fá c ilm e n te con e ste n u e v o e n fo q u e . La m a y o ría d e los o p o sito re s falla n e sta s p re g u n ta s añ o tras añ o , con lo q u e el uso d e e ste m a te ria l s u p o n e u na im p o rta n te v e n ta ja para e v ita r e ste p ro b le m a . E sp e ra m o s q u e e sta o b ra os sea d e u tilid a d para la p re p a ra c ió n d el M IR, o d e c u a lq u ie r o tro e x a m e n d e re sid e n cia . Por ú ltim o , no q u is ié ra m o s te rm in a r e sta s lín e a s sin d a rle s las g ra cia s a n u e stro s a lu m n o s. En re a lid a d , e llo s son los v e rd a d e ro s a rtífic e s d e esta o b ra . Sus o p in io n e s e in q u ie tu d e s son lo q u e h ace p o sib le q u e in te n te m o s m e jo ra r ca d a d ía.

Jorqe García Macarrón

5

www.FreeLibros.me

COLABORADORES FERNANDO AGUILAR RODRÍGUEZ Hospital Universitario Doce de Octubre Madrid RICARDO BRIME MENÉNDEZ Hospital Universitario de la Princesa Madrid JORGE GARCÍA MACARRÓN Hospital Universitario La Paz Madrid SARA MIREIA GARCÍA PTACEK Hospital Clínico San Carlos Madrid ARIANA GONZÁLEZ GÓMEZ Hospital Universitario de Getaíe Getaíe (Madrid) JUAN GROS OTERO Hospital Universitario Príncipe de Asturias Alcalá de Henares (Madrid) CRISTÓBAL MANUEL RODRÍGUEZ LEAL Fundación Hospital deAlcorcón Alcorcón (Madrid)

www.FreeLibros.me

CARDIOLOGIA Y CIRUGIA CARDIOVASCULAR DERMATOLOGÍA DIGESTIVO Y CIRUGÍA GENERAL ENDOCRINOLOGÍA, METABOLISMO Y NUTRICIÓN FARMACOLOGÍA GINECOLOGÍA Y OBSTETRICIA HEMATOLOGÍA ENFERMEDADES INFECCIOSAS NEFROLOGÍA NEUMOLOGÍA Y CIRUGÍA TORÁCICA NEURO LO G ÍAY NEUROCIRUGÍA OFTALMOLOGÍA OTORRINOLARINGOLOGÍA PEDIATRÍA PSIQUIATRÍA REUMATOLOGÍA TRAUMATOLOGÍA UROLOGÍA

11 43 63 87 113 119 141 159 185 201 225 251 267 287 303 323 339 351

Preguntas y respuestas más frecuentes

361

9

www.FreeLibros.me

C CAPITULOS 9-12.

a r d io l o g ía y c ir u g ía c a r d io v a s c u l a r

CARDIOPATIA ISQUEM ICA.

Recuerda que los datos más preguntados en esta patología son los que hacen referencia a las pruebas diagnósticas . . tratamiento. En el caso del infarto, hay que tener también muy presentes tanto las complicaciones más importantes ;omo el tratamiento de las mismas.

1. ANGINA DE PECHO. Lo más importante es conocer la clínica y el manejo del paciente con angina estable e inestable. Fíjate que el ■■¿sumen del manejo de la cardiopatía isquémica te indica qué pruebas dirigen al paciente hacia un tratamiento médico o "acia un cateterismo que nos señale si hay que realizarle algún otro procedimiento (ACTPo by-pass quirúrgico median_ injertos de safena o de mamaria).

Mane/o de la tardiopatia isquémita. A N G IN A ESTABLE

IAM

A N G IN A INESTABLE 4 -5 días si no com plicado

48h sin clínica

S i: ▼ - Arritm ias severas - Refractaria al tto. - FEV dism inuida - I. cardíaca - E C G en reposo alterado - IAM previo - Angina post IAM No concluyente/im posibilidad - Elevación de troponina para realizar ergometría/ -H d el ST con el dolor dudosa

Ergometría (valora la isquemia) Ecocardiografía (valora la f e v d

Si: - FEV dism inuida - I. cardíaca - Arritm ias severas

Cateterismo

3$í Individualizar con cada paciente

- Enferm edad 1 vaso - Enferm edad 2 vasos - Algunos casos 3 vasos - Tto. de reestenosis de injertos de m am aria y safena..

- Enferm edad del tronco de C .l. - Algunos casos 3 vasos (diabéticos y frecuentem ente si disfunción ventricular)

*

A

C^ACTP^)

Bypass

T



M ortalidad < 1 %

M ortalidad < 1%

IAM < 3%

5-10% (pequeños)

Éxito 1aria 85-90%

Éxito 1ario 85-90%

Reestenosis 30-45% 105 6 meses. (20-30% con el uso de stent). 5-10% con stents liberadores de fárm acos

10-20% al año en venosos

11

www.FreeLibros.me

ULTRA-RESUM ENES. Manual CTO de Medicina y Cirugía, 8.a edición

• •

Criterios de alto riesgo de la ergometría: Mala respuesta de la frecuencia cardiaca o tensiónarterialcon el Prueba positiva muy precozmente o a bajas cargas.



P e rsiste n cia d e la p o sitivid ad c lín ic a o e lé c tric a m ás allá d el m in u to

1

ejercicio (falta de elevación o descenso).

5de re c u p e ra c ió n .

C rite rio s d e alto riesgo d e eco d e e s tré s:

■ Aparición de defectos de contractilidad con el estrés, en muchos territorios. C rite rio s de alto riesgo co n gammagrafía de perfusión:

• • •

Aparición de importantes defectos de perfusión que se recuperan con el reposo. Defectos de perfusión en más de un territorio. Aumento de captación pulmonar con el Talio

1.

¿Cuál es el dato característico de la angina es­ table?

1.

Que la clínica aparezca con esfuerzos de inten­ sidad parecida en cada paciente.

2. ¿Cuándo es más frecuente la angina de Prinzmetal?

2.

Por la noche y en reposo.

2. INFARTO DE M IO CARDIO. Te vamos a presentar a continuación el manejo de un IAM con las actuaciones que se han demostrado de utilidad. Antes de mirar el algoritmo recuerda que el diagnóstico de IAM se hace con base a: Elevación y posterior caída de los niveles de enzimas cardíacas específicas, acompañada de al menos uno de los siguientes síntomas: 1. Dolor de características compatibles con etiología isquémica. 2. Cambios en el ECG compatibles con isquemia aguda (ascenso o descenso ST). 3. Desarrollo de nuevas ondas Q. 4. Intervención coronaria (ej: angioplastia). Hallazgos en la anatomía patológica compatibles con necrosis cardíaca.

Algoritmo de manejo de IAM P A C IE N T E C O N S O S P E C H A D Et IAM IAÍ

i

(Traducido en clínicacom o dolor con las características típicas que debes conocer)

N I C sublingual AAS E C G si posible

S E R V I C I O U R G E N C IA S - E C G (1) ¿Cuál es el E C G típico de un IAM? - P E T IC IÓ N E n z im a s M IO C Á R D 1 C A S (2) ¿Cuál es la enzim a m ás específica de IAM? - AAS (administrarla si no se hizo y continuar) - Analítica - A N A LG ESIA (generalm ente co n m ódicos) y N T G

IN G R E S O

E le v a c ió n d e S T o b lo q u e o d e ra m a iz q u ie rd a n u e v o c o n c lín jc a c o m p a tib le .

1 2 h e s p e c ia lm e n te si y a n o h a y d o lo r

T T O . R E P E R F U S IO N + B B i.v. + A s p irin a (d -c lo p id o g re l si A C T P ) + IE C A + e s ta tin a ’P T 11¡"

A lte r a c io n e s is q u é m ic a s sin e le v a c ió n d e S T o b lo q u e o d e ra m a n u e v o .

T R O M B O L I S IS (3) ¿ Q u é fá rm a c o se ad m in istra c o n ju n ta m e n te c o n la r-TPA o el T N K ?

* El m ayor beneficio del tratam iento se obtiene cuand o el paciente lleva m enos d e 6 horas con dolor. Entre las 6 y 1 2 horas el beneficio es m en os im portante. Entre las 1 2 y las 24 horas, la angioplastia puede ser útil pero no la fíbronilisis.

12 www.FreeLibros.me

A A S -Tclopidogrei Anti anginosos H N F o HBPM Inhibidores GPIIb/a si alto riesgo

S i: -is q u e m ia re c id iv a n te -d e p re sió n fu n ció n VI

R esto m ism o tto. y e rg o m e tría

-IA M p revio s

y e co ca rd io g ra fía a. los

-c a m b io s E C C exte n so s

7-1 U d ía s tras al m o n o s 4 8 Inoras sin d o lo r

C O R O N A R IO G RAFIA

C

a r d io l o g ía y c ir u g ía c a r d io v a s c u l a r

Respuestas al algoritmo de manejo del IAM.

1.

2. 3.

Ondas T altas que luego se negativizan. ST elevado: alteración electrocardiográfica más importante en la fase aguda del infarto. Ondas Q: aparecen cuando hay necrosis. Las troponinas T e l . La heparina, cuando el trombolítico administrado es la rTPA o el TNK.

Formatos usados en la prevendón setundaria del IAM. AAS: la aspirina usada en dosis de 100-300 mg/día, reduce la mortalidad tras un infarto. Indicada siempre, salvo contraindicación. lECAs: mejoran el pronóstico en pacientes con fracción de eyección menor del 40%, IAM previo e IAM anterior. Además previenen la dilatación ventricular (remodelado). Estatinas: indicadas para obtener unos valores diana de colesterol total 4 5 aunque asintornática

- FEV I < 50% aunque a s in to m á tic o - N iñ o : sie m p r e ciru g ía

FEVI < 50% o DTS > 55 aunque asintornática

Anatomía favorable: VPPB Anatomía desfavorable: prótesis

Intentar reparación, si no se puede: prótesis.

PRÓTESIS - Alternativa: operación de Ross - Niño: valvuloplastia

PRÓTESIS - Alternativa operación de Ross

20

www.FreeLibros.me

ESTENOSIS MITRAL: 1) TTO. MÉDICO: - Diuréticos (disminución de precarga y del "líquido de los pulmones") - Fármacos para controlar la frecuencia (betabloq., antagonistas del Ca, digoxina, especialmente indicada si hay FA asociada). - Anticoagulantes (si FA crónica, trombos en eco o tromboembolismos previos).

IN S U FIC IE N C IA AÓRTICA:

1) I l(). MÉDICO: - lECAs (muy útiles en esta valvulopatía) u otros vasodilatadores (nifedipina). - Inotropos (digoxina es útil en pacientes que esperan cirugía). - Diuréticos. 2) INDICACIÓN TTO. QUIRÚRGICO: En insuficiencia aórtica SEVERA si: - Aparece disminución delafunción ventricular o dilatación

2) INDICACIÓN TTO. MECANICO: Estenosis Mitral SEVERA en: - Pacientes sintomáticos en grado lll-IV NYHA. - Poco sintomáticoso asintomáticos siHTP severa o IC derecha.

importante del ventrículo izquierdo.

- Hay síntomas, con o sin disfunción ventricular. 3) TÉCNICA QUIRÚRGICA: - Sustitución valvular.

ESTENOSIS AORTICA: 1) TTO. MÉDICO: - Fármacos para control de la frecuencia ventricular, si fibrilación auricular. - Cuidado con fármacos vasodilatadores (prácticamente contraindicados) y con diuréticos (que pueden disminuir la precarga y el volumen sistólico) o inotropos negativos. 2)

INDICACIÓN TTO. QUIRÚRGICO: Estenosis Aórtica SEVERA en: - Pacientes sintomáticos. - Pacientes asintomáticos con disminución de FEV.

3) TECNICA: - Si "Score" favorableen eco, valvuloplastia conbalón. Contraindicada la valvuloplastia con balón si insuficiencia asociada o trombos en eco. - Si "Score” desfavorable o insuficiencia asociada sustitución valvular.

£ ¿ O a < < C

TO -o ^ >' ^ S q

INSUFICIENCIA MITRAL: 1) TTO. MÉDICO: - Se pueden aplicar los mismos fármacos que en EM con la excepción de vasodilatadores que no son útiles en laEM aislada y sin embargo son de mucha utilidad en la IM. (Sobre todo los lECAs).

3) TECNICA QUIRURGICA: - Anuloplastia o sustitución valvular.

^

^ N

O z

2) INDICACIÓN TTO. QUIRÚRGICO: -Momento ideal cuando el paciente comienza a presentar disminución de la función ventricular o dilatación importante del VI. - Si sintomáticos (aunque generalmente peores resultados porque ya suelen tener disminución de función V.I.).

3) TECNICA QUIRURGICA: - Sustitución valvular. En niños se puede hacer valvuloplastia.

^ >

N

O O

£ pa ^ ■

> -< p| ^ 5 cC

£ g ^ *

> ^ ^ ^

O

0

O

g

£ Hl

[— > ^3

www.FreeLibros.me

ULTRA-RESÚMENES. Manual CTO de Medicina y Cirugía, 8.a edición

1.

¿Cuál es el signo de Rivero Carvallo?

1. El aumento de la intensidad de los soplos origi­ nados en el lado derecho con la inspiración.

2.

¿Qué es el soplo de Austin-Flint?

2.

Es el que se oye en ocasiones en las insuficien­ cias aórticas durante la diástole, provocado por una “estenosis mitral" secundaria al impacto del chorro de regurgitación sobre la valva ante­ rior de la mitral.

Indicaciones generales sobre el uso de prótesis.

PRO TESIS B IO LO G IC A S: - M u je re s q u e desean te n e r hijos. - Enf. q u e co n tra in d ica n a n tico a g u la ció n . - P acie n te s m ayores d e 8 0 añ os. - P a cie n te s q u e no sigan un co n tro l co rre cto de la m e d icació n

PRO TESIS M ECAN ICAS: - M enores de 6 0 -6 5 añ os co n e sp e ra n za d e vid a m a y o r de 10 años. - E n fe rm e d a d e s q ue n ecesiten a n tico a g u la ció n . - N iñ o s y a d o lescen tes. - Estados con a u m e n to d e c a lc ific a c io n e s.

Complicaciones asociadas al uso de prótesis valvulares.

CO M PLIC A C IÓ N

ETIO LO G ÍA

CLÍN ICA

TRATAM IENTO

- O c u rre en los p rim ero s

EN DO CARDITIS PR EC O Z

6 0 días. - C o n ta m in a ció n d u ran te

Fu lm in an te . IC

la in te rv e n ció n .

A n tib ió tico s y cirug ía

- S. ep id e rm id is - Tras los 6 0 d ías. - M ism o s m ecan ism o s y

TARDÍA

TRO M BO SIS

g érm enes que en vá lv u la nativa. - Estrep tococos.

A n tib ió tico s. In su ficien cias va lvu la re s

C iru g ía si ha\ signos de d isfu n ció n val v illa r o d e sh ld e n cia de la prótesis.

Pacientes no a d e cu a d a m e n te

IC

an tico ag u lad o s

Su stll :ció n v a lv u la r urgente, ■ptimización de la c

c lación i tro m b o lisis según los ca-os.

EMBOLIA

ANEMIA H EM O LÍTICA D ISFU N CIÓ N PRO TÉSICA

2 -3 % m itra l. 1 - 2 % aó rtica.

C h o q u e de los h em atíes con la prótesis. - Fallo e strru ctu ral. - D e h isc e n cia s .

Actúe a-jular con IN R en tre 2 .5 - 3 .5 v a ñ a d ir tiagm gación plaq u etarla com e p reven ció n ). S ílm e anérr,



-

- Fo rm ació n d e trom tv-»

22

www.FreeLibros.me

m e n le no precisa.

Su stitució n valvu lar.

C

CAPÍTULO 29.

a r d io l o g ía y c ir u g ía c a r d io v a s c u l a r

CARDIOPATÍAS CONGÉNITAS.

Las cardiopatías congénitas más preguntadas hasta ahora han sido la CIA, la coartación de aorta y la tetralogía de rallot. Para estudiar este tema, es mejor que recuerdes primero los aspectos generales, sobre todo los que están en ación con las complicaciones que pueden presentar los shunt de Izquierda-derecha y de derecha-izquierda. Otro aspecto que no debes olvidar, por ser común al conjunto de las cardiopatías congénitas, es que el método diagnóstico d< elección para todas ellas es la ecocardlografía. REPASA:

[ C l a s i f i c a c i ó n f i s i o p a t o l ó g i c a de l a s c a r d i o p a t í a s c o n g é n i t a s , F i g u r a

179,

C a p ít u lo 2 9

d el M a n u a l C T O 8 a E d .]

CLÍNICA DE LAS CARDIOPATÍAS CONGÉNITAS. C O R T O C IR C U IT O D C H A . - IZ Q D A .

C O R T O C IR C U IT O IZ Q D A . - D C H A .

- C ia n o s is .

- I líp e r a f lu jo p u lm o n a r (a la larga H T P y s d . d e F ís e n m e n g e r ) .

- P o lig lo b u lia . - C r i s is h ip o x é n iic a s ( típ ic o d e l f a llo t ) .

- A u m e n t o d e in f e c c io n e s p u lm o n a r e s . - I n s u f ic ie n c ia c a r d ía c a .

- A b s c e s o s c e r e b r a le s (típ ic o d e l F a llo t ) .

Cardiopatías ton Shunt izquierda-deretha (atianótitas). C ARACTERÍSTICAS PROPIAS

TIP O S

C LÍN IC A

Tres tipos: - Tipo osliurn secun dum (la m ás írecu en le). - O sliu m p rim iim :

CIA

- G e n e ra lm e n te asintom ático s hasta 2- - 3d

- C u a n d o el cortocircuito es significativo Q p -Q s > 1 , 5 - 2

década.

indicad a cirugía.

- D esd o b lam ie n to am p lio y fijo del 2S ruido.

(asociado a sd. de D o w n e insuficiencia m itral co n frecuencia). - Seno venoso (asociado frecu en tem en te a DVPAP).

CIV

TRATAM IEN TO

- O stium p rim u m : eje izquierdo

- Tratam iento de D P y CIA

en E C C .

- O stium se cu n d u m : eje d e re ch o en E C C .

(sólo en oslium secundum )

- C ateterism o: salto oxim étrico en A D .

se puede realizar vfa sondaje percutáneo.

La m ayoría son pequeño s y cierran

Soplo síslólico "en barra" en m osocardio.

espon táneam en te.

C atororísm o: salto oxim étrico A D -V D

- La H T P grave con traindica cirugía. - En prem atu ros sintom áticos

DP

Shunt entre aorta y arteria pulm onar.

REPASA:

Soplo "en m aquinaría" en parte superior

con ductus es úti 1i el e m p leo de ¡n d o m etad n a.

del borde esternal izquierdo .

[ T ip o s de C o m u n i c a c i ó n i n t e r a u r i c u l a r , F i g u r a 1 8 0 , C a p í t u l o 2 9 d e l M a n u a l C T O

1.

¿Cómo se llama la asociación de CIA con estenosis mitral?

1. Síndrome de Lutembacher.

2.

¿Y la inversión del flujo, haciéndose derecha-izquierda tras el desarrollo de HTP?

2. Síndrome de Eisenmenguer.

8 a E d ].

Cardiopatías ton Shunt deretha-izquierda (tianógenas).

TIPOS

CARACTERÍSTICAS - A cab a lg am ie n to ao rta. - C IV

TF

- Estenosis pu lm o nar. - HVD. - H ip o a flu jo pu lm o n ar.

CLÍNICA

RX

La co m ú n a card io jtatías cia n ó tica s + : - Em b o lism o s p arad ó jico s. - A lte racio n e s de co ag u lació n .

- C o ra zó n en zueco .

Ciru gía antes de

- Abscesos ce re b rale s.

- Signo del h ach azo .

1 8 m eses.

Es la ca rd io p a tía cianóg en a m as fre cu e n te a jta rtir del añ o de ed ad. - M a n te n e r e l ductus

- A orta sale d e V D .

TGV

- A rt. P u lm . sale de V I.

C ian o sis m u y p recoz /la

Pedículo estrech o.

- Alguna zo n a de shunt

ca rd io p a tía cia n ó tica al

H ip e ra flu jo

n a c im ie n to m ás fre cu e n te .

pu lm o nar.

l-D (sobre todo CIA ).

TRATAMIENTO

p e rm e ab le (P C E 1 ), R ash kind . - C o rre cció n Q x el in te rca m b io arte ria l.

- V er fig ura.

23 www.FreeLibros.me

U LTRA -RESÚ M EN ES. M anual C T O de M edicina y Cirugía, 8 .a edición

Traspositión de los grandes vasos. Aorta

D u ctu s arterioso persistente

C ava

A. Pulm onar

C ava

Coartatión de aorta. Repasa las principales características de esta cardiopatía con estas preguntas:

1. ¿Cuál es la cardiopatía congénita más frecuen­ te asociada a coartación de aorta?

1. La valvulopatía aórtica de tipo bicúspide.

2. ¿Cuál es el tipo de coartación de aorta más fre­ cuente?

2.

La postductal.

3.

¿Cuál es el principal hallazgo exploratorio en una coartación de aorta?

3.

Un retraso y disminución de amplitud de los pulsos femorales.

4. ¿Qué sospecharía en primer lugar si un pacien­ te diagnosticado de coartación de aorta acude a urgencias con cefalea muy intensa, rigidez de nuca y fotofobia?

4.

Pensaremos que ha tenido una rotura de un aneurisma intracraneal. Otras alteraciones aso­ ciadas a coartación de aorta son la poliquistosis renal y el síndrome de Turner.

5.

¿Qué hallazgos típicos encontramos en una Rx de tórax de un paciente con coartación de aorta?

5.

Las muescas en la superficie inferior de las costillas y el "signo del 3". Sin embargo no te confundas, las pruebas de elección para el diagnóstico son la ecografía y la aortografia .

6.

¿Cuál es el periodo ideal para el tratamiento de un paciente con coartación de aorta?

6.

La edad ideal para la reparación está entre 2-3 años (no retrasarlo más de los 6 años). Recuer­ da que, en los adultos, la cirugía a veces no corrige la HTA.

7.

¿Qué síndrome asocia coartación aórtica, pterigium colli, hipertensión arterial, edemas en miembros superiores, disgenesia gonadal (XO)?

7.

Síndrome de Turner.

CAPÍTULO 31.

ANEURISMAS Y ENFERM EDADES DE LA AORTA.

La enfermedad más importante de la aorta es el aneurisma, del que vamos a repasar su forma más importante, el secundario a ateroesclerosis. Recuerda que el tipo más frecuente de aneurisma de aorta es el abdominal y su localización típica es infrarrenal. También te ofrecemos un algoritmo del manejo de la disección de aorta.

24

www.FreeLibros.me

C a r d io l o g ía

y c ir u g ía c a r d io v a s c u l a r

MANEJO DE LA D ISECCIÓ N DE AORTA.

Subclavia izda. Desgarro intima

Falsa luz Luz verdadera

DISECCION TIPO A (aorta ascendente)

SOSPECHA CLÍNICA

DIAGNÓSTICO

TRATAMIENTO

DISECCIÓN TIPO B (NO aorta ascendente)

Dolor típico: dolor intenso, brusco, desgarrador, que progresa a medida que se extiende la disección. Síntomas compresivos de estructuras adyacentes. Sangrado (ruptura de la pared). Síntomas isquémicos migratorios. Asimetría de pulsos. HTA (hipotensión si rotura).

- Eco transesofágica (1a elección) y TAC. - Rx tx: ensanchamiento mediastínico.

BAJAR la PR ESIÓ N

Contraindicado diazóxido e hidralazina ■ * y muy especialmente la anticoagulación. Contraindicado balón de contrapulsación.

A R T E R IA L

4 P BLOQ + nitropusiato IV

I

,

Tipo A

Tipo B

I

TTO. CONSERVADOR (médico)

4

CIRUGÍA *•

COMPLICACIONES

Mantenimiento

25

www.FreeLibros.me

ULTRA-RESÚMENES. Manual CTO de Medicina y Cirugía, 8.a edición MANEJO DEL ANEURISMA DE AORTA. Ateroesclerosis (causa m ás fre cu e n te , repasa tam b ié n los d e m á s tip o s de a n e u rism as con su lo ca liz a c ió n y fíjate q ue tod as tie n e n tto. q u irú rg ico de e le c c ió n ).

t

A S IN T O M Á T IC O S (lo m ás fre c .).

ANEURISM A (aorta ¡n frarrenal lo ca liza ció n m ás fre cu e n te )

Sín to m as co m p re siv o s de estru ctu ras ad ya ce n te s, E m b o lism o s. R O T U R A (causa m ás fre cu e n te de m u erte ).

SO SPECH A

Por sínto m as. B o rd e c a lcifica d o en una R x..

A O R T O C R A F IA (té cn ica m ás im p o rtan te an tes de ciru g ía).

D IA G N Ó S T IC O

O T R O S : E C O G R A F ÍA (de e le c ció n para co n tro le s en < 5 cm ). TC.

-S IN T O M Á T IC O S .

TRATAM IENTO: C IR U G ÍA

CAPÍTULO 32.

- > 5 cm . - C re c im ie n to ráp id o .

ENFERM EDADES A RTERIA LES.

La mayoría de las preguntas encuadradas bajo este epígrafe hacen referencia a la isquemia arterial aguda y crónica, por lo que deben ser dos temas que domines a la perfección.

CARACTERÍSTICAS DE LAS PRINCIPALES ENFERMEDADES ARTERIALES. S ÍN T O M A S R o b o d e la

- D ism in ución de! pulso de! lado afecto.

s u b c la v ia

- Síntom as de in su ficie n cia vertebro-basilar.

A tra p a m ie n to d e la a rte ría p o p lítea

- C lau d icació n in term itente de la extrem id ad afecta. - D ism in ución de pulsos con la con tracción del

- S O P L O C O N T IN U O +

m asa pulsátil.

- 1. C A R D ÍA C A si es de flujo m uy alto.

Sd. d e c o m p re s ió n

del tó rax

A rteriografía.

Arleriogra fía.

gem elo.

Fístu la A -V

de la sa lid a

D I A G N Ó S T IC O

A rteriografía.

- A lteraciones por com presión de integrantes del

M aniob ras.

paquete neurovascu lar (nervio, arteria, y vena).

Rx.

T R A T A M IE N T O

- Q u irú rg ico (en darte recto mía o d erivación ). - Q u irú rg ico (m iotom ía del gem elo interno). - O bliteración de co m u nicació n (generalm ente perculánea). - C O N S E R V A D O R (en la m ayoría). - Q u irú rg ico refractarias al tratam iento anterior).

- M icro ém b o lo s de colesterol procedentes de placas A te ro e m b o lia s

arteroescleróticas, frecuen tem en te tras procedim ien tos intervencionistas.

C lín ico .

Elim in ació n de la fuente em bolígena

- Sínd rom e del dedo azu l. En algunos pacientes son efectivos A rte ritis de T a k a y a su

- M ujeres jóvenes asiáticas. - O bstrucción de las grandes ram as de la aorta.

Angiografía.

los corticoides en la fase aguda. E n d a rte m ri. im ías y derivacion es quirúrgicas.

T ro m b o a n g e ítís

- C lau d icació n .

O b lite ra n te

- Fenóm eno de Raynaud.

(B u e rg u e r)

- Trom boflebitis migratoria de venas superficiales.

REPASA:

H- clínica \ exp lo ració n física. A rteriografía: afilam ien to de lo-* vasos distales.

Abstinencia del tabaco

[ D ia g n ó s t ic o d i f e r e n c ia le n t r e t r o m b o a n g e í t is y a r t e r o s c le r o s ís o b lit e r a n t e . T a b la 8 9 , C a p í t u l o 3 2 d e l M a n u a l

CTO 8a E d .]

26 www.FreeLibros.me

C

a r d io l o g ía y c ir u g ía c a r d io v a s c u l a r

En relación con el Raynaud, recuerda la acrocianosis, que tiene una clínica parecida, pero no cursa con alteraciones tróficas (el Raynaud puede hacerlo), no desaparece en los ambientes cálidos (aunque sí puede mejorar) y, como en el Raynaud, no existe un tto. eficaz.

A LGORITM O D IAG N Ó STICO TERAPÉUTICO DE LA ISQUEM IA ARTERIAL AGUDA Y CRÓNICA. O CLUSION ARTERIAL PERIFERICA

Aguda

Crónica

Sospecha em bolia:

Sospecha trom bosis

C laudicación

C laudicació n corta.

No antecedentes vasculares. Pulsos contralaterales conservados.

arterial aguda:

larga distancia

D olor de reposo,

Historia de claudicación

(estadio I ó lia).

lesiones tróficas

o enfermedad vascular.

(estadios IIb, III, IV).

Arteriografía

Arteriografía

i C ardiopatía

No cardiopatía

embolígena

em bolígena

(90%)

"

\

Valvulopatías Fibrilación auricular Válvulas mecánicas Disquinesias (1AM) j

Placas ateromatosas Aneurismas E. paradójicas

Isquemia mal tolerada (alteración sensitiva y motora)

Estenosis segmentarlas en sectores proxl males

Isquemia tolerada (sin alteración sensitiva ni motora)

Estenosis u oclusiones extensas o afectación territorio dlstal

ANGIOPLASTIA PERCUTÁNEA ±/■Stent

Estenosis residual odisección

Tratamiento médico. Medidas higiénicodietéticas.

Fibrinolisis Embolia: EM BOLECTOM ÍA

Angioplastia percutánea más prótesis endoluminal

Trombosis » Cirugía By-pass

con sonda de Fogarty

Trombosis: TROM BOENDARTERECTOM ÍA

REPASA:

[ E s t a d i o s c l í n i c o s de l a F o n t a in e , T a b l a

89,

REPASA:

[ D i a g n ó s t i c o d i f e r e n c i a l e n t r e e m b o lia y t r o m b o s i s , T a b l a 8 8 , C a p í t u l o 3 2 d e l M a n u a l C T O

C a p ít u lo

32

del M a n u a l

CTO 8a E d . ]

8 a E d .]

27

www.FreeLibros.me

L

ULTRA-RESÚMENES. Manual CTO de Medicina y Cirugía, 8.a edición

1. ¿En qué consiste el fenómeno de Raynaud?

1.

En una secuencia típica en relación con la ex­ posición al frío o con tensión emocional consis­ tente en palidez (vasoconstricción), cianosis y rubor (hiperemia reactiva).

2. ¿Cuál es la causa más frecuente de fenómeno de Raynaud?

2.

La enfermedad de Raynaud (el 50% de los f e ­ nómenos de Raynaud son enfermedades de Raynaud, es decir, de causa desconocida). De los de causa conocida, recuerda algunos muy importantes como la esclerodermia, la enfer­ medad de Buerger y los trastornos mieloproliferativos.

3. ¿Cómo se realiza el diagnóstico de Raynaud?

3.

Reproduciendo los síntomas, exponiendo las ma­ nos a agua helada.

4.

¿Cuál es la causa más frecuente de isquemia crónica de los miembros?

4.

La aterosclerosis.

5.

¿Cuál es la causa más frecuente de claudicación en los adultos jóvenes?

5.

El síndrom e aortoiliaca).

6.

¿Cuál es el sitio más frecuente de oclusión ateroesclerótica en las extremidades inferiores?

6.

A nivel de la arteria femoral superficial, en el interior del canal de Hunter.

7.

¿Cuándo se realizan injertos extraanatómicos?

7.

En pacientes de alto riesgo por enfermedad aso­ ciada grave y edad avanzada o en enfermos con escasa expectativa de vida.

CAPÍTULOS 7 Y 8.

de

L erich e

(o b lite ra ció n

ARRITMIAS.

La mayor parte de las preguntas se refieren a las taquiarritmias, especialmente la fibrilación auricular. A)

BRADIARRITMIAS.

Los mecanismos fundamentales para la producción de bradiarritmias son que haya una alteración del sistema de generación de impulsos (como ocurre, por ejemplo, en el síndrome del nodulo sinusal enfermo) o que haya una altera­ ción en el camino de conducción del estímulo que obligue a un marcapasos más lento a "tomar la responsabilidad" de marcar el ritmo del corazón. Recuerda que, cuanto más distal sea una interrupción del sistema de conducción, más grave es.

Caraíterístitas y tratamiento de las bradiarrtimias. ENFERM EDAD

Disfunción nodo sinusal

M AN IFESTACIO NES ECG

TRA TAM IEN TO

Pued e e x istir:

M a rc a p a so s en :

- B ra d ic a rd ia .

- S in to m á tico s.

- B ra d ic a rd ia - ta q u ic a rd ia .

- Pausas d iu rn a s > 3 sg.

ALTERACIO N ES C O N D U C C IÓ N A-V Bloqueo prim er grado

PR > 0 ,2 0 sg.

No p re cisa .

A S O C IA C IÓ N

Bloqueo segundo grado (Mobitz 1)

A u m e n to progresivo de PR

M a rc a p a so s si

IAM in ferio r.

hasta b lo q u eo de im p u lso .

sin to m á tico .

Fárm aco s.

M a rca p a so s.

IAM an terio r.

Bloqueo segundo grado (M obitz II)

Bloqueo tercer grado

LO C A LIZ A C IÓ N

N od o AV.

Fallo de co n d u cció n de algu nos im p u lso s (P no seguida de Q R S ).

D iso cia ció n A - V.

M a rca p a so s.

28

www.FreeLibros.me

H a z de H iss

C

a r d io l o g ía y c ir u g ía c a r d io v a s c u l a r

De los bloqueos, es importante conocer el concepto y su tratamiento (que es muy fácil, como ves). Pon un poco más de atención en el bloqueo de 2S grado, que es el nivel donde cambia el tratamiento, dependiendo de que sea Mobitz I ó II. (Por cierto, como te decíamos antes, el Mobitz II, que es más distal, más grave y además evoluciona más frecuen­ temente a BAV completo).

B)

TAQUIARRITM IAS.

Las taquiarritmias son más preguntadas que las bradiarritmias.Te vamos a mostrar los aspectos más interesantes.

Extrasístoles Extrasístoles auriculares

Extrasístoles ventriculares

P an te s de tie m p o .

Q R S an tes d e tie m p o .

6 0 % a d u lto s la p re se n ta n .

6 0 % ad u lto s la p re se n ta n . Im p o rta n c ia c lín ic a en a fe cto s d e c a rd io p a tía

Sin im p o rta n cia p ato ló g ica.

isq u é m ica si fre c u e n te s, c o m p le jo s o fe n ó m e n o R so b re T.

Q R S n o rm a l.

Q R S a n ch o s.

B e ta b lo q u e a n te s si m o le stias.

B e ta b lo q u e a n te s si sín to m a s m o lesto s.

Pausas no c o m p e n sa d o ra s.

Pausas c o m p e n sa d o ra s.

Taquitardia Supraventritular Paroxístita. Son taquicardias regulares de QRS estrecho. La causa más frecuente es por reentrada.

1.

¿Cuál será la primera medida que debemos tomar ante un paciente con una taquicardia supraventricular paroxística?

1.

El empleo de maniobras vagóles (Valsalva, ma­ sajecarotídeo...)

2.

¿Qué medidas adoptarías si estas no son efectivas?

2.

Uso de adenosina, ATP o verapamil i.v.

1.

Puede ser de las dos maneras, todo depende de que el impulso vaya por la vía normal y regrese por la accesoria (QRS normal), conducción ORTODRÓMICA (que es la más frecuente), o que vaya por la vía accesoria y regrese por la nor­ mal (QRS ancho, conducción antidrómica).

Síndromes de Preextitatión.

1.

¿Cómo es el QRS en una taquicardia supraventricular asociada a WPW, ancho o estrecho?

2. ¿Qué alteraciones hidroelectrolíticas cursan con QT largo?

¿Cuál es el tratamiento definitivo de estos sindromes de preexcitación?

2. La hipocalcemia, hipopotasemia (recuerda la onda U, que también puede aparecer caracte­ rísticamente en esta alteración) e hipomagnesemia.

3.

La ablación de la vía accesoria con un catéter de radiofrecuencia.

29 www.FreeLibros.me

ULTRA-RESÚMENES. Manual CTO de Medicina y Cirugía, 8.a edición

Fibrilatión auritular (FA). Debes tener claro el manejo de esta arritmia. Es esencial saber cómo se emplea el tratamiento anticoagulante, que se administra tanto si la FA es crónica como paroxística.

Manejo ante una fibrilatión auritular.

1Q. Control frecuencia cardíaca 1. Digoxina 2. Betabloqueantes 3. Verapam il/Diltiazem

2e. ¿Merece la pena revertir a ritmo sinusal? En función de dos parámetros: 1. Dilatación auricular: >55m m . de diámetro, no compensa 2. Tiempo que lleva en FA: >6-12 meses, no sirve para nada. Recidivaría

32. Tiempo de evolución de la FA En función de dos parámetros: 1. < 48 horas: cardioversión a. Farmacológica: antiarrítmicos de clase le (salvo IC o cardiopatía estructural impor­ tante, en la que se prefiere amiodarona) b. Eléctrica: Desfibrilador 2. > 48 horas: a. Anticoagular 3-4 semanas (Puede suprimirse este paso si en la ECO transesofágica se demuestra ausencia de trombos en aurícula o en orejuela) b. Posteriormente cardiovertir c. Después mantener anticoagulación 3-4 semanas más

4Q. Indicaciones de anticoagulación oral crónica: 1. Alto riesgo embolígeno

I

a. AP embolia o AIT b. EM

■No FR — > Antiagregar • 1 FR moderado — > Antiagregar o anticoagular

c. Prótesis valvulares

• > 2 FR moderados — > A CO

2. Factores de riesgo moderados

• 1 FR elevado — >

a. HTA b. DM c. IC d. >75 años e. FE < 35%

I

30 www.FreeLibros.me

A CO

C

a r d io l o g ía y c ir u g ía c a r d io v a s c u l a r

CO N CEPTO

ECG Q R S a n c h o ; Igual sie m p re : m o n o m o rfa ; d ife re n c ia s de un

Taq. ventricular

M á s de 3 la tid o s p ro ce d e n te s del

la tid o a o tro : p o lim o rfa.

v e n tríc u lo a m ás de 1 0 0 Ipm .

P u e d e e x is tir d iso c ia c ió n A V y a veces h ay P retró grad a. T a q u ic a rd ia v e n tric u la r so ste n id a (cu a n d o es > a 3 0 sg).

T V con Q R S p o lim o rfo s

Torsades d e Pointes

del M a n u a l C T O 7- Ed ició n )

de am p litu d c a m b ia n te .

O s c ila c ió n de la lín e a base en el cu rso de la ta q u ic a rd ia .

TV . Se c o n sid e ra signo de

RIVA

REPASA:

Q T largo p re vio . (M ira las ca u sa s de Q T largo en pág. 57

S im ila r a T V con fre c u e n c ia en tre 6 0 - 1 0 0 Ipm .

re p erfu sió n tras un IA M .

[ D a t o s a f a v o r d e lo r i g e n v e n t r i c u l a r o s u p r a v e n t r i c u l a r e n l a t a q u i c a r d i a r e g u l a r c o n T a b la

28,

C a p ít u lo

8

del M a n u a l

Q RS a n c h o ,

CTO 8a E d .]

Para que te acuerdes bien, ten claro que todas las arritmias que producen compromiso cardiovascular importante t enen como tratamiento de elección la cardioversión eléctrica inmediata.

CAPÍTULO 30.

HIPERTENSIÓN ARTERIAL.

El tipo más frecuente es la esencial. Sin embargo, en el MIR han aparecido muchas veces preguntas sobre las causas De HTA secundaria, que es lo primero que hay que descartar ante un paciente con aumento de presión arterial. Debes ■nocer bien cómo se diagnostica la HTA. Recuerda, además, que la HTA es un factor patogénico por sí mismo de a teraciones orgánicas. Hablamos de HTA cuando: ■ TAS>140mmHg. ■ TAD> 90 mmHg.

PASOS A SEGUIR ANTE UN AUM ENTO DE PRESIÓN ARTERIAL. M edición presión arterial

PAS > 1 4 0 mm Hg y/o PAD > 9 0 mm Hg

M edicion es repetidas seriadas en las 2-3 sem anas siguentes A Co nfirm ación H TA

► No confirm ación HTA

D ES C A R TA R H TA 2aria

Seguim iento

H T A 2aria (5-10% )

H T A esencial 1 s m edidas d ietéticas

T

Tto. etiológico

-sal

/D“ °ro| de otros factorOSde riesgo cardiovasculares

C O N T R O L HTA

* En caso de DM o afectación de órganos diana se comienza de entrada tratamiento farmacológico cuando ¡as cifras tensionales sean entre 140-130 de sistólica y/o 90/80 de cliastólica

NO C O N T R O L

Tto. M É D IC O * - Antag. calcio - lECA s o antagonistas receptores de anglotensina II - 13 bloquentes - D iuréticos - Alfa-1 -antagonistas

31 www.FreeLibros.me

U LTRA -RESÚ M EN ES. M anual C T O de M edicina y Cirugía, 8 a edición

ALTERACIONES ORGÁNICAS EN LA HTA. CARDIOVASCULAR

- H V I. - A te ro scle ro sis p e rifé rica . - R e tin o p a tía liip e rte n siv a

RETINA

(ver u ltra rre su m e n O fta lm o lo g ía ). - A C V (lie m o rrá g ico s e isq u é m ico s).

SNC

- E n c e fa lo p a tía liip e rte n siv a . - E n ferm ed ad de B in sw a n g e r. - A rte rio e sc le ro sis ren al (le sió n p arecid a

RIÑÓN

a la de e scle ro sis sisté m ic a ).

Para descartar las HTA secundarias, tienes que conocer qué patologías pueden cursar con un aumento de la PRE­ SION ARTERIAL (mira también cuál es la prueba diagnóstica de elección en cada caso).

CAUSAS DE HTA SECUNDARIA. REN O VA SCU LA R (el grupo más frecuente)

- E ste n o sis a rte ria l (m ás fre cu e n te en a n c ia n o s). - D isp la sia fib ro m iis c u la r (m ás fre cu e n te en m u je re s jó v e n e s ). - A n tic o n c e p tiv o s o ra le s (la cau sa e sp ecífica m ás fre cu e n te de H T A se c u n d a ria ). C a si to d o son liip e rfu n c io n e s : - H ip e rtiro id ism o . - H ip e rp a ra tiro id is m o .

EN D O CR IN A S

- A cro m e g a lia . - C u s liin g . - H ip e ra ld o ste ro n is m o . - F e o c ro m o c ito m a .

CA R D IO V A SC U LA R ES

- C o a rta c ió n de a o rta . - In su ficie n cia aó rtica (H TA sistó lica a isla d a ). - C ic lo s p o rin a .

FA R M A C O LÓ G IC A S

- E stero id e s. - A IN E s . - C o c a ín a , a n fe ta m in a s.

TRATAM IENTO. Hay diferentes fármacos de uso común en el tratamiento de la HTA. Debes conocer el tratamiento en algunas situaciones especiales. Además, sería bueno que recuerdes los efectos secundarios más importantes de los fármacos más usados en el tratamiento de la HTA. EFECTOS SECUNDARIOS

FÁRMACO D iu ré tic o s tia zíd ic o s y de asa .

D iu ré tic o s a lio ra d o re s de K +.

B e ta b lo q u e a n te s . A lfa b lo q u e a n te s . A n ta g o n ista s C a ++ tip o d ih id ro p irid in a s ( n ife cl¡p in a ). A n ta g o n ista s C a ++tipo ve ra p a m il o d iltia z e m . lE C A s. A n ta g o n ista s re ce p to re s A n g . II (tipo lo sartán )

H ip o K +. A lim e n to á cid o ú rico . H ip e rg lu c e m ia . 1 lip e rlip id e m ia . H ip e r K +. G in e c o m a s tia y d isfu n c ió n se x u a l en el varó n (éstos dos la e sp iro n o la c to n a ). B ro n c o e s p a s m o . D e p re sió n , in s u fic ie n c ia v a s c u la r p e rifé rica , liip e rtrig lic e rid e m ia . H ip o te n sió n o rto stá tic a , ta q u ic a rd ia . T a q u ic a rd ia , c e fa le a , e n ro je c im ie n to facia l, e d e m a s p re tib ia le s.

C ro n o e in o tro p ism o n e g ativo s, e stre ñ im ie n to . Tos irrita tiva , e ru p c io n e s , d isg eu sia.

M a re o .

32 www.FreeLibros.me

C

a r d io l o g ía y c ir u g ía c a r d io v a s c u l a r

1. ¿Qué fármacos se usan generalmente en el tra­ tamiento de la UTA del embarazo?

1.

La alfametildopa y la hidralacina.

2.

¿Cuál es el concepto de HTA maligna?

2.

La que produce edema de papila.

3.

En un anciano con enfermedad prostética leve y HTA serán considerados como muy útiles los...

3.

Los antagonistas alfa-1 (prazosín, terazosín),que además de hipotensores, se utilizan para au­ mentar el flu jo de orina.

4. ¿Por qué son de menor utilidad los betabloqueantes y los diuréticos en el tratamiento de HTA con el colesterol elevado y/o glucemias anor­ males?

4.

Porque estos dos tipos de fármacos alteran el perfil lipídico. En el caso de los betabloqueantes, pueden ser peligrosos en la DM porque en­ mascaran los síntomas adrenérgicos de la hipoglucemia. Los diuréticos tiacídicos pueden ade­ más producir hipopotasemia e hiperuricemia.

5.

¿Qué fármaco es considerado de elección en el tratamiento de la HTA vasculorrenal?

5. Los lECAs o los ARA-II. Son también de elección cuando hay DM, ya que parece que reducen la incidencia de nefropatía. Si hay estenosis bilate­ ral o existe sólo un riñón, están contraindicados.

6.

¿Cuál deberá ser tu 1a sospecha si un paciente con HTA (que no toma diuréticos) tiene el K* bajo?

6.

Podría ser un hiperaldosteronismo primario, que cursaría con K' bajo y alcalosis metabólica.

7.

¿En qué tipo de HTA secundaria se utiliza el renograma con captopril como prueba diagnóstica?

7.

En la HTA vasculorrenal.

8.

¿Cuándo es más probable la HTA secundaria?

8. Cuando la HTA aparece antes de los 30 años o después de los 50.

CAPITULO 2.

SEM IOLOGIA CARDIACA Y VASCULAR .

Lo más preguntado en este tema son las características del pulso, tanto arterial como venoso. ^

REPASA:

[ C a r a c t e r í s t i c a s d e l p u ls o A r t e r i a l y V e n o s o , A p a r t a d o s

2.2

y

2.3,

C a p ít u lo

2

del M an u al

CTO 8a E d .]

1. ¿Qué soplos aumentan con la maniobra de Valsalva?

1. Los soplos de la miocardiopatía hipertrófica obstructiva y el prolapso mitral.

2.

2. El soplo diastólico por estenosis mitral que aparece en el contexto de una insuficiencia aórtica.

¿Cuál

es el soplo de Austin-Flint?

Últimamente los soplos y otros ruidos cardíacos están tomando mayor trascendencia.

CAPÍTULOS 5 Y 6.

IN SU FICIEN CIA CARDÍACA._________________________________________

La insuficiencia cardíaca es la situación en la que se produce un gasto cardiaco inferior a las necesidades metabóli5S. El marcador pronóstico mas importante es la fracción de eyección, siendo la mortalidad al año del 50% en aquellos an clase funcional IV. REPASA: [ T ip o s de in s u f ic ie n c ia c a r d i a c a s e g ú n e l g a s t o c a r d í a c o , T a b la 16, C a p í t u l o 5 d e l M a n u a l CTO 8 a E d.]

1. ¿Cuáles son las principales causas de insuficiencia cardiaca?

1. La cardiopatía isquémica y la HTA.

2.

2. La hipertrofia ventricular izquierda y la isque­ mia miocárdica.

¿Y las principales causas de IC diastólica?

33 www.FreeLibros.me

ULTRA-RESÚMENES. Manual CTO de Medicina y Cirugía, 8.a edición CLÍNICA.

C O N G E S T I Ó N I Z D A . (F a llo re tró g ra d o )

[ ^ C O N G E S T I Ó N D C H A . (V . C a v a s u p e rio r)

D is n e a . O rt o p n e a . D is n e a p a ro x ís tic a n o c tu rn a . E d e m a a g u d o d e p u lm ó n

In g u rg ita c ió n y u g u la r

I N S U F IC IE N C I A A N T E R Ó G R A D A IZ D A . I 1 1 P O P E R F U S IÓ N : P e rífe ric a (fria ld a d c x tr c m id a d e s ). R e n a l (T a ld o s te ro n a , T líq u id o s ). C e re b ra l (c o n fu s ió n , a lt. m e m o r ia ...). [^ C O N G E S T I Ó N D C H A . (V , C a v a inferior)

^ IN S U F IC IE N C IA A N T E R Ó G R A D A D C H A .

E d e m a s e n z o n a s d e c liv e s . H e p a to m e g a lia d o lo r o s a , c irro s is ,

REPA SA :

[D

D e fe c to d e lle n a d o v e n lr ic u la r iz q u ie r d o .

if e r e n c ia s e n t r e l a in s u f i c i e n c i a c a r d í a c a c o n

T a b la

15,

C a p ít u lo

5

del M an u al

f u n c i ó n s i s t ó l i c a d e p r im i d a f r e n t e a c o n s e r v a d a .

CTO 8 a E d . ]

TRATAMIENTO. Las diferentes posibilidades que existen se resumen imaginando que el corazón insuficiente es como una persona débil, cargada con un peso muy grande que intenta empujar una gran piedra; por lo tanto es fácil deducir que le podemos ayudar transformándole en un atleta (fármacos inotrópicos positivos), quitándole parte del peso que lleva (fármacos reductores de la precarga) o cambiando la piedra que intenta mover por una más pequeña (reducción de la postcarga).

Formatos inotrópitos positivos. Digital, dobutamina, etc. Para recordar los aspectos más importantes de la digital, contesta tapando la columna de la derecha:

1.

¿Cuál es la indicación más clara para uso de la digital en la IC?

1.

2.

¿Cuál es la causa más frecuente de intoxicación digitálica?

2. Insuficiencia renal.

3.

¿Cuál es la arritmia más frecuente en la intoxi­ cación digitálica?

3. Las extrasístoles ventriculares.

4.

¿Y la más característica?

4.

Taquicardia auricular con BAV variable.

5.

Se detiene el tratamiento y se emprenden me­ didas para el control de las arritmias. En con­ creto, un marcapasos temporal puede ser ne­ cesario si el bloqueo no se soluciona de forma adecuada, y la lidocaina es muy útil para el tratamiento de las taquiarritmias ventricula­ res. Si hay hipopotasemia, evidentemente una parte importante de su tratamiento será su re­ posición.

5. ¿Cómo se realiza el tratamiento de una intoxi­ cación digitálica?

Miocardiopatía dilatada con FA.

34 www.FreeLibros.me

C a r d io l o g ía

y c ir u g ía c a r d io v a s c u l a r

La digoxina puede mejorar el pronóstico de los pacientes con insuficiencia cardíaca y fibrilación auricular, pero no ■a demostrado mejorar la supervivencia de la IC en ritmo sinusal. El resto de inotrópicos positivos no han demostrado una ~eioría del pronóstico, incluso pueden empeorarlo.

Formatos que disminuyen la pretarga o la posttarga. * •

Reducción de Precarga —» Disminución de las presiones telediastólicas —> Disminución de la congestión venosa. Reducción de Postcarga —»Aumento del gasto cardíaco.

V a s o d ila ta d o r e s :

Los más usados son los lECAs (vasodilatadores arteriales y venosos). Mejoran la IC y retrasan su desarrollo en pacien~ con disfunción ventricular izquierda. Mejoran el pronóstico de los pacientes con IC. Son el tratamiento de elección en ¿ miocardiopatía dilatada. Mejoran la supervivencia en pacientes con ICC de etiología isquémica y en pacientes con - ccardiopatía dilatada. últimamente se están introduciendo en la clínica los ARA II (antagonistas de los receptores de angiotensina II), que ;n el mismo efecto beneficioso de los lECAs, con la ventaja añadida de que no aumentan los niveles de bradicininas, as cuales se consideran responsables de algunos efectos secundarios de los lECAs, como la tos o el angioedema. Su uso - =sido aprobado en el tratamiento de la HTA y la ICC. N itra to s:

Son vasodilatadores preferentemente venosos. D iu r é t ic o s :

Reducen la precarga. El tratamiento con espironolactona en dosis bajas (25 mg/24h) ha demostrado disminuir la dualidad. Repasa la siguiente tabla acerca de los diuréticos.

MECANISMOS DE ACCIÓN

DIURÉTICO

Tiazidas (clorotiazida, upamtda, ndapamida).

Inh ib ición de

INDICACIONES

- Insu ficiencia ca rd ía ca leve o m o d erad a.

c an ale s en N a/ Cl en T C D .

- H TA .

EFECTOS SECUNDARIOS

CONTRAINDICACIONES

- H ip o k. - C u a n d o F C < 2 5 mi/ m in.

- H ip e ru rice m ia . - A um en to glucosa y lípidos.

- Estados ed em atosos

A54 furosemida, áddo etacrínico, oumetanida, ndacrinona).

Inh ib ición del tran sp o rtad o r 2 C I +

refractario s. - Estados ed em atosos agudos graves.

Na + k en asa de H e n le .

- F C < 2 5 mi/ m in.

-

H ipo K.

- A n u ria . - E m b ara zo .

-

H ipo Na.

-

- H ip ersen sib ilid ad .

-

H ip e ru rice m ia . H ipo M g.

-

O to to xicid ad .

- H TA grave.

-

A sociado a otros d iu rético s en estado ed em atoso o H TA .

Ahorradores de K+ Antagonistas de la aldosterona espironolactona).

B loq uea la acción

-

de la aldosterona en TCD .

-

H ip erald o stero n ism o s no corregibles co n cirug ía. Estados ed em atosos con h ip erald o stero n ism o (Sd. n e fró tico ,

2 Amiloride,

IC C , cirrosis).

-

-

H ip er K. 1. ren al.

-

Tratam ien to s con

-

-

Im p o ten cia, g inecom astia. H ip er k.

su p lem ento s de k.

triamterene. B lo q ueo de la ab sorción en T C D de N a.

- Lo s m ism os q ue el anterior.

- H ip er K.

- Sd. de Lidd le.

B e ta b lo q u e a n t e s :

Reducen la mortalidad a largo plazo en los pacientes con IC por disfunción sistólica. También son útiles junto con los catcioantagonistas verapamil y diltiazem, en el tratamiento de la disfunción diastólica. R E P A S A : [ In d ic a c io n e s y c o n t r a in d ic a c i o n e s d e lt r a s p l a n t e c a r d í a c o , A p a r t a d o

6.3,

C a p ít u l o 6 d e l M a n u a l

CTO

8 a

E d.]

35 www.FreeLibros.me

ULTRA-RESÚMENES. Manual CTO de Medicina y Cirugía, 8.a edición

1. ¿Cómo se hace el tratamiento de un EAP?

1.

Con vasodilatadores, diuréticos, 02y sulfato de morfina. Además se pueden administrar disi­ tálicos. En resumen, se intenta ayudar al cora­ zón por todos los medios, y además forzamos la oxisenación.

2.

¿Cuándo son de utilidad los betabloqueantes o los antagonistas del calcio tipo verapamil o diltiazem en el tratamiento de la insuficiencia cardiaca?

2.

Cuando hay fallo de relajación del ventrículo. (Fíjate en que son inotropos negativos).

3. ¿Qué prueba complementaria aporta más fre­ cuentemente datos sobre la etiología de la in­ suficiencia cardiaca?

3.

La ecocardiografía

4. ¿Qué tipo de alteración del patrón respiratorio es típico de la ICC?

4.

La respiración periódica o de Cheyne-Stokes.

5.

5. Restricción de sal, pérdida de peso (en obesos)...

Hemos hablado del tratamiento de la ICC en su vertientefarmacológica, pero ¿podrías decir qué medidas no farmacológicas son también de uti­ lidad?

6. ¿Te acuerdas de cuál es el efecto secundario más típico de los lECAs?

6.

La tos irritativa. Recuerda que también pue­ den producir hiper K* y descenso de la función renal.

Recuerda que: - En la IC sistólica se utilizan: betabloqueantes + espironolactona + lECA/hidralacina - nitratos. Otros: ARA II y digoxina. No dar verapamilo o diltracem. - En la IC diastólicase utilizan: betabloqueantes + verapamilo o diltiacem. No dar digoxina (salvo FA) y cuidado con los vasodilatadores y diuréticos. - El tratamiento del EAP es una movida: morfina, oxigenoterapia, vasodilatadores (si TA> 90 - 100 mmHg), inotrópicos (si existe hipotensión o el paciente no mejora), diuréticos, asentarse.

CAPÍTULOS 22-25.

MIOCARDIOPATÍAS.

Mira con atención el cuadro:

CARACTERÍSTICAS DE LAS MIOCARDIOPATÍAS.

M IO CARD IO PA TÍA

Dilatada

Hipertrófica

Restrictiva

V O LU M EN VI

FEV

D ISFU N C IÓ N

ESPESO R PARED

EJEMPLO M IO CARD IO PA TÍA SECU N DARIA

A lim e n ta

Dism inuye mucho.

Sistólica.

N o rm a l

A lc o h ó lic a .

Normal o aumenta.

D ia stó lic a .

m u ch o .

D ism in u y e m ucho.

D ism in u y e ,

N o rm a l o

n o rm a l o

d ism in u y e un

a u m e n ta .

po co.

P e rip arto . F á rm a co s.

Aumenta mucho. A sim é tric a m e n te

Friecl reich .

(St. septo). Fibrosis e n d o m io c á rd ic a . D ia stó lic a .

N o rm a l o a u m e n ta .

Enf. e n d o m io c á rd ic a e o sin o fílic a . Fib ro elasto sis e n d o c á rd ic a .

Fíjate como la única miocardiopatía que cursa con una función sistólica claramente deprimida es la dilatada, que por ello tiene una FEV muy disminuida.

36 www.FreeLibros.me

C

a r d io l o g ía y c ir u g ía c a r d io v a s c u l a r

A) M IOCARDIOPATÍA DILATADA. Piensa en ella cuando veas a un paciente con síntomas de insuficiencia cardíaca en un corazón muy dilatado. Es oastante característica de esta patología la aparición de arritmias y embolias (ya que se favorece el remanso sanguíneo rr las cavidades tan dilatadas). Los soplos eliastólíeos, las calcificaciones vasculares, la HTA, y la existencia de angina de aecho, en general te deben orientar a pensar en otra patología que esté produciendo esa insuficiencia cardíaca. Debes conocer qué procedimientos terapéuticos son útiles en un paciente con miocardiopatía dilatada.

Manejo délaMiotardiopatía dilatada. S I T U A C I Ó N EN Q U E S E U S A

T R A T A M IE N T O lE C A b

I. c a r d ía c a .

D ig o x in a

I. c a r d ía c a c o n F.A.

I D e sfib rilad o r im p la n t a d lo

A n t ic o a g u la c ió n

“ r a s p la n lo C a re lia n o

A r r it m ia s v e n t r ic u la r e s g ra v e s . - S it u a c io n e s q u e f a v o r e c e n f o r m a c ió n tr o m b o s en e s p e c ia l la FA. - T r o m b o s d e t e c t a d o s p o r e c o c a r d io g r a f ía . - E m b o lis m o s p re v io s. C u a n d o la in s u f ic ie n c ia c a r d ia c a e s re s is t e n t e a l t r a t a m ie n t o m é d ic o y no e x is t e n c o n t r a in d ic a c io n e s .

B) M IOCARDIOPATÍA HIPERTRÓFICA. La clínica es muy parecida a la de la estenosis aórtica, con la misma tríada sintomática. Sin embargo, aquí es la d is n e a , y no la angina, el síntoma más frecuente. La muerte súbita, generalmente en relación con el desarrollo de tm ias, puede ser el primer síntoma de una miocardiopatía hipertrófica, y es más típica en niños y adolescentes, _ i luciéndose más frecuentemente con o tras un ejercicio intenso. Es muy importante que domines los rasgos característicos de la auscultación y la ecocardiografía. Sin embargo, no te ea j .oques asumiendo que todas las miocardiopatías hipertróficas son obstructivas (de hecho, esto es más bien "raro , ya sólo aproximadamente 1/4 de los pacientes con esta enfermedad tiene obstrucción en el tracto de salida), a pesar de pregunten mucho por su traducción en la exploración física, que es el soplo. Con respecto a las maniobras que aumentan o disminuyen el soplo, date cuenta de que todo aquello que nenta la fuerza de contracción del ventrículo izquierdo favorece la obstrucción, y por lo tanto, aumenta el soplo r-íta es la razón por la que están contraindicados los fármacos que aumentan la contractilidad o que disminuyen la Z’ ~' o postcarga). De la misma manera, todo lo que descienda la cavidad del ventrículo izquierdo (descenso de e ~ado por maniobra de Valsalva, vasodilatadores, ortostatismo) aumenta el gradiente de salida.

C) MIOCARDIOPATÍA RESTRICTIVA. 3ara

acordarte de ella, piensa en un cuadro muy parecido a la pericarditis constrictiva, pero con ausencia de ca lc ifiy un engrosamiento simétrico del ventrículo. REPASA: [ D i f e r e n c i a s e n t r e l a c a r d i o m i o p a t í a h i p e r t r ó f i c a d i l a t a d a y r e s t r i c t i v a , T a b l a 6 8 , C a p í t u l o 2 2 d e l M a n u a l CTO 8* E d .]

ra cio n e s pericárdicas

-ara terminar de repasar las miocardiopatías, contesta a las siguientes preguntas: 1. ¿Qué miocardiopatías secundarias tienen caracte­ rísticamente una mayor incidencia de embolismos?

1.

2.

2. Amiloidosis.

Un corazón con aspecto moteado en la eco es típico de...

La miocardiopatía del periparto y la eosinofílica o de Loeffler.

3. ¿Cómo están las presiones de llenado de los ven­ trículos en las miocardiopatías?

3. Aumentadas en todas. En la dilatada, porque hay defecto de la sístole (y queda mucha sanare al f i ­ nal de la sístole), y en las restrictiva e hipertrófi­ ca, porque hay defecto de relajación.

4.

¿Qué fármaco ha demostrado aumentar la super­ vivencia de los pacientes con miocardiopatia dila­ tada para el tratamiento del sindrome de insufi­ ciencia cardiaca que padecen?

4.

Los lECAs. También ha demostrado aumentar la su­ pervivencia la combinación de vasodilatadores como n itra to s e h id ra la cin a , tam bién los betabloqueantes y la espironolactona.

5.

¿Puede presentar un paciente con miocardiopa­ tía hipertrófica angina o sincopes si no tiene obs­ trucción de salida?

5.

Sí. Fíjate en esto (es muy importante), los síntomas de la miocardiopatía NO tienen relación con obstruc­ ción (cosa que sí ocurre en la estenosis aórtica).

37 www.FreeLibros.me

ULTRA-RESÚMENES. Manual CTO de Medicina y Cirugía, 8.a edición

7. ¿Es patognomónica la aparición de SAM en la MHO?

7.

8. ¿Recuerdas alguna alteración típica del ECG de la MHO?

8. Las alteraciones en el ECG son muy habituales en la MHO (de hecho, pocos pacientes tienen un ECG normal). Lo más frecuente es que pre­ senten alteraciones de la repolarización. Es muy típica la presencia de ondas T invertidas gigan­ tes. Recuerda también que pueden apreciarse arritmias auriculares y ventricuiares.

CAPÍTULO 4.

No, puede aparecer también en otras patolo­ gías, pero cuando hay obstrucción en una miocardiopatía hipertrófica, siempre está presen­ te el SAM (recuerda que la otra característica ecocardiográfica importante es la existencia de hipertrofia asim étrica, generalmente pronun­ ciada en septo).

FÁRMACOS EN CARDIOLOGÍA.

Los fármacos más importantes que debes conocer son los usados en el tratamiento de la angina y de la insufi­ ciencia cardiaca, que tienes en los capítulos correspondientes de este ultrarresumen. IZ^ REPASA:

[ F á r m a c o s en C a r d i o l o g í a , A p a r t a d o s

4.1, 4.2

y

4.3

del M anual

CTO 8a E d . ]

EFECTOS DE LOS FÁRMACOS EN DISTINTAS PATOLOGÍAS CARDÍACAS. IE C A

C A L C IO A N T A G O N 1STAS

B -B L O Q U E A N T E S

T r a t a m ie n t o d e la IC

M e jo r a n la su p e rv iv e n c ia e n el t r a t a m ie n t o d e la IC .

D IU R É T IC O S

P ro lo n g a n la s u p e rv iv e n c ia y

In su ficie n cia C a rd ia c a . M io card io p a tia D ilata d a

p r e v ie n e n el d e s a r r o llo d e IC en p a c ie n te s c o n d is fu n c ió n v e n tr ic u la r (al igual q u e e s p ir o n o la c lo n a , b e t a b lo q u e a n t e s

d ia s t ó lic a .

M e jo r a n los s ín t o m a s d e

T r a ta m ie n to de la IC

N o e n sistó lic a

c o n g e s tió n d e la IC .

d ia s t ó lic a .

e h ic lra la z in a m á s n itra to s}

A n t ia n g in o s o s

C a rd io p a tía Isq u é m ica

A d m in is tr a d o s p r e c o z m e n t e en

(d ism in u y e n

IA M p r e v ie n e n el d e s a r r o llo d e IC y d e rein farto e n p a c ie n t e s c o n u n a

c o n tr a c t ilid a d y re d u c e n

FE < 4 0 % ,

T r a t a m ie n t o d e la an g in a

P re v ie n e n la d ila t a c ió n v e n tricu la r.

v a s o e s p á s t ic a (P rin zm e ta l).

p o stea rga).

- A n t ia n g in o s o s ( d ism in u y e n la

U tile s ju n t o co n los

c o n tr a c t ilid a d y la

n itra to s e n el

fr e c u e n c ia c a rd ia c a ).

tra ta m ie n to d e la IC

- En IA M r e d u c e n la m o r t a lid a d , la in c id e n c ia

s e c u n d a r ia a un IA M .

d e m u e rte s ú b ita , re in fa rto y angor.

- R e d u c e n la H T A ren al y v a s c u lo r e n a l sa lv o e s t e n o s is b ila te ra l o riñ ó n ú n ic o .

H TA

H T A e n d ia b é t ic o s .

- R e d u c e n la - H T A le v e o m o d e ra d a , - H T A e n el a n c ia n o (el i h i el ro p i r iel i ñas).

m o r b im o r t a lid a d a

m o r b im o r t a lid a d a largo

largo p la z o . - A c o n s e ja d o s en el

p la zo , - T r a ta m ie n to d e la

tra ta m ie n t o d e la

d is e c c ió n a ó r tic a .

H T A e n el a n c ia n o (tiaciclas).

A n tia r r ítm ic o s g ru p o II. T ra ta m ie n to d e v a ria s

A rritm ia s

P o d ría d is m in u ir la FA e n p a c ie n te s c o n IC C ,

- A n t ia r r ít m ic o s g ru p o IV. - E v ita r e n W P W e n FA o flutter,

a rritm ia s. Fren a do res d e l N AV . Q T largo c o n g é n it o , ta q u ic a r d ia s in u s a l sin t o m á t ic a , h ip e r t ir o id is m o ..,

1. ¿Cuál es el betabloqueante de menor vida media?

1. El esmolol.

2.

2. El sotalol.

¿Qué betabloqueante alarga el QT?

38 www.FreeLibros.me

i

C a r d io l o g ía CAPÍTULO 27. 1)

y c ir u g ía c a r d io v a s c u l a r

EN FERM EDADES D EL PERICARDIO.

PERICARDITIS AGUDA.

jbserva que esta enfermedad se puede confundir con un IAM, ya que clínicamente puede presentar: Dolor torácico. Alteraciones electrocardiográficas. Aumento de las enzimas mlocárdlcas (por afectación del epicardio). - j r ello, debes advertir en qué aspectos se diferencian ambas patologías: El dolor torácico aumenta en la pericarditis con algunas maniobras como el decúbito o la Inspiración, lo que no suele suceder en el IAM (características pleuríticas). Al Igual que en el IAM, hay elevación del segmento ST, pero en la pericarditis es cóncavo hacia arriba, mientras que en el IAM es convexo. Además, los cambios suelen ser más difusos en la pericarditis. Otras alteraciones que pueden aparecer en el ECG de la pericarditis son: negativlzaclón de ondas T, disminución del voltaje del QRS, descenso del Intervalo PQ y arritmias auriculares. En la auscultación se puede escuchar el roce pericárdico. Es poco frecuente que se eleven las enzimas mlocárdlcas en la pericarditis, y cuando lo hacen es porque hay miocarditis acompañante. Además, se elevan menos de lo que cabría esperar por la extensión de las alteraciones en el ECG. Hay muchas causas de pericarditis, pero la más frecuente es la ¡diopática (que es, en la mayoría de los casos, de i=.,sa vírica); repasa el tto. de esta y fíjate que en todas las pericarditis agudas están contraindicados los anticoagulantes. 2) DERRAME PERICÁRDICO

3)

1. ¿Cuál es el mejor método de diagnóstico del derrame pericárdico?

1. El ecocardiograma.

2.

2.

En un derrame sanguinolento, ¿en qué etiologías deberemos pensar?

En etiología tuberculosa o neoplásica.

TAPONAMIENTO CARDÍACO Y PERICARDITIS CONSTRICTIVA.

Son dos patologías del pericardio que debes diferenciar, por ello quizás es mejor que las estudies comparándolas irire sí y con la mlocardlopatía restrictiva.

DATOS C LÍN IC O S

PULSO V E N O SO

EC G

ECOCARDIO GRAM A

RADIOGRAFÍA DEL TÓRAX

TRATAM IEN TO - PERICARDIO-

Taponamiento

- Hipotensión. - Pulso paradójico.

- O n d a X prom inente. - O n d a Y dism inuida o ausente.

- D ism inución Q RS.

- Alternancia eléctrica.

C E \ T E S IS o ventana

- D erram e

pericártlica si ha)

perica rdico.

- A u m en to d e la

- C olap so de

silueta card íaca.

cavid ades

d e rram e purulento, urém ico o tuberculoso.

d erechas.

- C ontrain d icad o s diuréticos.

Pericarditis constrictiva

- C lín ica de I. cardíaca derecha crónica. - Signo de Kussmaul.

REPASA:

- 1/3 hay - O ndas X e Y prom inentes.

- Onda Y predom inante.

fibrilación auricular.

- QRS bajos. - A planam iento e

- A um enta el

- C alcifica cio n e s

espesor del

en 50% .

pericardio.

- Pericardiectom ía.

inversión onda T.

[ D i a g n ó s t i c o d if e r e n c ia l de la m io c a r d i o p a t í a r e s t r i c t i v a , T a b l a

72,

C a p ít u lo

25

del M a n u a l

CTO 8a E d .]

El taponamiento cardíaco suele aparecer en las preguntas como un evento agudo que requiere una Intervención urgente (en general, una perlcardlocentesis de urgencia). SI lo encuentras en relación con un IAM, debes diferenciarlo de un IAM de .entrículo derecho, que también cursa con hipotensión e ingurgitación yugular (fíjate si hay alteraciones en el ECG de precordiales derechas y recuerda que en el IAM de VD es raro que exista pulso paradójico). La pericarditis constrictiva cursa con síntomas de insuficiencia cardíaca derecha crónica. Debes diferenciarla de a miocardiopatía restrictiva, que también tiene morfología de raíz cuadrada en la curva de presión del ventrícu­ lo y signo de Kussmaul (observa que, sin embargo, en la pericarditis constrictiva pueden existir golpe pericárdico \ alteraciones de éste en la radiografía y en el ecocardiograma).

39 www.FreeLibros.me

ULTRA-RESÚMENES. Manual CTO de Medicina y Cirugía, 8.a edición

1.

¿De qué patología es típica la alternancia eléc­ trica en el ECG?

1. De aquellas que cursen con derrame pericárdico severo. Piensa sobre todo en taponamiento cardiaco.

2. ¿Por qué se produce el pulso paradójico?

2.

Por el desplazamiento del septo interventricular hacia la izquierda durante la inspiración, momento en el que llega más sangre al corazón derecho, y por ello, el corazón izquierdo aloja y expulsa menos sangre en la sístole y el pulso disminuye.

3.

¿A qué se debe la morfología en "Dip plateau" o en raíz cuadrada en la curva de presión ventri­ cular?

3.

Al cambio brusco de presión intraventricular durante la diástole, cuando el corazón se en­ cuentra con un pericardio poco distensible que frena su relajación.

4. ¿Por qué están contraindicados los diuréticos

4.

Porque dificultarían aún más el ya comprome-

CAPÍTULO 33.

EN FERM EDADES DE LAS VENAS.

TROM BOSIS VENOSA PROFUNDA. La causa más frecuente de TEP es la TVP; por ello, es bueno que relaciones ambas patologías.

De 3 a 6 m eses

40 www.FreeLibros.me

C

a r d io l o g ía y c ir u g ía c a r d io v a s c u l a r

ALGORITM O D IAG N Ó STICO DEL TROM BO EM BO LISM O PULMONAR.

ES TIM A C IO N D E LA P R O B A B IL ID A D C L ÍN IC A

Probabilidad m edia y baja

Probabili dad alta o pacientes ingresados

' IV D ím ero - D V

Alta

Normal

Técnicas de imagen

V

O bservación

Tratar

No IR o alergia al contraste

< ---- TV P

Normal o no el ¡agnóstico^"

IR o alergia al contraste

V

V

Angio TAC

y Grafía

Ultrasonidos de extrem ó dades

No diagnóstica

I Angiografía pulm onar

Debes recordar que un método analítico de utilidad para el diagnóstico del TEP es la medición del dímero D. Tiene un alto valor predictivo negativo. Basta una alta sospecha clínica para comenzar con el tratamiento anticoagulante.

1.

¿Qué habría que hacer después de instaurar tratamiento en un paciente con clínica sugerente de TVP y TEP?

1.

Continuar el estudio de su tromboembolismo venoso (ECO-Doppler, flebografía y/o gammagrafía).

41

www.FreeLibros.me

D

e r m a t o l o g ía

Al repasar la "Derma", presta mucha atención a los epónimos y a los datos característicos, y hazte una buena lista con ellos. No olvides que generalmente el dato "clave" que te lleva al diagnóstico es sólo uno, y que muchas veces es únicam ente la lesión elemental o su distribución en la piel lo que debes recordar.

C A P ÍT U L 0 1.

G EN ERA LID A D ES.

Com o ya sabrás, conocer las lesiones elementales en Dermatología es el principal paso para el diagnóstico, que es clin ico en la mayoría de los casos.

IZ ^

REPASA:

[L e s io n e s c u t á n e a s e le m e n ta le s , F ig u r a 6 , C a p ít u lo 1 d e l M a n u a l

CTO

8 a E d .]

1.

¿Qué diferencia a una pápula de un nodulo?

1.

La pápula es una lesión pequeña (< 1cm), palpa­ ble y levemente sobreelevada (liquen plano). El nodulo es profundo, y más palpable que visi­ ble (eritema nodoso).

2.

¿Y una escama de una costra?

2.

Las escamas son células secas (psoriasis); las costras, secreciones secas (impétigo).

3.

Lo caracterisitico del habón es...

3.

Su fugacidad ( Verdaderas: el liquen escrofuloso. > Facultativas: * Eritema nodoso (asociación frecuente a TBC, sobre todo en niños). * Eritema indurado de Bazin. 7. Z O O N O SIS Y ENFERMEDADES POR A RTRÓ PO D O S. Parásitos. Botón de o rie n te. Leishmania donovani var. infantum en España (vector: Phlebotom us). Mediterráneo.

- Niño con una lesión ulcero-costrosa en la cara de evolución crónica que deja cicatriz. - Diagnóstico por biopsia. Tratamos con antimoniales intralesionales. Sarna o escab iosis. El prurito nocturno y familiar y, a veces, los antecedentes (viaje a zonas tropicales) son los principales indicios para sc-pecharla. Es importante saberque la sarna humana excepcionalm ente afecta a la cabeza.

La lesión más específica es el surco acariño, en cuyo extremo está la eminencia acariña. La sarna noruega ocurre en inmunodeprimidos (muy contagiosa). El tratamiento es permetrina, lindane (contraindicado en niños y embarazadas). Ivermectina oral cuando hay resisten­ cia. La sarna nodular se trata con corticoides. Otros.

Recuerda:

-

Melanodermia de los vagabundos: piojo del vestido. Impétigo en sábana en el cuero cabelludo: descartar pediculosis. Máculas cerúleas: por P. pubis.

www.FreeLibros.me

53

ULTRA-RESÚMENES. Manual CTO de Medicina y Cirugía, 8.a edición

8.

ENFERMEDADES VIRALES.

TIPO

PRIMOINFECCIÓN

RECIDIVA

• + f r e c . a s in to m á tic o (9 5 % ).

- + f r e c . o r o la b ia l.

• C lín ic a :

HERPES EXTRAGENITAL

- En in m u n o d e p .

- + f r e c . g in g iv o e sto m a titis. VHS I

TTO.

= ú lc e ra c r ó n ic a .

- O tra s : Q u e r a to c o n ju n tiv itis .

P r im o in fe c c ió n :

- Si re c id iv a m u y

P a n a d iz o .

fre c u e n te tto

H e rp e s g la d ia to ru m . E ru p c ió n v a r ic e lif o r m e .

A c ic lo v ir 2 0 0 m g / 5 h 5 - 7 d ía s

su p re s o r 6 m e se s. V a la c ic lo v ir

• C a u s a + f r e c . d e ú lc e ra g e n ita l

HERPES GENITAL

8 0 % V H S II 20% V H S I

5 0 0 m g / 1 2 h de 5 - 1 0

e n o c c id e n te d e s p u é s d e las

d ía s

t ra u m á tic a s .

+ Levo .

- L e s io n e s v e s ic u lo s a s . - 3 0 % m e n in g itis a s é p tic a . H e rp e s zó ste r.

VARICELLA ZOSTER

V a ric e la

H e rp e s z ó s te r e n :

D o lo r in te n s o ,

In m u n o d e p r im id o s .

e ru p c ió n

> 5 5 a ñ o s.

m e ta m é ric a u n ila t

F o rm a s e s p e c ia le s .

Otros. VPH: condilomas acuminados: se asocian con más frecuencia a VPH 6 y 11. VPH 16, 18 se asocian a papulosis bowenoide y son los más oncogénicos. Tratamos con podofilino (no en embarazo), crioterapia, infiltraciones con interferón alfa, imiquimod.

1. Cuadro cutáneo mediado por inmunocomplejos que típicamente ocurre 10-15 días después de un brote de herpes simple.

1. Eritema exudativo multiforme.

2.

2.

Vesículas umbilicadas agrupadas en racimo so­ bre una base de eritema.

3. ¿Cuál es la complicación más frecuente de la varicela? ¿Y la más grave?

3.

Sobreinfección de lesiones. Neumonía.

4.

4. Carbamacepina, antidepresivos tricíclicos, capsaicina tópica.

Lesión típica del herpes.

Tratamiento de la neuralgia postherpética.

CAPÍTULO 7.

EN FERM EDADES ERITEM ATODESCAM ATIVAS.

Se identifican conociendo su localización típica y su lesión elemental. A veces en el MIR te dan alguna clave que confirma la sospecha (p.e. Wickham en el liquen ruber plano), pero saber qué y dónde te debería bastar para diagnosticarlas. Fíjate en el esquema (están numeradas según su frecuencia). Falta la pitiriasis rosada de Gibert, que por ser aguda, autolimitada y presentar el medallón heráldico, hace su diagnóstico fácil. Recuerda que es típica de jóvenes, en primavera/otoño, y por su aspecto y edad de aparición, obliga a descartar una sífilis secundaria. No requiere tratamiento. - Todas son crónicas o cursan con brotes, y salvo la dermatitis seborreica (que es patogénicamente bastante distinta), todas presentan fenómeno de Koebner y se tratan de forma sintomática (la seborreica con ketoconazol). - El prurito NO es un signo característico de la psoriasis, y sí lo es del liquen plano. - Los dos trastornos de la queratinización (pitiriasis rubra pilaris y Darier) tienen en común: el ser infrecuentes, y tratarse con queratolíticos y/o retinoides y no responderá los corticoides.

54 www.FreeLibros.me

D

e r m a t o l o g ía

Enfermedades eritematodestamativas. DERMATITIS SEBORREICA PITIRIASIS RUBRA PILARIS

• Áreas seborreicas: c cabelludo, cejas, surcos nasogenlanos, retroauocular, preesternal, Ingle

• Eritema + descamación amarillenta grasienta • Costra láctea, falsa tiña am lantácea, erltroderm la de Lelner

• Tronco y extrem id ad es con los típicos islotes blancos de piel Indem ne • Pápulas hiperqueratósicas foliculares (dorso dedos) junto a eritema anaranjado extenso. H lperq ueratosls palm oplantar am arillenta • Inicio brusco, luego cro n lflca d ó n . En m uchos casos d esaparece sola

• Intensa en enferm edades neurológlcas y SID A

DARIER • Caras laterales del cu ello, centro del tronco y áreas seborreicas • Pápulas pequeñas marronáceas malolientes. Peor con el sol • DISQUERATOSIS (típica) • D ep resio nes puntlform es en palm as y plantas; alts. ungueales

LIQUEN RUBER PLANO • Caras de flexión de antebrazo y pierna, m ucosa oral, sacro

• Pápulas poligonales rosadas, brillantes • Signos: estrías de Wickham, reticulado, cuerpos coloides de Civatte • Autoinmune (celularj • Poco frec pero típ ico : esp inocelular oral, pterigium ungueal, alo p ecia Irreversible

PSORIASIS • C odos, rodillas, cu ero cab ellud o, zonas de roce

• Placa eritematosa cubierta por escama blanco plateada • Signos (B ro cq ): bujía, rocío de Auspitz, membrana de Duncan Burque • Hlperqueratosls o rto qu eratóslca+ áreas de paraqueratosls • Papllom atosls, acantosls, hipogranulosis

• Neutrófilos (M unro y Kogoj)

Psoriasis. Piensa en ella ante cualquier eritema o descamación, porque es muy frecuente y no es raro que sea atípico en su esentaciÓn o localización (pustulosis palmar, en pliegues...). La biopsiaayuda bastante. Recuerda su patogenia: aumento de la velocidad de crecimiento de la piel y activación de la inflamación que explican la histología (fíjate). Forma más grave: pustuloso generalizado o von Zumbusch (después la eritrodérmica). La forma más frecuente es la psoriasis en pequeñas placas. Desencadenantes: litio, betabloqueantes, estrés, alcohol, suspensión de corticoides, infección estreptocócica, traumátismos. Mejora con sol, calor y humedad. La artritis suele asociarse a psoriasis grave y a afectación ungueal, y suele aparecer después del cuadro cutáneo. Onicopatía: piqueteado (más frecuente), mancha de aceite, onicólisis (más característico). Tratam iento de la psoriasis. [ h i s t o l o g í a d e l a p s o r ia s is y d e l li q u e n p l a n o , T a b l a

8,

C a p ít u lo

[ TRATAMIENTO DE LA PSORIASIS, TABLA 9 , CA P ÍTU LO 7 DEL M AN UAL

7 del

M anual

CTO 8 a E d .]

CTO 8 a E d .]

Ui

REPASA:

www.FreeLibros.me

ULTRA-RESUM EN ES. Manual CTO de Medicina y Cirugía, 8 .a edición

CA PITU LO S 21, 2 2 , 24, 25 Y 26. 1.

N EO PLA SIA S CU TAN EAS._______________________

EPITELIOMAS. REPASA:

[ D if e r e n c ia s e n t r e b a s a lio m a y c a r c in o m a e p id e rm o id e , T a b l a

25,

C a p ít u lo

22

del M an u al

CTO 8a E d .]

1.

¿Cuál es el factor causal más importante en los epiteliomas?

1.

La exposición solar crónica. También tener un fototipo de piel I y II.

2.

¿A qué corresponde la siguiente descripción his­ tológica: "Células pluripotenciales de capa basal de epidermis proliferando en forma de nidos y cordones celulares en empalizada?

2.

Epitelioma basocelular.

3. ¿En qué localización es más frecuente el espinocelular que el basocelular?

3. En las manos.

4. ¿Cúal es el tipo de epitelioma basocelular más destructivo?

4 El ulcus rodens.

Precancerosis. R E PASA: [ P r i n c i p a l e s P r e c a n c e r o s i s , T a b l a 24, C a p í t u l o 21 d e l M a n u a l C T O 8 a E d . ] Existen precancerosis susceptibles de cirugía que se pueden extirpar (nevus sebáceo de Jadassohn, nevus displásico...). Las que se deben a daño crónico de la piel se pueden tratar con retinoides y fotoprotección. Las queratosis actínicasse tratan con 5LU tópico, crioterapia, imiquimod tópico o cirugía.

2.

LINFOM AS.

Son LNH de baja agresividad. La mayoría son T. Los más frecuentes son la m icosis fungoide (ML) y el síndrome de Sézary, que constituyen la mayoría de los de células T. Líjate que los B suelen ser extracutáneos y "metastatizan" en la piel, dando una historia breve (10% células Sézary

- Q U IM IO T E R A P IA

POI.IQUIMIOTERAPIA

Un posible nuevo tratamiento es el bexaroteno.

56

www.FreeLibros.me

D

L-

e r m a t o l o g ía

.cerda que es de células T-helper (CD4) y que el diagnóstico lo aporta la histología típica del estadio en placas filtració n epidérmica con microabscesos de Pautrier; el estadio inicial es inespecífico y el final (tumoral) es genera. menos característico. : ■'ases finales puede haber transformación blástica e infecciones-sepsis por estafilococo. Sezary. : a leucémica de la MF.

¿Cuál es la leucemia que más habitualmente da clínica cutánea?

1.

La leucemia monocítica acucia, seguida por la leucemia mietocítica aguda (cloromas).

O T R O S TUM ORES.

4.

'

Un lactante de 2 meses presenta un cuadro cu­ táneo sugestivo de dermatitis seborreica en cuero cabelludo, cuello, axilas y área del pa­ ñal. A la exploración destaca la presencia de adenopatías, hepatoesplenomegalia y una otitis supurativa. Días después, se hace una analítica que evidencia anemia. ¿Qué te sugiere?

1.

Letterer-Siwe.

2.

Un niño con depresiones puntiformes en pal­ mas y plantas, asociadas a múltiples basaliomas cutáneos, que a su vez presenta retraso mental y cara de "síndrome" (con malformaciones ocu.ares y craneofaciales), tiene...

2.

Síndrome de los nevus basoceljlares Corlim.

3.

La inmunohistoquímica positiva para S-100 y OKT6 sugiere...

3.

Células de Langerhans (no olvides los gránulos de Birbeck).

Un nodulo cupuliforme de unos 2 cm, lobulado y duro, móvil, con cráter central cubierto por un tapón queratósico en la sien de un anciano.

4.

Queratoacantoma (involuciona solo, diferenciar de espinocelular y leishmaniasis).

5.

Una placa eccematosa en la aréola mamaria de una anciana, sugiere...

5.

Paget (descartar carcinoma ductal de mama aso­ ciado). Recuerda: extramamario más frecuen­ te = vulva.

ó.

Un neonato presenta un nodulo en la espalda que con el roce se edematiza y enrojece du­ rante horas...

6.

Mastocitoma. Es más frecuente la urticaria pig­ mentosa, otra mastocitosis que da un cuadro generalizado maculopapular y síntomas sistémicos si se roza (dolor abdominal, hipotensión, ta­ quicardia,...); distinguir de la roséola sifilítica.

7.

Los angioqueratomas son característicos d e...

7.

Fabry (esfingolipidosis).

8.

Región anatómica que desarrolla queloides con más frecuencia.

8.

Preesternal.

9.

El tumor anexial más frecuente e s...

9.

Pilomatrixoma.

DERMATOSIS PARANEOPLÁSICAS.

Recuerda el cuadro clínico típico del glucagonoma, síndrome de las cuatro D: diabetes, depresión, dermatitis (eritema errolítico migratorio) y deep vein throm bosis (TVP). La acantosis nigricans muestra piel aterciopelada grisácea en pliegues. La forma maligna presenta afectación de -_;co sa s e hiperqueratosis palmoplantar (no en la benigna). Causas de acantosis nigricans benigna: Hereditaria (infancia, A.D.). 5 tuaciones con resistencia a la insulina: obesidad, diabetes, acromegalia, Cushing y tratamientos con estrógenoácido nicotínico e hidantoínas.

57

www.FreeLibros.me

ULTRA-RESÚM ENES. Manual CTO de Medicina y Cirugía, 8 .a edición

CA PÍTU LO 11.

ACN É. ACNÉ

E T IO P A T O G E N IA

RO SÁCEA - L a b ilid a d v a so m o to ra - D e m o d e x f o llic u lo r u m - E n fe rm e d a d e s g a stro in te stin a le s

- A lt e r a c ió n d e q u e r a t in iz a c ió n in fu n d ib u la r. - P r o p io n ib a c t e r iu m a c n é s .

P O L IM O R F IS M O A b ie rto s

C L ÍN IC A

Com edones—> pápulas—> pústulas—> nodulos—> quistes C erra d o s LE V E

1.

N o com ed o nes F lu sh in g facia les E ritem a p e rs is te n te (cu p e ro sís) T e la n g ie c ta sia s L e s io n e s o ftá lm ic a s o h ip e rp la sía de te jid o s b la n d o s

GRAVE

D IA G N Ó S T IC O

T R A T A M IEN T O

-

C L ÍN IC O

C L Í N IC O

L E V E (c o m e d o n e s y p a p u lo so ): tra ta m ie n to tó p ic o , t e n d e n c ia a la re c u rre n c ia . - P e ró x id o d e b e n z o ílo - R e tin o id e s tó p ic o s - Á c id o a z e la ic o . - a n tib ió tic o s: d o x ix ic ic lin a al 1 % o e ritro m ic in a al 2% M O D E R A D O (p á p u lo p u s t u lo s a s y n o d u lo q u fstic o s ): - T ra ta m ie n to o ra l: a n t ib ió t ic o s : d o x ic ic lin a y m in o c ic lin a , n u e v o s d e riv a d o s d e la e ritro m ic in a . - A n tía n d ró g e n o s . - Iso tre tin ifn a o ra l. G R A V E ( c o n g lo b a ta y fu lm ín a n s) - A n tib ió tic o s o ra le s - C o r tic o íd e s o ra le s - Isotre tin o fn a o ral

¿Cómo se trata la dermatitis perioral?

1.

- E vitar e stím u lo s v a so d ila ta d o re s - D o x ic ílin a o m in o c ic lin a o ral de e le c c ió n . - M e tro n id a z o l tó p ic o c o m o m a n te n ím ie n L o . - C a s o s se v e ro s: is o tre tin o fn a oral

Con tetraciclinas orales. Aparecen en mujeres jóvenes y estresadas, con brotes de pápulas periorales que dejan en el área perilabial una banda libre.

58 www.FreeLibros.me

D CA PÍTU LO 20.

e r m a t o l o g ía

FACO M ATO SIS.

N EU RO FIBRO M A TO SIS TIPO I

ESCLEROSIS TU BERO SA

C ro m o so m a 1 7

C ro m o so m as 16 y 9 Facom as A strocitom a

C lio m a óptico

N o d u lo s d e Lisch (m uy esp ecíficos)

Arigiufibromas faciales Neurotibromas cutáneos

(Pringle) \

M

Rabdomíomas

Efélid es axilares (signo de C ro w e )

c a rd ia c o s N

Crisis convulsivas M a n ch a café con

(Sd. de West)/''""'

leche (> 6)

C ifo esco lio sis A n g io m io lip o m as b ilaterales 'e ocrom o citom a

Fibrom as ungueales (T. de Ko enen)

N EU R O FIBRO M A TO SIS T IP O II C ro m o so m a 2 2 M an ch as p o cró m icas

N e u ro fib ro m as b ilaterales del VIII p< Sord era n eurosenso rial re tro co clear A fe ctació n del facial El tu m o r surge de la ram a vestib u la r

P laca d e piel d e za p a (región lum bo sacra)

C lín ic a de fosa posterior

VON-HIPPEL-LINDAU

STU RGE-W EBER

C ro m o so m a 3

Angiomas en piel X

Hemangiomas ; en retina

F H IA 2 a

a

fe o r rom fie ito m a

Tu m ores cereb eloso s (H em ang io b lasto m a cereb eloso ) (ataxia, vértigo)

M a n ch a vin o d e O p o rto (trigém ino) A ng io m atosis lep tom en ín gea C a lc ific a c io n e s en "vía de tren " C risis e p ilé p tica s

C a rc in o m a renal y q uistes renales

www.FreeLibros.me

ULTRA-RESÚMENES. Manual CTO de Medicina y Cirugía, 8.a edición

CA PITU LO 23.

MELANOMA ¿En qué p o rce n ta je tie n e n m e tástasis?

C a ra c te rís tic a s típ icas del m ela n o m a :

- Es el cáncer que más ha aum entado su incidencia. - O curre a edades más precoces que los epiteliom as (40 años). - Afecta a áreas distintas: Cabeza y cuello en varones. Piernas y espalda en m ujeres. - M ás frecuente en m ujeres (2/3). - M u y m etastatizante.

Hasta el 65% (2/3). M e tá sta sis m ás fre cu e n te s:

Satélites en piel cercana (vía linfática), después ganglionares. También hem áticas: pulmón y S N C (1 - causa de muerte).

Signos alarm a (A B C D ): sim etría de la lesión o rd es irregulares am bio de color iám etro > 6 mm

L EN T IG O M ALIGNO

EX TEN SIO N S U P E R F IC IA L

10%

70%

M ujeres ancianas

M ujeres jóvenes

90% en cara/zonas fotoexpuestas de pieles envejecidas

30% nevus previo. Exposición intermitente. ■ — ' H om bre: espalda ^ M ujer: piernas

X

M ancha que crece durante muchos años (> 1 0 ), luego profundiza (nodulo). Epidérm ico.

/

M ácula con mosaico de colores que crece 4-5 años y luego Infiltra (nodulo). Metástasis 35-70% .

/ M EJOR PRO NOSTICO

M ELANOM A N ODULAR

M AS FRECUENTE

X

4

\ /)

15%

i

Hom bres de edad media Sobre piel sana. C u alquier zona. Aparición repentina. Nodulo negro uniforme. Rápidam ente invasor IB (crecim iento vertical sin radial). B Frecuente ulceración y sangrado.

5-1 0% (60% en negros y orientales)

'

M ujeres ancianas Planta del pie (talón), manos, lecho ungueal... Sin relación con fotoexposición.

1 i N ¡YOi tlv Clark

Inirai’picfónniro

III di B f; II

IV

V ftifAl

0 l'S Á É t0 :- fls lji

Xd,-

L EN T IG IN O SO A C R A L

/

V

IihIhc de Wm [>n-s|o\\ H

M ácula en mosaico (m ancha que crece). Algunos am elánicos. Mal pronóstico por diagnóstico tardío.

0 mm

M EN OS FRECUENTE

PEO R PRO NOSTICO EscjímI invasión ck’rmii pjiipiUr

0,75 mm

Invasión lot.il

1,5 mm

lierm Ei papiLir

Invasión tkrm is rcüc til.tr

4 mm

H ip o d e rm is (tej. ad ip o so )

Melanoma precoz (supervivencia a 5 años 99%): M elanom a ¡n situ (epidermis) o derm is < 1 mm

Tratamiento: extirpación quirúrgica;

Factores p ro n ó stico s (por orden):

- Profundidad (Breslow/ Clark). - Ne de ganglios afectados, satelitosis. - Localización BAN S, peor pronóstico. - M elanom a nodular. - Varones/ jovenes. - U lceración, respuesta inflamatoria ausente, índice m itótico, tipo histológico.

- Breslow < 1 mm; margen 1 cm . - Breslow > 1 mm; margen 2 cm . Cuan do Breslow > 1 mm hay que explorar el ganglio centinela. Ganglio +

IFN a 2 b

Ganglio Breslow > 4 m m Breslow < 4m m

60 www.FreeLibros.me

IFN ot2b nada

D CAPÍTULO 8.

e r m a t o l o g ía

ECCEM A Y DERM ATITIS ATÓPICA.

ECCEMA DE CO N TA CTO .

M ECAN ISM O

A LÉ R G IC O

IRRITATIVO (MÁS FRECU EN TE)

H ip e rs e n sib ilid a d tip o IV

No in m u n o ló g ico

Agudo: v e sícu la s-a m p o lla s. Subagudo: c o stra s-d e s ca m a ció n . Crónico: liq u e n iflc a c ló n -íls u ra c ló n .

C LÍN IC A

LO C A LIZ A C IÓ N

D o rso de m an o s

- T o d a la m an o - M á s fre c u e n te en a t íp le o s

• N íq u e l-m u je re s -b lsu te ría

SU STA N CIA Q U E LO PRO DUCE

• C ro m o - v a ro n e s-c e m e n to , b isu te ría , •

- Jabon es

c a lz a d o

- D e te rg e n te s a lc a lin o s

Rarafe n ile n d ia m i n a-tin te s-p e lu q u e ría s

- D iso lv e n te s

• M e d ic a m e n to s tó p ico s (n e o m lcín a )

D IA G N Ó S T IC O

2.

P ru e b a s e p lc u tá n e a s po sitivas

P ru e b a s e p lc u tá n e a s negativas

DERMATITIS ATÓPICA. No olvides que es frecuente. Ictiosis vu lg ar

A S O C IA C IO N E S .

A lo p e c ia are ata V itÍligo

FO RM AS M EN O R ES.

ESTIGM AS. -Hertogue -Dennie-Morgan -Cataratas -Lengua geográfica

Predisposición

(genética, inmunol, atópica)

-Piel seca, pálida -Infecciones cutáneas frecuentes

LA C T A N T E (2 m eses-2 años) Eritema vesicular

-Q ueratocono

en cara que respeta ángulos nasogenianos

-Pitiriasis alba -Eccem a num ular -Dishidrosis -Prúrigo sim ple y nodular

IN F A N T IL (2-10 años) Prurigo y liquenificación en flexuras

10-20% A D U L T O (10-25 años) Prúrigo y liq u en ificació n dorso manos y pies, cara y cuello



PRURITO siempre----------

Tto : A ntih istam ínicos generales, corticoides tópicos, em olientes Los graves: corticoides generales, ciclo sp o rina, P U V A , tacrolim us y p im ecrolim us tópico

Ten en cuenta que los eccemas crónicos son muy "susceptibles" y empeoran con todo (estrés, ambiente frío/seco estimulantes...) y que es imperativo cortar el prurito. Evita en general los corticoides sistémicos en las dermatitis cróni­ cas, porque éstas recidivan al suspenderlos. Recuerda que el prúrigo es un tipo especial de pápula que es pequeña, roja, redonda, con una vesiculita encima . mu; pruriginosa.

6'

www.FreeLibros.me

U LTR A -R ESÚ M EN ES. M anual C T O de M edicina y Cirugía, 8 .a edición

CA PÍTU LO 10. 1Z^REPASA:

TOXICO D ERM IAS.

[ D i a g n ó s t i c o d if e r e n c ia l de e rite m a s m u lt if o r m e s , T a b l a

12,

C a p ít u lo

10 d e l

M anual

CTO 8a E d . ] .

¿Qué es el herpes iris de Bateman?

1.

Lesión típica en diana del EEM con vesícula encima.

2. Causas más frecuentes de eritrodermia.

2.

Por orden: eccemas, psoriasis, fárm acos, linfomas.

3. Existe una diferencia fundamental entre el eri­ tema exudativo multiforme minor y mayor, ¿la recuerdas?

3.

El EEM mayor tiene afectación mucosa.

4. ¿En qué estrato de la piel se localiza la lesión de la necrólisis epidérmica tóxica?

4. En la capa basal de la epidermis. Recuerda que tiene signo de Nikolosky positivo.

1.

62 www.FreeLibros.me

D CAPÍTULO 32.

ig e s t iv o y c ir u g ía g e n e r a l

HEPATITIS VÍRICA S.

Recuerda que el cuadro clínico es sim ilar para todos los virus de la hepatitis: síntomas constitucionales, fiebre, y en a (ase de estado, ictericia y hepatomegalia, pero no olvides que es más frecuente la hepatitis anictérica. Recuerda ambién que, cuanto más sintomática sea, menor es la posibilidad de sufrir una hepatitis crónica. En ocasiones pueden presentarse formas fulminantes (encefalopatía e insuficiencia hepatocelular reflejada por un ■empo de protrombina menor del 40%), que son más frecuentes en la hepatitis B (sobre todo coinfección B-D y mtantes precore) y con la E en embarazadas.

VHB

VH A

M. transm isión

60-90

30

P. incubación

- Percutánea - Perinatal sexual

Fecal-oral

VHC

VHD

VHE

50

60-90

40

Como B

Fecal-oral

- Desconocido (40%) - Percutánea

Clínica

Cronicidad

< 1 % Hep. agudas 10% Hep. subclínicas

Nunca

- Coinfección = VHB - Sobre infección casi 100%

80% (20-35% van a cirrosis)

Edad

90% en RN 1% adultos

Nunca

C oinfección B-D

Particularidades

Diagnóstico - infección aguda - Infección crónica

- Infección pasada

La que más - Frecuente diarrea en niños frecuentemente tiene - STs + frecuente en adultos manifestaciones extrahepáticas (rash, - Colestasis artritis, GN ...)

- IgM anti VHA - No existe - IgC anti VHA

- ig - Vacuna virus

- Anti VHC Se usa el RNA-VHC pa ra : - Confirmar el DG - Monitorizar el tto. - IMD

- IgM anti HBc - HBsAg + IgG anti HBc - IgC anti HBs

Sobreinfección B-D

- >% fallo hepático severo (20%) - Cronifica casi 100%

- Colestasis - Hepatitis fulminante en 20% embarazadas

- Co infección; IgM anti HBc-f anti H D IgM - Sob re infección: IgG anti HBc+ IgM anti HD

- A n ti V H E - N o e xiste

- Medidas higiénicas

- Medidas higiénicas

Profilaxis

- La más importante de las hepatitis post transfusionales - Crioglobulinemia

- >% hepat. fulminante sobre todo en ADVP

inactivados

- Screening en

- Ig - Vacuna

donantes de sangre

- Igual que VHB

- M e d id a s hig ién icas

recombinante

Ten en cuenta que: - La hepatitis A y E son similares: transmisión fecal oral, no se cronifican, presencia de colestasis. - El VEHB es el único cuyo genoma es AD N , siendo los demás ARN virus. Es muy importante que tengas claro el diagnóstico serológico de la hepatitis B.

HBsAg

ANTI-HB s

ANTI-HB c

+

IN TERPRETACIÓ N

-

Ig M

In fecció n ag ud a.

+

-

Ig C

In fe cció n c ró n ic a .

-

-

Ig M

P erío d o v e n ta n a .

-

+

Ig C

-

+

-

Fase de re cu p e ra ció n d e la in fe cció n . Estado p o stv a c u n a c ió n . Falso p o sitiv o .

1.

¿Cuál es el primer marcador detectable en el suero de un paciente con hepatitis B aguda?

1. HBsAg.

2.

¿Qué tipo de IFN es el que se usa en el tratamiento de las hepatitis?

2. EL alfa.

63

www.FreeLibros.me

ULTRA-RESÚMENES. Manual CTO de Medicina y Cirugía, 8.a edición v además es positivo el HBeAg, la infección es muy contagiosa, pues es un indicador de la actividad replicativa, aunque el mejor marcador serológico de replicación viral es el ADN-VHB. Observa que tener el ant¡HBs+ es sinónimo de N O padecer la enfermedad (curación/vacunación). ^^REPASA:

[ F ig u r a s

97

y

98,

CAPÍTULOS 36 Y 37.

C a p ít u lo

32

del M anual

CTO 8 a E d .]

CIR R O SIS. COM PLICACIO N ES DE LA CIR R O SIS.

No olvides que el diagnóstico de cirrosis se realiza mediante una biopsia: fibrosis + nodulos de regeneración. En nuestro medio, la causa más frecuente es el alcohol, especialmente si asocia infección por VHC, seguido de las hepatitis víricas. Fíjate en que si los pacientes están compensados, no presentan clínica ni alteraciones analíticas. Para valorar la función hepática, se utiliza la clasificación de Child-Pugh: te será útil la regla nemotécnica BEATA. B Bilirrubina. E Encefalopatía. A Ascitis. T Tiempo de protrombina. A Albúmina. DA TO S DE LA B O RATO RIO Transa mi nasas.

A u m en ta C O T > CPT.

A lbúm ina.

D ism inuye en fases avanzadas.

Inm unoglobulinas.

A u m en to policio nal.

Factores de coagulación.

D ism inu yen todos m enos el VIII.

Act. fibrinolíticos.

Aum entan.

Plaquetas.

D ism inuye por hiperesplenism o.

C oleste rol.

Aum enta en las cirrosis biliares. Dism inuye en las cirrosis no biliares.

HIPERTENSIÓN PORTAL. La causa más frecuente de Hipertensión Portal (HTP) es la cirrosis. Sin embargo, debes conocer que hay otras causas de HTP que recogemos en el siguiente dibujo:

Tipos de hipertensión portal.

64 www.FreeLibros.me

D

ig e s t iv o

r

c ir u g ía g e n e r a l

1.

¿Cuál es la causa más frecuente de ascitis?

7.

La c ir r o s is h e p á tic a .

2.

¿Cuál es el síntoma más frecuente?

2.

E l a u m e n to d e l p e r ím e t r o a b d o m in a l.

3.

¿Cuál es el mejor método para confirmar su exis­ tencia?

3.

La e c o g ra fía a b d o m in a l.

A SCITIS.

Manejo de la ascitis. C lin ¡C

Sospecha diagnóstica Exploración

M atidez cambiante O leada ascítica

Confirmación:

e c o g ra fía (d e t e c t a h a s t a 100 mf

Paracentesis diagnóstica ( 500/mrrd Inflación

Hem atíes

> 50000/mm3neoplasia

Am ilasa

Ascitis pancreática

ADA

TBC

Trigicéridos

Ascitis quilosa

Masiva (a tensión)

A



No

Mejora

> 1,1: HT Portal

Albúm ina

* Recuerda que un gradiente a bómina érica-albúmina en líquido ascítico mayor o igual a 1,1 j^dl indica hipertensión portal.

D iu réticos ..

Gradiente

Paracentesis evacuadora (4-6 I)



, Paracentesis evacuadora

A"

No ▼ T iPs ^ Cirugía ‘ Shunt peritoneovenoso

A" Mejora

'A

M ejora

No

PERITO N ITIS BACTERIANA ESPONTANEA.

Diagnóstico. Leucos > 1 0 .0 0 0 Prot > 2 ,5

PB 2a

LD H > 225 Prot.

Sospecha clínica

Paracentesis

Leucos.

-D o lo r abdom inal

(Qué nos da e[ diagnóstico definitivo?

Cultivo A

-Fiebre

flora p o lim icro b ian a son frec. los anaerobios

C lu c < 5 0 mg/dl

Leucos > 5 0 0 PM N > 2 5 0

PBE

flora m on om icrobiana el más frec. E. coli

Tto. e m p írico —> ■cefas de 3á

14 Profilaxis*! ¡2 3

con n orfloxacino en pacientes de alto riesgo ¿recuerdas cuáles?: -HDA -Prot. en líq. ase. < I e, d -Antecedentes de PBE

¿En qué consiste la bacteriascitis m o n o m icro b iana?

► C u ltivo + con < de 2 5 0 PMN

¿Y su etiología más frecuente?

► Estafilococos

65

www.FreeLibros.me

ULTRA-RESÚMEIMES. Manual CTO de Medicina y Cirugía, 8.a edición

HEMORRAGIA DIGESTIVA ALTA (HDA) POR VARICES.

Manejo de la HDA. Cirrótico con sospecha vanees



, . r

► Diagnostico endoscopico °

.

p bloqueantes (hasta que ► disminuyan F.C. en 25%)

Prevención 1a-

_____

^ ¿Cuáí es ía primera medida a tomar? (1)

Y

Si hemorragia aguda ► Tratamiento etiológico -De elección: escleroterapia o ligaduras endoscópicas seguida de glipresin o somatostatina. -2- elección: -Somatostatina o gíipresina

7. _

-Taponamiento con balón, TIPS urgente o cirugía cuando fracasan las anteriores medidas.

.

Prevención 2a,ia

p bloqueantes + nitritos: la mejor opción. Escleroterapia. Ligadura con bandas. Shunt selectivo.

(1) Reponer la volemia

CIRUGÍA DE LA HTP. La indicación fundamental es el sangrado por varices esofágicas, no corregido con tratamiento conservador; otras son la ascitis refractaria o el síndrome de Budd-Chiari. Recuerda que el tratamiento quirúrgico nunca se hace si no ha habido síntomas. ^ ^ R EP A SA :

[C

i r u g í a d e l a h ip e r t e n s i ó n p o r t a l ,

F ig u r a 1 1 5 , C

a p ít u l o

37

d el

M

anual

CTO

8

a E d .]

Tratamiento quirúrgico. T ra n se cció n eso fág ica m e c á n ic a — > En h e m o rrag ia ag ud a en p a cie n te de alto riesgo

D ire cta s (p o co u tilizad as)

1Su g iu ra — > En tro m b o sis m asiva d el sistem a porta

/ / Técn icas / quirúrgicas

- P o rto ca va té rm in o -la te ra l — > Ú ltim o s re cu rso s, alta m o rta lid ad

\

- P o rto ca va la tero -lateral — > En a scitis reb eld e

N o se le ctivo s (alta

- M e s o c a v a — > N iñ o s

in c id e n c ia de

A scitis

e n ce fa lo p a tía hep ática)

B u d d -C h ia ri - E sp le n o rre n a l p ro xim a l o ce n tra l — > E sp le n o m e g alia con

D e riv a c ió n o

h ip e re sp le n ism o

sh un ts p o rto sisté m ico s

W a rre n S e l e c t iv a s

► ,, . , , (la m as u sada)

66

www.FreeLibros.me

i - ¡¡ D e e le c c ió n en c iru g ía e le c tiv a s

y ca n d id a to s a trasp la n te !! ■ 1 - N O e n ascitis

D

ig e s t iv o y c ir u g ía g e n e r a

Los TIPS son dispositivos colocados por vía endovascular, que hacen la función de una derivación portoca-.ade ib transitoria, hasta que llegue el trasplante. Nunca son un tratamiento definitivo. ENCEFALOPATÍA HEPÁTICA. No olvides que, simplificando, este trastorno se produce por una falta de eliminación o por un exceso de producción de amonio y otras sustancias: sedantes, HDA, alta ingesta de proteínas, estreñimiento, infecciones... El tratamiento consistirá en corregir los factores precipitantes:

Disminuir las proteínas de la dieta. Lactulosa, que al disminuir el pH intestinal, reduce la producción de amonio. Antibióticos: neomicina y metronidazol (sólo en agudas). Flumacenil. ¿Recuerdas por qué es útil?

Fisiopatología.

BY-PASS

CIRCU LATORIOS (evitan 1er. paso hepático)

SÍNDROM E HEPATORRENAL. Te interesa saber que, ante un cirrótico que presenta insuficiencia renal progresiva y tenga una FENa + de < 1%. jede tratarse de: Insuficiencia renal prerrenal: responde a la expansión de volumen. Síndrome hepatoirenal: falta de respuesta a la expansión de volumen. No se reconocen factores precipitantes, excepto la peritonitis bacteriana espontánea y hepatitis alcohólica.

www.FreeLibros.me

.

ULTRA-RESÚMENES. M an u al C T O de M edicina y Cirugía, 8 .a edición

CA PÍTU LO 33.

FÁRMACOS E HÍGADO.

1.

Relaciona cada fármaco con su lesión hepática:

1. 2. 3. 4. 5. 6. 7. 8.

Paracetamol. Anticonceptivos orales. Metotrexate. Metildopa. Ac. valproico. Cloruro de vinilo. Clorpromacina. Eritromicina.

a. b. c. d. e. /• 3h.

Necrosis tóxica. Fibrosis hepática. Esteatosis microvesicular. Colestasis. Angiosarcoma. HT portal. Hepatitis aguda (tipo vírica). Adenoma.

Respuestas: 1-a, 2-h, 3-b, 4-g, 5-c, 6-e-f, 7, d, 8-d.

CAPÍTULO 34.

H EPATITIS CRÓNICA.

La hepatitis crónica viene definida por la persistencia de la inflamación más allá de 6 meses. La causa más frecuente corresponde a las hepatitis víricas (B, C y D), y dentro de ellas, la C. Recuerda algunos fármacos que la producen: alfametildopa, ketoconazol, nitrofurantoína, oxifenisatina.

ANATOMÍA PATO LÓ GICA DE LAS HEPATITIS CRÓNICAS.

!

H EPATITIS C R O N IC A

ESPA C IO PORTA

ACTIVA

N ORM AL

Lesión en sacabocados *\d e lámina limitante mí'*‘

A. hepática V. portal C . biliar

V

Vena centfolobuHilar

/ /

Necrt ó l. .• i p, ■ ¡ I■ ' VX

sr .tíSL.

JK

■ )'

Nt; ■,, ' .

■ [¿5 /

V / Placa limitarle

'

ele .1

Mi im inut Ifsire-, limitado

HEPATITIS C R Ó N IC A PERSISTENTE r. aeioi/FZ

1.

¿Cómo es la anatomía patológica de la hepatitis crónica lobular?

1.

Igual que la de la persistente, es decir, con un infiltrado mononuclear limitado al espacio por­ ta con membrana limitante conservada, a lo que hay que añadir focos de necrosis e infla­ mación en el lóbulo hepático.

Repasa la hepatitis crónica autoinmune: Etiología desconocida asociada a autoanticuerpos e hipergammaglobulinemia. Asociación a múltiples enfermedades autoinmunes. - Tratamiento de elección: esteroides ¡Ojo! aquí no se pueden utilizar pautas en días alternos.

68

www.FreeLibros.me

D

CIRUGÍA GFNFEAI

ig e s t iv o y

Autoanticuerpos:

-

HAI tipo 1: antimúsculo liso. HAI tipo 2: anti - LKM 1. HAI tipo 3: anti - Ag soluble hepático (SLA).

1. ¿Qué otra enfermedad presenta los anti - LKM?

1.

La infección crónica por VHC.

2.

2.

Los niveles de ADN-VHB en suero son bajos y las transaminasas están elevadas.

3.

La infección se debe a cepas matantes precore.

La respuesta al interferón en el VHB es mejor si...

3.

Y es peor si...

CAPÍTULO 35. TRASTORNOS HEPÁTICOS ASOCIADOS AL CONSUMO DE ALCOHOL. Recuerda que la hepatopatía alcohólica representa u n espectro de posibilidades clínicas: esteatosis hepática-hepatitis cohólica-cirrosis, pudiendo ser reversible el paso de un nivel a otro. Fíjate que un nivel G O T/G PT> 2 es sugestivo de roatitis alcohólica.

1.

¿En qué otras enfermedades aparece la hialina de Mallory?

CAPÍTULO 15.

1. Deficiencia de alfa-1-antitripsina, Wilson, cirrosis biliar primaria, diabetes mellitus, esfe­ roides, amiodarona.

M ALABSORCIÓN.

Manejo del enfermo con malahsorción. Clín ica de sospecha -D iarrea

Screening: cuantificación grasas en heces/24h -



-Disminución de peso -M alnutrición -M alestar a b d o m in a l y distensión

r Esteatorrea (> 6 g en 24 horas) Y

P ru eb a d e la D -x ilo sa , R x b a rio , e stu d io ¡n m u n o ló g ico y m ic ro b io ló g ic o

N o rm a l

A n o rm a l

M a ld ig e stió n

(< 4 ,5 g en orina a las 5 horas

(in su f. p a n c re á tic a e x o c rin a )

de dar 25 8 de D-xilosa)

M a la b so rc ió n

A n c ia n o s

Falsos

A scitis

Test d e se c re tin a (la + se n sib le )

In su fic ie n c ia ren al

Test de p a n c re o la u ry l

Y

P ru e b a d e D x ilo sa C 14 lactulosa H i

Tto: enzim as pancreáticas

N o rm al

A n o rm a l

M a la b so rc ió n

S o b re c re c im ie n to , b a c te ria n o

Rx bario (flo cu lad ó n y fragm entación)

C u ltlvo de blopsla: co n firm ació n

Blopsla Intestinal, siem p re diagnóstica e n:

> I U ’ gorg/cul: so sp ech a

-Abetalipoproteinem ia: enterocito s llenos de I(pidos H ipogam m aglobulinem ia: au se n cia de cels. plasm .

> 10

gorg/cul: anorm al

p red o m in a n an aero b io s y coliform es

-Enf. d e W hipp le: m acrófagos co n Inclusiones PAS +

r

Tetraciclina

Tto: antibiótico

A m picilin a CTM X

www.FreeLibros.me

59

ULTRA-RESÚMENES. Manual CTO de Medicina y Cirugía, 8.a edición ^1

Etiologías importantes de la malabsortión.

Sobrecrecimiento bacteriano

Patogenia

Etiología

-Deconjugación de ác. biliares: lo + impte. -Consumo de vitamina B |2 -Daño directo de la mucosa Alteraciones anatóm icas -Alt. absorción de H. de carbono (divertículos, estenosis

H ip o d o rh id ria

Pancreatitis

Inm uno-

H ipom otilidad

crónica

deficiencias

(esd erod erm ia,

alt. postquirúrgicas), fístulas, resección de

pseudobstrucción, hipotiroidism o, am iloidosis,

válvula iliocecal.

-Malabsorción Biopsia diagnóstica: -Artritis no deformante -Fiebre ^ Macrófagos con inclusiones PAS+ -Linfadenopatías -Alt. neurológicas en mucosa y ganglios -Enteropatía pierde proteínas

neuropatía diabética)

Ziehl-Neelsen positivo

M . av¡um -¡ntracellulare

Ziehl-Neelsen negativo

Tropheryma whippelii

Bacilos

► Enf. de Whipple Sin tío. es mortal

Tto. de elección C T M X al m enos 1 año

CA PITU LO S 44 Y 45.

PAN CREATITIS AGUDA Y PAN CREATITIS CRONICA.

Pancreatitis aguda

Etiología

1. L itia s is b ilia r.

Pancreatitis crónica

1. A lc o h o lis m o c ró n ic o (fibrosis q u ístic a en n iñ o s).

2 . A lc o h o l.

D o lo r c ró n ic o .

Clínica

M a la b so rc ió n .

D o lo r típ ic o .

D M (p é rd id a 9 0 % del p á n cre a s). A m ila s a y lip a sa n o rm a le s (no sie m p re ).

BQ:

Diagnóstico

- A m ila sa a u m e n ta d a . - L ip a sa a u m e n ta d a .

C a lc ific a c io n e s en p ru e b a s d e im ag en . C P R E (e le cció n ). P ru eb a d e la s e c re tin a . A n a lg e sia si d o lo r.

Tratamiento

A n a lg e sia .

E n z im a s p a n c re á tic a s.

D ie ta a b so lu ta .

C iru g ía :

Si se ve ra y c a u sa b ilia r: p a p ilo to m ía

- D o lo r in c o n tro la b le (lo m ás fre cu e n te ). - Ic te ric ia o b stru ctiv a .

p o r C P R E en

3 6 -7 2 h o ra s.

V a lo ra r c o le c iste c to m ía .

- Im p o sib ilid a d d e d e sca rta r n e o p la sia . - C o m p lic a c io n e s . O b s tru c c ió n :

P se u d o q u iste s.

Com plicaciones

F le m ó n . A b sce so s.

- C o lé d o c o . - D u o d e n a l. P se u d o q u iste s. F ístu la s p a n c re á tic a s. T ro m b o sis d e la ve n a e s p lé n ic a .

70

www.FreeLibros.me

D CAPÍTULOS 4, 5 Y 6. 1^^ REPASA:

ig e s t iv o y c ir u g ía g e m f r a

PATOLOGÍA DEL ESÓFAGO.

[ D i a g n ó s t i c o d i f e r e n c i a l d e l a d is f a g ia , F i g u r a 4 , C a p í t u l o i d e l M a n u a l

CTO 8 a E d.].

TRASTORNOS MOTORES. La siguiente tabla te puede servir para diferenciar las distintas patologías gracias a la manometría.

1.

Registro del cuerpo

P. basal del EEI

Acalasia

T

n/T

EE difuso

T

N

N

Esderodermia

1

1

N

¿Cuál es la técnica diagnóstica de certeza en el

□^REPASA:

[ S u s t a n c i a s q u e in f lu y e n e n l a p r e s ió n d e l

Relajación del EEI

1. pHmetría de 24 horas. EEI,

T a b la

2,

C a p ít u lo

4

del M an u al

CTO 8 a E d.]

REFLUJO GASTRO ESOFÁGICO (RGE).

Tratamiento del RGE. TR ATA M IEN TO D E L RG E

Esofagills alcalina

Ácido :IBP (si refractorio)

T

1- Modificaciones estilo de vida. 2~ Neutralizar sales biliares; Colestiramina Hidróxido de Al Su eral tato

Resistencia al tratamiento médico (poco frecuente) ■Complicaciones

Cirugía

■Acompañando a miotomía Heller (acalasia) Hernia paraesofágica

HERNIA DE HIATO.

Frecuencia.

Característica.

Tratamiento.

Tipo 1

Tipo II

90%

10%

- U nión esófago-gástrica d e sp la/acU a través del hiato. - No saco herm ano.

- U nión esófago-gástrica no desplazarla. - Sí saco h erniano.

M édico y sólo si RG E sintom ático.

Tipo III

M ixta: tipo I + tipo II

Cirugía au nq ue esté asintom ático.

1.

Señala si es verdadero o falso respecto al Divertículo de Zenker: a. Se sitúa debajo del músculo cricofaríngeo. b. La técnica diagnóstica de elección es la endoscopia. c. El tratamiento de elección es la miotomía cricofaríngea más diverticulectomía.

1. Respuesta: a:F; b :F; c:V.

2.

La localización más frecuente del carcinoma eso­ fágico e s ...

2.

El tercio medio.

3.

La localización más frecuente del adenocarcinoma de esófago es...

3.

El tercio inferior.

í I

www.FreeLibros.me

U L T R A -R E SU M E N E S V

CAPITULO 16.

CT ; u

:20 mm anomalías árbol biliar)

Esfinterotomía p o r CPRE

'o *

Ác. ursodesoxicólico Litotricia biliar Asmtomático Cále, no calar. < 15 mm extracorpórea Vesícula funcionante Cále, no calaf. 600 mg/ di

- H iperglucem ia > 300 mg/ di - A cid osis m etabólica con anión gap T

- Hiperosm olaridad > 350 m Osm / Kg

- T C uerpos cetónicos en orina

- Acidosis m etabólica leve

- Hipertrigliceridem ia - T Transam inasas y C P K - Leucocitosis con desviación izda. C O M P L IC A C IO N E S

C O M P L IC A C IO N E S Fracaso renal agudo prerrenal - Pancreatitis

- Fracaso renal agudo prerrenal - Pancreatitis

- IAM

- C ID

- Infecciones - Edem a cerebral (en niños)

- Infecciones

- Hidratación (3-5 litros de salino fisiológico,

- H ID R A T A C IÓ N (10-12 litros de salino fisiológico o hiposalino, después glucosado) lo más im portante.

después glucosado) - IN S U LIN A i.v. (8-10 U /h o ra ) - Potasio (salvo hiperpotasem ia o insuficiencia renal)

- Insulina i.v. (a dosis bajas)

- Bicarbonato (sólo si pH < 7)

- Bicarbonato (si acidosis láctica)

- 1-2% m ortalidad - Causa más frecuente m uerte: IAM e infecciones

- Alta m ortalidad (hasta 50% en algunas series).

- Potasio (salvo hiperpotasem ia o insuficiencia renal)

93 www.FreeLibros.me

ULTRA-RESUM ENES. Manual CTO de Medicina y Cirugía, 8.a edición Aprovecha la ocasión para repasar los distintos tipos de acidosis.

Algoritmo diagnóstico de la acidosis metabólica. A C ID O S IS

... I .''.¿A nión g a p ? :

N(irmal _ L / C o n s u m o ele fá rm a co s?

(. -

(1) (1 ) (2) (2)

D iu r é t ic o s a h o rra d o re s d e p o ta sio I n h ib id o r e s d e la a n h id r a s a c a r b ó n

D ia r r e a U r e t e r o s ig m o id o s t o m ía A c id o s is tu b u la r H ip o a ld o s t e r o n is m o

> 300

A c id o s is lá c t ic a I. R e n a l - T ó x ic o s (B ig u a n ic la s ...) - I. R e s p ir a t o r ia

N o rm al o casi

I

I

I n t o x i c a c io n e s

C c t a c id o s is

-

E t ile n g lk o l

¿Alcoholemia?

d ia b é t i c a

- S a lic ila t o s M e ta n o l

I n d e l e c la b le

A lta

4

I

C e to a c id o s is tra s ay u n o p r o lo n g a d o

C e t o a c i d o s is a lc o h ó lic a

(1) P é r d id a d ig e s tiv a d e io n b ic a r b o n a t o : p H o á c id o ( < 5) (2) P é r d id a r e n a l d e io n b ic a r b o n a t o : p H o a lc a lin o (> 6 , 5)

TRATAM IENTO.

Enfoque terapéutico. D M tip o 1

D M tip o 2 D ieta + e je r c ic io

D ieta + in s u lin a + e je r c i c i o

< 8 años edad

I

i anos edad

i

i

Tto co n ve n cio n al

Tto in ten sivo

¿Buen control? H b glicosilada

¿Buen control?

¿Buen control?

G lucem ias capilares

G lucem ias capilares

NO

I

I

I

I

Seguir igual

T dosis

Seguir igual

t dosis

I

Seguir

£n esí:a cas¡||a n o hay acuerdo universal.

'§ ua^

Si no hay control con un fárm aco, se aum enta la dosis; si falla, se asocian varios fárm acos, y si falla, se em plean a.o. + insulina o insulina en m onoterapia. En determ inadas ocasiones sí hay preferencias:

NO

SI

NO

I

H b glicosilada

H b glicosilada



SI

El tratam iento intensivo consigue m enores H b A lC y por tanto menor in ciden cia de com plicaciones m icrovasculares. Sus inconvenientes son el m ayor núm ero de hipoglucem ias (aunque no más hipoglucem ias graves que con el tratam iento convencional) y m ayor ganancia ponderal. El tratam iento intensivo no se em plea en m enores de 8 años, porque aum enta el riesgo de hipoglucem ias que afectan al desarrollo intelectual. Otras contraindicaciones para el tratam iento intensivo son: -Neuropatía autonóm ica severa (las hipoglucem ias inadvertidas). -Edad avanzada. -Ictus agudo. -Enferm edad coronaria no estabilizada. -Alteraciones del com portam iento, depresión.

94

www.FreeLibros.me

Embarazo

Insulina está indicada si: - Glucemia basal > 95 - Glucemia 2 h después de. comer > 120 - Hidramnios - Macrosomía fetal

1. Renal

Insulina (disminuir la dosis)

Obesidad

B ig uanid as (efecto anorexígeno)

Diagnóstico con glucemia muy alta o descom pensación

Insulina

E

n d o c r in o l o g ía , m e t a b o l is m o y n u t r ic ió n

M E C A N IS M O D E A C C IÓ N

FÁ RM A CO

E F E C T O S S E C U N D A R IO S

H ip o g lu c e m ia (raros e l resto).

- A c tú a so b re el re ce p to r.

In s u lin a

- L ib e ra n in su lin a p re fo rm a d a en el p á n c re a s. A c c ió n p ro lo n g a d a . - A u m e n ta n el n ú m e ro d e re ce p to re s d e in s u lin a . - Efecto a n iv e l p o stre ce p to r.

S u lf o n ilu r e a s

R e p a g lin id a y n a t e g lin id a

- R e g u la la se cre c ió n d e in s u lin a p o r un re ce p to r d ife re n te a la s s u lfo n ilu re a s . A c c ió n rá p id a y tra n sito ria . -

B ig u a n id a s

A c a r b o s a y m ig lito l

H ip o g lu c e m ia p ro lo n g ad a (raros el resto ).

In h ib e n la g lu c o n e o g é n e sis h e p á tic a . D ific u lta n la a b so rc ió n in te stin a l de g lu co sa. P o te n c ia n la a c c ió n p e rifé rica d e la in s u lin a . Efe cto a n o re x íg e n o .

B a ja in c id e n c ia d e h ip o g lu c e m ia . - A c id o sis lá c tic a (ra ra , p ero típ ic a ), (n o h ip o g lu c e m ia ). - In to le ra n c ia g a stro in te stin a l (el m ás fre cu e n te ). F la tu le n c ia (no h ip o g lu ce m ia ). C o n to ro s A D O s p u e d e p ro v o c a r h ip o g lu c e m ia . T t o . co n g lu co sa o ral (n o sa ca ro sa ).

- In h ib e las a lfa g lu c o sid a sa s in te s tin a le s .

H e p a to to x ic id a d (ra ra), le u c o p e n ia leve. R e te n c ió n h ícl rica .

- D is m in u c ió n re siste n cia p e rifé rica . - C o n tro l d e l m e ta b o lism o lip íd ic o .

R o s ig lit a z o n a y p io g lit a z o n a

A D O : e n g e n e r a l, c o n t r a in d ic a d o s e n in s u f ic ie n c ia s ( r e n a l, h e p á t ic a , r e s p ir a t o r ia ,...) .

Medir la glucemia a las 3:00 a.m. Si está nor­ mal (o alta), significa que no llega suficiente insulina NPH hasta la mañana, y habrá que au­ mentar la dosis nocturna (fenómeno del alba). Si está baja, la hiperglucemia es un fenómeno de rebote y habrá que disminuir la dosis de insulina nocturna que ha producido dicha hipoglucemia (efecto Somogy).

1. ¿Cuál es la actitud ante un paciente diabético en tratamiento insulinico con hiperglucemias matutinas?

CAPITULO 4.

ENFERM EDADES P E LAS GLANDULAS SU PRARREN ALES.

SÍNDROME DE CUSHIN G.

Datos oríentativos en e l diagnóstico etiológico del sd. de Cushing. S. C u sh in g florido Sd . de C u sh in g

+ C o rtiso l i e n san g re y o rin a (es el único caso)

C U S H IN G Y A T R O G E N IC O

V iriliz a ció n (TAndrégenos)

Into leran cia g lu cíd ica

Edad infantil

A lca lo sis h íp o p o tasém ica

Co n m enos frecuencia:

D ebilid ad p roxim al

(O besidad central, HTA, acné, hirsutismo, am enorrea, osteoporosis, m iopatía, cara "luna llena",

Ginecom astia (varones)

giba de búfalo, estrías,

Hemorragia disfuncional

H iperpig m entación

fragilidad capilar...)

(mujeres)

cután ea

EN FER M ED A D DE

C A R C IN O M A

T U M O R S E C R E T O R DE

C U S H IN G H IP O FISA R IA

SU P RA R REN A L

A C T H E C T Ó P IC A

M E N O S FR E C U EN T E

M AS FR E C U E N T E

REPASA:

[D

ia g n o s t ic o b io q u ím ic o d e l s ín d r o m e d e

C

u s h in g

,

F ig

u ra

57,

www.FreeLibros.me

C

a p ít u l o

4

d el

M

an u a l

CTO 8a Ed.

U L T R A - R E S Ú M E N E S . M anual C T O de M edicina y C irugía, 8 .a edición

Diagnóstico del sd. de Cushing. S O S P E C H A D E C U S H IN G

S C R E E N IN G

D X d e H IP E R C O R T IS O L IS M O

D IA G N O S T IC O

“ C o rtiso l lib re u n ita rio (C L U ): ele v ad o - D X T 1 m g n o c tu rn o : no su p rim e

- D X T 2 m g n o c tu rn o : no su p rim e - C L U > 3 ve ce slo no rm al - C o rtiso l 23 = 24 h o ras: d e te c ta b le

S IN D R O M E D E C U S H IN G D IA G N O S T IC A D O

▼ ACTH

N o rm al o e le v a d a : ce n tra l o e ctó p ico

■S u p rim id a : A C T H in d e p e n d ie n te

í

T 8 -D X T

S u p rarre n al

T V e r Tab la 3 7, M a n u a l C T O 8 .a Ed.

T C abd

V e r Tab la 3 8 , M a n u a l C T O 8 .a Ed.

Tratamiento del sd. de Cushing. (La cirugía es el tratamiento de elección prácticamente siempre) T U M O R H IP O F IS A R IO

TU M O R PRO D U C TO R

AD EN O M A

PRO D U CTO RACTH

A C T H E C T Ó P IC A

SU PRARREN AL

R e s e c c i ó n t r a n s e s f e n o id a l

E x tir p a c ió n d e l tu m o r

E x tir p a c ió n del ad e n o m a

(hem i h ip o fise cto m ía si no se visu a liza el tu m o r)

|

C A R C IN O M A SUPRARREN AL

E x tir p a c ió n d e l tu m o r + M ito ta n e

| A L T E R N A T IV A S SI F R A C A S A C IR U G IA O P A C IE N T E IN O P E R A B L E

H ip o f is e c t o m ía to tal

S u p r a r r e n a le c t o m ía b ila te ra l

R T h ip o f is a r ia S u p r a r r e n a le c t o m ía b ila te ra l

+ R T h ip o f is a r ia

S u p r a r r e n a le c t o m ía q u ím ic a

I L_

S u p r a r r e n a le c t o m ía q u ím ic a (Ketoconazol, Aminoglutetimida, Metopirona)

96

www.FreeLibros.me

M ito ta n e _

E

n d o c r in o l o g ía , m e t a b o l is m o y n u t r ic io x

H IPERAN D RO G EN ISM O . Difícil diagnóstico en varones. Y en mujeres, el diagnóstico diferencial puedes verlo en el siguiente esquema:

Algoritmo diagnóstico del hirsutismo.

Sin vírilización

Con vírilización (clitoromegalia, alopecia de distribución masculina, agravamiento de la voz)

Hirsutismo familiar Hirsutismo idiopático Hiperplasia suprarrenal de aparición tardía Fármacos S.O.R

y ol ¡gome morrea

Predominio de DHEA (1 7 cetoesteroides en orina T)

Predominio TESTOSTERONA (1 7 cetoesteroides en orina N)

O rigen suprarrenal:

O rigen ovárico:

Carcinoma suprarrenal

Tumores S.O.R(raramente)

H IPERALDOSTERONISM O PRIMARIO.

Fisiopatología y clínica. ALDOSTERONATT

Se elimina K+ ,7 ,-------- ► Hipercaliuria

I HIPO POTA SEMIA

HTA

Debilidad muscular Calambres Poliuria/Polidipsia Alteraciones ECC (arritmias, QT largo, T picudas, ondas U prominentes) Hipotensión postural Bradica relia

sin edemas No hay edemas por un fenómeno de escape a los 3-5 días de exceso de aldosterona que detiene la reabsorción de sodio. Es debido al aumento del péptido natriurético atrial, otros péptidos y a mecanismos tubulares.

ALCALOSIS METABÓLICA

LO

El Na+ arrastra H .O

www.FreeLibros.me

O q3

0 tn _o

HIPOPOTASEMIA |

H ip o p o ta se m ia m a n te n id a tras su sp e n d e r el tra ta m ie n to p re vio y a d m in is tra r s u p le m e n to s d e p o tasio

ob

d u ra n te 2 se m a n a s

CU

Excreción urinaria de potasio O

75 jo u

O

c 'O y

> 30 mEq/d

< 3 0 m Eq/d

E

CU

-C

T to . d iu ré tic o p revio

cu => cr

QJ

ro en

Actividad renina plasmática

P é rd id a s dig estivas

t

jtí

"en

o u

2 O >,

I r

___________ i __________

'O c

Hiperaldosteronismo 29

c y

«3

H ip e rte n s ió n re n o v a s c u la r

O

H ip e rte n s ió n m alig n a

y

E

BJD

Cu

° £ .9 x _&

O b“ u "r* 3 C

£ .i S Q-

CU

O

2

LU

OS

p

Í5 S .= o E 2

LU

LO

AÍdosterona plasmática^

Aldosterona ->25 Actividad renina plasmática Q u e no se s u p rim e tras

ta

N e fro p a tía p ie rd e sal

'=3 o op £ 'en . 30% Cálculos renales en Rx o sintomáticos

Varón

M ujer

O b servación

Estrógenos

- i Masa ósea > 2 ,5 Desviaciones típicas(Osteoporosis) -Historia de un episodio de hipercalcem ia potencialm ente letal

C IRU G IA

A denom a

Hiperplasia

2 opciones

E xtirpació n del a d e n o m a y e x p lo ració n d e las d e m á s g lá n d u la s p aratiro id e as

P a ra tiroid ecto m ía

P a r a tiro id e c to m ía total

subtotal

e im p lan te e c tó p ic o d e una g lá n d u la

(se d e ja una o m edia)

(en lu g a r a c c e s ib le p a ra e x t ir p a r la si p e rs is te h ip e r c a lc e m ia )

1.

¿Cuál es la causa más frecuente de hipercalcemia?

1. El hiperparatiroidismo primario.

2.

Y, ¿cuál es la causa más frecuente de hiperpa­ ratiroidismo primario?

2.

Adenoma paratiroideo.

La hipercalcemia condiciona una cantidad de calcio filtrado muy alta, por lo cual, aunque la reabsorción sea mayor de lo habitual, sigue habiendo hipercalciuria.

102

www.FreeLibros.me

E n d o c r in o l o g ía ,

m e t a b o l is m o y n u t r ic ió n

Diagnóstico diferencial y tratamiento del hipoparatiroidismo. HIPOCALCEM IA

Hipofosfatemia

Hiperfosfatemia

Descartar:

Descartar I, Renal

- Déficit de Vit. D - Malabsorción

„ 1

- Antiepilepticos 2

^ -— r , C Niveles de PTH ....

C T e s t de Ellsworth-Howard, (infusión de PTH)

2-No t AM Pc orina

2- t AM Pc orina

No t Fósforo orina

2 - í AM Pc orina

N o t Fósforo orina

*

t Fósforo orina



Pseudohi poparatiroidismo tipo I

T

Pseudohipoparatiroidismo

Hipoparatiroidismo

tipo II (defecto en la respuesta a PTH más allá del receptor, en

A-

la proteínquinasa)

Morfotipo Alioright: A.

Talla ioaja

Morfotipo

Retraso mental

normal

Obesidad C ara redonda Acortamiento

i

4e metacarpiano

S H P tip o Ib

Pseudopterigium colli

(d e fe c to d ife re n t e en el re c e p to r



de PTH )

SH P tipo la (defecto en la subunidad C del receptor de PTH)

*

T R A T A M IE N T O

*

Calcio a dosis aíta^ - v ita m in a D a d o sis a lt a s 1 z l t i a c íd a s

a causa más frecuente de hipoparatiroidismo es la cirugía, todas las demás son raras.

CA PÍTU LO 7. 1.

NUTRICIÓN, PISLIPEM IA Y OBESIDAD.______________

N u trició n

REPASA:

[V

a l o r a c ió n

d e la

d e s n u t r ic ió n

,T

a b la

70,

C

a p ít u l o

7

d el

M

an u a l

CTO

8 a E d .]

Distribución de principios activos en la dieta. Grasas: 30-35% • Saturadas < 10% • M onoinsaturadas 15-20% • Poliinsaturadas < 1 0 % • Colesterol < 3 0 0 mg/'día

Proteínas: 10-15% • Valor nutricio nal: % de am inoácid os esenciales (no > 25% ) • Valor biológico: d epen de de lo e q uilibrad a que esté en cuanto a com p osición de am inoácid os y su el¡gestibil¡dad.

Hidratos de carbono: 55% . Se prefieren los com plejos (absorción lenta). • Fibra

— ► Solub le: en len tece vaciam iento gástrico y absorción de H C sim ples. Insoluble: efecto volum en en colon.

103 www.FreeLibros.me

ULTRA-RESUM ENES. Manual CTO de Medicina y Cirugía, 8.a edición

Soporte nutridonal: elección de la técnica ¿El paciente está malnutrido o corre riesgo de estarlo?

No



A lim e n ta c ió n norm al



No

Dieta adecuada ± suplementos orales

Nutrición enteral

2.

Nutrición parontcralj

O besidad.

La mayoría de los pacientes obesos tienen obesidad exógena. Solamente una minoría tienen obesidad secundaria. REPASA:

[E

t io l o g ía d e l a o b e s id a d

s e c u n d a r ia

, T

a b la

76,

C

a p ít u l o

7

La distribución d e la grasa corporal es un factor de riesgo card io vascular si:

d el

M

an u a l

C T O 8 a Ed.]

>1 cf

índice cintura/cadera I l > 0 ,85 9 Norm al 20 - 2 4 ,9 Sobrepeso 25 - 2 9 ,9

■Indice de masa corporal =

Peso (Kg)

Obesidad grado I 30 - 3 4,9

Talla (m 2)

Obesidad grado II 35 - 3 9 ,9 Obesidad m órbida o grado III > 4 0 O besidad extrem a o superm órblda > 50

Tratamiento de la obesidad (no dar fármacos si sólo hay sobrepeso) Anorexígenos Dieta y ejercicio

3.

Inhibidores de la absorción orllstat.

Fibra dietética saciante para disminuir el hambre.

Fluoxetlna si se asocia a bullmla o depresión.

Slbutramlna. Inhibidores de la recapacltaclón de 5H T y noradrenallna

Tratamiento quirúrgico: - Obesidad mórbida refractarla. - Comorbllldad severa - Técnicas mixtas (restrictivas parcialmente malabsortlvas): By-pass gástrico y de Roux

D islipem ias. REPASA:

[O

b j e t iv o s

d e c o n t r o l

d e

la d is l ip e m ia

, T

a b la

74,

C

a p ít u l o

7

d el

M

an u a l

CTO 8a Ed.]

Tratamiento. ALTERACIÓN LIPÍDICA PRINCIPAL - H ip e rc o le ste ro le m ía (a u m e n to L D L , T G y H D L n o rm ale s).

FÁRMACOS 1s ELECCIÓN

FÁRMACOS 23 ELECCIÓN

Estatlnas o resinas

Fibratos

E sta tin a s o fib ra to s

A c id o n ic o tín ic o

d ism in u id o ). - H ip e rtrig lic e rid e m ¡a .

R e sin a s + e sta tin a s E sta tin a s + e z e tim ib e E sta tin a s + fib rato s

- H ip e rlip e m ia m ix ta (a u m e n to de L D L c o n T G e le v a d o s y H D L

TRATAMIENTO COMBINADO

R e sin a s (puede e le v a r TA G ) + fib rato s

Fibratos

A c id o n ico tín ico

104

www.FreeLibros.me

Fib rato s + á c id o n ic o tín ic o

E

n d o c r in o l o g ía , m e t a b o l is m o y

M üaO O h

1. ¿Cuál es el objetivo en el tratamiento de la hipertrigliceridemia?

1. El objetivo es mantener el nivel de triglice^dos por debajo de 200 mg/dl.

2.

¿Cuál es la hiperlipoproteinemia más frecuente?

2.

3.

Relaciona ambas columnas: a. b. c. d. e.

Resinas. Ácido nicotínico. Estatinas. Fibratos. Probucol.

La hipercolesterolemia poligénica.

a. a.

Diarrea. Estreñimiento. Interacción con la absorción de otros fármacos, posible aumento de TAG. c. Molestias gastrointestinales leves, colelitiasis, mió patíos. d. LDL pequeñas y densas en cifras normales. e. Alteración de las transaminasas, miopatías.

Respuestas: a-2, b-4, c-5, d-3, e-1. 4. ¿Cuál es la alteración lipídica típica de la diabe­ tes mellitus tipo 2?

Hipertrigliceridemia, descenso de HDL y LDL pequeñas y densas.

Definición sd. metabólico (ATPIII)

3 c rite rio s efe lo s 5 s ig u ie n te s ■P re s ió n a rte r ia l > 1 3 0 / 8 5 A T P - lll

■T rlg llc e rld o s > 1 5 0 ■ HDL < 4 0 va ró n / < 5 0 m u je r ■G lu c e m ia a y u n a s > 1 1 0 ■C in tu ra > 1 0 2 v a ró n / > 8 8 m u je r

P re s e n c ia d e D M tip o 2 o IHC o G A A o h lp e rln s u lln e m la p o r e n c im a d e l c u a rtll s u p e rio r d e ía p o b la c ió n y 2 d e lo s s ig u ie n te s (si no h a y a lte ra c ió n d e la g lu c o s a se p re c is a n 3 c rite rio s ): ■P re s ió n a rte r ia l > 1 4 0 / 9 0 OMS

■T rlg llc e rld o s > 1 5 0 ■ HDL < 3 5 va ró n / < 4 0 m u je r ■ IM C > 3 0 y / o ICC > 0 ,9 v a ró n / > 0 ,8 5 m u je r ■ M lc ro a ib u m in u ria > 2 0 m c /m in

O b e s id a d c e n tra ! (c in tu ra > 9 3 va ró n / >79 m u je r e u ro p e o s , p a ra o tra s ra z a s s o n d is tin to s ) y 2 d e lo s s ig u ie n te s : ID F

■P re s ió n a rte ria ! > 1 3 0 / 8 5 ■T rig lic e r id o s > 1 5 0 ■ H D L < 4 0 va ró n / < 5 0 m u je r ■ G lu c e m ia a y u n a s > 1 0 0 o d ia g n ó s tic o p re v io de D M tip o 2

1

o tjl

4.

www.FreeLibros.me

E

n d o c r in o l o g ía , m e t a b o l is m o y n u t r ic ió n

1. ¿Cuál es el objetivo en el tratamiento de la hipertrigliceridemia?

7. El objetivo es mantener el nivel de triglicéridos por debajo de 200 mg/dl.

2.

¿Cuál es la hiperlipoproteinemia más frecuente?

2.

3.

Relaciona ambas columnas: a. b. c. d. e.

La hipercotesterolemia poligénica.

a. a.

Diarrea. Estreñimiento. Interacción con la absorción de otros fármacos, posible aumento de TAG. c. Molestias gastrointestinales leves, colelitiasis, miopatías. d. LDL pequeñas y densas en cifras normales. e. Alteración de las transaminasas, miopatías.

Resinas. Ácido nicotínico. Estatinas. Fibratos. Probucol.

Respuestas: a-2, b-4, c-5, d-3, e-1. 4. 4.

¿Cuál es la alteración lipídica típica de la diabetes mellitus tipo 2?

Hipertri gti ceri demi a, descenso de HDL y LDL pequeñas y densas.

Defíniíión sd. metabóliio (ATPIII)

3 c rite rio s d e lo s 5 s ig u ie n te s ■P re s ió n a rte r ia l > 1 3 0 / 8 5 A T P - lll

■T rig llc e r id o s > 1 5 0 ■ HDL < 4 0 va ró n / < 5 0 m u je r ■ G lu c e m ia a y u n a s > 1 1 0 • C in tu ra > 1 0 2 v a ró n / > 8 8 m u je r

P re s e n c ia d e D M tip o 2 o IHC o G A A o h lp e rin s u lln e m la p o r e n c im a del c u a rtil s u p e rio r d e la p o b la c ió n y 2 d e lo s s ig u ie n te s (si no h a y a lte r a c ió n d e la g lu c o s a s e p re c is a n 3 c rite rio s ): ■ P re s ió n a rte ria l > 1 4 0 / 9 0 OMS

■ T rig llc e rid o s > 1 5 0 ■ HDL < 3 5 va ró n / < 4 0 m u je r • IMG > 3 0 y / o ICC > 0 ,9 v a ró n / > 0 , 8 5 m u je r ■ M ic ro a lb u m in u ria > 2 0 m c /m in

O b e s id a d c e n tra l (c in tu ra > 9 3 v a ró n / > 7 9 m u je r e u ro p e o s , p a ra o tr a s ra z a s s o n d is tin to s ) y 2 de lo s s ig u ie n te s : ID F

■ P re s ió n a rte r ia l > 1 3 0 / 8 5 ■T rig lic e r id o s > 1 5 0 ■H D L < 4 0 va ró n / < 5 0 m u je r ■G lu c e m ia a y u n a s > 1 0 0 o d ia g n ó s tic o p re v io d e D M tip o 2

105

www.FreeLibros.me

ULTRA-RESÚMEINJES. Manual CTO de Medicina y Cirugía, 8.a edición

CAPÍTULO 2.

1.

EN FERM EDADES D E LA H IPÓ FISIS Y DEL HIPOTÁLAMO.

Entre las células hipofisarias productoras de hor­ monas, ¿cuáles son las más abundantes?

1.

Las células secretoras de GH, que representan el 50%.

2. ¿Cuál es el tumor hipofisario secretor más fre­ cuente?

2.

El prolacti noma.

3.

¿Cuál es el tumor hipofisario más frecuente del MEN-I?

3.

El prolacti noma, seguido del productor de GH, igual que en la población general.

4. ¿En qué pensarías ante un paciente con clínica de insuficiencia suprarrenal, con sodio bajo y potasio normal?

4.

En un déficit de ACTH. El potasio no se altera porque en la regulación de los mineratocorticoides es más importante el eje renina-angiotensina que la ACTH.

5. ¿Cuál es la causa más frecuente del déficit de ACTH?

5.

El tratamiento prolongado con esteroides (yatrogénico).

PROLACTINOMA.

Manejo. H ip e r p r o la c t in e m ia

---------- R M -----------M a c ro a d e n o m a

M ic r o a d e n o m a

N o rm a l

(sintom ático o no)

Y

Asintomático

Sintomático

'

A G O N IS T A S

A G O N IS T A S

Hiperprolactinemia idiopática (aunque no excluye

D O P A M IN É R G I C O S

O B S E R V A C IÓ N

responde

no responde

mantener Tto.

T dosis

D O P A M IN É R G I C O S

m icroadenoma oculto) Asintomático

Sintomático

O B SER V A CIO N

AGON ISTAS

re s p o n d e

no re sp o n d e

responde

T do

mantener Tto.

D O P A M IN É R G IC O S

responde

no responde* o no tolera el Tto.

-persisten defectos mantener Tto.

visuales

!

|~*Antes de la cirugía, puede intentarse tratam iento con otros inhibid ores de la p rolactina, como son la cabergolida o la quinagolida.

-no responde* -no tolera

i

m a n te n e r T to

resección

re s e c c ió n

transesfenoidal

transesfenoidal

La radioterapia tiene un papel muy limitado, reservándose para tumores de crecimiento rápido, pese a tratamiento médico y/o quirúrgico o tras la cirugía en pacientes no curados que no toleran la bromocriptina.

Recuerda que la PRL, que no tiene una hormona liberadora hipotalámica específica, es estimulada por TRH. Esto explica el aumento de PRL en el hipotiroidismo primario de larga evolución. Otros estímulos para la secreción de PRL son los estrógenos, que estimulan la síntesis y secreción, pero frenan el efecto sobre la mama, y la estimulación del pezón. Basalmente, la prolactina está inhibida por la dopamina.

1.

Ante un macroadenoma que cursa con eleva­ ción ligera de los niveles de prolactina, ¿qué deberías pensar?

2. Mujer con antecedentes de hemorragia obsté­ trica durante el parto hace cuatro días, refiere incapacidad para la lactancia. ¿Cuál sería nues­ tra sospecha diagnóstica?

1. En realidad se trata de un adenoma no funcio­ nante con hiperprolactinemia por compresión del tallo hipofisario. 2.

Déficit de PRL por infarto hipofisario postpar­ to (síndrome de Sheehan).

106 www.FreeLibros.me

E

3.

n d o c r in o l o g ía , m e t a b o l is m o y n u t r ic ió n

Pac. con cefalea intensa de inicio brusco, naúseas y hemianopsia bitemporal.

j

A p o p le jía h ip o fisa ria . neuroquirúnico u re n te .

Tratam iento

ACROMEGALIA. No olvides que hasta 1/3 tienen HTA y, la gran mayoría, debilidad y cansancio (los demás síntomas son fáciles de reconocer, ya que son debidos a crecimiento de tejidos en distintas localizaciones). Además la GH es una de las hormonas contrainsulares, por lo que el 20-40% de los pacientes tienen intolerancia a los hidratos de carbono y el 13-20% diabetes franca. Se asocian a esta enfermedad: * Pólipos nasales. * Pólipos colónicos. RIESGO AUM ENTADO DE CÁNCER DE COLON. * Aneurismas intracraneales.

1.

¿Qué tamaño suelen tener los adenomas productores de GH?

1.

Suelen ser macroadenomas.

2.

De nada.

¿De qué sirve una determinación aleatoria de GH?

Manejo de la acromegalia.

Sospecha clínica

i IGF-1 elevada

RM (o TC )

IGF-BP3 elevada

Campímetría

o

GH 120 minutos después de S .O .G . (10Og) >2pg/l

Pruebas de función

microadenoma

(o > 1 pg/l con IRMA)

hipofisaria

(menos frecuente)

M ACROADENOUA

V

Cuando el paciente rechaza la

CIRUGÍA*

drueía o está contraindicada

A Curación No controla:

(Sólo o coadyuvante

HTA

de somatostatina)

No respuesta

Pegvlsomant

completa

Hlperglucemía

^ (antagonista rec. GH)



No curación

RT

► Curación

▼ No curación,

Análogos de la

t----------------

somatostatina

rechaza o

mientras hace efecto

NOTA: los agonistas dopaminérgicos inhiben la secreción de GH en los acromegálicos (no en personas normales), re es mucho menos eficaz que la cirugía, la RT o el octreótide.

CURACIÓN: GH < 2 (< 1 con IRMA) tras SOG. sobrecarga oral de glucosa.

H IPOPITUITARISM O . REPASA:

[D

ia g n ó s t i c o y t r a t a m ie n t o d e l

H ir o r iT U iT A R iS M O , T a b l a 1 8 , C

a p ít u l o

2

d el

M

anual

CTO 8 a E

d

.]

107

www.FreeLibros.me

ULTRA-RESÚM ENES. Manual CTO de Medicina y Cirugía, 8.a edición

DIABETES INSÍPIDA.

Fisiopatologia. Razonando los datos clínicos y de laboratorio, no tendrás que esforzarte en atemorizarlos.

Con ADH

Sin ADH

(norm al)

(diabetes insípida)

H ace perm eables al agua los túb ulo sco nto rnead o

Los túbulos son im perm eables

distal y colector



T

El agua de la orina que llega al T C D

Toda la orina que llega al T C D es

(que es siem pre diluida) pasa de los

elim inad a a los uréteres.

túbulos al intersticio y de ahí a la sangre.

La orina es muy diluida La o rin a queda más

La sangre queda más

concentrada

diluida

-densidad < 1 0 1 0 -osm < 3 0 0 mosm/kg

T Esto es lo que ocurre, de m anera

poliuria hipotónica

La sangre queda más concentrada -osm > 2 9 0 mosm/kg

▼ sed

exagerada, en la S IA D H : dism inución de

?

la diuresis, osm . urinaria y sodio en orina

polidipsia

aum entados, hiponatrem ia dilucional y niveles bajos de otras sustancias en la sangre.

La causa más frecuente de DI central es la idiopática, pero dado que un 55-70% se deben a alguna causa subyacente (en primer lugar, cirugía u otros traumatismos), es importantísimo hacer un estudio exhaustivo del paciente antes de clasificar la diabetes insípida como idiopática. CAUSAS DE DI

CENTRAL. - Idiopática (25 - 30%) - Tumores hipofisarios o cerebrales, cirugía, traumatismos. - DI familiar (AD). - Sel. Wolfram (AR): DI, DM, cófosis, atropia óptica. NEFROCÉNICA - Adquirida (lo más frecuente): hipercalcemia, hipopotasemia, tubulopatías intersticiales, Li, demeclocilina. -Recesiva ligida a X .

Diagnóstico.

1. ¿Cuál es la tríada típica del craneofaringioma?

1.

Déficit de GH + diabetes insípida + calcificaciones supraselares.

Prueba de la deshidratación:

Ante un paciente con poliuria y polidipsia, esta prueba intenta distinguir al que bebe mucho para recuperar las pérdidas urinarias del que bebe mucho por otro motivo, y secundariamente, orina mucho.

108

www.FreeLibros.me

E

n d o c r i n o l o g í a , m e t a b o l i s m o y n u t r i c i o -.

A L G O R I T M O D I A G N O S T I C O D IA B E T E S IN S IP ID A

S O S P E C H A D I: pol¡dipsia, p o liu ria , o rina d ilu id a (O sm o < 300) ¿P ac. D esh id ratad o? O sm p > 295 N a +p > 145

No



▼ P ru eb a d e sh id ra tació n (4-12 h)

Dx d e DI

■f

AA u m e n ta



Infusión A D H ¿A u m enta O sm o ?

¿O sm u rinaria?

No varía

< 7% D IN

(> 3 00)

i 7 ,5 0 % D IC

> 50%

parcial

co m p leta

D IC

P O T Ó M A N O : no está in d icad o in fu n d ir A D H . Si se h iciera, la O sm o no au m e n taría (sim u lan d o una D IN )

Tratamiento. DI central: DDAVP intranasal. DI central parcial: carbamacepina, clofibrato o clorpropamida. DI nefrógena: restricción salina + tiacidas y evitar sobrecarga hídrica. Tratamiento alternativo: indometacina.

SIADH. Secreción inadecuada de ADH. REPASA:

[C

a u sa s

d l

s e c r e c ió n

in a d e c u a d a d e

AVP (SIADH),

T

a b la

19,

C

a p ít u l o

2

d el

M

a n u a l

CTO 8 a Ed.]

Tratamiento del SIADH. Corregir la causa desencadenante si es posible. Restricción hídrica. Salino fisiológico + furosemida. En hiponatremia aguda severa (convulsiones, coma): salino hipertónico. Reposición gradual para evitar mielinolisis central pontina. En SIADH crónico: litio, demeclociclina.

1. El síndrome SIADH nos obliga a descartar de­ terminados déficits hormonales con similares síntomas y hallazgos analíticos, ¿sabes cuáles son?

1. El hipotiroidismo y la insuficiencia suprarre­ nal.

2. ¿Existen edemas en el SIADH?

2.

3.

¿Qué fármaco se puede usar en el tratamiento del SIADH?

No, porque el exceso de ADH provoca reten­ ción de agua libre.

La demeclociclina, que bloquea la acción de la ADH a nivel renal. Recuerda que puede ser causa de diabetes insípida nefrogénica.

O)

www.FreeLibros.me

ULTRA-RESÚM ENES. Manual CTO de Medicina y Cirugía, 8.a edición

CAPITULO 4.

EN FERM EDADES

DE

LAS

GLANDULAS

SU PRA REN A LES.

FEO CROM O CITOM A

Manejo.

C e fa le a , p a lp ita c io n e s e h ip e rte n s ió n y/o p a ro xism o s h ip e rte n s iv o s c o m o

C o le litia sis

sín to m a s m ás fre cu e n te s.

H ip e rc a lc e m ia

H ip o te n sió n o rto stá tica

P o liu ria

P é rd id a d e peso , fie b re , su d o ra c ió n

R a b d o m ió lisis

In to le ra n c ia g lu cíd ica

O RIN A DE 24h: Catecolaminas libres Ac. vanilmandélico Metan efrinas

E le v a d a s p o r o tra c a u sa

Catecolaminas:

Otras pruebas Je

F E O C R O M O C IT O M A

- Le v o d o p a , M e tild o p a

(uso e x c e p c io n a l,

M etane frin as:

só lo si alta

- IM A Ü , p ro p ra n o lo l

so sp e c h a c lín ic a )

O tro s: - C a rh id o p a , c ie r to s a lim e n to s, estím u lo fisio ló g ic o , á c . V a n ilm a n d é lic o

N egat

del tu m o r

Aortografía abdominal

(FEN O XIBEN Z AM INA, íentoiainina sólo en crisis)

(T u m o re s e x tra su p ra rre n a le s g e n e ra lm e n te irrig a d o s por una a rte ria a b e rra n te ).

La administración de suero salino previene la hipotensión arterial pos) quirúrgica.

MIBG: Mela-yodo-benzil-guanidina

CAPITULO 9.

(Control de paroxismos con fentolamina ± nítroprusialo. Control de arritmias con propranolol!.

TRASTORNOS N EOPLASICOS ÓRGANOS ENDOCRINOS.

QUE

AFECTAN

A

M ULTIPLES

SINDROMES DE NEOPLASIAS ENDOCRINAS MULTIPLES. De este tema sólo es imprescindible saber qué neoplasias constituyen los síndromes MEN-I, MEN-lla y MEN-llb. Es uno de los asuntos en los que más se agradecen unas buenas reglas nemotécnicas (te animamos a que te hagas las tuyas personales). También te puede ayudar el siguiente diagrama:

MEN.

M EN I

M EN NA

MEN IIB

HIPERPLASIA CÉLULAS DE LO S ISLOTES CRO M O SO M A 11 M enin a

CMT HIPERPLASIA PARATIROIDES

HIPERPLASIA ADENOM A HIPO FISARIO

FEOCROMOCITOMA

\

CROMOSOMA

\ FEO CRO M O CITO M A

RET

CARCIN O ID E LIPO M AS

110

www.FreeLibros.me

RASGOS M ARFANOIDES

E

n d o c r i n o l o g í a , m e t a b o l i s m o y n u t r i c i o -.

Las características de cada síndrome (clínica, diagnóstico, tratamiento) no son más que la suma de sus . ■ ~-r alvo algunos datos, que son los que debes recordar: La causa más frecuente de hiperparatiroidismo, tanto en el MEN-I como en el MEN-II, es la hiperplasia p arar/(mientras que es el adenoma en el hiperparatiroidismo aislado). El tumor pancreático más frecuente es el gastrinoma, seguido del insulinoma, aunque la secreción pancreática frecuentemente incrementada es la de polipéptido pancreático. El feocromocitoma es bilateral en el 50% (10% en el feocromocitoma aislado) y produce más adrenalina que noradrenalina o incluso sólo adrenalina (ambas hormonas por igual en el feocromocitoma aislado).

Streening y manejo del CMT en el Men II. Proceso de detección selectiva de CMT en el MEN 2 A n a liz a r el A D N d e un fa m ilia r afe cto d e C M T p a ra c o n o ce r la m u tació n c o n c re ta G E N R E T

▼ B u sc a r la m u ta ció n en

(se re q u ie re el po sitivo

el resto d e los fa m ilia re s

en d o s d e te rm in a cio n e s)

Ese fa m ilia r no p a d e c e rá C M T y no re q u ie re n ingún e stu d io

Tiroidectomía total en la in fa n c ia (< 6 años). P o te n cia lm e n te m e jo r tasa d e cu ració n y ah orro d e d in e ro

1.

¿Qué tríada presenta, entre otros síntomas, el PGA I?

2. ¿Qué sospecharía en un paciente con hipoti­ roidismo e insuficiencia suprarrenal?

1.

Candidiasis mucocutánea, hipoparatiroidismo e insuficiencia suprarrenal. Gen AIRE (AR)

2. Alteración hipofisaria o bien, PGA tipo II --> Polisénica (HLA DR3/DR4).

III

www.FreeLibros.me

ULTRA-RESÚMENES. M anual C T O de M edicina y Cirugía, 8 .a edición

TABLAS DE RESUMEN. Pruebas diagnósticas en las enfermedades por déficit de secreción hormonal. H orm ona

Enferm edad

¿C u á l e s el estím u lo habitual?

¿Con qué prueba podem os sim ular dicho estím ulo?

H orm onas tiroideas (T3,T4)

H ip o tiro id ism o .

TSH

N o son n e cesarias p ru e b a s de e stim u la c ió n .

CHRH

H orm ona de crecim iento (GH)

D éficit de G H .

Estím u lo con c lo n id in a , arginina o

v ariad o s e stím u lo s, entre ellos la

hip o g lu cem ia in su lín ica.

h ip oglu cem ia).

H orm ona antidiurética (ADH)

D ia b e te s in síp id a.

Cortisol

A d d iso n .

Aldosterona

H ip o a ld o ste ro n ism o .

Renina

(q ue a u m e n ta b ajo n u m ero so s y

A u m e n to de o sm o la rid a d , d ism in u c ió n de v o lu m e n p la sm ático . ACTH

Prueb a de e stim u la c ió n co n A C T H .

R e n in a -a n g io te n sin a .

Pru eb a d e d e a m b u la c ió n + fu ro se m id a.

D is m in u c ió n d e la perfusión renal

H ip o rre n in e m ia .

P ru eb a d e la d e sh id ra ta ció n .

(d ep leció n de volu m en).

Pru eb a d e d e a m b u la c ió n + fu ro sem ida.

Horm ona paratiroidea (PTH)

H ip o p a ra tiro id ism o .

H ip o c a lc e m ia

D ia b e te s m ellitus.

H ip erg lu cem ia

N o son n e cesarias p ru e b a s d e e stim u lació n .

- Sobrecarga oral d e g lu cosa.

Insulina

- M e d ic ió n d e p é p tid o C tras e stím u lo co n glucagón (rara v e z se e m p le a , d isting u e D M tipo I y tipo II),

Pruebas diagnósticas en las enfermedades por hipersecreción hormonal. Hormona

Enfermedad

¿Cuál e s el inhibidor habitual?

H orm onas tiroideas (T 3 J 4 )

H ipertiroidism o.

T3 y T4 in hib en TSH .

H orm ona de crecim iento (GH)

A crom eg alia.

su stan cias y situ acio n e s (entre

D ism in u ció n d e osm o larid ad , S IA D H .

Cortisol

C ushin g .

Horm ona paratiroidea (PTH) Insulina

Sobrecarga oral d e glucosa.

ellas hip erglucem ia).

H orm ona

Renina

viendo que no respon de al estím ulo con T R H .

Som atostatín a y m u ch a s otras

anti diurética (ADH)

Aldosterona

¿Con qué prueba podem os sim ular dicha inhibición? Podem os com p rob ar cóm o la T S H está in hib id a por T3 y T4

a u m e n to de v olu m en plasm ático.

Prueba d e sobrecarga hídrica (sólo usada en caso s dudosos).

Cortisol (y otros glucocorticoides)

Pruebas de su presión con d e xa m e taso n a

in h ib en A C T H .

(débil o fuerte).

Hiper-

Sobrecarga d e so d io o

Sobrecarga d e so d io o e xp a n sió n

ald ostero nism o .

e xp a n sió n d e v olu m en.

de v olu m en.

H ip erren in em ia.

Hiperparatiroidism o. Insulinom a.

A u m en to de volu m en del LEC. Ingesta d e potasio. PAN.

N o se utilizan pru ebas d e supresión.

H ip ercalce m ia.

N o se u tilizan pru ebas de su presión .

H ipog lu cem ia.

P rueb a d e ayu n o prolongado.

112

www.FreeLibros.me

F arm CAPÍTULO 6.

a c o l o g ía

(SECCIÓ N ONCOLOGÍA Y PA CIEN TE TERMINAL). TRATAMIENTO D EL PACIEN TE TERMINAL. CUIDADOS PALIATIVOS. AIN ES.

Lo importante de este tema es que diferencies bien los AINEs propiamente dichos del paracetamol. La siguiente tabla te explica el funcionamiento de estos fármacos.

PROPIEDADES DE IOS AINEs. Analgésica

- Inhibe SNC a nivel subcortical. - Inhibe síntesis y liberación de los mediadores del dolor.

Antitérmica

- i la temperatura. - No hipotermia en apiréticos.

Antiinflamatoria

Su efecto es independiente de la causa de la inflamación.

Urico eliminadora

- Dosis bajas (c2g/d) -» Hiperuricemiantes. - Dosis altas - » i eliminación renal de ácido úrico.

Antiagregante plaquetario

Inhibe tanto la prostaciclina como el tromboxano, pero el efecto final es una inhibición de la agregación plaquetaria.

‘i la contractilidad uterina y prolonga el trabajo del parto.

Tocolrtico

1.

¿Cuál es el efecto secundario principal de la vincristina?

1. Neuropatía periférica.

2.

¿Qué quimioterápico es de elección en los sindromes mieloproliferativos?

2. Hidroxiurea.

OPIÁCEO S. AGONISTAS PUROS; •CODEINA: Junto con paracetamol •TRAMADOl: No es estrictamente un opiáceo.

Escalón 2 de la OMS • Dolor moderado

•MORFINA: Vida media corta •FFNTANIIO: Se puede administrar en parches transdérmicos

Escalón 3 de la OMS • Dolores intensos

•MCTADONA: Vida media larga. Deshabituación de Heroína

AGONISTAS/ANTAGONISTAS. Se pueden dar en dolor biliopancreático No dar después de haber empleado Morfina

•PFNTAZOCINA •MEPERIDINA

ANTAGONISTAS PUROS •NAIOXONA: Antídoto de la intoxicación aguda •NAITREXONA: Deshabituación a largo plazo

REPASA:

[E y

s c a l a a n a l g é s ic a

P

d e

la

a c ie n t e t e r m in a l d e l

OMS M

(m

an u a l

o d if ic a d a

CTO

8 a E

), F d

ig u r a

13,

C

a p ít u l o

6,

S

e c c ió n

d e

O

n c o l o g ía

.]

117 www.FreeLibros.me

G

in e c o l o g ía y o b stettm o a

OBSTETRICIA CA PÍTU LO 2 2 .

EVALUACIÓN GESTACION AL._________________

Semanas

EVO LU CIÓ N Y PRUEBAS DIAGNÓSTICAS EN UN EMBARAZO NORMAL.

3

5

8

4

4

A

12

4

14

18

20

24

28

30

32

34

37

40

4

4

4

4

4

4

4

*

4

4

42

¡ 1 Fecha última

pHCC

ECO

An alítica general,

en sangre trans-

i * E c q Am nlo-

Serología

centesis

' C ordo-

2* ECO

[

Tea

"I

icentesis

¡ OSullivan

Cromosomopatías

Despistaje DM

3* ECO

Cariotipo AFP Triple test

Bíopsia coríal

Doble

1AmntOSCOpiaj M icro tom a

toma tardía detección estreptococo B

preco z

vaginal

Am nio centesis

Registro | card lo to c o g rá fico 1

Madurez pulmonar Bilirrubina en LA

fetal, m on ito rizació n intraparto Sufrimiento fetal

Las pruebas realizadas a lo largo de la gestación pueden dividirse en:

a) No invasivas: • Ecografía (principal método de evaluación gestacional).

ECOGRAFIA TERCER TRIMESTRE

PRIMER TRIMESTRE

Valora alteraciones del crecimiento fetal (CIR):

1Confirm a gestación intraútero

Simétrico (tipo I): DBP/DA normal

1 Determina Edad Gestacional 1Valora morfología uterina y

porque el numerador y el denominador están i

gestación múltiple 1Valora flujo sanguíneo del ductus

se produce por cromosomopatía o noxa antes de

1 Detección de marcadores de

Asimétrico (tipo II): DBP/DA

la 26a semana

cromosomopatía

'ti

Se produce por noxa tras 32a semana.

V SEGUNDO TRIMESTRE Braquicefalia • Diagnóstico mofológico

Edema nucal

- Biometría fetal (DPFJ LF, diámetro abdominal) -Valoración de anejos ovulares L- Estigmas y malformaciones (ej.: síndrome de Down)

Arteria umbilical única

Dilatación pieloureteral

SINDROME DE DOWN Fémur corto ( LF/DBP i )

Triple test. Determ inación de:

- Alfafetoproteína. En el síndrom e de D o w n: - HCG. - Estrió! no conjugado.

- A FP i - HCG t - Estrió! i

110

www.FreeLibros.me

ULTRA-RESÚMENES. Manual CTO de Medicina y Cirugía, 8.a edición

___________

b) hm sivas:

>35a Indicaciones ^

AP/AF cromosomopatías Screening BQ/eco (+ )

2y Trimestre Biopsia corial

Amniocentesis

Funículo Centesis

8 - 1 2 sem

1 2 - 1 6 sem

> 18 sem

- La + precoz

- La + segura (< 1% de abortos)

- Permite tratamiento

Momento de realización Ventajas

- Resultado rápido

Inconvenientes

- Resultado rápido

- t Lenta (3-4 sem)

- El mayor n2 de

- Riesgo intermedio de abortos

-T é c Fis: >rápido pero icrs,

abortos/mosaicos

(hemorragia/trombo)

solo si sospecha concreta

Fosfalidilglicerol

Amiocentosis tardía (>20 sem) Si

- Estudio de madurez pulmonar - Valorar bilirrubina en iso inmuniz Rh - Tío polihidramnios - Sd transfusión Feto-Fetal

Inm aduro m adurez incompleta m adurez completa

2 >2

No existe No existe Existe

M O N ITO RIZA CIÓ N FETAL. Valora cuatro parámetros: frecuencia cardiaca fetal (FC F), variabilidad, ascensos, deceleraciones.

Tipos de detelerationes. DIP II

DIPS UM BILICALES

S in c r ó n ic a .

D e c a la je 1 0 a 4 0 seg .

D e c a la je v a r ia b le .

Morfología

E s p e c u l a r c o n la c o n t r a c c ió n .

E s p e c u l a r c o n la c o n t r a c c ió n .

V a ria b le .

Causa

E s t ím u lo v a gal p o r c o m p r e s ió n c e f á lic a ( c e d e n c o n a t r o p in a ) .

H ip o x ia .

C o m p r e s ió n d e l c o r d ó n .

Gravedad

Leve.

M u y g r a v e (v e r p H e n s a n g r e f e t a l) .

V a r ia b le s e g ú n f r e c u e n c ia e i n t e n s id a d .

DIP 1 Relación con la contracción

R

epasa

:

[P

r o n ó s t ic o

eeta l seg ú n

l a m o n it o r iz a c ió n

, A

pa rta d o

22.4,

120 www.FreeLibros.me

C

a p ít u l o

22

d el

M

a n u a l

C TO 8 a Ed.]

......................................G

in e c o l o g í a y o b s t e t r ic ia

Es m uy preguntada la actitud obstétrica en función de esta evaluación. Repásala en el siguiente esquema.

Astitud obstétrica según el registro cardiotocográfico intraparto.

CA PÍTU LO 21.

I

l

i

Normal

Repetir microtoma en 15 minutos

Extracción fetal inmediata

FISIO LO G ÍA D EL EMBARAZO.

En el gráfico de la página siguiente tratamos de esquem atizar los cambios más relevantes que se producen en el organismo de la m ujer em barazada. No obstante, en las siguientes líneas resum im os los aspectos más importantes de este tem a: - Dilatación del músculo liso a todos los niveles por el aum ento de progesterona (a ello se debe la dism inución de TA inicial, la dism inución de motilidad intestinal con el consiguiente R G E, estreñim iento y litiasis biliar, y la crisis renoureteral sin litiasis). - Anem ia relativa.

- i de iones y proteínas plasmáticas (excepto fibrinógeno y trasportadoras que T). - T de hormonas pero i de su acción (ej. insulina).

CARDIOVASCULARES Volumen vascular

Aumenta durante toda la gestación

TA

1 en 1 c y 29 trimestre T en tercero

Auscultación

Posible soplo sistólico funcional.

ENDOCRINAS

Tamaño cardíaco

Desviación del eje a izquierda.

Frecuencia cardíaca

Aumenta

Hipófisis Tiroides

t GH í TSH t ACTH t Prolactina Aumenta el tamaño

PULMONARES Ventilación

Aumenta. Causa alcalosis respiratoria

NEFROLÓGICAS Función renal

Aumenta el filtrado glomerular i creatinina y urea sérica.

www.FreeLibros.me

121

ULTRA-RESÚMEIMES. Manual CTO de Medicina y Cirugía, 8 .a edición :H

C A P IT U L O 23.

HEM ORRAGIAS D EL 1er T R IM ESTR E: ABORTO. GESTACION ECTÓ PICA. ENFERM EDAD TRO FO BLÁ STICA .

CO N CEPTO S. 12

_ J __

Neonatal U rdía

fatMO» natal O.

0 c M ortalidad peri natal

t .

1

& ■o 3

M ortalidad p e riru t*! am pliada

i 500 g.

A b o rto d ife rid o

A borto en curso

i

*

!

A bo rto consumado: - C om pleto • Incom pleto

1000 g. - A m e n a z a d e a b o rto : sangrado vaginal-. O rific io c e rv ica l in tern o COCI) c e rra d o .

-Aborto p re c o z: en las p rim eras 12 sem anas. -Aborto ta rd ío : e n tre las sem a na s 12 y 2 0 .

- A b o rto e n cu rso : co n co n tra cc io n e s u te rin a s fuertes. A so m a n restos.

-Parto in m a d u ro ; en tre sem a na s 2 0 y 2 8 . Feto in v ía b le ( < ‘1 0 0 0 g).

- A b o rto in m in e n te : O C I a b ierto . C o n tra c cio n e s fuertes.

-Parto p re té rm in o : en tre se m a na s 28 y 3 6 . Feto via b le {> 1 0 0 0 g).

- A b o rto c o n su m a d o : cesan las c o n tra cc io n e s y e l do lor. - A b o rto c o m p le to ; e xp u lsió n total d e los restos. - A b o rto in co m p le to : persisten restos en la c a vid a d u terin a. - A b o rto d ife rid o : p ersisten restos 4 sem a n a s. Riesgo d e C ID .

D IA G N Ó STIC O DIFERENCIAL DÉ LAS HEMORRAGIAS DEL 1 er TRIMESTRE. EMBARAZO ECTÓ PICO

ABORTO

ENF. TROFOBLÁSTICA Amenaza - Escasa.

Hemorragia

Dolor

- R o ja , con coágulos. Leve.

- Ú tero gestacional norm al. - O C I cerrado.

Bcta-HCC

Ecografía

- R oja, con coágulos. - Co ntinu a.

- D ilatación cervical. - O C I abierto.

Complicado

Escasa, interm itente, oscura.

Co ntinu a.

Leve.

EN P U Ñ A LA D A .

- Variable. - A veces con V E S ÍC U LA S (esto es diagnóstico). Leve.

- FIE B R E . - A FE C T A C IÓ N D EL ESTA D O G E N E R A L . -A N E M IA . - H IP O T E N S IÓ N . - SH O CK.

Signos subjetivos de gestación.

- Ú tero m enor que am enorrea, tacto vaginoabdom inal

- A nem ia. - H IP ER EM ESIS. - P R EEC LA M P SIA . - (H ipertiroidism o).

- Tacto vagino­ abdom inal m uy

Útero m ayor que am enorrea.

doloroso, signos de irritación peritoneal.

doloroso, a veces se palpa masa an exial.

- Cifras bajas para am enorrea.

D ism inución progresiva.

Latido fetal presente.

En curso

Con las contracciones.

C O N T R A C C IO N E S U TER IN A S.

Otros síntomas

Exploración

Inminente

M U Y ELEV A D A .

- Crece m enos del 6 6 % a las 48 h.

- Latido ausente. - En incom pleto o

- Latido ausente. - No saco gestacional.

Saco gestacional en trom pa, útero vacío.

- Imagen "en copos de nieve".

¿SHOCK? Sí

M-

-

-

►No

i

i

4 0 añ os D e se o s g enésico s cu m p lid o s

E xp lo ra ció n c lín ic a re p e tid a H iste re cto m ía

D e te rm in a c io n e s sem a n a le s d e P - H C G

sim p le co n -

-

Leg rad o co n asp ira ció n

R x tó ra x, p ru eb as d e fu n ció n h e p ática,

m ola in situ

T C c ra n e a l (si c lín ic a )

P - H C G e le v a d a 8 sem an as

R e m isió n co m p le ta

d e sp u é s d e la e v a cu a ció n C e s e h e m o rrag ia U te ro in vo lu c io n a d o A n e jo s n o rm ales P - H C G n o rm al 3 se m an as

P - H C G m e n su al 6 m eses

R x tó ra x

P - H C G b im e n su a l otros 6 m eses

P ru eb as d e fu n ció n hepática

A n tic o n c e p tiv o s 1 añ o

T C cra n e a l

N o rm al

A lte ra cio n e s

E n fe rm e d a d lo c a liz a d a

E n fe rm e d a d m etastásica*

E n fe rm e d a d

M o la in vaso ra o

B ajo

A lto

tro fo b lástica

c o rio c a rc in o m a

riesgo

riesgo

M e to tre xa te

M e to tre xa te

M e to tre x a te

P o liq u im io te ra p ia

+ le g rad o /h iste re cto m ía

+ h iste re cto m ía

p ersistente

* E n la e n fe rm e d a d m etastásica la h iste re cto m ía no tien e u tilid ad .

www.FreeLibros.me

123

ULTRA-RESÚMENES. M an u al C T O de M edicina y Cirugía, 8 .a edición ¡ > M

CA PÍTU LO 31. REPASA:

ESTADOS H IPERTEN SIVO S D EL EMBARAZO.

[flSIOPATOLOGÍA

DE

LA PREECLAMPSIA, F |G U R a 7 6 , CAPÍTULO 3 1

D EL

MANUAL

CTO

8 a Ed .]

Tratamiento. Lo verdaderam ente eficaz para tratar a la m adre es poner fin a la gestación. Ahora bien, tam bién hay que considerar el grado de m adurez fetal antes de provocar el parto. A) Tratamiento sintom ático. - Antihipertensivos: >Hidralacina: de elección, uso intrahospitalario. >a-metildopa: en casos leves, control am bulatorio. >N ifedipina, labetalol. - Anticonvulsivantes: >Sulfato de magnesio: puede provocar hiporreflexia y depresión respiratoria. Antídoto: gluconato cálcico.

1.

¿Qué sospecharías ante un diagnóstico de preeclampsia antes de la 20a semana de gestación?

1.

Embarazo molar o ¡estación múltiple.

B) Tratamiento fisiopatológico.

Leve: TA > 140/90 en al menos 2 ocasiones

Eclampsia

Grave

- Ingreso hospitalario. - Reposo en decúbito lateral

pretérm ino.

izquierdo. - Tratamiento am bulatorio con alfa-m etildopa oral.

FIN A LIZA R G ESTA C IÓ N

- M aduración pulm onar si - Tratamiento i.v. con hidralacina

>' Vaginal

« Cesáreas

y sulfato de magnesio.

- Alteración materna - Alteración fetal

- Si persiste TA > 160/110,

- Parto no evoluciona

interrum pir gestación.

REPASA:

[C

r it e r io s d e g r a v ed a d

d el

M

a n u a l

CA PÍTU LO 24.

C T O

e n l o s e s t a d o s h ip e r t e n s iv o s d e l e m b a r a z o

, A

31.4, C a p i t u l o 31

parta d o

8 a Ed .]

HEMORRAGIAS D EL T E R C E R TR IM ESTRE___________________

PLACENTA PREVIA Y A BRUPTIO PLACENTAE D IA G N Ó STICO DIFERENCIAL HEMORRAGIAS D EL TERCER TRIMESTRE



IP E R T O N ÍA

A b ru p tio placen tae

A T O N IA

R otu ra U te rin a

¿ T O N O M U S C U L A R ? :. C e s á re a urgente

D o lo r A b d o m in a l V asos previos No

• ¿ S U F R IM IE N T O FETA L? No

^REPA SA:

[D

ia g n ó s t ic o

d el

M

a n u a l

d ie e r e n c ia l

C T O

d e l a s h e m o r r a g ia s d e l

P la ce n ta p r e v i a

3er t r i m e s t r e ,

T

a b la

► A ctitu d va ria b le

26,

C

a p ít u l o

24

8 a E d .]

A C TIT U D ANTE LA PLACENTA PREVIA. Se resume en:

-

Esperar mientras no haya riesgo ni para la madre ni para el feto. Inducir el parto cuando aparezca riesgo o cuando el feto esté m aduro. Preferir la vía vaginal siem pre que la placenta no obstruya el canal.

¡NO DAR TOCOLÍTICOS SI EXISTE HEMORRAGIA SEVERA! ¡NO HACER TACTO VAGINAL HASTA HABER INSPECCIONADO CON VALVAS!

124

www.FreeLibros.me

G

in e c o l o g ía y o b s t e t r ic ia

OTROS CUADROS PARA EL DIAGNOSTICO DIFERENCIAL. - Abruptio: Com o norma general, una vez diagnosticada, se finaliza la gestación (generalm ente cesárea urgente).

- Rotura de vasa previa: Precisa cesárea urgente. Alta mortalidad fetal.

- Causas de hemorragia puerperal: Desgarros

H erida placentaria (80%)

( 20%)

Restos ptaceníarios

Atonía uterina

Parto

Placenta anómala

nversion Coaguuterina lopatías

2 h.

24 h.

6 sem.

Alumbramiento

1.

¿Cuát es la causa más frecuente de rotura ute­ rina?

1.

La dehiscencia de una cicatriz de una cesárea previa.

2.

¿Qué hemorragia del 3er trimestre es caracte­ rísticamente de sangre fetal?

2.

La rotura de vasos previos; se manifiesta como hemorragia y sufrimiento fetal en la amniorrexis, sin afectación materna.

3.

¿Cuál es el diagnóstico de sospecha y la actitud ante un parto en el que al realizar la amniorrexis aparece hemorragia de sangre roja y sufri­ miento fetal?

3.

Rotura de vasa previa; es una urgencia obsté­ trica y precisa cesárea inmediata.

CA PITU LO 32.

D IA BETES GESTACIO N AL.

FISIOPATOLOGIA , i insulina

►1a mitad del embarazo

Hipoglucemia materna ("la glucosa se está guardando"!

"Fase de reservas"

ANABOLISMO 4- necesidades de glucosa

t depósitos grasos

Metabolismo de la gestante

— Insulinorresistencia

2- mitad del embarazo *"Fase de aprovechamiento ( de las resen/as"

T glucemia (NO SIEMPRE HAY HIPERCLUCEMIAi

Uso de ácidos grasos (Lipolisis)

CATABOLISMO f de las necesidades de insulina

T cuerpos cetónicos

www.FreeLibros.me

125

ULTRA-RESÚMENES. M a n u a l C T O de M edicina y C irugía, 8 .a edición >

PRUEBAS DIAGNÓSTICAS DE LA DIABETES GESTACIONAL. TEST DE OSULLIVAN

PERFIL GLUCÉMICO

SOBRECARGA ORAL DE GLUCOSA

Indicación

- Screening a todas las gestantes.

- O'Sullivan patológico. - Anamnesis positiva.

- Sobrecarga patológica.

Momento

- Semana 24 de gestación.

- En los 3 trimestres si es preciso.

- Cualquier momento.

- Dar SOgde glucosa.

- Dar 100gde glucosa.

- Dar dieta de 1800 cal.

- Glucemia 60 min La más im portante, la esterilidad. >La más frecuente, las molestias pélvicas crónicas.

TUBERCULOSIS GENITAL. Repásala con las siguientes cuestiones.

132

1.

¿Qué vía de diseminación utiliza?

1.

Hematógena.

2.

¿Qué es el S. de Netter?

2.

Oclusión de la cavidad uterina como secuela de una endometritis tuberculosa.

3.

¿Cuál es el método diagnóstico más adecuado?

3.

Microlegrado uterino.

4.

¿Cuál es la localización más frecuente?

4.

Localización tubárica.

www.FreeLibros.me

___________________G CA PÍTU LO 19.

i n e c o l o g í a y o b s t e t r ic ia

CÁ N C ER DE MAMA.

Es la neoplasia más frecuente en la m ujer (y probablemente por eso m uy preguntado en el MIR). El cáncer de m am a se considera sistém ico desde que sobrepasa la m em brana basal. Por lo tanto, requiere para su tratam iento un abordaje local y un abordaje sistém ico. Sólo nos limitaremos al tratam iento local (cirugía y radioterapia) cuando la linfadenectom ía sea negativa (resección de al menos 10 ganglios, siendo todos ellos negativos) y no haya factores de mal pronóstico.

SECUENCIA DIAGNÓSTICA. Ha Clínica ___________^ Mamografía Exploración ^ Ecografía (en 2. No obstan­ te, es obligatorio medir las cifras de andróge­ nos para asegurar su etiología (podría ser tu-

moral...).

136

www.FreeLibros.me

G CA PITU LO S 12 Y 13.

iim e c o l q g ía y o b s t e t r ic ia

PATOLOGIA D EL C U ELLO UTERINO.____________________________

La ectopia no es una lesión premaligna, sin embargo, la metaplasia puede evolucionar a displasia en un pequeño porcentaje de los casos. D IA G N Ó STIC O Y TRATAMIENTO DE LA PATOLO GÍA CERVICAL.

Negativa

Positiva

C^Repetir anulmente^}

Negativa

Insatisfactoria (no se visualiza zona de transición)

Positiva

Neoplasia intraepitelial cervical

Repetir citología

SIL Bajo grado SIL

Alto grado

Crioterapia, láser o termocoagulación

Curación * * .No curación

Conización *

C^JJiopsiaT^) y estadificar

1.

La presencia de epitelio cilindrico en zonas de epitelio plano estratificado (distal a la unión escamocolumnar) se denomina...

1.

Ectopia.

2.

La presencia de epitelio plano estratificado en zonas de epitelio cilindrico (proximal a la unión escamocolumnar) se denomina...

2.

Metaplasia.

3.

¿Qué se considera una colposcopia positiva?

3.

En la tinción con ácido acético: leucoplasia con o sin punteado rojo (base) o enrejado (mosai­ co). En la tinción con lugot (test de Schiller): zonas no teñidas.

4.

¿Y una colposcopia negativa?

4.

Para considerarla negativa, hay que ver la unión escamocolumnar. En caso contrario, la prueba es insatisfactoria o indeterminada.

5.

¿Cuándo se debe hacer una citología?

5.

Desde el comienzo de las relaciones sexuales hasta los 35 años anualmente, después hay me­ nos acuerdo hasta los 65.

6.

¿Y en las pacientes sin relaciones sexuales?

6.

Aunque el riesgo es muy bajo, existen casos de cáncer de cérvix en vírgenes. Además la citolo­ gía detecta otras alteraciones.

www.FreeLibros.me

137

ULTRA-RESUMENES. M an u a l C T O de M edicina y Cirugía, 8 .a edición

_____

CÁNCER DE CÉRVIX.

L

( E S T A D IO S N O Q U IR Ú R G IC O S

ESTADIOS Q U IRU RG ICO S

Si deseos genésicos no cum plidos: conización En caso contrario: histerectomía lh lia

W ertheim Meigs (histerectomía ampliada linfadenectomía ilíaca y obturatriz). Si invasión linfática: RT externa

lili Illa lllb

,O

Braquiterapia + RT externa + quimioterapia

/

IVa IVb

Por lo tanto, un carcinoma de cérvix deja de ser quirúrgico cuando afecta parametrios.

Este es un buen momento para recordar las características más importantes de las principales neoplasias ginecológicas, puesto que año tras año son preguntadas en el MIR. Fíjate en la tabla de la página siguiente, que recoge todas las características fundamentales de cada tipo de tumor, incluyendo factores de riesgo y métodos diagnósticos.

1. ¿Cuál es el método de screening para el cáncer de Cérvix?

1.

La citología.

www.FreeLibros.me

G

in e c o l o g ía y o b s t e t r ic ia

E N D O M E T R IO

CÉRVIX

O V A R IO

M AM A

- 6 0 a ñ o s (tu m o re s

Edad

4 0 añ o s. (20 a ñ o s d isp lasias)

e p ite lia le s).

55 añ o s.

- 2 0 años

A u m e n ta riesgo co n la e d a d .

(g e rm in o m a s)

- N u l ¡gesta. - VPH. - P ro m isc u id a d .

F. Riesgo

- A n te c e d e n te s

- E stró g en o s no

- Tab aco. - In ic io p re c o z de re la c io n e s se x u a le s.

c o m p e n sa d o s.

p e rs o n a le s / fa m ilia ­

- O b e s id a d .

res.

- M e n o p a u s ia ta rd ía .

- N u l¡g esta.

- M e n a r q u ia te m ­

- SO R

p ra n a /m e n o p a u sia

- N u lip a rid a d .

ta rd ía .

- D M , H TA

- R a d ia c ió n . - R a z a b la n c a .

|

F. Protectores

No p ro m isc u id a d .

Diagnóstico: - Screening

(cito lo g ía)

- Definitivo

Biopsia.

CA PÍTU LO 7.

Sí.

-A C O .

-A C O .

- C e s ta c ió n an te s

- M u ltíp a ra s .

- SO R

d e lo s 3 0 añ o s.

N o.

No.

S í (m a m o g ra fía ).

Le g ra d o .

P o stq u irú rg ico .

B io p sia.

EN D O M ETRIO SIS.

"A más menstruación, mayor riesgo": menarquia precoz, ciclos cortos, menstruación abundante... Sospecha clínica (dismenorrea secundaria, infertilidad)

Ecografía

AINE

pt -wW-, ^ Si persiste el dolor

Anticonceptivos orales (potentes, androgénicos, anovulatorios) Si persiste el dolor tras 6 meses de tratamiento

Laparoscopia eliminando todos los implantes posibles (Si muy extenso: cirugía abierta) + Análogos de GnRH 6 meses

Seguimiento (ecografía + CA 125)

www.FreeLibros.me

139

H CA PÍTU LO 1.

e m a t o l o g ía

CO N CEPTO Y EVALUACIÓN DE LA S ANEMIAS.

Las anem ias constituyen uno de los temas estrella en el M IR. La m ayoría de las preguntas hacen referencia a aspectos clínicos, analíticos y de diagnóstico diferencial.

ESTUDIO GENERAL DE LAS ANEMIAS. ANEMIA

HIPOPROLIFERATIVA

(Prod u cción in eficaz)

HEMORRAGIA/ HEM OLISIS (t destrucción)

f

H em orrag ia

D EFECTO M ADURACIÓN

( i pro d u cció n)

D añ o m ed ular

D efectos citoplásm ico s • Talasem ia

Deficiencia de hierro

H e m o lisis

1 _

• Sideroblásticas

V A uto in m u n es

i'

Inflamación

Hemoglobinopatías

l Estím ulo

D efecto s n ucleares

- Enferm ed ad renal.

• D éficit de fó lico

- Enferm ed ad m etab ólica.

• D é ficit de vitam in a B

Reticulocitos

TT

fo t

Nof

N

t oN

Nof

N

T o N

N ot

N,4oT

LDH Bilirrubina

D efecto m em brana/en zim ático

j oN

ir

VCM



Recuerda que, una vez que has diagnosticado la anem ia m ediante el descenso de hemoglobina, debes clasificarla mediante el V C M , porcentaje de reticulocitos y frotis de sangre periférica, en el que pueden aparecer hematíes con morfología típica que orientan a la causa.

^

REPASA:

[ A n e m ia s en f u n c ió n d e l

REPASA:

[ D if e r e n t e s t ip o s d e c é l u l a s s a n g u ín e a s , F ig u r a

VCM d e l

h e m a t íe , T a b l a

1,

1,

C a p ít u lo

C a p ít u lo

1

1del

M an u al

d el M a n u al

CTO 8a E d .]

CTO

8 a E d .]

Une los siguientes procesos con su causa más fre­ cuente: 1. 2. 3. 4. 5. 6.

Anemia Microcitosis eritrocitaria Anemia normocítica Macrocitosis Anemia megaloblástica Aplasia adquirida

a. b. c. d. e.

Anemia de trastornos crónicos. Alcoholismo. Déficit de folato. Idiopática. Ferropenia.

Respuestas: fe, 2e, 3a, 4b, 5c, 6d.

CAPÍTULOS 4 Y 6.

ANEMIAS FERROPÉN ICA Y M EGALOBLÁSTICA Y SU DIAGNÓSTICO DIFERENCIAL.

ANEMIA FERROPÉNICA. La ferropenia es la causa más frecuente de anem ia. En el M IR suelen preguntar el diagnóstico diferencial entre ferropenia, talasem ia, anem ia de los trastornos crónicos y anem ia sideroblástica. Repásalo en el siguiente algoritmo.

141

www.FreeLibros.me

ULTRA-RESÚMEIMES. M an u a l C T O de M e d ia n a y Cirugía, 8 .a edición

Anemias microcíticas: diagnóstico diferencial. Anemia microcítica (VCM< 80 fl)

Ferritina sérica

1

l Baja

Normal o elevada

I Siderem ia + Transferrina

Ferropenia

f Dism inuidas

S: N ó t T: N

▼ Hb A 2 y Hb F Valores

Anem ia de trastornos crónicos

Hb A ,: 2-3% HbF: 20 años

Neumococo* Meningococo

C 3C ± Vaneo*

Ancianos

Neumococo* Listeria

C 3C + Ampi ± Vaneo*

Embarazado (muerte fetal) Listeria (además de los propios de Puérpara su edad, etc.) Alcohólico Oncológico Inmunosuprimido celular

Añadir Ampi (a pesar de que el tto. ya incluya vaneo)

B C P e n LCR

Listeria

Ampi

Déficit C5-C8 (más frec.) Déficit de properdina

Meningococo (menor mortalidad)

C 3C

Fractura de la base Fístula LCR Meningitis recurrente Patología O RL

Neumococo*

C 3C ± Vaneo*

Endocarditis TCE abierto

S. aureus

Derivación LCR

S. epidermidis

Neuroci rugía Nosocomial

S. aureus R aeruginosa

Vaneo + cefepime

Tbc (proteínas, mantoux, epi, etc.)

Tuberculostáticos + CC

Criptococo (mayor IS - CD4 < 100, criptolátex-).

Anfo B + 5 fluociosina

IS celular

Vaneo o cloxa

* Dependiendo del ámbito comunitario, a veces es resistente o no.

175

www.FreeLibros.me

ULTRA-RESÚM EN ES. Manual CTO de Medicina y Cirugía, 8 .a edición '-A

Attitud ante la sospetha d e meningitis.

PMN

i glucosa proteínas

linfodtos

i glucosa proteínas

C L ÍN IC A 1 O • Fiebre • Cefalea • Confusión

hipertensión

• Vóm itos

intracraneal:

• Signos meníngeos (rigidez de nuca, Kernig, Brudzinsky) • Crisis com iciales

Fondo de ojo

-Meningitis (M) bacteriana -M. tuberculosa o viral en fase precoz

CAUSAS INFECCIOSAS

D escartar

PUNCION * LUMBAR

(no e d em a de papila), TC

-M. químicas -Enf. de Behget

CAUSAS NO INFECCIOSAS

linfodtos glucosa normal proteínas

-M. tuberculosa -M. por hongos -M. por listeria, leptospira y algunos Virus - Brucelle -T. pallidum

-M* q encefalitis viral -Infecciones parameníngeas (otitis, mastoiditis) -Parásitos (toxoplasmosis, triquinelosis, cisticercosis)

-Carcinomatosis meníngea -Sarcoidosis meníngea

-Encefalomielitis postinfecciosa -Enfermedades desmiel inizantes -M. de Mollaret

Adem ás de para la bioquím ica y recuento celular, el LCR obtenido mediante punción lum bar puede utilizarse para: - Cultivo: diagnóstico definitivo. Detección de antígenos: neum ococo, H. influenzae, m eningococo A y C ... Estudio d e hongos: antígeno criptocócico, tinta china, cultivo ... - Serología: virus, Brucella, sífilis (V D R L). - Citología: carcinom atosis meníngea. - Gram: el resultado del Gram orienta hacia el germen causal.

Aspecto microscópico de los distintos gérmenes en la tinción Gram.

C o co s G ra m + en racim o

Estafilococo

C o co s G ra m + en cad ena

Estreptococo

D ip lo c o c o s G ra m +

N eu m o co co

B a cilo s G ra m +

Listeria

C o co s G ra m -

M ening ococo

B a c ilo s G ra m -

Enterobacterias

C o c o b a cilo s G ra m -

H aem o p h ilu s

Profilaxis. a) Inmunoprofilaxis. Existe vacuna frente a: -

Haem ophilus influenzae B : incluida en el calendario de vacunación sistemática. M eningococo A y C . No hay vacuna frente al tipo B, que es el responsable de la mayor parte de las meningitis meningocócicas. Está indicada en situaciones de epidem ia, sobre todo en menores de 1 5 años. Streptococcus pneumoniae.

b) Quimioprofilaxis. Ante un cuadro de meningitis meningocócica, es necesario hacer profilaxis a los contactos íntimos, tanto niños como adultos. Se usa rifam picina (alternativa ciprofloxacino, o floxacino o (ceftriaxo n ay espiram icina: embarazadas y niños). - Niños mayores de un mes y adultos: 10 mg/Kg (hasta 600 mg) cada 12 horas durante dos días. - Niños menores de un mes: 5 mg/Kg cada 1 2 horas durante dos días. Si el cuadro es una meningitis por H a e m o p h ilu s, se debe realizar profilaxis a contactos íntimos de fam iliares o de guardería menores de 6 años. Se usa rifam picina: - Niños m enores de un mes: 1 0 mg/Kg/día durante 4 días. Niños mayores de un mes: 20 mg/Kg/día (hasta 600 mg) durante 4 días.

176 www.FreeLibros.me

E 1.

¿Qué virus producen típicamente hipoglucorraquia?

1.

Parotiditis, CMV, coriomeningitis linfocitaria.

2.

¿Qué te sugiere una celularidad en LCR » 50.000 cls/mm3?

2.

Un absceso cerebral roto a ventrículos.

3.

¿Podrías enumerar alguna causa de me­ ningitis no infecciosa que curse con LCR hipoglucorráquico, con aumento de pro­ teínas y de PMN?

3.

Las meningitis químicas y la meningitis de la enferm edad de Beh2 ó 3 semanas (aunque el intervalo es variableJO hematuria. El tratamiento se basa en: > Plasmaféresis (el único que mejora la función renal) + corticoides o inmunosupresores. > Puede realizarse trasplante siempre que se haga fuera del período de actividad de la enfermedad.

188

www.FreeLibros.me

IMe f r o l o g í a 1.

¿Qué prueba diagnóstica pulmonar es caracte­ rística?

1.

La DLCO está aumentada, lo que es debido a la presencia de sangre en alvéolos.

2.

¿Cuál es el patrón de afectación renal?

2.

Desde glomérulos casi normales a CN focal proliferativa y necrotizante con semilunas. La IFD m uestra un depósito lineal de IgG (son Ac. AMBG, presentes en el 90% de los casos).

3.

¿Qué HLA se asocia en el 80-85% de los casos?

3.

HLA-DR2.

Púrpura de Schónlein-henoch. Aunque este tem a lo vas a ver más detenidam ente en Reumatología, lo m encionam os brevemente aquí.

Señala la falsa respecto a esta patología: a. Asocia artralgías, púrpura no trombopénica, dolor abdominal y glomerulonefritis. b. Requiere la realización de plasmaféresis aso­ ciada a esteroides en dosis altas. c. Cursa con niveles de IgA elevados en el 50% de los casos y de FR en el 60%. d. La enfermedad de Berger es una forma monosintomática de la enfermedad. e. La afectación renal cursa con hematuria y proteinuria.

R espu esta: La falsa es la b; la plasmaféresis sólo se emplea en casos graves; generalmente sólo suete requerir tratamiento sintomático. Los corticoides no mejoran su evolución. El resto son características de este cuadro.

Nefropatía diabética. En un paciente diabético, el diagnóstico de nefropatía requiere la detección de m icroalbumlnurla (20-200 pg/min) en más de dos muestras en un periodo de 3 a 6 meses.

Evolución de la nefropatía diabética. E V O L U C IO N

T E R A P E U T IC A

▼ In ic io h ip e rg lu c e m ia 10-15 años



I- C o n tr o l m e ta b ó lic o

M ic ro a lb u m in u ria

*

- D is m in u ir H T A (e le c c ió n IE C A )

( > 2 0 pg/m in) 3-7 años ^ P ro te in u ria m an ifie sta

C o n tro l e stricto

[

(M a c ro p ro te in u ria > 3 0 0 mg/d)

d e la H T A (e le c c ió n IE C A )

N e fro p a tía c lín ic a 5-7 años ^

I -D iá lisis (c u a n d o a c l. C r < 1 5 m l/m in)

I.R . T e rm in a l

[-Trasplante

1.

Cita dos cuadros clínicos que se den con más frecuencia en los pacientes diabéticos que en la población general.

1.

Infecciones urinarias y necrosis de papila.

2.

¿Sabrías decir por qué en la nefropatía diabéti­ ca existe mayor riesgo de hiperpotasemia?

2.

Por un lado, se debe al hipoaldosteronism o hiporreninémica (acidosis tubular tipo IV) y por o t r o , al uso h a b itu a l d e IECA co m o antihipertensivo. El riesgo aumenta cuando es necesario restringir la sal (edemas, HTA).

189

www.FreeLibros.me

ULTRA-RESÚMEIMES. Manual CTO de Medicina y Cirugía, 8.a edición

■■■■

3.

¿En qué enfermedad puede verse una lesión sim ilar a la de Kim m estiel-W ilson (glomeruloesclerosis nodular periférica)?

3. En la nefropatia por cadenas liseras.

4.

¿Puede ser corregida la microalbuminuria?

4. El control de la HTA (con lECAs) y la hiperglucemia co ra sen la microalbuminuria en las fa ­ ses iniciales.

M ediante unas autocuestiones, repasa otras patologías con afectación renal:

1.

En la PAN, el riñón se afecta en el 90% de los casos. ¿Cuál es la manifestación más frecuente?

1.

Hipertensión arterial.

2.

¿Cuál es la manifestación renal más frecuente de la granulomatosis de Wegener?

2.

Hematuria y proteinuria en ranso variable.

3.

¿Recuerdas qué anticuerpos eran sensibles y específicos de esta enfermedad?

3.

Los c-AN CA; p e rm ite n e l d ia s n ó s tic o del Wesener y su sesuimiento.

4.

¿Qué tipo de lesiones renales puede producir la A.R.?

4.

Una amiloidosis secundaria, una GN membra­ nosa (por sales de oro y penicilamina) o proliferativa, afectación renal por vasculitis reumatoide o necrosis papilar y nefropatia tubulointersticial (por uso de analsésicos).

5.

¿En qué consiste el síndrome de Alport?

6.

¿Qué dato característico aparece en el ME?

6.

Membrana basal deslaminada en "capas de ho­ jaldre".

7.

La manifestación clínica renal más frecuente de la amiloidosis, ¿cuál es?

7.

Proteinuria.

8.

Cita algunas patologías que puedan cursar con amiloidosis renal secundaria.

8.

Osteomielitis, Mieloma Múltiple, AR, tbc.

CAPÍTULO 3.

5. - Sordera neurosensorial. - Nefritis hereditaria (hematuria más proteinuria). - Esferofaquia o lenticono. - Recuerda que en estos pacientes los anticuer­ pos antiMBG no pueden reaccionar, salvo que sea sobre un riñón trasplantado.

IN SU FICIEN CIA REN AL AGUDA.

Respecto a este cuadro, lo más importante es que sepas reconocerlo y distinguir entre IRA prerrenal, parenquimatosa y postrenal. Para esto, además de los datos clínicos, son importantes los datos analíticos. IRA P re rren a l 7 0 -8 0 %

IR A P are n q u im a to sa 1 0 -2 0 %

D escenso de perfusión re n al: * 4-Vol. sangre efectivo: - A b so lu to : hem orragia, pérdidas ren ales, G l o cutáneas 3er espacio. - R elativo: IC C , cirrosis, síndrom e nefrótico. * O clusión arterial: - Trom boem bolism o. - Aneurism a a. renal o aorta. • A IN Es, IECA.

Factores fa vo reced ores: - Edad avanzad a. - D eshid ratación. - Enf. renal subyacente. Factores de sen ca d en a n tes: - Isquem ia reanl m an tenid a: hem orragias, ICC , fárm acos vasoconstrictores. - Agentes nefrotóxicos: - Am inogucósidos. - Contrastes radiológicos (IRA no oligúrica). - H em olisis (Hb).

190 www.FreeLibros.me

IR A P o stre n al 10%

- Litiasis renal. - H iperp lasia prostética.

IMe f b o l o g ía Fíjate en los datos (véase Tabla 2, Capítulo 3 del Manual CTO 8a Ed.): en la IRA prerrenal indican todos una orina con­ centrada, porque el riñón trata de recuperar el poco agua que le llega; en cambio, en la parenquimatosa fracasa en esta función y los datos indican una orina muy diluida (aunque tenga oliguria) por déficit intrínseco de la función renal. Para el diagnóstico diferencial también nos sirve de ayuda la historia clínica del paciente (sus antecedentes), pues nos puede orientar respecto a la etiología del cuadro, y por tanto, al tipo de insuficiencia renal que se trate.

Tratamiento de la insuficiencia renal aguda.

IRA OLIGURICA TRATAMIENTO

J T Si hiponatrem ia: restricción hídrica Control del peso * D ieta:

A C ID O S IS

-Proteínas 0 ,7 g/Kg/día.

-Bicarbonato IRA

IRA

oral o i.v.

P R ER R EN A L

P A R EN Q U IM A TO SA

-M antener

-Carbohidratos 100g/día.

4 Control ingesta

H C 0 3> 15m m ol/l. . H IP ER P O TA SEM IA

de sal y agua.

Si sobrecarga de volum en: restricción de sal + diuréticos

Exp an dir volem ia.

Tto. específico:

(m antener

G N , S H U , PTT, etc

PV C 5-14 cm H 20 ) K + < 6 ,5 mEq/l EC G normal

▼ D IU R ESIS

D IU R ESIS

AD ECU AD A

IN SU FIC IE N TE

K + 6 ,5-7

K+>7

T picudas

arritm ias, Q R S anchos desaparición ondas P

v - Resinas intercam bio

A ñadir: - G luconato calcico. - Diálisis. - D iuréticos

Seguim iento

Furosem ida

catiónico.

y

a dosis altas.

- Restricción K +

- Bicarbonato. - S. glucosado. + insulina.

- Diuréticos

- Salbutam ol se.

observación.

v

A ñadir:

- D iuréticos IRA P O L IU R IC A

IRA O L IG U R IC A

Habrás observado que no aparece en el esquema la IRA postrenal, y esto es debido a que su tratamiento específico es muy intuitivo. ¿Qué harías en un caso así?... Efectivamente, habría que salvar la obstrucción; si es en vías altas, mediante cateterismo percutáneo; y si es en vía baja, mediante sondavesical. Si se produjese un síndrome de pérdida de sal, habría que administrar solución salina i.v. Más tarde se intentaría el tratamiento etiológico.

1. ¿Cuál es la causa más frecuente de necrosis tubular aguda?

í. Isquemia renal mantenida.

2. ¿Y la causa más frecuente de fracaso renal poliúrico?

2.

Toxicidad por aminoglucósidos.

3. ¿Qué técnica de imagen es la más útil en el diag­ nostico de la IRA posrenal?

3.

Ecografía.

4. La aparición de cilindros hemáticos en el sedi­ mento urinario es sugerente de....

4.

Olomerulonefritis o vasculitis.

5. ¿Cuál es la causa más frecuente de muerte en la IRA?

5.

Infecciones.

6. ¿Cuál es la causa más frecuente de anuria?

6.

Uropatía obstructiva.

191

www.FreeLibros.me

ULTRA-RESÚM ENES. Manual CTO de Medicina y Cirugía, 8.a edición

7.

La mejor prueba diagnóstica para diferenciar una IRA de una IRC es...

7. La ecografía.

8.

¿Cuál es la causa más frecuente de IRA postrenal?

8. Litiasis.

9.

¿Sabrías decir tres causas de eosinofiluria?

9. Nefritis por hipersensibilidad y émbolos de colesterol y Churg-Strauss.

^

REPASA:

[In d ic a c io n e s d e d iá lis is , T a b la

CA P ÍT U LO 4.

5,

EN FERM ED A D

C a p ít u lo

3 del

M an u al

CTO 8a E d .]

R E N A L C R Ó N IC A .___________________________________

Las preguntas que han caído sobre la IRC en las últimas convocatorias se han centrado en sus características clínicas y analíticas, posibles complicaciones y su tratamiento conservador.

Manifestaciones clínicas y analíticas de la IRC.

t Cr y urea en plasma t moléculas intermedias

Alteración función leucocitos y plaquetas

Alteración del intercambio iónico transmembrana

Pol¡neuropatía urémica (MM11, dolor urente; i vel. conducción) Encefalopatía urémica

i t T, de hemorragia

H i posten uría

I Jiperglucemia (por intolerancia a la glucosa)

Hipertrigliceridemia con colesterol normal y 4 H D L

Arteriosclerosis (t IAM, ACVA)

Pérdida de la función endocrina del riñón

En fase inicial: Poliuria y nicturia

En fases tardías: retención de agua y sal + oliguria

i síntesis de eritropoyetina

i Anem ia normocítica normocrómica

HTA volumen-dependiente

Adm. EPO recombinante humana (rHuEPO) Trasfusiones

Pérdida de la capacidad para

H i per potase mía

Fósforo, Calcio y hueso (mirar osteodistrofia renal)

Adm. 20 mEq/día de bicarbonato v.o.

Restricción de potasio

192

Acidosis metabólica (náuseas,, anorexia, p. peso, Kussmaul, desmineral ilación ósea)

www.FreeLibros.me

IM e f r o

l q g ía

Entre las complicaciones a destacar de la IRC se encuentra la osteodlstrofla renal, m uy preguntado en las últimas convocatorias, por lo que te presentamos el siguiente esquem a:

Osteodistrofía renal i síntesis 1-25 dihidroxicolecalciferol por el riñón



i FG al 25%

Osteomalacia/raquitismo Retención ele fosfato

(las co stilla s son los

I absorción intestinal Ca

hueso s m ás afectad o s)

i aporte de P f pérdidas renales de Ca ++

Resistencia periférica 'C avanzada -Yl a la PTH en IRC

(^ h ^ PO C A LC EM I^ ) ^---"—

Carbonato calcico sí Ca ’ ‘ < 7 '5 mg %

R e a b so rció n su b p e rió stica falang es)

Suplementos de Ca, vit. D y quejantes de P

Osteítis fibrosa quística

G Eritropoyetina

4-

C rá n e o en "sa l y p im ie n ta "

+

Q u is te s ó seo s p a rd o s

Calcificación metastásica (si Ca x P > 70)

El com ienzo del tratamiento sustitutorio de la fundó n renal (diálisis) no depende del nivel de B U N , urea o filtrado dom erular, como podrías pensar en principio, sino más bien de la calidad de vida del paciente y de la valoración riesgo/ oeneflclo.

Complitadones d el tto. dialítito. Dementia dialítita/Sind. d el desequilibrio. Demencia dialítit a

Síndrome del desequilibrio

Por a c u m u lo n e u ro n a l

Por d iá lisis rá p id a s o

d e a lu m in io

co n líq u id o d e d iá lis is in a d e c u a d a s

P é rd id a d e l in te le c to

E d e m a c e r e b r a l: s o m n o le n c ia ,

y d e la m e m o r ia , t e n d e n c ia

c o m a y m u e rte

■r

1 al s u e ñ o , c o m a y m u e rte

T ra ta m ie n to : E D T A

T ra ta m ie n to : re p e tir d iá lisis le n ta m e n te + m a n ito l a l 2 0 %

1.

Sabes que la insuficiencia renal crónica puede ser producida por múltiples causas; ¿recuerdas las más frecuentes?

1.

DM y en segundo lugar HTA.

2.

Las complicaciones de la IRC pueden llegar a ser mortales; una de ellas es la principal causa de muerte, ¿cuál?

2.

El IAM, ya que hay tendencia a la aterosclerosis (hipertrigliceridemia y disminución de HDL).

3.

¿Cuál de las siguientes manifestaciones clínicas de la IRC mejora con el tto. renal sustitutivo: disfunción sexual, hipertrigliceridemia, prurito, anorexia o alteraciones del sueño?

3.

La anorexia.

4.

En la IRC son característicos los riñones peque­ ños, pero en algunos casos esto no se cumple, ¿los recuerdas?

4.

Enf. poliquistica, amiloidosis, trombosis ve­ nosa renal, diabetes m ellitus, tesaurosmosis (E. Fabry) e infiltración linfomatosa

5.

¿Por qué existe un aumento de hemorragia di­ gestiva alta en los pacientes con IRC?

5.

Por hipersecreción ácida y/o trombocitopatia.

www.FreeLibros.me

193

ULTRA-RESÚMEIMES. Manual CTO de Medicina y Cirugía, 8 .a edición >1

6. ¿Recuerdas en qué consistía la enfermedad ósea adinámica?

CA PÍTU LO 11.

-

fr:

6. Es aquella causada por la interposición de alu­ minio (agua de diálisis, hidróxido de aluminio) en la matriz ósea, impidiendo su remodelado; s ó lo s e v e en p a c ie n t e s en d iá lis is tra s paratiroidectomia.

TRASTORN OS TU B U LA R ES Y Q U ÍSTICO S.

Los cuadros más preguntados de este grupo son: Enfermedades quísticas renales. Acidosis tubular renal. La siguiente tabla muestra las principales enfermedades quísticas.

CARACTERÍSTICAS DE LAS ENFERMEDADES QUÍSTICAS.

^■Aquística del adulto

HERENCIA

CLÍNICA/ COMPLICACIONES

DIAGNÓSTICO

EVOLUCIÓN

TRATAMIENTO

A.D. Cr. 16

-Hematuria, nicturia litiasis, ITU, quistes hepáticos, aneurismas intracraneales. Poliglobulia, HTA

Ecografía

Lenta a I.R.

El de las complicaciones

r

DIAGRAMA

,v

v° / .Q V

í/ ^

S to S

V

poliquística del niño

A.R. Cr. 6 (- frec.)

Masa abdominal bilat. HTA; incapacidad concentrar orina; HTP

A.R. A.D.

Ecografía

I.R. en poco tiempo

El de las complicaciones

- Poliuria, anemia nefropatía pierde sal - En la forma A.R.: degeneración retiniana

Ecografía, TAC

A.R. I.R. en 20a

- Alta ingesta de agua, Na, bicarbonato - Corregir anemia - Tto. de HTA

- Aparición bimodal - Asintomático (cálculos, ITU) - Nefrocalcinosis (benigna)

Pielografía i.v. Rx: "imagen de cepillo"

' %

c f

r i. , (>i,’í'-J Enfermedad quística medular

Riñón en esponja medular

Esporádico A.D.

Benigna No hay HTA

De la litiasis (si aparece) Evitar deshidrataciones

í

/ ].

/

t

/

Hay unos trastornos en los que está alterado el transporte de electrólitos en los túbulos.

DIABETES INSÍPIDA NEFROGÉNICA. Suelen preguntarla en forma de caso clínico; en él, te presentan una serie de datos analíticos de sangre y orina junto con síntomas típicos de diabetes (poliuria, polidipsia, etc.), pero no tiene hiperglucemia ni glucosuria. Es más frecuente en ancianos y lactantes (por no tener acceso libre al agua) y cursa con una deshidratación hipertónica y orinas hipotónicas. Su tratamiento consiste en hidratación + diuréticos tiacídicos.

1. ¿Con qué patologías hay que hacer el diagnósti­ co diferencial del síndrome de Bartter?

- Toma subrepticia de diuréticos. - Abuso de laxantes. - Vómitos. • Nefropatias crónicas. - Gitelman.

¿Cómo diferenciar un síndrome de Bartter de in síndrome de Gitelman?

2.

En el 1" caso nos encontramos con hipercalciuria, m ientras que en el 2 o hay Ca" en orina normal.

194 www.FreeLibros.me

IMe f r o l o g í a T R A ST O R N O S TU BU LA RES H ER ED IT A R IO S . SIN D R O M E

S IN D R O M E

D E RA RTTER (A.R.)

DE LID D LE

(hiperaldosteronismo hiperreninémico asíntomático)

(pseudohiperaldosteronismo)

I

Mutación en canal Na en tu bulo colector cortical

Defecto en transportador Na+:K +: 2C I+ en rama ascendente gruesa de Henle No reabsorbe K

Hiperplasia ap. yuxtaglomerular

Hipopotasemia

1

Expansión de volumen

© eje RAA

i

y H~

i

Alcalosis hipopotasémica

HTA

© e je R A A *

4- aldosterona

I

Vasodilatación

Hiperaldosteronism o

l

hiperrreniném ico



sin H TA ni edemas *■

No HTA

que se intercambia con K +y H +

El N a' se intercambia por el Ca++ y

t PG

T reabsorción de Na

No reabsorbe N a '

H ipercalciuria (lo diferencia del Gitelm an) Nefrocalcinosis (a veces)

Alcalosis

Tto.: Triamtirene o amiloride +

y acidiuria paradójica

restricción de Na+

Clínica: debilidad, poliuria, alcalosis hipo-K e h ip o -C i, hipo Mg, nefropatía pierde sal

S. de Lkldle

Tto: Suplementos de K +, Cl y N a+

\

Indometacina; diuréticos ahorradores de K+

ALDC ALDOSTERONA

y

HIPOPOTASEMIA

Acidosis tubular renal. Sobre este tema te suelen pedir que hagas el diagnóstico diferencial entre los distintos tipos de acidosis tubulares; fíjate en las últimas tres filas, pues te permitirán diferenciar unas de otras. REPASA:

[ A c id o s is t u b u la r e s r e n a le s , T a b la

40,

C a p ít u lo

11

d el M an u al

C TO 8 a E d .]

Por último, te recordamos su tratamiento: - AT tipo I: bicarbonato hasta que se eliminen la acidosis y la hipercalciuria. - AT tipo II: bicarbonato + suplementos K +/- tiacidas. - AT tipo III: corregir la acidosis + resinas atrapadoras de potasio +- fluorhidrocortisona.

1. ¿Qué entidades pueden aparecer asociadas a una acidosis tubular tipo II o proximal?

1.

Cistinosis, Mietoma Múltiple, Waldestróm, sarcoidosis, LES, toma de tetraciclinas caducadas...

2. Cita las principales causas de AT tipo IV

2.

■Nefropatía diabética ■Nefroansioesclerosis por HTA. - Nefropatías tubulointersticiales crónicas.

3. ¿En qué patología pensarías ante un niño con retraso de crecimiento, nefropatía pierde sal y anemia hemolítica? ¿Cómo la tratarías?

3.

Síndrome de Fanconi. Suplementos de P, K y HC03.

195

www.FreeLibros.me

ULTRA-RESÚM ENES. Manual CTO de Medicina y Cirugía, 8.a edición

CA PÍTU LO 6.

SÍNDROME N EFRÓTICO.

Señale cuál de las siguientes NO esta presente en el síndrome nefrótico: a) proteinuria >3 o 3,5 g/dia b) hipoalbuminemia P a C O ,. >Temperatura. >2,3 difosfoglicerato (D P C ). >Hidrogeniones (descenso de pH ). Efecto Bohr.

[ F ig u r a

1 7 y

T a b la 2

en e l A p a rta d o

3 .3 , C a p ít u lo

• DLCO: recuerda que depende de: -

202

Superficie alveolocapi lar. Volumen de sangre capilar pulmonar. Espesor m em brana alveolocapilar. Grado de discordancia V/Q. Grado de Hb.

www.FreeLibros.me

3

d el M an u al C T O

8 a E d .]

IM e u m o lo g ía y c i r u g í a

t o r á c ic a

Estas alteraciones en el intercam bio gaseoso pueden llevar a la hipoxem ia (PaO n< 8 0 mmHg). Los mecanismos causantes de la hipoxem ia se pueden diferenciar con el siguiente esquem a:

Algoritmo diagnóstico de la hipoxemia. HIPOXEMIA PaO2< 80 mmHg

Alta

Normal o Baja

I G ra d ie n te alveo lo arterial de

HIPOVENTILACIÓN

02

NO

CA PÍTU LO 16.

SI

l

¡

SH UN T

- ALTERACIÓN V/Q - ALTERACIÓN DIFU SIÓ N

EN FERM ED A D ES DE LA PLEU R A .

Este se ha convertido en el segundo tem a más importante de la asignatura de neumología. Procura estudiarlo en paralelo con la cirugía torácica; debes dom inar las indicaciones d eto raco cen tesisy la colocación de un tubo de tórax en los derram es pleurales.

1.

DERRAME PLEURAL.

Aprende a distinguir los derram es por las características del líquido y a distinguir los trasudados de exudados.

REPASA:

[ C a r a c t e r ís t ic a s d e e x u d a d o y t r a s u d a d o , A p a r ta d o

1 6 .1 , C a p ít u lo

1 6

d el M an u al

CTO

8 a

E d .]

Contesta ahora las siguientes autocuestiones sobre el derram e pleural:

1. ¿Cuál es la manifestación más frecuente?

1.

Dolor pleuritico.

2. ¿Cuál es el hallazgo radiológico más frecuente? ¿Y el más típico?

2.

Más frecuente: borramiento ángulo costofrénico posterior. Más típico: menisco de Damoisseau.

3. ¿Cuáles son las causas más frecuentes de exu­ dado y trasudado?

3.

Exudado: paraneumónico (TBC en ciertas zonas). Trasudado: insuficiencia cardiaca.

4. Además del aumento de ADA, ¿qué otro paráme­ tro es rentable en el diagnóstico del derrame tuberculoso?

4.

El aumento de interferón gamma.

www.FreeLibros.me

203

U LTR A -R ESÚ M EN ES Manual CTO de Medicina y Cirugía, 8.a edición A

Caraeterístitas de los príntipales derrames pleurales. CARACTERÍSTICAS

TIPO

Paraneumónico

- Segunda causa de exudado, y causa mas frecuente de derrame sanguinolento. - Frecuencia: pulmón >mama> linfoma. - Linfocitos, célula5 malignas. - Si disnea intensa: tubo de drenaje y pleurodesis con talco.

Neoplásico

- El exudado más frecuenté en algunas zonas. - Suele ser unilateral. - Linfocitos y ADA elevados,Escasez de células mesotellales.

TBC

- Ruptura conducto torácico. - Lechoso, colesterol normal o bajo. - Quilomlcrones palognomónico. - Triglicéridos >110 mgídl.

Quilotórax

Pseu doqu ilotórax

Hemotórax

2.

- Causa más frecuente de exudado. - Indicaciones de tubo de drenaje: - Presencia de pus. - Microorganismos en Cram o cultivo, - Glucosa 200 ml/h durante 4-5 h consecutivas. - Sangrado en 24 horas >1500.

NEUM OTORAX.

El tipo más preguntado es el espontáneo primario. Conocer los tipos de neumotorax y sus tratamientos.

Algoritmo terapéutiío del neumotorax.

respiratorio

Observación Reposo

4 No resolución en 5 días

respiratorio >20-30%

i Tubo de tórax con aspiración

4 Reparar pared torácica

I

No resolución (fuga aérea más de 7 días)

4 Tubo de tórax con aspiración

neumotórax espontáneo

(herida soplante)

Tratar como neu rrrotórax espontáneo

- Cirugía

- Pleurodesis en pacientes inoperables

204

www.FreeLibros.me

Tratar como neumotórax espontáneo

Si recidiva: anovulatorios y/o pleurodesis

Meu m

3. -

o l q g ia y c ir u g ía t o r á c ic a

1. ¿Cuál es la causa más frecuente del neumoto­ rax espontáneo secundario?

1.

EPOC.

2. ¿Y cuál suele ser la causa del primario?

2.

Ruptura de bullas subpleurales apicales.

3. ¿De qué neumotorax es típico el "bamboleo" mediastínico?

3.

Neumotorax traumático abierto.

4. ¿Cuál suele recidivar?

4.

El espontáneo prim ario y e l catamenial.

NEOPLASIAS PLEURALES.

La más frecuente: metástasis. M esoteliom a maligno: asociado al asbesto. No hay tratamiento eficaz. Recuerda: las calcificaciones lineales en pleura con parénquima intacto indican sólo exposición al asbesto. M esoteliom a benigno: sin relación con la exposición al asbesto. Suele ser asintomático. Se asocia a hipoglucemia y osteoartropatía hipertrófica.

CAPÍTULO 19.

N EOPLASIAS PULM ONARES.

En el pulmón son mas frecuentes los tumores primarios que los metastásicos. Mas del 90% son tumores malignos. Representa la prim era causa de muerte por cáncer. Al dejar de fumar dism inuye el riesgo, que ya a los 15 años se aproxim a al de los no fumadores, aunque nunca se ¡guala al de un no fumador. Tanto la clínica como la radiología de tórax dependen de la localización del tumor (central, periférico).

C a v it a c ió n > 2 0 % .

C a . e p id e rm o id e

M a s a c e n tra l ( a t e le c t a s ia ) .

A n a p lá s ic o d e cé lu la s p e q u e ñ a s

E l p e o r p r o n ó s t ic o .

- El m á s fre c u e n te .

A d e n o c a rc in o m a

- P u e d e a se n ta r s o b r e c ic a t r ic e s . - N o r e la c ió n t a b a c o .

A n a p lá s ic o de c é lu la s g ra n d e s

N o d u lo o m a s a p e r if é r ic a .

C a v it a c ió n 2 0 % .

1. ¿En qué consiste el síndrome de Pancoast?

1.

Destrucción radiológica de 1a y 2a costillas y afectación de nervios C8, T I y T2 con dolor (a veces también e xiste un síndrome de BernardHorner por afectación del sim pático cervical).

2. ¿Qué tumor lo produce con más frecuencia?

2.

Carcinoma epiderm oide que asienta en e l vér­ tice pulmonar.

3. ¿Dónde metastatiza con más frecuencia el car­ cinoma pulmonar?

3.

Hígado, suprarrenales. Adenocarcinoma: ten ­ dencia también a hueso y cerebro.

4. ¿Qué tipo de tumor produce síndromes paraneoplásicos con más frecuencia?

4.

El carcinoma de células pequeñas (oat-cell).

5. ¿Cuál es la causa más frecuente de síndrome de la cava superior?

5. El carcinoma de células pequeñas.

205

www.FreeLibros.me

ULTRA-RESÚMEIMES. Manual CTO de Medicina y Cirugía, 8.a edición Una vez diagnosticado, es fundamental determ inar la localización y el grado de extensión del tumor: - Carcinom a M icrocítico: • Enfermedad Localizada: confinado al tórax, ganglios regionales (incluye m ediastínicoshiliaresy supraclaviculares contralaterales), afectación del nervio recurrente y síndrome de vena cava superior. Indica que la enfermedad puede ser abarcada por radioterapia. • Enfermedad Avanzada: no RT. - Carcinom a N O de células pequeñas: se usa el estadio TN M .

1=^ REPASA:

[C

l a s if i c a c ió n

TN M

pa raelC

N C P , T a bla

26,

C a p ít u l o

19

del

M

anual

CTO 8a E d . ]

Tratamiento. Fíjate en los criterios de irresecabilidad e inoperabilidad. El principal criterio de inoperabilidad se basa en el FEV1 del paciente: < 1 1es inoperable. Entre 1 y 2 I se debe calcular el FEV1 postoperatorio.

1=^REPASA:

[C

r it e r io s d e in o p e r a b il id a d ,

T a bla

28,

C a p ít u l o

19 d e l

M

anual

CTO 8a E d .]

Para el tratamiento, debemos distinguir entre carcinom a de células pequeñas y de células no pequeñas.

Tratamiento del tartinoma no mitrotítito. TI

T2

T3

T4

Tratamiento del tartinoma mitrotítito. S e p u e d e a ñ a d ir c iru g ía en la (T i No Mo)

í L im it a d a al t ó r a x



Q T y R T t o r á c ic a ^

Extendida

► QT

>

S i r e s p u e s t a : R T h o lo c r a n e a l p r o filá c tic a

1. ¿Cuál es el tumor benigno más frecuente del pulmón?

1.

El adenoma, y dentro de éste el carcinoide (aun­ que en la actualidad el carcinoide no se consi­ dera benigno, sino de baja malignidad, ya que hay algunos carcinoides metastásicos).

2. ¿Cuál suele ser la presentación radiológica de los tumores benignos?

2.

Los adenomas suelen presentarse como masas centrales y el hamartoma como masa periférica.

3. ¿Dan clínica estos tumores?

3.

El adenoma puede producir tos, hemoptisis, (el carcinoide metastásico produce síndrome car­ cinoide); el hamartoma, suele ser asintomático.

4. ¿Qué ventajas aporta realizar una PAAF ante un nodulo pulmonar solitario cuando la fibrobroncoscopia no es diagnóstica?

4.

Si la PAAF demuestra malignidad, el paciente se som ete a mediastinoscopia para estadificación, y si esta es positiva (N2, estadio IIIB), se "ahorra'' la toracotom ía, pues el tratam iento de este estadio ya es sistém ico (RT+QJ). No obs­ tante, dados sus frecuen tes falsos negativos (ya que la muestra que obtiene es de citología), algunos autores aún proponen hacer toracoto­ mía de entrada a todos los pacientes.

206

www.FreeLibros.me

IMe u m

o l o g ía y c ir u g ía t o r á c ic a

NODULO PULMONAR SOLITARIO. Densidad radiológica de 1 a 6 centímetros rodeada de parénquima pulmonar sano.

Algoritmo diagnóstko del nodulo pulmonar solitario. • Paciente < 3 5 años, no fum ador.

Módulo ton criterios de benignidad

• C alcificació n en "palom ita de m aíz" o nido central denso. • Sin crecim ien to en 2 años.

Nodulo de signifiiado dudoso

1 Paciente < 3 5 años, fum ador 1 En crecim ien to o tam año grande. Sintom ático.

• D u plicación 1 6 meses.

Confra/ Rx: ■P rim e r añ o : cad a 3 m eses. ■Po ste rio rm e n te : anual. D iagnóstico etiológ ico : Variación

Estable

Fibrobroncoscopia

Si el nodulo es periférico radiológicam ente tam bién se

D iagnóstico

No D iagn óstico

puede intentar broncoscopia con biopsia transbronquial.

PAAF

En N. periféricos

-Broncoscopia-

En N. centrales

I No diagnóstico

* ■ Toracotomía

CAPÍTULO 4.

ENFERM EDAD PULMONAR O BSTRU CTIVA CRÓN ICA (EPOC).

En este tema debes centrarte en: - Diferenciar la EPOC con predominio de enfisema de la bronquitis crónica. - Tratamiento: indicaciones de oxigenoterapia y principales características de los fármacos. La siguiente tabla te indica las principales características clínicas: ENFISEMA

BRONQUITIS CRÓNICA

Disnea

G rave.

Leve,

Tos y expectoración

Escasa.

A bu nd an te,

Infecciones bronquiales

Pocas, pero graves.

Frecuentes.

Aspecto físico

A stén ico, sonrosado, taq uipn eico.

Sob repeso , cianótico.

Auscultación

D ism in ución del m urm ullo vesicular.

Roncus gruesos y sibilancias.

Rx. tórax

H iperinsuflación .

A um en to tram a broncovascular, corazón agrandado.

Cases

H ip o xem ia leve.

H ip o xem ia severa. H ipercapnia.

Hipertensión pulmonar

R eposo: no. Ejercicio : m oderada.

Reposo: intensa. Ejercicio : em peora.

Cor pulmonale

R aro salvo en fase term inal.

Frecuente.

Capacidad de difusión

D ism in uida,

N orm al.

Hematocrito

Norm al.

Elevado.

207

www.FreeLibros.me

ULTRA-RESÚM EN ES

y

8.a

Recuerda estas asociaciones: - Enfisema centroacinar —>tabaco —> lóbulos superiores. - Enfisema panacinar —> déficit de alfa-1 -antitripsina —> bases. Con respecto al tratamiento, debes conocer los fármacos disponibles y algunas características importantes de ellos. Repasa las características de los fármacos. Aquí te indicamos las principales:

1. BRONCODILATADORES: 1 .1 .

B e t a a d r e n é r g ic o s :

1.2. -

- RESORCINOLES (terbutalina) y SALIGENINAS (salbutamol). > Selectivos beta-2 (exceptoorciprenalina). > Vía inhalatoria (de elección, dado el comienzo de acción mas rápido y ser la vía con menores efectos secundarios) o parenteral. > Efectossecundarios:temblor, taquicardiay nerviosismo. > T1/2 corta: salbutamol, terbutalina y fenoterol. > T1/2 larga: salmeterol y formoterol. M etilx a n tin a s (teofilina): Vía oral: en pacientes ambulantes, usada como preparados de liberación retardada. Vía intravenosa: aminofilina. De uso ocasionalmente en pacientes ingresados por reagudización. Niveles plasmáticos: • 10-20 mcg/ml: niveles terapéuticos. • >20 mcg/ml: efectos secundarios leves. • >30 mcg/ml: efectos secundarios graves —» convulsiones, arritmias e hipotensión.

REPASA:

^

1 .3 .

-

[F a c to r es

q u e in f l u y e n en el m e t a b o l i s m o d e la t e o f i l i n a ,

T a bla

6,

C

a p ít u l o

4

del

M

anual

CTO 8a E d

]

A n t ic o l in é r g ic o s ( b r o m u r o d e ip r a t r o p io , b r o m u r o d e tio t r o p io )

Vía inhalatoria. Efecto en 30 min. Efectos secundarios: xerostomía, breves paroxismos de tos, glaucoma...

2. CORTICOIDES: -

En agudizaciones que no responden a tratamiento broncodilatador, se pueden utilizar porvíasistémica en pauta corta. Vía inhalatoria cuando la prueba esteroidea oral fue positiva.

3. MUCOLÍTICOS (N-acetilcisteína...), EXPECTORANTES: -

Reducen la viscosidad o aumentan el aclaramiento del moco.

4. ANTIBIÓTICOS: -

La causa más común de descompensación en los pacientes con EPOC son las infecciones respiratorias. Iniciar tratamiento antibiótico si hay 2 ó más de los siguientes datos: > Aumento del volumen del esputo o que éste se vuelva purulento. > Incremento de la disnea o de la tos. > Fiebre o aparición de ruidos torácicos intensos.

5. Indicaciones de la OXIGENOTERAPIA CRÓNICA: -

208

Pacientes con tratamiento broncodilatador completo (3 líneas), en situación estable, con abandono del hábito tabáquicoy: > PaO, respirando aire ambiente es menor o igual a 55 mmHg. > PaO, entre 55-60 mmHg y existe alguna de las siguientes situaciones: - Hipertensión pulmonar. - Cor pulmonale. - Insuficiencia cardiaca congestiva. - Trastornos del ritmo cardiaco. - Policitem¡a(hematocr¡to>55%).

www.FreeLibros.me

INIe u m

■■■nna

o l o g ía y c ir u g ía t o r á c ic a

Algoritmo terapéutito de la EPOC Clasificación EPOC EPOC LEVE FEV, >70%

EPOC MODERADA 50 - 70% SEVERA < 50%

. V alo rar d isnea

T IN TER M ITEN TE

CONTINUA

t

y

B ET A A G O N IST A S

A N T IC O L IN É R G IC O S

Y

y EPO C

EPO C

M O D ER A D A

S EV ER A

T

Y

DISN EA

D IS N EA

IN TER M ITEN TE

C O N T IN U A

AN A NTII T IC O L IN E R G IC O S

de de Ibase a todos, y generalm ente A N T IC O L IN E R G IC O S

BETAAGONISTAS

a d em anda

BET A A G O N IST A S

a d em anda

a dem anda

T

Y

No m ejoría

Y

No mejoría

N o m ejoría

T



Asociarlos

A sociarlos y

Asociar

a veces

T E O F IL IN A B ET A A G O N IST A

T E O F IL IN A

de larga duración.

Y



No mejoría

N o m ejoría

PRU EB A ES T ER O ID E A (con esteroides orales) Si la respuesta es positiva

C O R T IC O ID E S IN H A L A D O S

NOTA: Esta clasificación de la gravedad es de la Sociedad Europea. H ay otras sociedades, como la am ericana y la española, donde el punto de corte para la gravedad varía un poco.

1.

¿Cuál es el fármaco que NUNCA puede faltar en el tratamiento de un EPOC severo?

1.

Un anticolinérgico. Los EPOC moderados tam­ bién suelen requerirlo, pero algunos se contro­ lan sólo con betaagonistas.

2.

¿Cuáles son los agentes causantes de infeccio­ nes respiratorias más frecuentes en el EPOC?

2.

Agentes más frecuentes son los rinovirus; dentro de las bacterias, H. influenzae y S. pneumoniae.

3.

¿Qué antibiótico darías?

3.

Amoxicilina + ácido clavulánico.

4.

¿Qué medidas aumentan la supervivencia en el EPOC?

4.

Abandono del tabaco y cumplimiento de la oxigenoterapia cuando esté indicada (>16h).

5.

¿Cuándo está indicado oxígeno domiciliario?

5.

Sólo si el paciente no fum a, y tras recibir todo el tratam iento broncodilatador, cumple los cri­ terios ya indicados.

CAPÍTULO 15.

TROM BOEM BOLISM O PULMONAR._______________________

Debes conocer bien su manejo diagnóstico y terapéutico, ya que es un tema "en auge" en el M IR.

REPASA:

[ A l g o r it m o

d ia g n ó s t ic o d e l

TEP,

F ig u r a

52,

C

a p ít u l o

15 d e l

M

anual

CTO 8a E d .]

209

www.FreeLibros.me

ULTRA-RESÚM EN ES

y

Repasa este tem a por medio de las siguientes autocuestiones:

1. ¿Cuál es la causa más frecuente de TEP?

1.

Suele ser consecuencia de la trombosis venosa profunda de MMII.

2. ¿Cuál es el hallazgo radiológico más frecuente del TEP?

2.

Lo más frecuente es que la radiografió tensa alteraciones inespecíficas (80%).

3. ¿Qué otros hallazgos puede haber?

3.

- Elevación de un hemidiafrasma. - Joroba de Hampton. ■Atelectasias laminares. - "Corte" brusco de un vaso. - Vaso en forma de "cola de rata" (en la arteriosrafia). - Hiperclaridad (signo de Westermak).

4. ¿Cuál es el test de screening más útil?

4.

La gammagrafía de perfusión + de ventilación pulmonar o el TAC con contraste (en muchos casos está sustituyendo a la gammagrafía). La arteriografía pulmonar.

5. ¿En qué momento se debe iniciar el tratamiento?

5.

Cuando se tiene la sospecha por la clínica, Rx, gasometría; sin esp e ra r la confirm ación.

6. La presencia de úlcera péptica activa, ¿contra­ indica el tratamiento?

6.

Es una contraindicación relativa, puedes repa­ sar las contraindicaciones en la Tabla 20, Capí­ tulo 15 del Manual CTO 8 a Ed.

¿Y la prueba considerada "gold standard"?

CA PÍTU LO 5.

ASMA.

En este tem a, céntrate so b reto do en el tratamiento y también aprende a distinguir una crisis leve de una grave. Recuerda que existen dos formas de asm a: ASMA EXTRÍN SECA

ASMA IN TRÍN SEC A

AF y/o AP alérgicos

Frecuentes

Raros

Edad

Niños y jóvenes

Adultos

Aumentada

Normal

Reacción cutánea a Ag 'gE

210

Eosinofilia en sangre y esputo

+■

+ ++

Reacción de hipersensibilidad tipo 1

+

-

Asociación a intolerancia a la aspirina

-

Sí, ± 1 0%

1. ¿Qué fármacos se asocian con frecuencia a cri­ sis de asma?

1.

Aspirina, betabloqueantes, compuestos de azu­ fre y tartracina.

2. ¿Cuál es la tríada clásica del asma sensible a as­ pirina?

2.

Rinitis vasomotora, poliposis nasal y asma. Re­ cuerda que hay reactividad cruzada con algu­ nos AINEs.

3. ¿Cuál es el hallazgo necrópsico más caracterís­ tico en una persona fallecida por una crisis de asma?

3.

Pulmones hiperdistendidos y ausencia de co ­ lapso al abrir la cavidad pleural.

4. ¿Cómo se refleja la obstrucción en las pruebas funcionales?

4.

El VR puede ser del 400% durante la crisis, la CVF 1 /sem ana y < 1 /día. - C risis n o c tu rn a s > 2 /m e s. - B a sa l: P E F > 8 0 % del te ó rico .

- SI sínto m as n o ctu rn o s: betaadrenérglcos de acción larga. - C rom og llcato o n edocrom ll altern a tiva a co rtico id e s en n iñ o s y jó ve n e s (au n q u e m enos eficaz).

A sm a p ersisten te m od erad o - S ín to m as a d ia rio .

- C o rtico id e s ¡n h . d osis m ás altas.

- C risis n o ctu rn as > 1 /se m an a .

- B ro n c o d lla ta d o re s acció n so sten id a. - B etaad ren érg lco d e acción co rta a d e m a n d a .

- P E F b a sal: 6 0 -8 0 % del te ó rico . A sm a persisten te severo

- C o rtico id e s ¡n h. d o sis altas.

- Sín to m as co n tin u o s. - C risis n o ctu rn as fre cu e n te s. - P EF basal < 6 0 % del te ó rico . - A ctiv id a d física lim ita d a .

- C o rtico id e s o ra les (a veces). - B ro n c o d lla ta d o re s acció n so ste n id a. - B e taad ren érg lco a d e m a n d a .

Para que lo recuerdes mejor, y simplificando mucho, podemos decir: - Asma persistente leve: sólo síntomas en crisis ocasionales. - Asma persistente moderado: crisis a diario, empiezan a alterar la actividad y el sueño; PEF 80-60%. - Asma persistente grave: limita la actividad cotidiana por los continuos síntomas; PEF No han demostrado utilidad Gammagrafía con galio ?

Tratamiento específico

215

www.FreeLibros.me

ULTRA-RESÚMEIMES

Características de las principales enfermedades intersticiales.

FIBROSIS PULMONAR IDIOPÁTICA

- Disnea de esfuerzo y tos no productiva. - Rx: patrón reticular o retlculorodular con preferencia en campos Inferiores. - TCAR: diagnóstico en fases precoces. - Patrón ventllatorlo restrictivo. - Tratamiento: cortlcoldes. SI no respuesta, se pueden añadir ¡nmunosupresores (azatloprlna o c¡clofosfam¡da).Trasplante en situación avanzada. LES: Pleuritis ± derrame pleural. Raro enfermedad Intersticial crónica.

Más frecuente es la afectación pleural. 20% enfermedad Intersticial: mas frecuente en varones con niveles elevados de FR ENFERMEDAD INTERSTICIAL Enfermedad apical bilateral flbrobullosa en fases avanzadas. ASOCIADA A ENFERMEDAD La que con mayor frecuencia presenta enfermedad Intersticial. Predomina DEL COLÁGENO en LLII. Puede aparecer HTP Infiltración llnfocítlca Intersticial. DM: mas frecuente cuando hay anticuerpos antl Jo-1. - Mas frecuente en varones jóvenes. - Neumotorax que recidiva en el 25% de los casos. - LBA: >3% de células de Langerhans. - Aumento de volúmenes.

HISTIOCITOSIS X

- Mujeres premenopáuslcas. - Disnea. LINFANGIOLEIOMIOMATOSIS - Neumotorax espontáneo que puede ser bilateral y recurrente. Qullotorax. - Se presenta con aumento de volúmenes pulmonares y patrón obstructivo o mixto. - Tratamiento: Progesterona + Ovarlectomla.

PROTEINOSIS ALVEOLAR

1.

¿En qué enfermedades intersticiales el diagnóstico es posible con el BAL?

CAPÍTU LO 10. 1.

- Acúmulo de material protelnáceo PAS +. - Patrón radiológico que recuerda el edema pulmonar. - Riesgo aumentado de Infecciones por Nocardla, M. avium y R carinii. - Tratamleto: lavado pulmonar completo.

1.

En la linfangitis carcinomatosa, carcinoma de células alveolares, linfoma y neumonía por P. Carinii. También en la histiocitosis X (si > 5% de célu­ las de Langerhans); proteinosis alveolares (PAS+); eosinofilias (>20%).

EO SIN O FILIA S PULM ONARES.

CARACTERÍSTICAS GENERALES:

Todas las enferm edades de este grupo com parten, como rasgos generales: - IN FILTR A D O S PU LM O N A R ES CO N EO SIN Ó FILO S. - P U ED E A PA RECER EO S IN O F ILIA PERIFÉRICA .

2. -

^

216

ASPERGILOSIS BRO N CO PULM O N AR ALÉRGICA (ABPA). Historia de atopla. Agente responsable: Aspergillus fumigatus; no infección, sino reacción inmunológica a la colonización crónica de la vía aérea (hipersensibilidad tipo I, III). Rx con Imagen en dedo de guante.

REPASA:

[C

r it e r io s d ia g n ó s t ic o s d e l a a b p a ,

T a bla

16,

C

a p ít u l o

www.FreeLibros.me

10

del

M

anual

CTO 8a E d .]

M

■■■■Mi

3. -

e u m o l o g ía

y

c ir u g ía

t o r á c ic a

EOSINOFILIAS PULMONARES DE ETIOLOGÍA DESCONOCIDA. Repasa las eosinofilias pulmonares en el siguiente cuadro:

CLÍNICA

RX

TRATAMIENTO

Criterios principales: - Asma. - Infiltrados pulmonares transitorios. - Eosinofilia>1000 ¡j\. - Reacción cutánea a A. fumigatus inmediata. - Precipitinas frente a A. fumigatus. - IgE sérica elevada. - Bronquiectasias centrales. - IgE o IgG específicas elevadas.

- Infiltrados transitorios recurrentes. - Bronquiectasias centrales. - Imagen en dedo de guante.

- Corticoides vía sistémica.

- Más frecuente: nitrofu rantoína. - Tos y disnea. - Fiebre y escalofríos.

- Patrón reticulonodular de predominio en bases.

- Retirar fármaco (corticoides).

- Manifestaciones mínimas.

- Infiltrados pulmonares transitorios.

N.E. Aguda

De forma aguda: - Fiebre y mialgias. - Tos y disnea - Crepitantes - Hipoxemia grave

- Infiltrados alveolointesticiales bilaterales (similar al SDRA) - Derrame pleural (a veces)

- Corticoides: presenta buena respuesta

N.E. crónica

- Fiebre, sudoración nocturna. - Tos, anorexia. - Antecedentes ASMA.

- Infiltrados periféricos ("negativo del edema de pulmón").

- Corticoides: rápida mejoría pero recurrencia al retirarlos.

ABPA

N.E. inducida por fármacos

Sd. de Loeffler

Sd. hipereosinófilo

CA PÍTU LO 22.

-Afectación multiorgánica: corazón, pulmón, hígado, piel y SN.

- Corticoides y/o hid ro xiurea.

SÍNDROM E D E D ISTR ÉS RESPIRA TO RIO AGUDO.

Se sospecha cuando hay una insuficiencia respiratoria aguda severa, un infiltrado difuso alveolointersticial y en la gasometría aparece: -

P aO , < 5 5 mmHg con FIO, > 0 ,5 . P aO , < 5 0 mmHg con FIO, > 0 ,6 .

El siguiente esquem a te ayudará a entender la clínica.

217

www.FreeLibros.me

ULTRA-RESÚM EN ES

y

8 .a

Clínica del síndrome de dificultad respiratoria aguda (SDRA) 1. Aumento de la permeabilidad alveolo-capilar por causa determinada: - Sepis: más frecuente. - Inhalación de sustancias. - Intoxicación por opiáceos. - Transfusión de hemoderivados. - Shock hipovolémico. 2. Insuficiencia respiratoria grave: P C O ,/ FiC O , < 200. Shunt. 3. Radiografía compatible: Patrón alveolointersticial difuso. 4. Excluir EAP cardiogénico: Presión enclavamiento (Fase aguda: infiltrados nodulares. >Fases avanzadas: infiltrado reticulonodular, patrón en panal. - Estudios de función pulmonar: patrón restrictivo, aunque en fases avanzadas asocia con frecuencia un defecto obstructivo leve. Fíjate en estos detalles sobre el diagnóstico: - Neutrofilia y linfopenia. No existe eosinofilia ni está elevada la IgE. Precipitinas séricas elevadas (aunque ésto sólo indica exposición, no enfermedad). - BAL: en la fase aguda ( en LU I.

peso. Insuficiencia respiratoria y muerte en 15a tras el com ienzo de la exposición

O curre incluso cuando la exposición al sílice ha cesado.

Progresión más rápida que la forma crónica. Frecuente encontrar conectivopatías y enferm edades autoinm unes.

Sim ilar a forma crónica.

Asintom ática o tos y disnea de esfuerzo progresiva.

- Nodulos redondeados en LLSS. - Adenopatías calcificadas en "cáscara de huevo".

- Disnea de esfuerzo y tos. - Patrón restrictivo con disminución de la D L C O ; a veces asocia obstrucción. - Pueder haber infecciones bacterianas recurrentes.

Coalescencia de los nodulos formando grandes conglomerados (> 1 0 mm). M ás en LLSS.

2. Exposición al asbesto. Latencia: 10-20 años. Clínica: > El síntoma más precoz y común es la disnea. > También cursa con tos y expectoración.

Enfermedades asociadas a la exposición al asbesto.

Asbestosis

I

Enfermedad intersticial. Rx: opacidades lineales en campos inferiores. Placas pleurales solas indican exposición.

Ca. Pulmón

j

Mesotelioma

- Epidermoide o adenocarcinom a. - M ínim o 15-19 años

- M áx. riesgo 30-35 años de exposición. - Rx: derrame pleural.

de exposición. - Efecto sinérgico con el tabaco.

- M uerte por extensión local.

1. ¿Qué alteración precoz de la asbestosis indica enfermedad grave?

1.

La disminución de la capacidad de difusión.

La silicosis predispone a padecer infecciones por...

2.

M. tuberculosis y micobacterias atipicas.

3. Otra neumoconiosis es la producida por antra­ cita, ¿qué hallazgos radiológicos encontramos?

3.

En la forma simple hay opacidades redondas de 1-5 mm de diámetro. La forma complicada presenta nodulos mayores de 1 cm, de predo-

2.

www.FreeLibros.me

IMe u m

o l o g ía y c ir u g ía t o r á c ic a

4. ¿Cuál es el tumor de pulmón que aparece con más frecuencia en la asbestosis?

4.

5. ¿Y el más característico?

5. Mesotelioma pleural maligno.

6. ¿Conoces alguna otra alteración pulmonar debi­ da a la exposición al asbesto, además de las ci­ tadas?

6.

CA PÍTU LO 14.

El Ca. epidermoide.

Derrame pleural benigno, placas pleurales y asbestosis.

HIPERTEN SIÓN PULMONAR.

Elevación de la presión m edia de la arteria pulmonar por encim a de 25 mmHg en reposo o 30 mmHg en ejercicio. Puede ser un proceso primarlo (H A P primarla) o aparecer en el seno de enferm edades respiratorias crónicas que cursan con hlpoxem la o enferm edades cardíacas. Piensa en este cuadro en pacientes, sobre todo m ujeres en la 3- y 4- décadas, con disnea de esfuerzo progresiva y signos físicos, radiológicos, EC G junto con ecocardiogram a sugestivos de hipertrofia ventricular derecha.

1. ¿Cuáles son las principales causas de HTP se­ cundaria?

1.

EPOC y tromboembolismo pulmonar.

2. ¿Cuál es el tratamiento de la HTP primaria?

2.

Antagonistas del calcio en dosis altas mejoran los síntomas y la supervivencia en algunos pa­ cientes con test de vasorreactividad agudo po­ sitivo. Si no responden o si el test es - : clase fu n cion al /-//: A n tico a gu la ntes. Clases III: A n tic o a g u la n te s , B o se n ta n , S ild e n a filo , t e p r o s t in il. C la se IV : A n tico a g u la n te s + prostaciclina.

3. ¿Está elevada la presión de enclavamiento?

3.

Sólo en

4

4.

Suelen aparecer defectos parcheados no segmen­ tarios y no sugerentes de TEP.

¿Qué datos aparecen en la gammagrafia de per­ fusión de la HTP primaria?

5. ¿Qué otras patologías producen un patrón simi­ lar a la de la HTP primaria?

CA PÍTU LO 6.

estadios avanzados.

5. La hipertensión portal, la infección por HIV, la in h a la ció n de c o ca ín a , los fá rm a co s anorexigenos y el aceite de colza desnaturali-

BRO N Q U IECTA SIA S.

Cuando repases este tem a, debes recordar de forma especial la flbrosls quístlca, ya que es una de sus etiologías y es :xeguntada con relativa frecuencia.

1. ¿Qué virus se asocian con la producción de bronquiectasias?

1.

Adenovirus y virus influenza.

2. ¿En qué cuadro pulmonar aparecen bronquiectasias por mecanismo inmune?

2.

Aspergitosis broncopulmonar alérgica.

3. Cita cuatro síndromes congénitos que incluyan bronquiectasias.

3.

S. de Kartagener (sinusitis, bronquiectasias, situs inversus, esterilidad). S. de William-Campbell (defectos del cartílago). S. de Mounier-Kiihn (traqueobroncomegalia). S. de Young (azoospermia obstructiva, sinusitis y bronquiectasias).

221

www.FreeLibros.me

ULTRA-RESÚM EN ES

y

8 .a

:« ■ ■ ■ ■ ■ ■ ■ ■ ■ ■ ■ ■ •:

4. ¿Cuáles son las causas más frecuentes de hemoptisis?

4.

5. ¿Qué signo físico aparece con frecuencia en las bronquiectasias y no en el EPOC?

5. Las acropaquias.

Las bronquiectasias y la bronquitis crónica.

Actitud diagnóstica y tratamiento de las bronquiectasias. l -Tos persistente, esputo purulento

Sospecha clínica^ Rx

-Hem optisis -Crepitantes gruesos, roncus, sibilancias

rR funóonalesj

A unq ue generalm ente , , „ , . . „ ^ ° , | -Imagen de rail de tranvía se ve mal en - . , , . , . -Imágenes quisticas radiografía simple

n i- • • -P obstructivo al inicio n . .... -Patrón restrictivo

TC alta resolución

A

Infecciones bacterianas T A TB em píricos

Bronquiectasias localizadas y clínica severa con fracaso de tto.

Hemoptisis masiva

m é d ic o d u ra n te

-Em bolización

1 año

arterial bronquial

p.e.: neumonías graves repetidas con compromiso vital

I

pers¡ste hemopti:

1

\ Cirugía

Cirugía

Fíjate que en el esquema no aparece la broncografía; actualmente rara vez es necesaria para el diagnóstico, aunque para algunos es Importante a la hora de planificar la cirugía, si bien la TC de alta resolución la ha llevado a un segundo plano.

CAPITULO 1.

RECU ER D O ANATOMICO.

Este tema no es muy preguntado, son más Importantes los aspectos fisiológicos. Hay tres partes; debes centrarte en lo siguiente: División de la vía aérea: lo más Importante es la distinción zona de conducción-zona respiratoria, y los conceptos de aclno y lobul¡lio. Histología: recuerda las principales diferencias en la estructura entre los distintos tramos de la vía aérea (tipos de células y localización, presencia de cartílago, epitelio pseudoestratlflcado/cuboldeo, etc). - Circulación: fíjate en la doble circulación pulmonar.

CIR CU LA CIÓ N

Bronquial

Pulmonar

ORIGEN

. A orta . A . interco stales

V .D .

RETORN O

TIPO VASO

SANGRE

A rterias

O xig e n ad a

V enas

D eso xig en ad a

Arterias

D e so xig en ad a

V enas

O xig e n ad a

Vv. Pulm on ares

A .l.

222

www.FreeLibros.me

IMEUMOLOGÍA y CIRUGÍA

T O R A C IC A

De todo esto te resaltamos algunos detalles con las siguientes autocuestiones:

1. ¿Dónde termina la zona de conducción?

1.

En los bronquiolos terminales.

2. ¿Qué es el acino?

2.

La unidad anatómica situada distal al bronquiolo terminal.

3. ¿Dónde son más abundantes las células calici­ formes?

3.

En tráquea y bronquios, disminuyen hacia la periferia.

4. ¿Dónde se localizan las células de Clara?

4.

En los bronquiolos fundamentalmente (algunas en los grandes bronquios).

5. ¿Dónde se originan las arterias bronquiales?

5.

En la aorta y arterias intercostales.

6. ¿Qué vasos son los que más contribuyen a las resistencias?

6.

Las arteriolas.

7. El cartílago recubre las vías aéreas hasta...

7.

A partir de los bronquios lobares deja de for­ mar un anillo continuo y ya no hay en los bron­ quiolos.

8. ¿A partir de qué zona el epitelio respiratorio se hace cuboideo?

8.

Bronquiolos respiratorios.

9. ¿De qué tipo es el epitelio alveolar?

9.

Escamoso.

CA PITU LO

2.

M ALFORM ACIONES.

De este tema sólo te destacamos el secuestro bioncopulmonar. Recuerda que consiste en tejido pulmonar con aporte sanguíneo sistémico, sin comunicación con la vía aérea; puede tener pleura propia (extralobar, el 90% en relación con hemidiafragma izquierdo) o no (intralobar). Recibe aporte sanguíneo de la circulación sistémica, aorta torácica o abdominal, lo más frecuente (a veces de una arteria intercostal). Es asintomático, salvo si se infecta. Requiere extirpación.

CAPITULO 24. REPASA:

TRASPLANTE DE PULMON.

[ I n d ic a c i o n e s

d e tra spla n te d e p u l m ó n ,

A

partado

24.1,

C a p ít u l o

24 d e l

M

anual

CTO 8a E d .]

Aquí te comentamos las complicaciones; fíjate que las muertes ocurridas en el primer mes tras el tratamiento están relacionadas con la cirugía; pasado éste, se hallan en relación con el rechazo y la infección (principal causa de mortalidad en la actualidad).

1. El rechazo suele manifestarse de forma similar a un proceso infeccioso, ¿cómo debe hacerse diagnóstico?

1.

Biopsia transbronquial, es más sensible y espe­ cifica para el rechazo agudo.

2. ¿Qué gérmenes predominan en la bronconeumonía tras el trasplante?

2.

Gramnegativos y estafilococos.

3. ¿Cuál es la infección oportunista más frecuente?

3.

Neumonía por citomegalovirus.

La cirugía de reducción de volumen pulmonar es una técnica paliativa para los pacientes EPOC tipo enfisema con afectación heterogénea del parénquima pulmonar. Esta técnica pretende la descompresión del parénquima vecino y que el tórax vuelva a una posición más fisiológica para los músculos respiratorios. Los mejores resultados se obtienen en casos de bulla única localizada. El peor pronóstico se asocia a hipercapnia y vejez. La complicación más frecuente es la fuga aérea.

223

www.FreeLibros.me

N CA PÍTU LO 1.

e u r o l o g ía y n e u r o c ir u g ía

INTRODUCCIÓN : ANATOMÍA, D EL SISTEM A N ERVIOSO.

SEM IO LO GÍA

Y

FISIO LO G ÍA

La Neurología es una asignatura difícil de estudiar, pero facilita m ucho las cosas dom inar la semiología. Si lo consi­ gues, no tendrás que m em orizar largas listas de síntomas en cada enfermedad o síndrom e, pues m uchos de ellos se pueden deducir. Por otro lado, ten en cuenta que es el tem a más preguntado, de m anera explícita o im plícita, en forma de caso clínico. No debes obsesionarte con la neuroanatom ía. Para el M IR sólo tienes que conocer los datos que te vamos a destacar en este capítulo, en e í que encontrarás las bases para razonar la clínica de muchos trastornos neurológicos.

1.2. y 1.9. ALTERACIONES D E LAS FUNCIONES SUPERIORES Y SÍNDROM ES LOBARES. Es necesario que sepas localizar una lesión hemisférica a partir de su sem iología, pues los síndrom es focales suelen caer im plícitos en casos clínicos. Si los aprendes bien, tendrás estudiada la clínica de muchos trastornos, sobre todo enfermedades cerebrovasculares y tumores. Repásalos en el siguiente esquem a, teniendo en cuenta b siguiente: - Fíjate en que cada lóbulo cerebral se encarga de controlar una fun dón distinta: • Frontal: actividad motora y conductas sociales. • Parietal: sensibilidad táctil. • Tem poral: m em oria, aprendizaje y audición. • O ccip ital: visión. - Habitualm ente sólo se afecta el lenguaje en lesiones del hemisferio dominante (izquierdo en pacientes diestros). El derecho es un hemisferio "más práctico" (apraxia de la construcción y del vestido, recuerda m elodías), mientras que el izquierdo es "más intelectual" (habla, lee, escribe, suma, interpreta partituras). El trastorno del lenguaje causado por daño cerebral, con integridad de las estructuras neurom usculares forma doras del mismo, se denom inan afasia.

REPASA:

[D

ia g n ó s t ic o d i f e r e n c ia l d e l a s a f a s ia s ,

T a bla

1,

C a p ít u l o

1 del

M

anual

CTO 8a Ed .]

- Las representaciones visual y auditiva son bilaterales. Por eso, mientras que puede haber parálisis e hipoestesias en lesiones de un hemisferio, para que haya sordera o ceguera completa de origen cortical son necesarias lesiones de ambos. LOBULO PARIETAL

LOBULO FRONTAL Centre ée la mirada REFLEJO CORNEAL A B O LID O

-rv rv ru

t Bulbo raquídeo

RECUERDA:

Puntiformes reactivas

REFLEJO NAUSEOSO A B O LID O

ATAXICA DE 8IO T (agónica)

LA PRESENCIA DE REFLEJOS O C U LO C EFÁ LIC O S (M OVIM IENTO CONJUGADO DE LO S OJO S EN D IRECCIÓN OPUESTA A LA ROTACIÓN DE LA CABEZA) INDICA INTEGRIDAD DEL TRONCO DEL ENCÉFALO.

CA PÍTU LO 4.

EN FERM ED A D ES V A SC U LA R ES C E R E B R A L E S .

Para su estudio es básico que hayas com prendido bien la semiología en el capítulo anterior. C LIN IC A : Puedes razonarla, en el caso de las lesiones más frecuentes, apoyándote en los dibujos del capítulo de semiología de este Ultrarresum en, y teniendo en cuenta lo siguiente:

IZ^ REPASA: 1.

[t e r r it o r io s

va sc u la r es c erebra les,

F ig u r a

21,

C a p ít u l o

4

del

M

anual

CTO 8a E d .]

Lesiones Vasculares Isquémicas:

Síndromes Vasculares: 1.

La clínica típica de los AIT de CARÓTIDA INTERNA es la amaurosis fugax, por oclusión de la arteria oftálm ica. Su oclusión suele compensarse por circulación colateral, aunque puede producir embolism os y clínica de A C M . Es típico de la disección de la a. carótida interna la asociación de amaurosis fugax, dolor cervical y síndrom e de Horner.

230 ;

www.FreeLibros.me

N

2.

e u r o l o g ía

y

n e u r o c ir u g ía

U tiliza el esquem a de los síndromes focales del prim er apartado de este Ultrarresum en para repasar la clínica de los

ACV de ARTERIAS CEREBRALES SUPERFICIALES. En ese esquema encontrarás 1os territorios de distribución de las tres arterias principales. Por ser el más frecuente de los síndromes cerebrovasculares, te recordamos la clínica del A C V de a. cerebral m edia:

(Imita del ACV de arteria terebra! media - Hem iparesia y hemihipoestesia contralaterales de predominio faciobraquial. - Hem ianopsia homónima contralateral. - Parálisis de la mirada conjugada al lado opuesto (los ojos se desvían al lado efe la lesión). - Afasia en lesiones del hemisferio dominante (Broca, W ernicke o global).

3.

Algo más difícil es acordarse de las RAMAS PROFUNDAS. Lo más relevante es la irrigación del tálamo, a cargo en su mayor parte de la a. cerebral posterior (2/3 posteriores) y en menor medida de la a. comunicante posterior (1/3 anterior).

1. ¿Recuerdas qué hay en el fondo de ojo de los pacientes con ACV de carótida interna?

1. Émbolos de colesterol (si el trombo llega a ar­ teria oftálmica).

2. La hemiparesia y hemihipoestesia de predomi­ nio faciobraquial son típicas de...

2. Lesión en el territorio de la arteria cerebral media.

3. La hemiparesia y hemihipoestesia de predomi­ nio crural son típicos de...

3. Lesión en el territorio de la arteria cerebral anterior.

4. La cuadrantanopsia homónima inferior es típica de...

4. Lesión de radiaciones ópticas pariétáles (a. Ce­ rebral média).

5. La cuadrantanopsia homónima superior es típi­ ca de...

5. Lesión de radiaciones ópticas temporales (á.

6. La hemianopsia homónima contralateral sin res­ peto macular es típica de...

6. Lesión de cintillá óptica o radiaciones ópticas parietal y temporal (a. cerebral media).

7. La hemianopsia homónima contralateral con res­ peto de la visión macular es típica de...

7. Lesión en el territorio de la arteria cerebral posterior.

8. En los ACV de cerebral posterior con afecta­ ción de la circulación proximal también puede aparecer...

8. Síndrome talámico.

9. La aparición de afasia en un ACV indica...

9. Lesión en el territorio de la cerebral media

cerebral medía).

izquierda.

10. La presencia de alexia con agrafía indica...

10.

Lesión del territorio de la arteria cerebral media.

11. La presencia de alexia sin agrafía indica...

11.

Lesión en el territorio de la arteria cerebral posterior

4. Los síndromes más importantes del SISTEMA VERTEBROBASILAR los tienes en el capítulo de síndromes troncoencefálicos de este Ultrarresum en. Recuerda que los procesos isquémicos a este nivel dan lugar a síndromes cruzados (afectan a vías largas contralaterales y a pares craneales ipsilaterales). Ahora estás en disposición de contestar algunas autocuestiones, que recogen los datos más preguntados en el M IR : 5. ¿Recuerdas los SÍNDROMES LACUNARES más frecuentes y la localización típica d e la lesión?

231

www.FreeLibros.me

ULTRA-RESÚMENES. Manual CTO de Medicina y Cirugía, 8.a edición --fM

Ataxia-hemiparesia Brazo anterior de la cápsula interna

Disartria-mano torpe

Brazo posterior de la cápsula interna

Idus motor puro

Núcleo ventral posterolateral del tálamo (VPL)

Idus sensitivo puro

Base de la protuberancia

Recuerda que la TROMBOSIS DE SENOS VENOSOS suele debutar con un síndrome de hipertensión intracraneal (la cefalea es el síntoma más frecuente), y después aparece focalidad neurológica.

REPASA:

[C a u sa s

d e t r o m b o s is v e n o sa c e r eb r a l ,

T a bla

14,

C a p ít u l o

Sospecha clínica ACV

4

del

M

anual

CTO 8a E d .]

AIT (24 horas)

TA ECG Analítica Rx tórax

/Normal pero dioica evidente?' V ^ r e p e t ir e n 72 horas

TAC iranea!: elección en fase aguda. Haremos RM si clínica de fosa posterior o sd. lacunar o sospecha de trombosis venosa

Descarta ACV *

o to ló g ic o )

ISQUÉMICO (80%)

HtMOm&CO(20%) Hiperdensidad

24 horas Borramiento estructuras Hipodensidad focal Efecto masa

m on eo

ikmbótko

INMUTO LACUNAR

TROMBOSIS VtHOSA

OTROS Arterltls: temporal, Takayasu,...

Procesos sépticos Embarazo y puerperio Deshldrataclón (ancianos)

O

U

Aterosclerosis

Fibrilación auricular

HTA

bifurcación carotídea

Trombo mural (IAM)

(Llpohlallnosls)

(HTA, tabaco, colesterol)

Valvulopatía (EM) Suelen Ir a cerebral

E ^ o —

Anticonceptivos orales Traumatismos craneales Procesos hematológlcos RM

Angiograíía

media 180%)

c-'Sc £2 0c « a>

ECO doppler carotídeo Anglografía

1 ’o yJ

ECO transesofáglco ECG

Signo de la delta vacía

Antiogregaaon ron/US

en TAC con contraste

Aniitoagulatión

Itlclopldina o clopidogrel de 2- elección)

¡diferida si

Estenosis carotídea

Infarto extenso)

Sintomática

Aslntomátlca

>70% Tromboendarterectomía 30-70%

N?

AAS

4 0 0 . T o m a an tico a g u la ció n o ra l. S o sp e c h a H S A a u n q u e T A C n o rm a l. ! C o n v u lsio n e s. H a y riesgo d e tra n sfo rm a c ió n h e m o rrá g ic a

2.

Hemorragia Intraparenquimatosa.

ACV HEMORRÁGICO ( 2 0 % ) ■■- ■

.

.

.

—| HEMORRAGIA SUBARACMOIMA \

TAC tra n e a l: h ip e r d e n s id a d

| HEMORRAGIA INTRAPARENQUIMATOSA]

HEMORRAGIA HIPERTENSIVA

HEMORRAGIA LOBAR ESPONTÁNEA

■Rotura microaneurismas de Charcot-Buchard.

■Déficit neurológico focal de comienzo brusco y curso progresivo.

PUTAMEN

Hemiparesia y hemihipoestesia contralaterales. Desv. ojos lado lesión. Dism. nivel conciencia.

TÁ LA M O

Sd. talámico + hemiplejía contralaterales

CEREBELO

Conciencia preservada inicial. Cefalea occipital, ataxia, vómitos. Hidrocefalia por compr. IV ventr.

PUENTE

Ver esquema del coma

S u sta n c ia b la n c a su b co rtica l

i

----------- 1------------

ANGIOPATIA AMILOIDE

MAtFORNIACION VASCULAR

A n c ia n o

Causa más frec. de hem. no HTA en el anciano. Recidivante

■Tratamiento: control TA (evitar cambios bruscos).

Jo v en

n i Otras

Angioma venoso: la más frec. asintomática Malf. arteriovenosa: la más frec. sintomática Angioma capilar (telangiectasias) Angioma cavernoso 1------------------1— Sospéchalas en paciente joven con hemorragia en un lugar atípico e historia de cefalea unilateral y pulsátil con convulsiones. En estos casos debe hacerse una angiografía.

Tratamiento de la H TIC Cerebelosa (> 3 cm de diámetro) Cirugía si Herniación

233

www.FreeLibros.me

ULTRA-RESÚM ENES Manual CTO de Medicina y Cirugía, 8.a edición

1. Se prefiere la TC para...

1.

2. Se prefiere la RM para...

2. Ver fosa posterior, infartos lacunares, trombo­ sis venosa o patología medular, e ictus en jóve­ nes.

3.

3.

Etiología embotica. Se produce 12-36 horas des­ pués de la embolia y suele ser asintomática.

1. ¿Cuáles son las que con mayor frecuencia dan síntomas?

1.

Las malformaciones arteriovenosas.

2. ¿Cuál es la malformación vascular cerebral más frecuente?

2.

Los angiomas venosos.

3. ¿Qué malformaciones vasculares sangran con menos frecuencia?

3. Angiomas venosos.

3.

La transformación hemorrágica de un infarto blanco cerebral sugiere...

Ver hemorragias y calcificaciones.

Malformaciones Vasculares.

4.

Hemorragia Subaracnoidea (HSA). - Causa más frecuente: Traumatismo craneoeneefáKco. - HSA espontánea más frecuente en > 20 años: rotura de aneurisma sacular (80% ). En < 20 años esta es más frecuentemente producida por malformaciones.

90% CONGENITOS

(SACULARES) Zonas de bifurcación Localización más frec.: confluencia de a. comunicante anterior y a. cerebral anterior

- C LÍN IC A :

Enfermedades sistémicas que pueden asociarlos:

OTROS Traumático Ateroscl erótico M IC Ó T IC O : endocarditis bacteriana

- Poliquistosis renal - Ehlers-Danlos - Marfan - Pseudoxantoma elástico - Displasia fibromuscular - Acromegalia - Coartación de aorta - Anemia de células falciformes

Rotura: tefalea súbita muy intensa (forma de presentación más frecuente} + sd. irritativo meníngeo con o sin pérdida de conciencia. Puede haber síntomas premonitorios o centinelas por compresión según la localización (ej: III par + midriasis arreactiva si se localiza en la comunicante posterior}. Diagnóstico diferencial con punción traumática: (prueba de los 3 tubos).

- D IA G N Ó ST IC O : TC CRANEAL sin tontraste (primera elección}.

Punción lumbar (más sensible, pero sólo se hace si alta sospecha y TC no concluyente}.

Angiografia de 4 vasos cuanto antes. - TRATAM IENTO:

Ingreso en UCI e iniciar tratamiento médico. E m b o liz a c ió n (siempre que sea posible}

Ctipaie (precoz en < 48-96 horas si grados I, II o III de Hunt y Hess y tardío a los 10-14 días si IV o V. Muy discutido}.

- C O M P LIC A C IO N ES Neurológicas

Vasoespasmo

Para excluir el aneurisma de la circulación general y prevenir el resangrado.

Sistémicas: SIADH, arritmias, mionecrosis cardíaca.

Responsable de la morbímortalidad Máximo el 62-82 día Clínica según vaso afectado

Profilaxis: nimodipino Tratamiento: triple H (aumenta perfusiónl pero también riesgo de resangrado}

Resangrado: primeras 24-48 Horas o una semana después. H idro teM ia Otras: infarto cerebral, hematoma subdural ...

254 :

www.FreeLibros.me

BIB B80

N

e u r o l o g ía

1. ¿Cuál es la localización más frecuente de los aneurismas micóticos?

1.

2. ¿Y de los aneurismas fusiformes?

2. Sistema vertebrobasilar.

CA PITU LO 17.

y

n e u r o c ir u g ía

Distalmente a la primera bifurcación de las ar­ terias principales del poligono de Willis (es de­ cir, más periféricos que los congénitos).

TUM ORES IN TRACRAN EALES.

Adem ás de ser uno de los tem as más preguntados en Neurología, los tumores son el tem a estrella de las preguntas de Neurocirugía. Suelen preguntar sobre la clínica o peculiaridades anatomopatológicas de algunos de ellos. Debes fijarte en la localización más frecuente de cada uno para deducir las m anifestaciones clínicas (utiliza como referencia el apartado de semiología de este ultrarresum en).

FRECUENCIAS: Lee con atención las siguientes premisas y fíjate en cóm o cam bia la respuesta al m odificar una sola palabra del enunciado:

1. Tumores sólidos más frecuentes en niños.

1.

Tumores del SNC.

2. Tumor intracraneal más frecuente del adulto...

2.

Metástasis.

3. Metástasis cerebral más frecuente en el varón...

3.

Ca broncogénico.

4. Metástasis cerebral más frecuente en la mujer...

4.

Ca mama.

5. Tumor que más frecuentemente metastatiza al SNC...

5.

Melanoma.

6. Tumor intracraneal primario más frecuente en adultos...

6.

Glioblastoma m ultiform e.

7.

7. Astrocitoma.

Tumor intracraneal más frecuente en niños...

8. Tumor neuroectodérmico primitivo más frecuen­ te...

8.

9. Tumor maligno más frecuente en niños...

9. Meduloblastoma.

10. Tumor supratentorial más frecuente en niños...

10. Craneofaringioma.

11. Tumores primarios con tendencia al sangrado...

11. Glioblastom a m ultiform e, m eduloblastom a, oligodendroglioma y adenoma hipofisario.

12. Tumores metastásicos con tendencia al sangra­ do...

12. Coriocarcinoma, melanoma, pulm ón, riñón y tiroides.

13. Tumor cerebral más epileptógeno...

13. Oligodendroglioma.

14. Tumor que más frecuentemente debuta con HTIC...

14. Meduloblastoma.

15. Tumor cerebral de peor pronóstico...

15. Glioblastoma m ultiform e.

16. Localización más frecuente en el adulto...

16. Supratentoriales (80%).

17. Localización más frecuente en niños...

17. Infratentoriales (60%).

18. Tumor intrarraquídeo más frecuente...

18. Metástasis.

Meduloblastoma.

235

www.FreeLibros.me

ULTRA-RESÚMENES. M anual C T O de M edicina y Cirugía, 8 .a edición CLÍNICA: Como esquema general puede servirte:

DATOS ANATOMOPATOLÓGICOS CARACTERÍSTICOS: -

Cordoma: células fisalíferas. Meduloblastoma: rosetas de Homer-Wright. Retinoblastoma: rosetas de Flexner-Wintersteiner. Oligodendroglioma: células en huevo frito. Pineocitoma: rosetas de Borit. Quiste coloide: material PAS (+). Meningioma: cuerpos de Psammoma.

DIAGNÓSTICO DIFERENCIAL POR LA LOCALIZACIÓN: (destacamos además algunas otras características relevantes de cada tumor).

Crisis + focalidad neurológica

UHfOMA I* M I SHt

MFTASTAStS

d iio b la ito m a

Mejor pronóstico el pilocítico y el subependimario de células gigantes (esclerosis tuberosa)

O íig o d in d n g fia m a

El de peor pronóstico

- Mujeres - Convexidad parasagital - Hiperóstosis cráneo y blistering - Aumento vascularización (único intradural vascularizado por la art. carótida externa) - Hormonodependencia - Si infancia o múltiples (NFM II)

lóbulo frontal. - Crisis y calcificaciones

Adultos

Niños

Línea media (cerebelo, tronco, n. óptico (NFM I)) Raro en hemisferios

O K B llO

MtNINWOMA

* Inmunodeprimidos (VIH) * Multicéntricos - Buena respuesta a corticoides - RT de elección

- Más frec. de tumor supradiafragmático que infradiafragmático - Unión corticosubcortical

Sust. blanca subcortical lóbulo frontal y temporal

n o H to m a ru ico s

ANGULO POHTOtBttBÍLOSO

n rvtH T tku io

ípefídim om a

Astror'rtoma Hemisférico

El mixopapilar en filum termínale

Medvlablastoma Vermiano Posible diseminación por LCR y/ó metástasis Niños

C olesteafom a (q uiste epiderm oide

Hemisférico Poliglobulia Von Hippel-Lindau

o tum or p e rio d o )

Adultos

UGLÓN QU¡ASMATICA

RIGIÓN

m ui

de hipófisis Más frec. el prolactinoma Hemianopsia bitemporal + clínica endocrinológica

plexos toroideos En niños en ventrículos laterales

Misma clínica campimétrica Calcificaciones supraselares

Neurohipófisis

Adultos

w w w .FreeLibros.m e

- Más frecuente el £ - Hidrocefalia + sd. Parinaud - El más radiosensible

N -

e u r o l o g ía y n e u r o c ir u g ía

Debes recordar además: La asociación entre determinadas neoplasias del SN C y enfermedades sistémicas (muchas de ellas facomatosis):

E n h m e d a d S h té m k a

N c e p h t ia d e l SN C

Esclerosis tuberosa

A strocitom a gigantocelular subep end im ario

Neurofibrom atosis tipo 1

C lio m a del nervio óptico

Neurofibrom atosis tipo II

N eu rin o m a bilateral del V III par

Enf. de von H ipp el-Lind au

H em angioblastom a cerebeloso

Enf. de Sturge-W eber

A ngiom a leptom eníngeo

M EN 1

H iperp lasia o ad enom a de hipófisis

Sd. de Klippel-Trénaunay

M alform ación vascular d e la m éd ula espinal

Sd. de Turcot

A strocitom as, m ed uloblastom a...

Infecció n por V IH

Linfom a cerebral prim ario

- La clín ica típica de la hipertensión intracraneal (HTIC): cefalea que em peora por la noche y con los cambios posturales, vómitos y edem a de papila. A veces hay diplopía por afectación del VI par (el más sensible a los incrementos de presión por su largo recorrido en la base del cráneo). - La clínica de la herniación uncal: midriasis ipsilateral y hem ianopsia hom ónim a contralateral (signos más precoces) y evoluciona a hem iplejía contralateral con Babinski, rigidez de descerebración y com a. Puede aparecer como conse­ cuencia de procesos expansivos supratentoriales e hidrocefalia. Recuerda que un III par con midriasis arreactiva sugiere etiología com presiva, mientras que si la pupila es normal hay que pensar en isquem ia (diabetes mellitus). - Atención al pseudotumor cerebral (también llamado hipertensión intracraneal benigna), en el que hay clínica de H T IC con aum ento de la presión del LCR (que es de composición normal) y pruebas de imagen normales (sin dilatación ventricular ni tum or cerebral objetivable). La prueba diagnóstica de elección es la punción lumbar.

REPASA; -

[C a u sa s

m á s fr e c u e n t e s d e eleva c ió n d e la p r e s ió n in tracraneal ,

T a bla

57, Ournno 15 d e l M a n u a l CTO 8a Ed.]

N U N C A realices una punción lum bar si hay signos de H T IC porque el paciente puede enclavarse. Sólo hay una excepción: el pseudotumor cerebral.

^

REPASA:

[M

a n e jo t e r a p é u t i c o g e n e r a l d e l p s e u d o t u m o r c e r e b r a l ,

CA PÍTU LO 12.

F ig u r a

76,

C a p ít u l o

15 d e l M a n u a l CTO 8 a E d .]

EN FERM ED AD ES DE LA PLACA MOTORA.

Debes conocer tres trastornos fundam entales de cara al M IR : la miastenia gravis, el sd. de Eaton-Lambert y el botulismo. Los suelen preguntar en form a de caso clín ico, y te obligarán a hacer el diagnóstico diferencial entre ellos.

1=^REPASA:

[D

ia g n ó s t ic o d if e r e n c ia l d e l o s

S

ín d r o m e s m ia s t é n i c o s ,

T a bla

50,

C a p ít u l o

12

del

M

anual

CTO 8a Ed.]

Fíjate especialm ente en la distinta form a de presentación: M U S T E N U G RA V IS EA TO N -U M BER T BOTUUSHIO

M úsculos c ra n e a le s M iem bros



► M iem bros (asimétrico) ► Pares craneales (sim étrico) ------- ► M iem bros (sim étrico)

Músculos craneales

N o clínica autonómica Disautonomía Disautonom ía

(no afecta pares sino músculos).

En el siguiente esquem a de tratamiento de M iastenia gravis, fíjate en: - El timo se quita en los pacientes con tim om a o formas generalizadas (entre la pubertad y los 55 años, aunque no haya tim om a). - La plasmaféresis o la Ig i.v. son de efecto breve, por lo que sólo se usan en crisis agudas o para preparar la tim ectom ía en pacientes de alto riesgo quirúrgico. - La inmunosupresión sólo se utiliza en pacientes en los que no dan resultado otras medidas.

237

www.FreeLibros.me

ULTRA-RESÚMEIMES. Manual CTO de Medicina y Cirugía, 8.a edición TRATAMIENTO DE LA MIASTENIA GRAVIS.

TRATAMIENTO DE LA MIASTENIA GRAVIS

Fonnn ocular exctusiva

Fcirma rj»:oorat1?n«1a

A nticollnesterásico

A n ticollnesterásico

Cr»s>: derm atitis, diarrea y demencia (se confunde con un cuadro psiquiátrico).

8. ¿Qué sospecharías ante un paciente que pre­ senta neuropatía sensitiva, disminución de la agudeza visual y dermatitis orogenital?

8.

Síndrome de Strochan o neuritis jamaicana.

9. ¿Qué te debe sugerir la aparición de un síndro­ me cordonal posterolateral en un paciente ané­ mico?

9.

Degeneración subaguda combinada de la m é­ dula por déficit de vitamina Bl2.

10. ¿Qué defecto sensitivo será más frecuente en el paciente anterior?

10. La alteración de la sensibilidad vibratoria y posicional (cordones posteriores).

11. Si un paciente que ha sufrido anoxia cerebral de la quese ha recuperado, desarrolla varios días después un cuadro de deterioro progresi­ vo con espasticidad y coma,pensarías en...

11. Encefalopatía postanóxica retardada por desmielinización difusa.

12. ¿Qué encefalopatías cursan con edema cere­ bral?

12. Hipercápnica, hepática, síndrome de Reye y sín­ drome de desequilibrio.

13. ¿Qué encefalopatía se precede habitualmente de hiperactividad adrenérgica?

13. La encefalopatía hipoglucémica.

14. ¿Qué sospecharías en un paciente alcohólico que ingresa en Urgencias con hiponatremia severa y desarrolla tetraplejia y anestesia, con­ servando solamente la mirada vertical y el par­ padeo?

14. Mielinólisis central pontina, como consecuen­ cia de un tratamiento demasiado rápido de la hiponatremia (suero hipertónico).

15. ¿Cuál sería tu sospecha diagnóstica en un pa­ ciente nefrópata que, 3 horas después de la hemodiálisis, desarrolla un cuadro de cefa­ lea, náuseas, agitación, somnolencia y con-

15.

7.

¿Cuál es la tríada de la pelagra?

— > Ataxia — > Síndrome

8, antes que el suero

Síndrome de desequilibrio.

244 www.FreeLibros.me

N CA PÍTU LO 7.

e u r o l o g ía y n e u r o c ir u g ía

E P IL E P S IA .

De este tema debes conocer:

1.

ETIO LO G ÍA POR GRU PO S DE EDAD. 35 años

Hipoxiae isquemia Crisis febriles. Idiopáticas. Tumores > TCE ECV

Z CLÍN ICA Y TRATAMIENTO DE ALGUNAS CRISIS Y SÍNDROM ES ESPECÍFICOS. ED A D

1 TÍP IC A S

G eneralizadas < 10 minutos No recidivantes

6 mesesS años

a ts a m u u s

I

Focales

1

> 1 5 minutos Recidivantes

A TÍP ICA S

- Espasmos infantiles en flexión

U . W tS T

Primer año

U . U N H O X - G A S T tU T

3-6 años

- Crisis muy variadas - Retraso psicomotor - Punta-onda lenta a 2-2,5 H z en vigilia y ritmo rápido a 10 H z durante el sueño

Adolesc.

- G eneralizadas - Con contracciones m ioclónicas en cintura escapular proxim al y MM SS - Al despertar - Fotosensibles - EEG : punta-onda a > 3 H z

u

w

a

( c p t l c p iio m io d o n H a fW&nH)

- Retraso psicomotor - Hipsarritm ia interictal

TÍP IC A S

a tis ts

m ausencia

T R A T A M IE N T O

C A R A C T E R ÍS T IC A S

Paracetam ol para la fiebre Profilaxis con diacepam rectal

A C TH Á c. valproico

Clonacepam Vigabatrina

Á c. valproico, y si no m ejora politerapia Felbamato

Á c. \alproico

G eneralizadas 5-10 segundos Sin pérdida de tono Varias al d ía EEG : punta-onda a 3H z

Etosuxim ida o ac. v a i ^ x o

6-14 años Asocia otros tipos de crisis ATÍPICAS

EEG: poüpuntas focales

attSIS PAROAUS COMPLEJAS 0 O S IS

Com ponente tónico EEG : punta-onda m ás lenta Refractaria a tratamiento Peor pronóstico

Carbam acepina (o fenitoína)

Ácido valproico (o Clonacepam )

MIOCLÓMKAS

Fenitoína, fenobarbital, carbam acepina y ác. valproico

ansa t o h k o -c ió h ic a s

3.

Suelen req u erir politerapia

Recuerda que un EEG anormal en ausencia de clínica NO DEBE tratarse con anticomiciales Por último, los EFECTOS SECUNDARIOS de cada antiepiléptico: FÁRMACO

EFECTOS SECUNDARIOS Hirsutismo, hiperplasia gingival

C a r b a m a c e p in a

H epáticos y hematológicos

T o p ira m a to

Som nolencia. Litiasis renal.

C to s u x im id a

Rash y hemato lógicos

fe n o b a r h r t a i

Sedación. N iños: hiperactividad

Repásalos en psiquiatría H epáticos, hematológicos, Ml

v a íp r o k o

pancreatitis hemorrágica, alopecia Trastornos dei com portam iento

V ig a b a t r in a

Trastornos del cam po visual

la m o tr ig in a

Exantema

G a b a p e n tin a

Intolerancia gastrointestinal

F e lb a m a to

Hematológicos

245

www.FreeLibros.me

ULTRA-RESÚMENES. Manual CTO de Medicina y Cirugía, 8.a edición -4

1. ¿Recuerdas qué antiepiléptico, cuando es to­ mado por la embarazada, se asocia con defec­ tos del tubo neural?

1.

El ácido valproico, pero también la carbamacepina.

2. ¿Y con defectos de la línea media y malforma­ ciones cardiacas?

2.

La difenilhidantoína y el fenobarbital.

3. ¿Cuál es el antiepiléptico de elección en au­ sencia simple?

3.

Acido valproico o etosuximida.

CAPITULO 8.

EN FERM EDADES D EGEN ERATIVAS D EL SISTEM A NERVIOSO.

Forma más frecuente: esporádica (80-90%).

ClÍNICA: Debilidad muscular lentamente progresiva con sígaos de I a y 2 a motoneurona

Afectación Afectación Afectación Demencia Disfunción

musculatura extraocular esfinteriana sensitiva autonómica

PRO N Ó STICO : supervivencia media de 3 años desde el inicio. La causa de la muerte es la debilidad de la muscula­ tura respiratoria.

TRATAMIENTO: no tiene en el momento actual. El riluzole prolonga discretamente la supervivencia.

CAPÍTULO 9.

EN FERM EDADES V IR A LES Y PRIÓNICAS D EL SNC.

Clínica característica de las encefalitis: disminución del nivel de conciencia y focalidad neurológica. - Causa más frecuente de encefalitis esporádica: herpes simple tipo I. No debes dejar de estudiar la ENCEFALITIS HERPÉTICA. Recuerda que presentará clínica, alteraciones radiológicas y alteraciones electroencefalográficas de focalidad frontotemporal. Ante la sospecha, debes administrar aciclovir de forma empírica. En el momento actual la PCR del LCR se ha convertido en el procedimiento diagnóstico de elección para las meningoencefalitis víricas, incluida la herpética. La biopsia cerebral se debe reservar para encefalitis de mala evolu­ ción, diagnóstico confuso o no respuesta a tratamiento. Hay otras enfermedades virales que debes conocer:

Virus asoc.

P o r a p o r e s ia

e s p a s t ic a

t ro p ic a l

le u t o e n t e fa lo p a t ia m u H H o ta l

HTLV-1

V iru s JC

p r o g re s iv a

Clínica característica H ip erreflexia y espasticidad progresivas de M M II

Tr. visuales (hem ianopsia hom ónim a), d em en cia y tr. personalidad M al ren dim ien to escolar y trastorno

P a n e n c c fa lrtis e s c le r o s a n te su b a g u d a

Saram pión

de personalidad. D espués deterioro n e u ro ló g ico y tetraparesia espástica

Fíjate en que todas afectan de algún modo a la sustancia blanca y no hay tratamiento para ninguna de ellas.

246 www.FreeLibros.me

N

e u r o l o g í a y im e u r o c ir u g ía

Recuerda que las ENFERMEDADES PRIÓNICAS tienen como agente patógeno una proteína infectiva llamada proteí­ na priónica o PrR Las formas familiares se deben a mutaciones de esta proteína en el cromosoma 20.

Características clínicas Canibalismo

Subaguda

Forma tlásita

Esporádica (85%) Familiar (AD) Yatrógeno (neurocirugía, extractos hipofisarios, cirugía corneal)

Rápidamente progresiva, supervivencia media 5 meses

Mueva vanante

Brote de encefalopatía espongiforme bovina

Subaguda, supervivencia media 15 meses

Kurv

CreutzfeldtJakob

G en tmann-StráussIerU heinker

Familiar

Insomnio fam iliar fatal

Familiar (AD)

Demencia + ataxia + mov. anormales

Demencia + ataxia + mioclonías

Debut con trastornos psiquiátricos, y posteriormente parestesias en M mii y atáxia Disfunción cerebelosa sin dem encia ni mioclonías. Ceguera, sordera y parálisis mirada conjugada

Rápidamente progresiva

Insomnio intratable, hiperactividad simpática, trastornos endocrinos, disartria y trastornos motores

1. ¿Cuál de ellas es frecuente en pacientes con SIDA?

1.

Leucoencefalopatía multifocat progresiva.

2. ¿En cuál es típico un patrón periódico con des­ cargas de ondas lentas de alto voltaje seguido de un periodo de aplanamiento de la actividad eléctrica cerebral?

2.

Panencefalitis esclerosante subaguda.

3. ¿Cuál presenta característicamente acelutaridad con proteínas normales o ligeramente elevadas en el LCR?

3.

Panencefalitis esclerosante subaguda.

4. ¿Cuál es la prueba diagnóstica de elección para la enfermedad de Creutzfeldt-Jakob?

4.

La biopsia cerebral (degeneración espongifor­ me y placas de amiloide PrP*).

5. ¿Qué diferencia a la enfermedad de CreutzfeldtJakob secundaria a extractos hipofisarios de las formas clásicas?

5. Son pacientes más jóvenes y predomina la ata­

CAPÍTULO 18.

xia sobre la demencia.

TRAUMATISMOS CR A N EO EN CEFÁ LICO S (TCE).

Como en todo paciente traumatizado, debes tener en cuenta las prioridades en el manejo de esta patología. El nivel de conciencia, valorado mediante la escala de Glasgow, es el principal factor pronóstico:

Glasaow

TCEleve TCE moderado TCE severo

14-15 9-13 3-8

CONCEPTOS: - Conmoción cerebral: pérdida de conciencia breve y transitoria sin alteraciones parenquimatosas en la TC ni RM. - Daño axonal difuso: pérdida de conciencia precoz y mantenida. En ocasiones se puede observar en la TC signos indirectos de DAI como pueden ser hemorragias puntiformes en zonas subcorticales, cuerpo calloso o porción rostral del tronco del encéfalo. - Contusión cerebral: en la T C se aprecian áreas hemorrágicas (hiperdensas) en el parénquima con escaso efecto de masa, que suelen localizarse en los polos frontal, temporal u occipital.

247

www.FreeLibros.me

ULTRA-RESÚM ENES. Manual CTO de Medicina y Cirugía, 8.a edición H

2.

En un paciente que tras un traumatismo pre­ senta hematoma periorbitario bilateral, equimo­ sis retroauricular, otorragia y licuorrea sospe­ charías...

1.

Fractura de ta base del cráneo.

Y si presentara exoftalmos pulsátil, quemosis conjuntival y lesión del VI par craneal, y el pa­ ciente comentase que oye un ruido dentro de la cabeza...

2.

Fístula carótido-cavernosa.

Meses después de un golpe en la cabeza, tu paciente se queja de cefalea y mareos. La ex­ ploración neurológica es normal, pero el pa­ ciente se muestra irritable y no puede concen­ trarse en sus tareas, sospecharías...

3.

Síndrome postraumático.

Com o complicación de los T CE destacan los hematomas epidural y subdural, que debes aprender a diferenciar, para lo cual te proponemos la siguiente tabla:

HEMATOMA EPIDURAL

HEMATOMA SUBDURAL Rotura de w . corticales. Agudo: primera sem ana.

Sangrado arterial (85%).

Origen

Lo m ás frecuente rotura d e la a . meníngeo media.

Subagudo: 7-10 días postTCE. Crónico: T C E trivial o no identificado en 50%. Típico de ancianos y alcohólicos.

Conmoción

cerebral

Clínica

Intervalo lúcido

Herniación uncal (coma de rápida evolución)

Agudo: clínica de herniación uncal progresiva de rápida evolución.

Crónico: cefalea y dem encia progresivas (parecido A un q u e < 3 0 % se presentan con la c lín ica clásica.

Hiperdensidad en forma de lente biconvexa.

TAC

Frecuentem ente efecto de masa.

Lesión parénquima

En general m enor y más tardía (por com presión)

Mortalidad

Agudo: hiperdensidad en forma de semiluna. Subagudo: isodenso. Crónico: hipodensidad en forma de sentí/una. En general m ayor y desde el principio (la sangre está en contacto con el parénquim a cerebral).

C on diagnóstico y tratamiento precoz

Las formas agudas tienen una mortalidad del

la mortalidad es aprox. 10%.

50-90% .

Evacuación quirúrgica mediante craneotomía.

Tratamiento

a A C V isquém ico pero fluctuante).

Agudo; evacuación quirúrgica m ediante craneatomia. Crónico: evacuación quirúrgica mediante trépano con o sin drenaje subdural.

H e m a to m a

H e m a to m a

e p id u r a l

su b d u ral

Línea de fractura

A. meníngea

Herniación del

Compresión del III p.

Herniación de la amígdala cerebelosa

248

www.FreeLibros.me

N CA PÍTU LO 20.

e u r o l o g ía y n e u r o c ir u g ía

PATOLOGÍA RAQUIM EDULAR.

HERNIA DISCAL. -

Es la patología neuroquirúrgica más frecuente. Localización más frecuente: colum na lum bar (L5-S1 > L4-L5), y en segundo lugar, cervical (C5-C 6 > C 6-C 7). C lín ica: contractura y dolor que aum entan con la flexión de la colum na + dolor irradiado a una extrem idad.

Motor

C5 C6 C7

Separación hombro

Sensitivo

Flexión codo

Hombro y cara lateral del brazo

Flexión codo Extensión muñeca

Cara lateral antebrazo 1® y 2® dedos

Extensión codo Flexión muñeca

Reflejo Bicipital

S

Bicipital Estilorradial

3® dedo

Tricipital

1

Tricipital

«»

------

1

a

Flexión dedos

4S y 5S dedos Cara medial antebrazo

TI

Aproximación y separación dedos

Cara medial antebrazo y medial brazo

Extensión rodilla

Cara anterior rodilla Cara medial pierna

Dorsiflexión tobillo

Cara anterolat. pierna

Extensión 12 dedo

Dorso pie hasta 19 dedo

Flexión plantar

Cara post. muslo y pierna Planta y cara lat. pie

* Excepcionalmente puede haber disminución del Aquíleo.

1.

Un cuadro de claudicación neurógena de MMII con lumbociáticas bilaterales en el que el dolor aumenta con la hiperextensión de la columna, te sugiere...

1.

Estenosis d e canal raquídeo.

Fíjate en que el dolor de la hernia discal aum enta con la flexión, m ientras que el de la estenosis de canal lo hace con la extensión.

MANEJO DE LUMBALGIA Y LUMBOCIÁTICA AGUDA. - En prim er lugar, tratamiento conservador: reposo en cam a unos 2 días (m ovilización precoz), A IN Es, miorrelajantes. - No indicado estudio radiográfico ni analítica en el prim er episodio, excepto si se sospecha causa grave (fiebre asociada, existencia de cán cer sistém ico, duración del cuadro más de 1 mes, A D V R ..) - Estudio de imagen (RM) urgente si: síndrome de cola de caballo y/o pérdida de fuerza en pierna de aparición brusca.

1=^REPASA:

[ C u a d r o s c o m p r e s iv o s d e l s is t e m a n e r v i o s o p e r if é r ic o , F i g u r a del M anual

67, C a p í t u l o 4 ,

S e c c ió n de T r a u m a t o lo g ía

CTO 8a E d .]

Pon especial atención al estudiar el síndrome del túnel del carpo (n. m ediano).

TRAUMATISMOS RAQUIMEDULARES. -

Fractura de Jefferson. A nillo del atlas. Fractura de Hangm an. Pedículos de C2 con subluxación anterior C 2-C 3 . Luxación atloaxoidea. Recuerda la relación con el sd. Dow n y la artritis reum atoide.

CA PÍTU LO 19. -

A B S C E S O C E R EB R A L Y EM PIEM A SU BD U RAL.

ET IO LO G ÍA : M ixtos (aerobios 60% y anaerobios 40% ) SID A : Toxoplasma gondii y hongos. Neutropénicos: Gram negativos y hongos.

www.FreeLibros.me

ULTRA-RESÚM ENES. Manual CTO de Medicina y Cirugía, 8.a edición H CLINICA: Tríada clásica: fiebre + cefalea + déficits focales. Sólo una minoría se presenta así. Lo más frecuente es cefalea (70%). Con frecuencia no hay fiebre. DIAG N Ó STICO : TAC ó RM: lesión hipodensa/hipointensa con fina pared que capta contraste en anillo. TRATAMIENTO: Cefalosporina de 3a + metronidazol (añadir vancomicina si sospecha de Staph.). Cirugía: si es único, encapsulado y produce H TIC o efecto de masa. - Aspiración estereotáxica (sobre todo abscesos múltiples y profundos). - Escisión (encapsulados y hongos).

1. ¿Conoces otras lesiones que capten contraste en anillo?

1.

2. ¿Recuerdas el manejo del absceso cerebral en pacientes con SIDA?

2. T ra ta m ie n to e m p íric o p a ra Toxoplasm a (sulfadiacina+pirimetamina). Si no responde, se hace biopsia cerebral estereotáxica para des­ cartar linfoma u otro germen menos frecuen-

CAPITULO 21.

Toxoplasmosis, linfoma cerebral primario, $iioblastoma m ultiforme y metástasis.

ANOMALIAS D EL D ESARRO LLO .

Craneosinóstosis: la más frecuente es la escafocefalia (sutura sagital). 1=^

REPASA:

[C

r a n e o s in ó s t o s is ,

A

partado

21.1,

C

a p ít u l o

21

d el

M

anual

C TO 8a E d .]

Mielomeningocele: forma más frecuente de espina bífida manifiesta. Recuerda su relación con el déficit materno de ácido fólico y el aumento de alfafetoproteína en suero materno y líquido amniótico. Malformación de Arnold-Chiari: la forma más frecuente es la tipo I y se relaciona con la siringomielia. Repasa las otras formas en el Manual.

www.FreeLibros.me

O C A PÍTU LO 11.

f t a l m o l o g ía

V ITR EO Y RETIN A.

RETINOPATÍA DIABÉTICA. R E C U ER D A : antes de la pérdida del tono vascular se produce una pérdida d e com petencia de la barrera hematorretiniana, siendo el signo más precoz que es sólo detectable mediante A FG .

P é rd id a d el to n o v a scu la r

H ip o x ia

P é rd id a d e la fu n ció n d e b arrera

Miicro a n e u rism a s

H e m o rra g ia

1er Signo c lín ic o v isib le

P u ntifo rm es o e n " lam a"

E x u d a d o s d u ro s

E x u d a d o s alg o d o n o so s

1y T ra d u c e n e d e m a d e retina

T ra d u c e n ¡isq u e m ia retin a

i

M ACU LA

Si las áre as d e isq u e m ia so n e xten sas

ed em a m a cu la r

N EO VA SO S

Si a p a re c e n e n la

P é rd id a progresiva d e visió n

1. LA SER LO C A L (Argón) 2. T R IA M IC IN O L O N A IN T R A V IT R E A o a n t ic u e rp o s a n ti-V E G F

<

Sin edema macular -

-► Observación

Con edema macular ■

-► Láser focal

G la u c o m a N e o v a s c u la r

— •—

H e m o rra g ia vitrea

P A N F O T O C O A G U L A C IÓ N (P F O

No proliferativa

Proliferativa —

,

Aguda

Pérdida de visión en un diabético

* Aparecen neovasos -

-+ PFC

Indolora: hemorragia vitrea

<

Con edema macular: glaucoma neovascular Meses: edema macular

1Crónica » Años: cataratas

1. ¿Cuál es la causa más frecuente de ceguera irre­ versible en: -Occidente -Edad laboral -El mundo

1.

Respuestas: - Degeneración macular senil. ■Retinopatia diabética. - Tracoma (algunos proponen el glaucoma cró­ nico sim ple).

www.FreeLibros.me

251

ULTRA-RESÚM ENES. Manual CTO de Medicina y Cirugía, 8.a edición 'A

2.

¿Y de ceguera reversible?

3. La retinopatía diabética no es la única manifes­ tación oftalmológica de la DM. ¿Recuerdas algu­ na otra?

2.

Cataratas.

3.

En los párpados son frecuentes el chalación y orzuelos de repetición. Las cataratas aparecen a edades más precoces y asimismo son frecuen ­ tes las parálisis de pares craneales (III y VI).

RETINOPATÍA ESCLEROHIPERTENSIVA. Alteraciones vasculares por arterieesclerosis -Signos de cruce -Alteraciones de la transparencia (hilo de cobre, hilo de plata) -Alteraciones del calibre

Alteraciones de la HTA -Estrecham iento arterial -Rectificación del curso de los vasos -Esclerosis reactiva

Alteraciones por extravasación (indicadores de malignización) -Exudados duros y algodonosos -Edem a de papila y retiniano -Hem orragias puntiformes y en llama

1.

¿Qué nos hace definir la hipertensión como maligna?

1.

La presencia de edema de papila; no la definen otros signos de repercusión orgánica, otros sig­ nos oftalmoscópicos, ni los valores de la TA.

La clasificación de Keith-Wegener-Barker pretende valorar la supervivencia; de ella debes conocer que el estadio incluye hemorragias en llama y exudados algodonosos, y el IV el edema de papila.

I

PATOLOGIA VASCULAR RETINIANA. Para com prender algunos hallazgos que aparecen en las oclusiones vasculares, es importante que vascularización de la retina: T

recuerdes la

an terio res: para m úsculos y ú vea an terio r

Arterias ciliares |

largas: para m úsculos y úvea an terio r posteriores

Argyll-RObertsOn—z> Pupila miOtica bilateral A:

Ú"

255

www.FreeLibros.me

ULTRA-RESÚM ENES. Manual CTO de Medicina y Cirugía, 8.a edición

En el defecto pupilar AFERENTE no hay ANISOCORIA. S i existe ANISO CORIA, es porque hay una alteración en la respuesta (vía eferente).

Igual con la luz que en oscuridad -------► Anisocoria esencial

Pupila

ANISOCO RIA '

-► Más marcada con luz -------► Fallo en el mecanismo de Miosis Más marcada en oscuridad

Fallo en el mecanismo de Midriasis

Tamaño de la pupila depende de BALANCE... MIDRIASIS

MIOSIS

ISOCORIA

i SIMPÁTICO

PARASIMPATICO

(Adrenérgico)

i OSCURIDAD / STRESS

LUZ

Feos sistémicos simpaticomiméticos

Feos sistémicos colinérgicos

ANISOCORIA

CON PUPILA CNMIDRIASIS

Ixteso simpático

Defecto parasimpátíto

•Utilización colirio simpaticomimético (Fenilefrína, cocaína)

• Utilización colirio anticolinérgico (Atropina, ciclopentolato, tropicamida) • Pupila ADIE responde lentamente a la luz • LESION III p.c no responde a la luz • Argy II Robertson (no responde a la luz pero sí a la acomodación)

Exceso parasimpatiio • Exceso colirio acetilcolina

Síndrome FHorner

NERVIO OPTICO.

Papilitis. Neuritis óptica anterior. CAUSA

EDAD

Virales

Infantiles

Idiopáticas o esclerosis múltiple

Adulto joven

Isquémica (forma arteriosclerólica)

+/- 60 años

Isquémica (forma arterftica)

+/- 75 años

Neuritis retrobulbar.

CAUSA DE FORMAS AGUDAS CAUSA DE FORMAS CRÓNICAS

256

www.FreeLibros.me

Esclerosis m ú ltip le es la m ás fre cu e n te . H e re d ita ria s, tó x ico -m e ta b ó lica s, a lco h o l-n ico tín ica s.

O

f t a l m o l o g ía

Neuritis ópticas. Diagnóstico diferencial entre la papilitis y la neuritis retrobulbar.

Clínica: -Disminución de la visión central rápida (horas-días) -Defecto pupilar aferente Fondo de Ojo ------

í

Papila edematosa

Normal

Papilitis

Neuritis retrobulbar (dolor con el movimiento ocular)

1

1

Tratamiento etiológico

PAPILEDEMA

NOA

NOP

Enfermo

Enfermo

Normal

11

11

No existe

Existe

* Nervio mojado"

NERVIO ÓPTICO

SANO

AGUDEZA VISUAL DEFECTO PUPILAR AFERENTE

Existe

- ITIS: existe,

con movimien­

- ITIS: existe,

con movimien­

DOLOR

No existe

tos oculares. - PATÍA: no existe

tos oculares. - PATÍA: no existe

FdO (EDEMA)

+++

+

Ni el médico ni el paciente ven nada.

CAMPO VISUAL

T mancha ciega

Defecto centrocecal

Defecto centrocecal

- ITIS: - Niños: viriasis. - < 50 años: idiopático

ETIOLOGÍA

- > 50 años: isquémica "patía" a) Arterítica: si existe - Síntomas arteríticos o - signos de arteritis o - T V SG : corticoides b) A rterosclerótica.

H TiC

REPASA:

[ D i a g n ó s t i c o d i f e r e n c i a l e n t r e l a s c a u s a s d e p a p i la e d e m a t o s a , A p a r t a d o 1 3 . 3 y T a b l a 2 1 , C a p ít u lo

13

del M anual

CTO 8a E d . ]

Si (lega a Urgencias una mujer joven quejándo­ se de dolor ocular con el movimiento de los ojos y pérdida de la agudeza visual unilateral de instauración rápida. 2.

Esclerosis múltiple

Neuritis retrobulbar como manifestación p re ­ coz de esclerosis múltiple.

En el caso anterior, ¿qué esperas encontrar en el fondo del ojo?

Nada, recuerda que la neuritis es retrobulbar,

y que, por lo tanto, "ni enferm o ni médico ven nada".

3. Además de edema de papila bilateral (papiledema), ¿qué otra manifestación puede dar la hi­ pertensión intracraneal en el ojo?

3.

Diplopía por parálisis. Recuerda que las princi­ pales causas de parálisis de pares craneales si­ guen orden alfabético: - Hipe: DM y Aneurismas de ACP. - IVpc: Traumatismos. - Vlpc: Tumores que produzcan HIC.

www.FreeLibros.me

257

ULTRA-RESÚM EN ES. Manual CTO de Medicina y Cirugía, 8.a edición -H

CA PITU LO 9.

GLAUCOM A.__________________________________________________________________

En este tema, es de vital importancia aclarar que sólo podemos hablar de glaucoma cuando existe un daño del nervio óptico; es decir, al contrario de lo q ue se suele pensar, glaucoma no se define como aumento de presión intraocular. G LA U C O M A PR IM A RIO DE A N G U L O ES T R E C H O . E ST A D IO S

-

-

P IO

C L ÍN IC A - N o h a y s ín t o m a s . - A n g u lo ir id o c o r n e a l e s t r e c h o (hiperm ótrojM »).

P re g la u c o m a

N o r m a l.

A t a q u e s p r o d r ó m ic o s

E le v a d a t r a n s it o r ia m e n t e . V u e lv e a la n o rm a lid a d e sp o n tán e a m e n te .

A ta q u e a g u d o

E le v a d a d u r a n te lo d o e l tie m p o q u e d u ra el a t a q u e .

- V is ió n b o r r o s a , h a lo s c o lo re a d o s . - C e f a le a o c a s io n a l. -

O jo ro jo d o lo ro so en m id ria sis m e d ia . S ín to m a s v eg eta tiv o s. D u r e z a p é tre a d e l g lo b o . D e s p u é s d e c a d a a t a q u e se r e d u c e c o n c é n t r ic a m e n t e el ca m [H J visu a l (si se d e m o r a e l Ira ta m ií'n to ).

Tratamiento de la crisis*(transitorio). Administración de diuréticos osmóticos. Analgésicos. Acetazolamida. Colirio de mióticos (se usan una vez que ha bajado la PIO). Colirio de esteroides. Colirios hipotensores. *Recuerda que está contraindicado todo aquello que produzca midriasis, ya que ésta dificulta la circulación del humor acuoso (atropina, ADTs, simpaticomiméticos,...). Tratamiento definitivo. Iridotomía periférica (quirúrgica o con láserYAG). También tratar el ojo contralateral. G L A U C O M A PRIM A RIO D E Á N G U L O A B IER T O .

D e fe c t o en f o reabsorción del humor acuoso S c re e n in g (rr d e ¡a P IO )

D ia g n ó s t ic u

P IO elevad a (más de 24 m m Hg, 2 0 -2 4 sospechoso)

A u m en to de la e xcav ació n papilar (más de 1/3 del diám etro papilar)

C am p im e tría

T ra ta m ie n to

T — /V\édico ( c o l i r i o s ) d e

2 -

e le c c ió n :

Si f r a c a s o :

- T ra b ecul o p Iast ia (láser de argón) o - T rab ecu lo cto m ía

+ A nálogos de Prostagland i na F. (-prost): de e le c ció n : - C o n tra in d ica c io n e s: glaucom a agudo, uveitis, gestación, herpes intraocular. - Efectos secu n d a rio s: h iperpigm en tació n iris e h i pertriq u iaris. -l- B etabloqueantes (Tim olol). 4- Inhibido res de anhid rasa, o a d r e n é r g ic o s .

1. ¿A partir de qué edad habría que hacer una tonometría a la población general, con el fin de detectar precozmente el glaucoma?

1.

A partir de ios 40 años.

2. ¿Qué disposición adoptan los vasos papilares en el glaucoma?

2.

Debido ai aumento de la excavación papilar adoptan una disposición en bayoneta. En este caso tienen que "bajar un escalón"; en el ede­ ma de papila "tenían que subirlo".

258

www.FreeLibros.me

O CAPÍTULO 10.

f t a l m o l o g ía

U V EÍTIS.

IRIDOCICLITIS O UVEÍTIS ANTERIOR

Clínica

Tratamiento Etiología más frecuente

COROIDITIS O UVEÍTIS POSTERIOR

- O jo rojo doloroso. - Precipitados queráticos, hipopion y fenóm eno de Tyndall. - Miosis.

- Moscas volantes, visión con niebla. No dolor si sólo posterior. - Lesiones exudativas, hemorragias cicatrices en coroides, vasculitis.

- Midriáticos tópicos. - Corticoides tópicos.

- Corticoides sistémicos.

- Idiopática.

- Toxoplasmosis.

- Otros tratamientos según etiología.

1. Un varón de 30 años, con antecedentes de rigi­ dez en la columna lumbar, llega a Urgencias por un cuadro compatible con uveítis. ¿Qué pien­ sas que padece este paciente?

1.

Una espondilitis anquilopoyética. La uveítis es la complicación extraarticular más frecuen te de esta enfermedad.

2. ¿Cuál es la forma de uveítis anterior más fre­ cuente?

2.

A pesar de todas estas interesantes asociacio­ nes, debes ten er claro que la form a más habi­ tual es la idiopática, y que sólo a p a rtir del segundo o tercer episodio se inicia un "protoco­ lo de uveítis~ con el fin de determ inar una p o­ sible causa.

3, En la sarcoidosis puede aparecer uveítis asocia­ da a ...

3.

Fiebre, parotiditis, y a veces, parálisis facial (triada de Heerfordt).

Para orientar la etiología, es útil clasificar las uveítis anteriores en granulomatosas y no granulomatosas.

CARACTERÍSTICA

NO GRANULOMATOSA

GRANULOMATOSA

Instauración

Aguda

Sobagoda

Intensidad de los síntomas

M arcada

M ás leve

Precipitados queráticos

Finos

Gruesos (en grasa de carnero)

En la cámara anterior predomina

Tyndall intenso

Tyndall m oderado

Ejemplo

Espondilitis anquilopoyética

Sarcoidosis

M E IA N O M A DE C O R O ID E S . De este tumor debes saber que es el tumor primario ocular maligno más frecuente en el adulto y que las metástasis ocurren casi siempre por vía hemática, siendo su localización más frecuente el hígado.

C A PÍTU LO 8.

CR IST A LIN O .

De las cataratas, debes saber que el tipo más frecuente es la catarata senil y que el tratamiento de elección es quirúrgico, mediante facoemulsificación de la catarata e implante de una lente intraocular en cámara posterior. Complicación postoperatoria más frecuente: la opacificación de la cápsula posterior, que se trata mediante capsulotomía con láser YAG. Complicación postquirúrgica más grave: la endoftalmitis, que se trata con antibióticos intravítreosy/o vitrectomía.

259

www.FreeLibros.me

ULTRA-RESÚM EN ES. Manual CTO de Medicina y Cirugía, 8.a edición :H

1. ¿Qué fármacos son los que más frecuentemen­ te ocasionan catarata?

1.

Corticoides

2. ¿Sabes en qué enfermedad muscular están afec­ tados de cataratas hasta el 90% de los pacientes?

2.

La distrofia miotónica de Steinert.

3. ¿Qué tipo de alteración pupilar producen las cataratas?

3.

Ninguno.

4. ¿En qué patologías aparece diplopia monocular?

4.

En la subluxación del cristalino.

5. ¿Cuál es la única causa de catarata reversible?

5.

La que se produce en la galactosemia.

CAPÍTU LO 7.

y mióticos.

CÓRN EA Y E S C LE R A .

Queratitis: inflamación del epitelio y estroma subyacente. Ulcera corneal: pérdida de sustancia del epitelio y estroma. Debes recordar que presenta diagnóstico diferencial con otras causas de ojo rojo doloroso.

P A T O L O G ÍA C O R N EA L. CA R A CTER ÍSTIC A S Y TR A T A M IEN TO .

Queratitis i

Infiltrado estromal con edema. De bordes poco claros; aspecto grisáceo y sucio. Hipopion (pus estéril en cámara anterior).

1

Úlcera dendrítica; ausencia de dolor por anestesia

“Afectación deliestroma corneal (no tiñe con fluoresceína). Son por mecanismo autoinmune

I

Queratitis que afecta al área correspondiente a la hendidura palpebral.

Antecedente de lesión del trigémino.

Queratitis de exposición.

Queratitis neuroparalítica.

corneal. Trás varios episodios

1 Absceso bacteriano

Queratitis herpética.

1 Colirio de antibiótico y ciclopléjicos (relajación atropínica del músculo ciliar).

Aciclovir tópico. Ciclopléjicos. Antibióticos profilácticos.

Lubricantes. Oclusión ocular. Antibióticos tópicos.

Afectación estromal de forma discoide secundaria a una queratoconjuntivitis herpética (tras varios brotes).

Edema estromal generalizado; por ejemplo en el contexto de una sífilis congénita.

I Queratitis estromal disciforme

260

Queratitis estromal difusa

T

+

M idriáticos. Corticoides tópicos.

M idriáticos. Corticoides.

www.FreeLibros.me

Oclusión ocular. Antibióticos tópicos.

O 1. La úlcera por Acanthamoeba se relaciona con el uso de...

1.

f t a l m o l o g ía

Usuario de lentillas que han nadado en agua dulce.

2. Establece las siguientes relaciones: 1. Artritis crónica juvenil oligoarticular. 2. Artritis reumatoide. 3. Enfermedad de Reiter.

A. Conjuntivitis. B. Uveitis. C. Queratomalacia. R esp u esta s: 1-B, 2-C, 3-A

y B.

3. De las inflamaciones del polo anterior del ojo, ¿en cuál no harías oclusión ocular?

3.

En las de origen infeccioso.

4. ¿Por qué en las queratitis herpéticas están con­ traindicados los corticoides y sin embargo se usan en las queratitis estromales y disciformes herpéticas?.

4.

Porque estas últimas son reactivas (por meca­ nismo autoinmune), no por herpes activo.

CAPITULO 4.

ORBITA.

P A T O LO G IA IN FLAM ATO RIA.

- O fta lm o p a tía tiro id e a : es la cau sa m ás fre cu e n te d e e xo ftalm o s en el ad u lto . ■C e lu litis preseptal

» - C e lu litis o rb ita ria

> _ T ro m b o fleb itis d el

(cau sa m ás fre c u e n te

sen o cave rn o so

d e e xo ftalm o s en n iñ o i.

SÍNTOMA

cE u u u n s PRESEPTAL

c a ju im s ORBITARIA

T R O M M SB O a SENO CAVERNOSO

Edema palpebral inflamatorio







Exoftalmos

No

Sí (M O N O LA TER A L)

Sí (BILATERAL)

Dolor local a la presión



IM PO R TA N TE

Tolerable

Dolor con la movilidad ocular

No





Repercusión general

Leve

M oderada

M UY GRAVE

P se u d o tu m o r in flam ato rio .

1. Recuerda cuál es la vía de acceso más frecuen­ te para que se desarrolle una celulitis orbitaria.

1.

2. ¿Cuál es el germen más frecuente encontrado en las tromboflebitis del seno cavernoso?

2. El Staphylococcus aureus.

Los senos paranasales, en especial desde el etm oidal.

P A T O LO G ÍA VASCU LAR.

1. ¿Qué sospecharías si llega a Urgencias un padente con un TCE severo que presenta un exof­ talmos axial de instauración rápida, pulsátil y ojo congestivo?

1.

Fistula carótido-cavernosa.

261

www.FreeLibros.me

ULTRA-RESÚM ENES. Manual CTO de Medicina y Cirugía, 8.a edición

2. Llega a consulta un niño con exoftalmos severo, intermitente, que aparece cuando el niño llora. Tú piensas que se trata de...

2. Varicocele orbitario.

3. ¿Qué debemos sospechar ante un exoftalmos que se instaura en segundos (muy brusco).

3.

Hemorragia orbitaria.

P A T O L O G ÍA TU M O R A L. Los tumores más frecuentes en el niño y en el adulto son los de comportamiento benigno. BENIGNO

MALIGNO

METÁSTASIS MÁS FRECUENTE

Niño

H o m ang io m a ca p ila r

R ab d o m io sarco m a

N eu ro b lasto m a

Adulto

H e m an g io m a cavernoso

Lin fo m a

M a m a , pu lm ó n.

1. ¿Cuál es la patología más frecuente que se presenta como un tumor?

1. El pseudotumor orbitario. Esta patología es más frecuen te que cualquier tumor.

2. ¿Cuál es la causa más frecuente de exoftalmos de dirección inferomedial?

2. Tumor de la glándula lacrimal principal.

SEM IO LO GIA. D IA G N O S T IC O D IF ER E N C IA L D EL O J O R O JO D EL LACTAN TE.

LAGRIMEO, FOTOFOBIA BLEFAROESPASMO

T Secreción purulenta o m ucopurulenta con

■Epífora crónica desde el nacimiento.

- Aumento del diámetro corneal.

form ación de légañas

■Episodios de secreción

- Aum ento de la PIO .

m ucopurulenta por

- Excavación papilar

los puntos lacrimales.

Conjuntivitis del lactante

aumentada.

Dacriocistitis del recién nacido

Glaucoma congénito (buftalmos)

-M asaje del saco la crim a l.

C iru g ía : restablecer

-R e p e rm e a b iliza r la vía

circu la ció n del

si a los 6 m eses no ced e

h. acuoso

esp o ntáneam en te

REPASA:

262

[D

ia g n ó s t ic o

D

if e r e n c ia l d e l a s c o n i u n t iv it is d e l l a c t a n te ,

www.FreeLibros.me

T a bla

6,

C a p ít u l o

6

del

M

anual

CTO 8 a E d . ]

O

Diagnóstíto

f t a l m o l o g ía

d ea n irltf e las príndpalesde hiperemia reattiva.

HIPEREMIA

Ciliar

VISIÓN

Pósito

i ó norma!

DOLOR

+-» profundo

• ++ muy intenso

PUPILA

Miosis

Miosis - Bradicoria

Mídriasis media arreactiva

CÁMARA ANTERIOR

Normal o Tyndaíl leve

Tynciv ■+ células

i si rochada

v

■‘

wmmmmammmmmmmrn

i ■|

. I í§é ¿duele?

NO

¿ P i* ia ?

¿Pupia?

Glaucoma agudo de ángulo estrecho

• Inyección buibar y tarsal • Pupila normal

Tinción con Fiuoresceína

• Inyección ciliar o mixta • Pupila normal

Inyección ciliar Pupila miótica Posible hipopión No alí. corneales

263

www.FreeLibros.me

ULTRA-RESÚMENES Manual CTO de Medicina y Cirugía, 8.a edición DIAGNÓSTICO DIFERENCIAL DE LAS PRINCIPALES CAUSAS DE PÉRDIDA DE AGUDEZA VISUAL. - Ametropías (mejora la visión con agujero estenopeico*). - Medios opacos (leucocoriasy trasilum inación alterada). - Alteraciones retinocoroideas (exploración oftalm oscópica alterada). - Lesión de vía óptica (alteraciones en la cam pim etna). - Ambliopía (disminución de agudeza visual sin alteraciones orgánicas detectables). * El estenopeico es un pequeño orificio empleado para explorar la agudeza visual, pues consigue dism inuir la aberra­ ción óptica que producen los rayos que atraviesan la pupila por su parte periférica.

CAPÍTULO 6.

CONJUNTIVA.

DIAGNÓSTICO DIFERENCIAL DE LAS CONJUNTIVITIS.

SIGNO

BACTERIANA

VIRAL

ALÉRGICA

Inyección

Marcada

Moderada

Ligera

Exudado

Purulento o mucopurulento

Mucopurulenta o mucinosa

Acuosa

Papilas

A veces

No



Folículos

No



No

Adenopatía preauricular

Raro



No

1. ¿Con qué relacionas a la conjuntivitis papilar y a la conjuntivitis flictenular?

í.

Con el uso de lentillas y con hipersensibilidad a antígenos del S. aureus, respectivamente.

2. ¿Cuál es el germen causal más frecuente de las conjuntivitis agudas bacterianas? ¿Y de las con­ juntivitis crónicas?

2.

En ambos casos el S. aureus.

3. ¿De qué tipo de conjuntivitis son típicas las papilas?

3.

De las alérgicas.

264

www.FreeLibros.me

O CAPÍTULO 2.

f t a l m o l o g ía

REFRACCIÓN.________________

AMETROPÍAS. CARACTERÍSTICAS Y TRATAMIENTO.

Esféricas

No esféricas

(curvatura del dioptrio uniforme)

(curvatura de las superficies no uniforme)

J Hipermetropía

Miopía

Globo corto o dioptrio

Globo largo o dioptrio

No hay punto de imagen

poco convergente

muy convergente

sino dos imágenes lineales

(imagen tras la retina)

(imagen delante de la retina)

Astigmatismo

Corrección con

Corrección con

Corrección con

lentes convexas

lentes cóncavas

lentes cilindricas

(convergentes)

(divergentes)

1.

Un paciente miope de 6 dioptrías, con cuaren­ ta y seis años de edad, se queja de astenopía durante las horas de trabajo. Se valora su pres­ bicia en + 1,25 dioptrías. ¿Cual será tu prescripción?

1.

Visión lejana: -6 dioptrías, visión próxima: -4,75 dioptrías. Se calcula sumando algebraicamente para la visión cercana, sin que haya modifica­ ción en la visión de lejos.

2. Un enfermo de 70 años desarrolla cataratas. An­ teriormente sufría presbicia. ¿Cómo se modifi­ cará esta?

2. La catarata miopiza (aumento del índice de re­

3. Un niño de 2 años tiene un estrabismo conver­ gente. ¿Qué es lo primero que debes descartar?

3.

Una hipermetropía. Un hiperm étrope compen­ sa su defecto de refracción forzando la acomo­ dación. El reflejo de acomodación no aparece aislado, sino que lo hace asociado a la contrac­ ción de los músculos rectos internos (conver­ gencia) y del músculo esfínter de la pupila (miosis). Eso explica que al niño se "fe vaya el ojo hacia adentro “ cada vez que lee. En el caso de la miopía podrá aparecer estra­ bismo divergente.

¿Por qué todo niño con estrabismo debe ser valorado por un oftalmólogo?

4.

Por varías razones, para evitar el desarrollo de ambliopía, d etecta r un defecto de refracción (hipermetropía) y descartar un posible retinoblastoma (recuerda que esta es, tras la leucocoria, la forma de debut más frecuente).

4.

fracción del cristalino y engrosamiento de este). Por eso puede debutar como una m ejora de la presbicia.

www.FreeLibros.me

265

ULTRA-RESÚM ENES Manual CTO de Medicina y Cirugía, 8.a edición -H

5. Relaciona los siguientes cuadros con la ametropia asociada: 1. Glaucoma crónico simple 2. Glaucoma de ángulo estrecho 3. Cataratas 4. Estrabismo divergente compensatorio 5. Estrabismo convergente compensatorio 6. Pseudopapilitis 7. Astenopia acomodativa 8. Degeneración de retina por elongación 9. Pterigion 10. Queratocono 6.

a. Miopía b. Hipermetropía c. Astigmatismo

Respuestas:

1-a; 2-b; 3-a; 4-a; 5-6; 6-6; 7-6; 8-a

9-c; 10-a y c.

Relaciona cada fármaco con su efecto secun­ dario: 1. Vitamina D. 2. Atropina. 3. Mióticos. 4. Latanoprost. 5. Corticoides. 6. Tamoxifen. 7. Cloroquina. 8. Amiodarona.

A. Catarata. B. Queratopatía. C. Hiperpigmentación de iris. D. Glaucoma crónico simple. E. Ataque agudo de glaucoma. F. Maculopatías. Respuestas: 1a,6; 2e; 3a; 4c; 5a, d; 6f; 7f; 8a,6.

www. FreeLibros. me

O

t o r r in o l a r in g o l o g ía

Consejos antes de repasar la ORL_________________________________________________________

-

En ella son m uy prácticas e importantes las tablas que diferencian y comparan distintas patologías, ya que es una especialidad muy sistem ática. Los cuadros clínicos y signos suelen corresponder a patologías concretas y e sp ecíficas, y tú debes saber distinguirlas (por ejem plo, un Plaut-Vincent de una neoplasia de amígdala, ambos unilaterales ; les cuadros inespecíficos o genéricos han sido m uy poco preguntados. Pon m ucha atención en los datos-clave, y en razonar la semiología. Por último, ten en cuenta que es difícil asegurar todas las preguntas (entre otras cosas, porque aquí no suelen salir de fuentes tan claras como lo es el Harrison en m uchas otras asignaturas), así que no te agobies y ve a lo esen cial.

CAPÍTULO 2.

OTOLOGÍA.

1. INTRODUCCIÓN. Repasa la anatomía y fisiología del oído en la página siguiente y responde a las siguientes preguntas:

1.

¿Cuál es la zona de Ramsay-Hunt? ¿Y el síndrome? ¿Y la neuralgia de Hunt?

1.

Pared posterosuperior de CAE y concha (nervio fa cia l). Herpes zo ster ótico. Irritación del ganglio geniculado, con dolor en te rrito rio s d ista le s: preauricular, CAE, suelo boca; y alteraciones gustativas.

2.

¿Qué estructuras baña la endolinfa?

2.

Laberinto membranoso (utrículo, sá c u lo ). con­ ducto coclear; es sim ilar al líquido endoceiular (rico en potasio) y drena en el saco endolinfático, que da al espa- ció epidural.

3.

Estructuras que intervienen en el mantenimiento del equilibrio.

3.

A feren tes desde vestíbulo, vías propioceptiva y visual y cerebelo.

4.

El huesecillo que se disloca más fácilm ente en los traumatismos e s ...

4.

Et yunque.

5.

Diferencias entre máculas y crestas amputares

5.

M áculas: - en utrículo y sáculo - d etecta aceleraciones lineales - se estimulan p o r variaciones de presión que ejercen los oto litos sobre las ce l. ciliadas. C re s ta s am putares:

- en cond. sem icirculares - detectan aceleraciones angulares - se estimulan por movimientos de la endolinfa que desplaza los estereocilios de las cel. ciliadas. 6.

-

-

La afectación del facial es constante (100%) en un tipo de fra c tu ra ...

6.

La oblicua, cuya clínica es una mezcla en tre la fra ctura transversal u longitudinal.

No olvides algunos epónim os (búscalos): nervio de Jacobson, m em brana de Reissner, ganglios de Scarpa, Gasser y espiral, nervio de W risberg... N. Jacobson: Conducto auditivo m edio (CAM ). Rama de IX par craneal. Glanglio Scarpo: Pares vestibular del VIII p.c. G. Spiral: Pars auditiva del VIII p.c. G . Gasser: V p.c. (porción sensitiva). N. Wrisberg: N. cuerda del tím pano (VIII). Gusto 2 3 anteriores. El fascículo longitudinal medial es la vía que conecta entre sí los núcleos oculomotores (III, IV, VI) y permite la mirada conjugada (es de gran im portancia; explica el nistagmo y se afecta en la esclerosis m últiple, dando oftalm oplejía internuclear y nistagmo disociado). Oftalmoplejía internuclear. Causa más frecuente: • Jóvenes: enferm edad desm ielinizante. • Ancianos: A C V A . • C lín ica com patible sin que realm ente exista: m icistenia gravis.

www.FreeLibros.me

267

O I D O IN T E R N O

O ID O EX T ER N O

c 'O y

Tj

CU

nj co en 3

u I\¡ c y T3

cu

cu TD O h~ u ~(V c (V - F u n ció n d e la T. d e E u s ta q u io :

IA

m Z m § O I/> m ££

Aireación y equilibrio de presiones. - ¿ R e c u e rd a s la fu n ció n del CAE?

Am plificador del sonido. - R e la c io n e s p o s te r io r e s d e l CA E.

- In e rv a c ió n :

Simpática: plexo carotídeo (V Sensorial y parasimpática: VII y IX

Mastoides y 3 a porción facial.

- Función d e la c ó c le a .

Interpreta intensidad y frecuencia del estím ulo sonoro. - ¿ Q u ie n irrig a e l o íd o in te rn o ?

Ramas de la arteria cerebelosa anteroinferior (AICA)

V Y= Vena yugular FCM = Fosa craneal media aa= aditus ad antrum A TM = Articulación temporomand¡bular.

V R = Ventana redonda V O = Ventana oval C .l.= Carótida interna C .E .= Carótida externa

SE= Saco endolinfálico C .A m = Crestas ampulares RV= Rampa vestibular RT= Rampa timpánica

co co

OJ

www.FreeLibros.me

O

t o r r in o l a r in g o l o g ía

Long

Transv

Frecuencia

70 - 80%

15 - 20%

Perforación tim pánica

Frecuente

Rara

Otorragia

Frecuente

Rara

Raro

Frecuente

Transmisión

Perceptiva (neurosensorial)

20% transitoria (neuroapraxia)

50% perm anente (neurotmesis)

Raro, leve y transitorio

Frecuente y severo

Hem otím pano

Hipoacusia

Parálisis facial

Vértigo

Dx

TC

Proyección en radiografía Recorrido Escalón en paredes del CAE

shüLLer

s Tenvers

CAE y OM

CAI



\o

Pruebas funcionales de la audición: (muy importantes). Rinne: monoaural, com para vía aérea y vía ósea de un mismo oído. W eber: biaural, comparando la vía ósea de am bos oídos. Schw abach: compara vía ósea del paciente con la del explorador.

© O

: m ejor vía aérea. : m ejor v ía ósea.

Rinne

W eber

N o rm a l

+

In d ife re n te

T ra n sm isió n

-

Laclo e n fe rm o

A la rg a d o

N e u ro s e n s o ria l

+

Laclo sa n o

A c o rta d o

U n ila te ra l

-

Laclo sa n o

A c o rta d o

B ila te ra l

-

In d ife re n te

A c o rta d o

Schwabach Igual a e x p lo ra d o r

C ó fo sis*

* Cófosis: hipoacusia neurosensorial profunda

Hipoatusias. Hipoacusia neurosensorial

H ipoacusia de conducción

A u d io m e tría to n a l

A u d io m e tría v e rb a l

■C r a p ó se o -c é re o

■C a e n a m b a s v ía s en to d o s A G U D O S

■T o n o s G R A V E S

■M E M É R E : to n o s G R A V E S

In te lig ib ilid a d ig u al, pero a u m e n ta el u m b ra l

In te lig ib ilid a d peor.

www.FreeLibros.me

269

ULTRA-RESÚMENES, Manual CTO de Medicina y Cirugía, 8.a edición >



B M -:

G e l lé p a t o ló g ic o N O RM A L,

J

R . e s t a p e d ia l (-)

O T O S C L E R O S IS

'

P a r a c u s ia s E sco to m a d e C a rh a rt.

A U D IO M E T R IA S U P R A L IM IN A R A m á s i n t e n s id a d b a ja in t e lig ib ilid a d

^

R e clu ta m ie n to :

O T O T U B A R IT IS

c o c l e a r e s

(Fowler. SISI)

]

(S e a lt e r a in t e n s id a d )

e n A .V . (R o ll o v e r ). M é n i é r e : i n i c i o e n g ra v e s y f lu c t ú a . M e t z ( + )• D i s c r e p a n c i a t o n a l- v e r b a l

D IS R U P C IO N O S IC U L A R

^Fatiga/adaptación: 1RETfcOC-QCL EARES (D e c a y , B e k e s y )

(S e a lt e r a s a tu r a c ió n !* !

( p e o r la V e r b a l). R. e s t a p e d ia l ( - ) . P o t e n c ia le s e v o c a d o s a lt e r a d o s . S ig n o H it s e lb e r g e r .

O T IT IS S E R O S A S , o to e s c le r o s is . L a s c u r v a s in d ic a n d if e r e n t e s t ip o s d e t im p a n o m e t r ía s

1.

¿Recuerdas que utilidad tenia la exploración del reflejo estapedial?

1.

- Valoración del OM, e j . : ausente en otosclerosis. - Diagnóstico topográfico de parálisis facial.

Reglas mnemotécnicas 1.

En la C O N D U C C IÓ N , el R IESG O es N EG A TIV O y si W EB ES te vas al lado enf. Hipocusia Rinne O W eber enf. conducción

2. Los accidentes de C O N D U C C IO N producen lesiones GRAVES. H. conducción Frecuencias graves 3. Estás tan FA TIG A D O que te deja ATRÁS hasta un C A R A C O L. Sordera retro - coclear Fatiga---------------► Vértigo central

Vértigo periférico L e n to

Ataxia eerebelosa V a r ia b le

Ataxia sensitiva V a r ia b le

In ic io

B ru sco

D u ra c ió n

C o rt a

V a r ia b le

V a r ia b le

V a r ia b le

E v o lu c ió n

E p is ó d ic a

P ro g re s iv a

P ro g re s iv a

P ro g re s iv a

S e n s a c ió n d e g iro



N o . M a re o e in e s ta b ilid a d

N o . In e s t a b ilid a d y

N o . In s e g u rid a d e n la m a r c h a

m a l d e fin id a

d e s c o o r d in a c ió n

N e u ro ló g ic o s

I N C O O R D I N A C IÓ N :

• C e r r a r lo s o jo s : e m p e o r a .

d is m e t r ía , d is a r t r ía ,

• A u m e n t o b a s e s u s t e n t a c ió n .

d is d ia d o c o c in e s ia ...

• P a to lo g ía c a r e n c ia l B 1 2 .

d e o b je t o s P o s ib le s s ín t o m a s

H ip o a c u s ia . A c ú fe n o s

a s o c ia d o s

• N e u r o p a t ía p e rifé ric a

+

+

R o m b e rg

+

C a íd a fija

C a íd a v a r ia b le

H o r iz o n t o - r o t a to r io

• N o h o r iz o n te - r o t a t o r io

• N o h o r iz o n te ro t a t o rio

U n id ir e c c io n a l y e n re c o rte

• V a r ia b le

• V a r ia b le

In h ib e

E m p e o ra

E m p e o ra

C a fa s F re n z e l

E m p e o ra

In h ib e

In h ib e

R e la c ió n

A rm ó n ic a

D is a r m ó n ic a

D is a rm ó n ic a

C a r a c t e r ís t ic a s

C a íd a v a r ia b le NO

C o m p o n e n t e r á p id o a l la d o s a n o N

I S T A G H

F ija c ió n o c u la r

-

d e s v ia c ió n c u e rp o

O R e la c ió n

P r o p o r c io n a d a

D e s p r o p o r c io n a d a

D e s p r o p o r c io n a d a

-

in t e n s id a d s in to m a to lo g ía E tio lo g ía

VPPB

Ic tu s v e r t e b r o b a s ila r

Ic tu s c e r e b e lo s o

• N e u r o p a t ía p e rif.

N e u r o n it is v e s tib .

EM

F e n it o ín a

• Tabes

M e n i e re

T u m o re s

P o s t- in fe c c io s a ( W Z )

• D e g e n e r a c ió n c o m b in a d a

T r a u m a tis m o s

E p ile p s ia

A lc o h o l

N e u r in o m a V III

M ig r a ñ a

M e d u lo b a s t o m a

270

www.FreeLibros.me

s u b a g u d a m é d u la . • L e s ió n t a lá m ic a

O

t o r r in o l a r in g o l o g ía

En general, el oído afecto es un oído hipodom inante, y esto explica la lateropulsión en las pruebas al lado enfermo (Romberg, Untemberger, m archa...) y la fase lenta del nistagmo (la rápida-compensatoria va al lado sano). Una excepción que ha caído en el M IR es la laberintitis serosa que genera un nistagmo irritativo. En una persona normal, cuando realizamos la prueba calórica provocamos un vértigo periférico (cuyo nistagmo hu\ e del frío"). En los pacientes con patologías causantes de vértigo periférico, en el oído afectado, casi siempre hipodominante, la pruébase cataloga de hiporrefléxica. Esto no ocurre en el VPPB, dado que no es un oído con hipofunción, sino una litiasis .

2. PATOLOGÍA DEL OÍDO EXTERNO. Factores de riesgo Baño en piscina, uso de bastoncillos, cuerpo extraño, diabetes mellitus.

Clínica OTALGIA (aumenta con masticación, signo trago ®). Otorrea. Sensación de taponamiento. Prurito.

Otoscopía

Hitas negruzcas o blanquecinas

Edema del CAE,

Reagudizaciones

secreción

Otorrea persistente

serosa-purulenta

Tejido de granulación

Forúnculo en CAE

A s p e r g illu s niger,

Dermatitis seborreica

fu m ig a tu s

característico

P. aeruginosa

S. aureus

NO MOJAR Cotas ciprofloxacino o genta-dexa

Cuidado.- locales,

P. aeruginosa, en

C a n d id a

Limpiar y aspirar, alcohol boricado, antimicóticos tópicos

OTOMICOSIS

Afecta a pares craneales

CAE eccematoso

Corticoides tópicos N O MOJAR

rara \ e z : c io rc a o xa c '

doxa oral \ drenaje

vo

OTITIS EXT. AGUDA (Difusa)

O TITIS CRONICA (Externa eccematosa)

NO TO C A R

O T I T IS EXT. L O C A L IZ A D A

xarones diabéticos •>C ¡s ‘ ‘ Ca 6 C iprofloxacino i.v. o ro b ra -

c a r b e n ic ili n a

OTITIS EXT. MALIGNA

OEB bacteriana

O E F (fú n g ica)

Diagnóstico diferencial rápido

OEB

OEF

ECCE

ONU

O ta lg ia

+

-

*

-

P ru rito

-

+

+

-

O to rre a

+

+

-

H ip o a s u b ia

+

+

-

- - tras p e rfo ra ció n

+

271 www.FreeLibros.me

ULTRA-RESÚMENES. Manual CTO de Medicina y Cirugía, 8.a edición

Una colección hem ática subpericóndrica, tras un traumatismo en el pabellón, ¿qué tratam ien­ to necesita?

1.

Desbridar y drenar + antibióticos locales (antipseudomonas) y generales por riesgo d e necro­ sis del cartílago y pericondritis.

2.

¿Qué tipo de otitis causa el virus de la gripe?

2.

M iringitis ampollar hemorrágica.

3.

La causa más frecuente de pericondritis, otitis externa difusa aguda y de otitis externa malig­ na e s ...

3.

Pseudomonas aeruginosa.

4.

La miringitis bullosa es producida por...

4.

Mycoplasma pneumoniae.

5.

El tumor maligno más frecuente del oído exte r­ no e s ...

5.

El carcinoma epiderm oide.

6.

Ante un cuadro de p arálisis fa c ia l, otalgia, vértigo e hipoacusia, junto con la aparición de lesiones cutáneas en concha auricular, pensa­ ríamos e n ...

6.

Zóster ótico o síndrome de Ramsay-Hunt.

3.

OÍDO MEDIO.

1

.

Recuerda que la hipoacusia ele conducción es un signo localizador de cierto valor, pero no identifica n i caracteriza a una patología concreta del oído medio. Además, algunas afecciones del O M en ciertas ocasiones dan cierta hipoa­ cusia de percepción (por ejem plo, colesteatoma, otosclerosis). Los temas más preguntados han sido la otosclerosis y las otitis (O M A y O M C ).

OM A

Etilogía

S. p n e u m o n ia e , H . in flu e n z a e M o ra x e lle c a ta rrh a lis - Fase d e c o le c c ió n : o talg ia + M E G +

Clínica

tím p a n o a b o m b a d o . - Fase d e o to rre a : p e rfo ra ció n + m e jo ría

P s e u d o m o n a s d , S. a u re u s

- O to rre a c ró n ic a o re c id iv a n te . - S IN o talg ia. - H ip o a c u s ia .

■O M A d el la c ta n te :

O M C c o le ste a to m a to sa :

- P re d o m in a s in to m a to lo g ía g e n e ra l.

- O to rre a fé tid a p e rsiste n te .

- Sign o d el trag o + .

- Sign o d e fístu la (c lín ic a ve rtig in o sa ).

- T ra ta m ie n to v ía in tra v e n o s a .

- P e rfo ració n a tic a l y m arg in al.

■O M A n e c ro tiz a n te : - En ID o e n fe rm e d a d e s sisté m ica s.

T im p a n o s c le ro s is o h ip o a c u s ia d e tra n sm isió n

- C o m p lic a c io n e s p o r o ste ó lisis. A m o x i.c la v u lá n ic o v.o. Si O M A re c u rre n te s: p ro fila x is d e m a n te n im ie n to T ra ta m ie n to

;

- R x : m a sto id e s e b ú rn e a (o p aca)

c lín ic a .

F o rm a s especiales

OMC

A M O X (b a ja d osis) +

T im p a n o p la s tia co n m a sto id e c to m ía

a d e n o id e c to m ía + m irin g o to m ía co n d re n a je s

272

www.FreeLibros.me

'

O

t o r r in o l a r in g o l o g ía

COMPLICACIONES DE OMA y OMC Infección que dure > 2 semanas. © IN T R A T E M P O R A L Mastoiditis • Com plicación más frecuente de la O M A . • 2 fases 1. Mastoiditis aguda coalescente: M EG , fiebre, dolor edem a retroauricular, opacidad radiológica de la celda mastoidea. 2. Mastoiditis exteriorizada (absceso subperióstico). • Tto: ceftriaxona i.v. + cirugía. Petrositis • Sd de Gradenigo: otorrea, neuralgia del trigém ino, paresia del VI pe. • Tto: atb iv + mastoidectomía am pliado. Laberintitis difusa. • Serosa: reversible. Hipoacusia neurosensorial, vértigo espontáneo y nistagmo hacia el lado enfermo. • Purulenta: irreversible. Cófosis, vértigo y nistagmo hacia el lado sano. Laberintitis circunscrita. • Fístula del conducto sem icircular (frecuentem ente el lateral) secundario a colesteatoma. • Vértigo. Parálisis facial. • Niños: secundario a O M A . Instauración aguda. Tto: atb y miringotomía. • Adultos: secundario a O M C colesteatomatosa. Instauración lenta. Tto: atb y cirugía. © IN TR A CR A N EA LES. Por orden de frecuencia: • Meningitis otógena (S. pneum oniae). • Abscesos epidural, subdural y cerebral. • Tromboflebitis de seno sigmoide.

Características principales de la otosclerosis. C lín ic a : -— - 9 2 0 - 3 0 a ñ o s c o n h ip o a c u s ia d e t r a n s m is ió n p r o g r e s iv a — a c ú fe n o s ; b ila t e r a le s e n 8 0 % . '

? 2:1

- N o o ta lg ia ni o to r r e a . - P a r a c u s ia s d e W illis y d e W e b e r .

Empeora con el em barazo Autosóm ica dom inante

’v -7 - H ip o a c u s ia p e r c e p t iv a a s o c ia d a e n 1 0 % p o r -

afectación c o c le a r.

- Tratamiento: ■ Estapedectomía c .

D iagnóstico d iferencial - E n fe r m e d a d d e P a g e t. - O s t e o g é n e s is im p e r f e c t a .

Exploración:

- O s t e o p e t ro s is .

- R in n e 0 y W e b e r la te r a l iz a d o a l o íd o e n f e r m o _

- H ip o a c u s ia h e r e d it a r ia . - O t it is a d h e s iv a c r ó n ic a .

- M u e s c a d e C a r h a r t (c a íd a v ía ó s e a 2 0 0 0 H z )

- Dislocación cadena osicular-

- R e f le jo e s t a p e d ia l a u s e n t e

1.

Ante la presencia de un pólipo en CAE proce­ dente del ático, siempre se debe descartar...

1.

Colesteatoma.

2.

La causa más frecuente de hipoacusia en niños menores de 6 años e s ...

2.

O titis media serosa.

3.

El paraganglioma es el tumor más frecuente del oído medio, ¿recuerdas sus características más importantes?

3.

- Tumor benigno muy vasculanzado. ■Clínica según localización: timpánico (acúfeno pulsátil) o yugular (afect. de pares craneales). - Diagnóstico por RM + arteriografia. - Tratamiento quirúgico previa embolización. - Descartar funcionalidad (catecolaminas en orina 24 h)

273

www.FreeLibros.me

ULTRA-RESÚMENES. Manual CTO de Medicina y Cirugía, 8.a edición A 4. O ID O INTERNO. Se puede decir que se afecta el oído interno cuando existe hipoacusia neurosensorial, vértigo periférico o ambos debes saber ya como identificarlos). Intenta llegar tú al diagnóstico tapando la colum na correspondiente en el cuadro. Al estudiar la tabla de la página siguiente, detente en 3 detalles: Hay cierta analogía en la conm oción laberíntica y en el V P P B , ya que existe en ambos un com ponente traumático (menos claro en VP P B) y canalitiasis que explica parte de los síntomas. Hay otros dos cuadros algo parecidos tam bién, uno que afecta al vestíbulo y el otro ala cóclea, que tienen patogenia poco clara (¿virus?, ¿m icrocirculación?). Son la neuronitis vestibular y la hipoacusia súbita, y en ambos se observa con frecuencia el antecedente de una infección (respiratoria). En la sordera súbita puede haber vértigo. En la neuronitis vestibular no hay sordera.

1.

¿Recuerdas cuáles son los dos fármacos cuya ototoxicidad tiene un predominio vestibular?

1.

Estreptom icina y gentamicina.

¿Cuál es la causa más frecuente de vértigo peri­ férico?

2.

Vértigo posicional paro xistico benigno (VPPB).

DATO CLAVE

Coclear unilateral en agudos.

H. n eu ro s e n so ria l +

Escotoma a 4.0 0 0 Hz. (coclear). Coclear bilateral, inicio en agudos, progresiva, simétrica.

DIAGNÓSTICO

H ip o a cu sia sú b ita .

Idiopátíca lo más frecuente

P re sb ia c u sia

> 50 años.

(unilaterales).

A m bas +

- Crisis paroxísticas de vértigo. - Acúfenos intensos. - H. coclear. - 20% bilaterales. - 30-50 años.

O titis media que se complica.

S ín d ro m e

Crisis de vértigo por movimientos cefálicos de extensión y giro.

N e u rin o m a acú stico .

E n ferm e d ad d e M é n ié r e {h id ro p s en d o lin fá tico id io pático ).

Crisis repetidas de horas.

L ab erintitis.

VPPB

\ertiginoso periférico + Crisis única de vértigo de varios días.

Acúfeno intenso (70%) y sensación de presión. Alteración del equilibrio 40%.

T rau m a a cú stic o . Antecedentes; Acúfenos + hipoacusia. acúfenos (agudo) profesión; Reclutamiento positivo. progresiva (crónico).

- H. retrococlear ± inestabilidad, acúfenos - Síntomas de vecindad (compresión V, VII (signo de Hitselberg) y pares bajos).

OTROS/CONFIRMAN

v estib u la r.

COMENTARIOSTRATAMIENTO - En niños frec antecedente de parotiditis. - Corticoides y vasodilatadores (precozmente). - No tratamiento (prevención laboral).

Peor la audiom etría verbal.

Esquizoacusía: comprensión mejor con monosílabos. Prótesis, labiolectura.

- Potenciales evocados PEATC (screening). - Reflejo estapedial (-). - RM con gadolinio (de elección) confirma (ramo vestibular). -A lt. intelígilibilídad (disociación tono-verbal).

Si bilateral: neurofibromatosís II. - Signos tardíos: hidrocefalia, compresión de tronco. - Tratamiento: cirugía. La localización más frecuente es intracanalícular a expensas del nervio vestibular.

- No otros síntomas neuro lógicos. - Se repite, dejando hipoacusia fluctuante para tonos graves.

- Reposo + antieméticos + antivértigo + dieta sin sal y diuréticos + vasodilatadores. - Quirúrgico si el vértigo es incapacitante y según audición (el paciente decide): • Buena: drenaje saco endolinf. o neurectomía vestibular. • M ala: laberintectomía.

Nistagmo al lado enfermo (!!!) en fase imitativa (serosa).

Al lado sano en fase destructiva (purulenta).

- Latencia. - Corta duración (min). - Agotabilidad. - Reversibilidad.

Canalitiasis / cupulolitíasis. Más frec. cond. semicircular post. Tto. = maniobra de liberación o de reubicación canalicular.

- Audición normal. - Dato único y típico la

- Frecuente antecedente de infección respiratoria alta. - Antívertigínosos.

www.FreeLibros.me

O

5.

t o r r in o l a r in g o l o g ía

N ERV IO FACIAL.

Respecto a este apartado, son tres los temas más preguntados: - Recorrido del facial para poder localizar el nivel de la lesión (ver el dibujo de la página siguiente). - Causas de parálisis facial: H Z ótico, O E maligna, O M C colesteatomatosa, O M A ... ET IO LO G ÍA PARÁLISIS FACIAL. Por frecuencia. 1 .Idiopática, de Bell o «a t'igore». 2 .Traumáticas. 3.Ram say Hunt o Zóster ótico. • V V Z en ganglio geniculado. • Otalgia intensa y vesículas en pabellón auricular (zona de Ramsay Hunt). • Mal pronóstico. • Tto: aciclovir. 4.Tum oral. • Instauración progresiva. 5 .0 titica . 6 .Sd de Melkerson - Rosenthal. • Brotes recurrentes de parálisis facial con edem a hemifacial y lengua geográfica. 7.Sd de Heerfordt (fiebre ureoparotídea). • Parálisis facial + ureítis anterior + parotiditis + fiebre. Diagnóstico diferencial entre parálisis central/périferica (en la p.supranuclear se respeta la musculatura frontal

1.

Una parálisis facial con test de Schirmer normal se lo caliza...

1.

D ista l a l g a n g lio g e n icu la d o .

2.

Una alteración aislada de toda la musculatura facial ind ica...

2.

L e sió n d ista l a l o r ific io e s tilo m a s to ia e o .

3.

Si hay parálisis de musculatura de la cara, con movimientos de la frente, lacrimación, gusto y sensibilidad intactos e s ...

3.

Una p a rá lis is fa c ia l c e n tr a l.

4.

¿Cómo es la inervación de la zona dorsal del nú­ cleo motor superior del facial?

4.

Bilateral.

,

Recorrido del nervio facial. C o n d u c to a u d itiv o in tern o

G a n g lio g e n icu la d o (1 s co d o )

Glándulas lacrimales 22

codo Glándulas

salivales

Orificio estilomastoideo R EG LA M N E M O T E C N IC A El nervio facial es Lacrimal Estribo Lengua O re ja (Ramsay Hunt)

Gusto

Area de Ramsay-Hunt

Musculatura facial

275

www.FreeLibros.me

ULTRA-RESÚMENES. Manual CTO de Medicina y Cirugía, 8.a edición A Parálisis de B ell (a t'rigore o idiopática): Es más frecuente en hipertensos, diabéticos y al final del embarazo. Es de inicio súbito y el 80% tienen buen pronóstico (neuroapraxia). ¿Herpesvirus, CM V? ¿Alteraciones m icrocirculación?¿Edem a? Damos corticoides y aciclovir (en los zoster óticos) junto con protección ocular, y si en existe degeneración axonal significativa: descompresión ¡ntratemporal.

CAPITULO 6.

6

semanas no mejora y/o

PATOLOGIA DE LA LARINGE.

1. GENERALIDADES.

1.

Función del cricotiroideo (anticus).

2.

¿Quién inerva sensitivamente la subglotis?

3.

1.

Tensa la glotis (también el m. vocalis), y es el único inervado p o r el laríngeo superior (rama externa).

2.

El recurrente (recuerda que glotis y supraglotis por el laríngeo superior).

3.

La glotis carece de linfáticos (im portante, ya que el pronóstico del cáncer en esta localiza­ ción es m ejor).

¿Dónde drenan los linfáticos de la glotis?

4.

A n o m a lía co n g é n ita laringología...

en

4.

Laringomatacia (75%), luego la parálisis recurrencial. N o re q u ie re tto.

5.

¿Quién inerva todo el paladar, salvo el tensor del velo?

5.

El vago; al tensor del velo lo inerva V3.

.

¿Recuerdas qué estructuras atravesaban el trián­ gulo de los escalenos?

6.

El triángulo de los escalenos lo formaban los músculos escaleno anterior y medio y lo cruza­ ban el plexo braquial y la arteria subclavia. No olvides que la vena subclavia es anterior al es­ caleno anterior, y que el nervio frénico descien­ de p or la superficie externa de este músculo.

7.

Único músculo que dilata la glotis.

7.

El posticus (cricoaritenoideo p o ste rio r). E ste concepto está puesto en duda.

8.

Epitelio que existe en la laringe.

8.

E p ite lio resp ira to rio (cilindrico pseudoestratific d a d o ) e x ce p to cu erd a s v o c a le s: plano poliestratificado no queratinizante.

9.

En la traqueotomia de urgencia se hace la inci­ sión a nivel d e ... ¿Y en la traqueotomia reglada?

9. Ambas entre el 2 o y 3er anillos traqueales (sólo se hace en membrana cricotiroidea (coniotomía) ante cuadros de extrem a urgencia, por­ que las estenosis posteriores son la regla).

10. La intubación endotraqueal NO se recomienda prolongarla más d e ...

10. 48 a 72 horas, por el peligro de estenosis tra­ queal (hacemos traqueotomia después).

11. Complicación más frecuente de la traqueotomia.

11. La hemorragia (precoz); estenosis (tardía).

12. Si sospechas un cuerpo extraño laríngeo, ¿qué

12. Radiografía en inspiración y espiración lo p ri­ mero, y si es posible, laringoscopio.

6

harás?

más

fre c u e n te

276

www.FreeLibros.me

O

t o r r in o l a r in g o l o g ía

2. CÁNCER LARÍNGEO. En cierto sentido, tiene analogías con los de oro e hipofaringe: también se da en varones fumadores (algo más jóvenes), pero suele presentarse precozmente con disfonía, por lo que el pronóstico es francam ente mejor (unido a que la zona es quirúrgicamente "agradecida"). En nuestro entorno, el pronóstico es algo peor que en países anglosajones, donde el glótico es más frecuente (con más precocidad diagnóstica y menos ganglios afectados). Del tratam iento, recuerda que siempre haremos el vaciamiento cervical profiláctico, excepto en T y T2 glóticos. Sesuele recomendar profilaxis postquirúrgica con betacarotenos o ácido retinoico, ya q u e cada año aparecerán nuevas neoplasias en un 3%. La laringitis crónica hiperplásica, con o sin queratosis, y los papilomas laríngeos del adulto son lesiones premalignas. Es fundamental el estadiaje, para dar el tratamiento más conservador posible (por ejem plo, laringuectomía horizontal supraglótica en tumores pequeños de supraglotis).Tto: cirugía +/- vaciam iento cervical +/- Rt. No: profiláctico N + : funcional o radical. Los estadiajes T N M en O R L han caído en el M IR recientem ente, aunque son muy difíciles de retener. Intenta al menos relacionar algunas cosas de u n vistazo rápido: - Por ejem plo, T corresponde en general a afectación de 2 regiones. - T 3 se asocia a la palabra "fijación" o "desborda". - T 4 es sinónimo de afectación de órganos de la vecindad (piel, hueso,...). - La dem ostración de afectación de ganglios (N ) nos obliga a considerar el tumor avanzado (estadio III o mavor). 3.

PARÁLISIS FARINGO-LARÍNGEAS.

Laríngeo superior

P a r á lis is

supranucleares

■Aspiraciones, voz d é b i. • Posición normal con acortan ento

Posición paramediana

Ram a interna

Posición paramediana.

Aspiraciones y disfonía (voz aérea).

Gran riesgo de aspiraciones y disfonía {voz aérea).

Posición intermedia.

Posición intermedia.

Posición normal fonatoria

Posición normal respiratoria

Ra m a extern a

Ram as traqueales

N . re c u rre n te

1. ¿Qué clínica esperarías encontrar en un pacien­ te con lesión de la rama motora del nervio laríngeo superior?

1.

Fatiga de la voz, exclusivamente.

2. ¿Recuerdas cuáles son las causas más frecuen­ tes de parálisis faringolaringeas?

2. Estrumectomía, carcinomas de tiroides, esófago o pulmón, aneurismas aórticos, cuerpos extra ­ ños...

277

www.FreeLibros.me

ULTRA-RESÚMENES. Manual CTO de Medicina y Cirugía, 8.a edición :H 4. LESIONES BENIGNAS. LARINGOSCOPIA

PRESENTACIÓN

DIAGNÓSTICO

COMENTARIOS

Profesora, cantante con disfonía.

Nodulos bilaterales en unión tercios ant./medio.

N o d u lo s v o c a le s .

Foniatría / microcirugía. Formación benigna más frecuente en m ujeres.

Varón de edad media con disfonía.

Lesión única en borde libre anterior. Unilateral localizado.

P ó lip o c u e r d a .

Formación benigna más frecuente en general y en varones. Tratamiento microquirúrgico.

Fumador empedernido con diplofonía, voz grave, estridor

Bilateral, toda la cuerda edematosa entre epitelio y ligamento vocal. Si es unilateral: descartar tumor ventricular.

E d e m a d e R ein k e .

Decorticación secuencial.

Habla dolorosa y tos en un postintubado.

Granulom as bilaterales tercio posterior.

L e sió n p o r in tu b a ció n .

Evitar intubaciones >72 h. Microcirugía.

Habla dolorosa.

Ulcera en un lado y dureza contralateral reactiva, tercio posterior.

U lce ra /p a q u id e rm ia d e c o n ta cto .

Edema laringe (zona post); monocorditis de aspecto ulceroinfiltrante.

TBC.

Disfonía P a p ilo m a .

Masa exofítica. Hombres.

(HPV) Disnea / disfonía

Por sobrecarga fonatoria. Foniatría / microcirugía. Hacer Dx diferencial con cáncer (biopsia). La T B C siempre conserva la m ovilidad de la cuerda. Premaligno en adultos. Frecuentes recidivas. Tto. con láser de C O r

_

Recuerda que la disfonía es un síntoma constante, puesto que las cuerdas están afectadas. El tratamiento de todas estas situaciones es la m icrocirugía, salvo en los nodulos vocales, en que "reeducamos" la voz y en la TB C , que se com bina con tuberculostáticos.

1.

¿Recuerdas alguna otra lesión laríngea premaligna?

1.

-L a rin g itis cró n ica , e sp e cia lm e n te e l tip o queratósico. -Papiloma laríngeo del adulto.

2.

¿Qué no se te puede olvidar hacer ante una disfonia de más de 15 dias de evolución?

2.

Laringoscopia indirecta para descartar un carcinoma.

5. PROCESOS QUE CURSAN CON DISNEA EN LA INFANCIA. La disnea con estridor es el principal síntom a de las laringitis en el N IÑ O . En el A D U LT O , los procesos infecciosos se caracterizan por la disfonía. - Superior: epiglotitis, D x con Rx lateral cervical para partes blandas. LA R IN G O S C O P IA C O N T R A IN D IC A D A . criterio de U C I, cuya causa es H.influenzae b, pero m ucho más rara y leve que en el niño.

- Inferior: laringitis aguda catarral, de causa viral o, con menos frecuencia, estreptococo o neum ococo, y cuyo tratamiento se hace com binando reposo fonatorio, evitar los predisponentes -muy importante-, AINEs y, a veces, antibióticos. Diagnóstico con laringoscopia indirecta. Las laringitis crónicas deben controlarse periódicam ente por el riesgo de malignización. Estridor insp y esp: afectación traqueal.

278

www.FreeLibros.me

O

t o r r in o l a r in g o l o g ía

Patología laríngea en el niño. Cualquier edad con cuadro tóxico

Estridor inspiratorio en un neonato

^ y disnea progresiva por pseudomembranas

L A R IN G O M A L A C IA

C R U P D I F T É R I C O (rarísimo)

N IN O C O N D ISN EA

(D escartado cuerpo extraño)

M ás frecuente y leve

M ás raro y grave

M enor de 3 años. Cuadro progresivo de disnea disfonía, tos perruna; leve; deglute secreciones.

2 a 5 años. Dram ática instauración de Estridor insp. Sentado y babeando. Sensación de gravedad odinofagia, "voz de patata caliente".

▼ '■ ‘Cuerdas rojas y edematosas en laringoscopia.

Epiglotis rojo cereza (No tocar, posible espasmo) Rx lateral confirma.

R x la te ra l n o rm a l. L A R IN G IT IS S U B C L Ó T I C A A G U D A

E P IG L O T IT IS A C U D A

Virus paraíníkienzae 1

Streptococus, S. Aureus, I IiH ya no es tan importante

Tranquilizar, am biente húmedo, evitarso b reinf. con antibióticos; es raro intubar.

Cefalosporina 3a i.v . 4- corticoides hasta 2/3 se intuban Con aire frío y seco puede desencadenarse una crisis nocturna de espasmo laríngeo q u e c e d e so la . L. E S T R 1 D U L O S A

CAPÍTULO 5.

PATOLOGÍA DE LA FARINGE.

1. GENERALIDADES.

1.

¿Quién inervaba sensitiva y sensorialmente la len­ gua?

í.

Los dos tercios anteriores la cuerda del tímpa­ no (facial); el tercio p osterior el glosofaríngeo y en parte el vago.

2. La rinolalia abierta es típica de...

2.

La insuficiencia velopalatina (alteraciones en pares IX y X).

3. Un joven acude a Urgencias con escalofríos e hipertermia. Aqueja dolor cervical intenso en el lado derecho, pero no podemos conocer otros síntomas por la postración del enfermo. Se aprecia taquipnea y petequias en la piel, y el análisis de sangre revela gran leucocitosis y elevada VSG. Su madre nos dice que se acaba de recuperar de una faringoamigdalitis. ¿Qué te parece?

3.

Síndrome de Lem ierre o sepsis postanginosa (su causa más frecu en te es el absceso parafaríngeo, una complicación de las am igdalitis).

2. HIPERPLASIA ADENOIDEA. Sospecharla en un niño con rinolalia cerrada, respiración orai ■ por obstrucción nasal), ronquido nocturno y típica facies. Confirm a el diagnóstico la Rx lateral de cavum v la fibrorrinoscopia posterior. Indican la adenoidectomía: otitis serosa, infecciones nasosinusales o bronquiales de repetición, m aloclusión, apneas del sueño. La cirugía está contraindicada si existe insuficiencia v elopalatina.

3. HIPERPLASIA DE AMÍGDALAS PALATINAS ( anginas1). REPASA:

[I

n d ic a c io n e s

y

c o n t r a in d ic a c io n e s

d e c ir u g ía . T a b la 1 6 , C a p í t u l o s

d el

M

a n u a l

CTO

8 a E d .]

279

www.FreeLibros.me

ULTRA-RESÚMENES Manual CTO de Medicina y Cirugía, 8.a edición A 4. FARINGOAMIGDALITIS AGUDA. Es común a todas: odinofagia, disfagia y fiebre. A favor Obstrucción nasal con rinorrea. Fiebre no muy alta. O M A repetición, laringitis, sinusitis (si es vírica). Lo más frecuente es que sea vírica y su tratam iento es sintom ático. - Si es bacteriana, el estreptococo del grupo A es el protagonista (otras veces Arcanobacterium A favor Fiebre alta. MEG. Faringitis aislda. El cuadro es bastante llam ativo, con inflam ación im portante, adenopatías submandibulares y placas blanquecinas ("amigdalitis pultácea") no adheridas ni sangrantes, localizadas en criptas amigdalares y/o pared posterior de la faringe, acom pañado de M EG + fiebre alta. Su tratam iento es la inyección intram uscular de Penicilina-Benzatina en monodosis.

Complicaciones de las amigdalitis: Recuerda que las OMA-sinusitis agudas son las com plicaciones más frecuentes de una amigdalitis. El resto pode­ mos dividirlas en dos clases: - Lo ca les: aquí te repasamos el tto. de la más frecuente, el flemón periamigdalino: Se localiza entre el constrictor superior de la faringe y la mucosa. Pus = ABSCESO -» Incisión + drenaje + penicilina i.v. + corticoides

<

Inflamación difusa sin pus = FLEMÓN -» Penicilina i.v. + corticoides

-

Sistém icas: GN,' fiebre reumática y sepsis postangina. Esta última consiste en una tromboflebitis séptica de la vena yugular interna +/- embolismos sépticos en P U LM Ó N , hígado o piel; el tto. consiste en drenar el absceso + antibió­ ticos i.v. y, en ocasiones, ligadura de vena yugular.

Relaciona: 1.

Absceso en suelo de la boca.

a.

Fiebre tifoidea.

2.

Unilateral, con úlcera + exudadoblanquecino, en un gran fumador con BEG.

b.

Plaut-Vincent (asociación fusoespirilar, tipico de bocas sépticas).

Falsas membranas, gran afectación y frotis con

c.

D ifteria (klebsiella o neumococo dan pseudomembranas también).

3.

BGP. 4.

Participación de toda la boca y leucopenia en anciano con MEG.

d.

Herpes. Toda la boca (labios, paladar, lengua, mucosa yugal, amígdalas).

5.

Niño con gingivoestomatitis asociada y poca afec­ tación amigdalar.

e.

Mononucleosis (VEB).

f. 6.

Bilateral con vesículas.

Herpangina (Coxsackie A). Sólo veto del paladar y amígdalas.

7.

Anginas + adeno, hepato y esplenomegalia.

g.

Schuttze (agranutocítica).

8.

Úlcera superficial + estado tífico.

h.

Ludwig.

9.

Otras faringoamigdalitis.

R e sp u e sta s: 1-h, 2-b, 3-c, 4-g, 5-d, 6 f, 7-e,

280 www.FreeLibros.me

8 -a.

O

t o r r in o l a r in g o l o g ía

5. ONCOLOGÍA FARÍNGEA. En general, los tumores son más comunes en varones, y tienen mal pronóstico, porque cursan aslntomáticos bastante tiempo. Fíjate que, salvo el nasofaríngeo, que tiene algunas peculiaridades (en China y asociado a otros factores de riesgo), el resto de tumores son bastante similares en cuanto a patogenia, clínica, diagnóstico y tratamiento. Un tumor de hipofaringe en una mujer escandinava y de localización retrocricoidea debe hacernos pensar en la existencia de un Plummer-Vinson asociado (y con ferropenia). Los tumores de cavum se han preguntado bastante.

NASOFARINGE

OROFARINGE

Adenopatía cervical. 2 . Otitis serosa ( 2 5 % ). Epistaxis de repetición 3. Insuficiencia respiratoria en varón joven. nasal ± Epistaxis. 4. Afect. pares craneales: V?, V II, III, IV , V I.

HIPOFARINGE

LARINGE

1.

Síntoma presentación

Dx

Rinoscopia posterior. Angiografía. TC , RM. (NUNCA BIOPSIA).

Rinoscopia posterior + T C /R M .

A N G IO FIBRO M A . No adenopatías.

CA-CAVUM Ca. epidermoide indiferenciado (Schmincke).

A.P.

Regresa tras los 20 años de edad. - Su comportamiento es "maligno". - Recidiva a veces. - Aunque es raro es el tumor benigno más frecuente del cavum. -

Epidemiología

Tratamiento

Cirugía, previa embolización.

-

-

Grupo A . Relación con V EB, humos, salazones. En el Cantón chino x 100.

- H o m b r e s = m u je r e s . - N o r e la c ió n c o n t a b a c o o a lc o h o l.

RT

-

-

Q T).

S i p e r s is t e n a d e n o p a t ía s , v a c ia m ie n t o c e r v ic a l

P a r e s te s ia fa rín g ea

Disfagia Adenopatía cervical. Si amígdala: aumento unilateral o lesión ulcerada que no cura en > tres semanas.

Biopsia. En 20-30% desarrollarán una segunda neoplasia en otra localización.

Disfonía > 3 s e m a n a s (sobre todo lo s g l ó t i c o s . La tos es in f r e c u e n t e .

L a r in g o s c o p ia \ b iio p s a 1 0 % d e s a rro Ie r a n

lo c a l iz a c ió n .

CA. EPIDERM OIDE

T a b a c o y a lc o h o l.

qu ím icos. H PV

- A m íg d a la ( 2 a m á s fr e c u e n t e d e

cabeza \ cuello . - Luego base lengua. • Luego pa la d a r b lan d o , pared poste rio r. - C ir u g í a -

-

- RT

- SupragJófico* en latinos.

Seno piriform e. - Gíó-icos en anglosajones Luego pared posterior. raras las a d en o p a tía s v por ta m o , m ejor P \ Reírocricoideo en - Cáncer más frec d e Pl um m er-Vínson. ca b ez a \ cu ello.

QT.

L a R T a is la d a e s ig u a l d e e f ic a z e n T1

\0

a m ig d a la r e s o d e s e n o p ir if o r m e .

- v a c ia m ie n t o

c e r v ic a l , in c lu s o en lo s N 0

- G r u jí a con \ a c ia m ie n t o RT. - En c u e r d a \ o c a l - T 1 : R T \ cordectomía son igual de e f ic a c e s .

1.

¿En qué entidad pensarías ante un paciente de 60 años, con epistaxis de repetición, cuyo pa­ dre o hermano fallecieron por una hemorragia digestiva masiva?

1.

Enferm edad de Rendu-Osler o telangiectasia hemorrágica hereditaria.

2.

¿Cuáles son las únicos tumores que no precisan vaciamiento cervical de entrada?

2.

Ca. cavum, tumores glóticos Tt y T 2, labio in fe­ rior T, y T2 fo sa s nasales y senos paranasales.

3.

Aunque no pertenezca a este tema, ¿sabrías de­ cir cuáles son los factores de riesgo para pade­ cer cáncer de labio?

3.

Raza blanca; rayos UV. Sexo varón.

CAPÍTULO 3.

una

s e g u n d a n e o p ía s a e n o t

RINOLOGÍA.

1. GENERALIDADES. No olvides algunos puntos importantes:

Estructuras y orificios de la pared lateral nasal. -

M eato superior: aq uí drenan las celdas etmoidales posteriores. El seno esfenoidal drena algo más atrás. Meato medio: desembocan las celdas etmoidales anteriores y senos frontal y maxilar. Meato inferior: con el orificio del conducto lacrimomucosonasal.

Repasa la vascularización nasal y fíjate en que es m ixta: la carótida interna, a tra\és de la oftálmica, da las etmoidales, y la carótida externa, a través de la m axilar interna, da la esfenopalatina v palatina ascendente.

281

www.FreeLibros.me

ULTRA-RESÚMENES. Manual CTO de Medicina y Cirugía, 8.a edición Pasa algo parecido con el drenaje, por eso los forúnculos del vestíbulo y los abscesos del tabique pueden dar tromboflebit s del seno cavernoso (nunca m anipular estas lesiones). Los senos paranasales se desarrollan en el siguiente orden: eTmoidal >eSfenoidal > M axilar >Frontal (las letras en negrita van en orden inverso al alfabético).

1.

Cita las relaciones anatómicas de las fosas nasales.

1. Superior: fosa craneal anterior. In ferior: paladar. La tera l: senos y ap. lacrimal.

2.

Recuerda la triada clínica de la fx. del suelo de la órbita.

2. Enoftalm os + d ip lejia + alteraciones sensitivas.

2. SÍNDROMES NASOSINUSALES. Se pueden clasificar en 2 grupos: a) Urgencias: epistaxis y patología traumática. b) Cuadros caracterizados por obstrucción y rinorrea +/- dolor. Respecto a la epistaxis, es esencial conocer el protocolo de su tratamiento (por si no cede con las medidas usuales) y hay que tener en cuenta esos pocos casos infrecuentes pero posibles con presentación atípica (sangrado posterosuperior) y asociación a factores de riesgo serios, especialmente en el anciano. Y en cuanto al 2 - grupo (ver síndromes nasosinusales en la pág. siguiente), intenta fijarte en el síntoma predominante, la presentación aguda o crónica y el aspecto rinoscópico, ya que los pasos en el Dx son: historia/exploración —» rinoscopia y Rx —» TC y RM —» arteriografía y, en último lugar, biopsia.

3.

SINUSITIS.

Las agudas suelen ser monobacterianas (gérmenes iguales a O M A ; neumococo, H. inHuenzae,...) las crónicas son polimicrobianas (anaerobios, al igual que en sinusitis de origen dentario); en SIDA el hongo Pseudoalleschería (dar ketonazol).

-

Localización de la sinusitis: por radiografías y puntos dolorosos. En personas M ayores, el M axilar es el seno más afectado (luego etmoides, frontal y esfenoides), y en Enanos (niños), el Etmoides.

-

Tratamiento de la sinusitis crónica: punciones, antibióticos, y al final, cirugía endoscópica.

-

Complicaciones: orbitarias a partir del etmoides (las más frec.) e intracraneales (meningitis, absceso epidural).

1.

La rinorrea unilateral fétida y purulenta es indi­ cativa d e ...

1.

Cuerpo extraño nasal (niños), carcinoma epidermoide o papiloma invertido nasosinusal (adultos) y de un tipo poco frecuente de sinusitis, la odontógena.

2.

Si se ve lesión ósea radiológica puede se r...

2.

Nunca una sinusitis, más bien una neoplasia.

3.

La anosmia definitiva es complicación de un tipo de rinitis aguda...

3.

La gripal.

282

www.FreeLibros.me

Predomina Rinorrea

(< 4 semanas)

AGUDOS

Dolor

Predomina Obstrucción

SINDROMES NASOSINUSALES

CRONICOS

(> 3 meses)

Dolor variable

Predominan obstrucción y ri ñorreas periódicas. Veladura de senos en Rx y Rinoscopia -

Cefalea facial que empeora al agacharse junto a M E G (no fiebre).

Rinorrea unilateral + dolor a la palpación. V eladura d e sen o s y niveles h idro-aéreo s en la Rx

SIN U SIT IS C R Ó N IC A

S IN U SIT IS A G U D A

Cirugía

Vasoconstrictores. Anioxi-clavuláílico 10-14 d

i ASOCIACIONES:

.%í

B ro n q u ie c ta s ia s :

C ee

'COs

+ tap o nes ep id é rm ic o s en M O UN 1ER KU HN

P ic o r y rin o rre a m u y

joven, ju n t o a conjuntivitis y estornudos en salvas. acuosa en

( risis d e rin o rre a a cu o sa

Catarro autolimitado

Obstrucción a lte rn a n te .

P re d o m in a la

Con

edad media. Pruebas alérgicas —.

co n sín to m a s g en erales

R in o la lia c e rra d a .

sequedad, costras.

a n o sm ia , "n o se h u elen ").

p re d o m in a n te en

y rin o rre a serosa y

E x u d a d o d e n so .

E p ista x is fre c u e n te s .

M u jer, jo v e n , raza

c u a d ro in f l. c ró n ic o .

Luz nasal estenosada, tumefacción

Atrófica, seca, costras

R . C R Ó N IC A

R. SEC A A N T E R IO R

p.H le n te

lueg o m u c o p u ru le n ta .

pálidos y b rilla n te s

Mucosa tumefacta y levemente enrojecida

R . A L É R G IC A

R. V A S O M O T O R A

C o rn e t e s e d e m atoso s,

I to tó p ic o : c ro m o g lk a lo ,

A n tico lin é rg ico s

A n t iH I , co rtic o id e s

N e u re c to m ía v id ¡ano

Mucha tumefacción y

K A RTA G EN ER

la v a d o s

moscatel P Ó L IP O S - m ú ltip le s, b ila tera les - b en ig n o s

P o m a d a s o leo sa s,

tu rb in e c lo m ía

Aspecto de uvas de

Atrófic a (sequedad, costras verdosas, cavidad nasal ancha)

O CEN A C o n c h o to m íii/

T to . sin to m á tico

+ situ s in ve rso en

O b s tru c c ió n

a m u rilla

enrojecimiento C O R IZ A

fetidez (caco sm ía y

Q d e Eyro

- o rigen in fla m .-a lé rg ico - m u y re c id iv a n te s

O

- lo c a liz a c ió n m á s fre c u e n te : se n o e tm o id a l E m b a ra z o , A C O ,

1 li|M itiro id ism o

v a so co n stric to re s,

R e s e rp in a

A A S , h id a n to ín a ,

T ío : p o li p e e tom ía (ciru g ía o n d o s o )pica)

ASÍ X I A l IO N LS

b e ta b lo q u e a n te s.

?

en jo v e n e s.

Único y7unilateral

N iñ o co n

A so m a d e sd e el se n o

s in u sitis + p ó lip o

m a x ila r a co a n a .

u n ila te ra l: F Q

K IL L IA N -A N T R O C O A N A L

O en d o se ó pica

p o lip o sis nasal

y d e fo rm id a d fa c ia l:

Woakes

hO oo co

www.FreeLibros.me

Alergia a AAS I Asma TRÍADA ASA (Widal)

t o r r in o l a r in g o l o g ía

( u m b io s It u s c o s T a

ULTRA-RESÚMEIMES, Manual CTO de Medicina y Cirugía, 8.a edición A

4. TUMORES DE NARIZ Y SENOS. La pirámide nasal presenta los mismos tumores que la piel: basalioma >espinocelular > melanoma. Diagnóstico tumoraciones de fosas y senos: anamnesis + exploración física; dx. por TC, biopsia. Es importante ;ener en cuenta el orden de frecuencia según la localización: seno m axilar>fosas nasales>seno etmoidal.

Radiología y tumores rinológitos. Mucocele:

Osteoma:

Seno frontal, radiotransparente, fluctúa, desplaza al ojo.

(Tumor benigno más frecuente) Seno frontal, calcificado en la Rx. Varón joven con cefalea.

Adenocarcinoma (30%):

Cuerpo extraño:

Etmoides. Serrín im p lica d a

En meato inferior. N iño con obstrucción unilateral y m o rre a fétida purulenta.

Granuloma piogénico: Septum . Tum or vascular benigno.

Ca. epidermoide (60%): (Tumor maligno más frecuente) Contorno óseo irregular en la Rx.

Quistes dentarios: Radicular. Inflamatorio.

Igual que c. extraño en un adulto. Adenopatías infrecuentes.

Folicular: Em briológico (germen dentario en Rx).

Recuerda que: - Los tumores benignos son raros. - En niños, el rabdomiosarcoma y luego la histiocitosis X son los tumores m ás frecuentes (dar RT y Q T). - El carcinom a epiderm oide tiene mal pronóstico, ya q u e 2/3 son en el seno maxilar, donde suele cursarsin adenopa­ tías y asintomático. Ni el tabaco ni el alcohol se relacionan con una mayor incidencia de este cáncer. - En el tabique hay muchos procesos que pueden dar crecim iento y destrucción: granulom a maligno m ediofacial, Wegener, lúes, T B C , M uco r... Son, en general, radiotransparentes y su diagnóstico es histológico (biopsia, cu ltivo ...).

1.

Recuerda las características del papiloma inver­ tido nasosinusal.

2.

Relaciona las siguientes columnas:

1.

Lesión premaligna, destructiva y recidivante, localizada en cornete y meato m edio que apa­ rece en varones en la 5a-6a decada de la vida. Tto. Qx con márgenes am plios. Degenera en carcinoma espinocelular.

1.

Diabético con cetoacidosis.

a.

Klebsiella rhinosclerom atis.

2.

Supuración en "granos de azufre".

b.

Granuloma maligno m ediofacial (tratam iento con RT).

3.

Rinitis que deja extensas cicatrices y "nariz de tapir", común en Asia y Sudamérica.

c.

Mucor (cirugía + anfotericina intravenosa).

Nariz en silla de montar.

d.

Lúes.

5.

Proliferación linfomatosa muy destructiva (destruye hueso).

e.

Actinom yces.

6.

Cuadro de rinitis y obstrucción nasal pro­ gresiva con epistaxis y perforación del ta ­ bique (sin destruir hueso).

f.

Wegener.

.

R e sp u e sta s: 1-c, 2-e, 3-a, 4-d, 5-b, 6 -f,

284

www.FreeLibros.me

O CAPÍTULO 7.

t o r r in o l a r in g o l o g ía

PATOLOGÍA DE LAS GLÁNDULAS SALIVALES.

1. ANATOMOFISIOLOGÍA. IZ^ REPASA:

[ E s q u e m a DE LAANATOM IAY FISIO LO GIA DE LAS GLÁNDULAS SAUVALES, FlG U RA del M a n u a l

1.

24,

CA P ÍTU LO

1

CTO 8a L d .]

Relaciona las siguientes columnas: 1. Parótida 2. Glándula submaxilar 3. Glándula sublingual

a) C.Stenon, IX PC, secreción serosa b) C.Wharton, facia l, secreción seromucosa c) Facial, secreción mucosa en reposo

Respuestas: a-1, b-2, c-3.

2.

2.

¿Recuerdas por qué la litiasis salival es más fre ­ cuente en la glándula submaxilar?

3.

Porque su secreción es espesa, con pH alcalino y el conducto de Wharton es largo y tortuoso.

3.

¿Qué antibióticos tienen eliminación salival?

4.

Betalactám icos y macrólidos.

4.

Manifestaciones clínicas de litiasis salival

5.

Parótida: síntomas de infección Glándula subm axilar: síntomas de cólico.

PA TO LO G ÍA : G L A N D U L A S SA LIV A LE S |

P A T O L O G IA IN F L A M A T O R IA

P A T O L O G IA T U M O R A L

C r ó n ic a

Ben ig n a

Maligna C a e p id e rm o id e , 1 0 % d e los m aligno s, p a n o tid a, m u y ag re siv o , u lce ra c ió n d e la p iel, V II p e, a so cia d o a ra d io tera p ia p revia

V ir a l:

B a c t e r ia n a

P a p e ra s .

S ín d r o m e s o c u lo s a l¡v a r e s

N o lit iá s ic a :

L itiá sic a

- S i a l o a d e n o s is ( i n d o lo r a y

C ilin d r o m a

(in v a sió n n eu ral)

S U B M A X IL A R r e c id iv a n t e ). - S jó g r e n ( t u m e f a c c i ó n ) . - M i k u l i c z ( in f a r t o in d o l o r o ) .

.

.

M u c o e p id e r m o id e (Maligno m ás frecuente) O tro s : - c é lu l a s a c i n a r e s , - m ix t o , - a d e n o c a r c in o m a .

- W a r t h in 10% , cf

.

-

El m ás fr e c u e n t e .

|_ ^ ( j j eres 8 0 /6 r E s u n t ip o d e a d e n o m a m o n o m o r f o l in f o m a t o s o . L H o m b r e s , e x c lu s iv o p a ró tid a . l q 5 % b ila t e ra l.

- O n c o c it o m a (e n la s e n e c t u d , c f ).

- H e e r f o r d t ( s a r c o id o s is ) .

- H e m a n g ijs m a ; e l m á s f r e c u e n t e e n n iñ o s . P A R O T ID A

285

www.FreeLibros.me

ULTRA-RESÚMENES. Manual CTO de Medicina y Cirugía, 8.a edición

CAPÍTULO 8.

■■

PATOLOGÍA CERVICAL.

MASAS CERVICALES. M ASAS C ER V IC A LES :

Q u is te s b ra n q u ia le s (más frecuentes)

Lateral

Suprahioideos (prim erarco). Infrahioideos (segundo arco). M alform ación más frecuente del cuello.

Linfangioma (hacer RM). Laringocele (aumenta con Valsalva; 10% asocian un tum or de ventrículo).

J Sincrónico con el pulso, no se desplaza en el eje vertical Paraganglioma (glomus) j pQr |a bifurcación carotídea. Quiste túnico (base cuello). Neuromas del vago.

_ . . , ,f Asciende con la deglución. H acer siempre gammagrafía por si existe Q u is te tir o e lo s o (mas frecuenten . . . . . , . . , , . . , ,. . . 0 l tiroides ectopico (extirpar incluyendo al cuerpo del hioides). M edial Quistes derm oides y epiderm oides. Teratoma en neonato con obstrucción aérea.

A m b as

Linfangioma, hemangioma invasivo, adenopatía tumoral (linfoma, metástasis epiderm oides).

Preguntas para repasar: 1.

La hipoacusia de conducción más frecuente e s ...

1.

La que ocurre tras perforación timpánica. Des­ pués va la otosclerosis. En niños, la más f r e ­ cuente es la o titis media serosa.

2.

La existencia de perforaciones múltiples en el tímpano en la otoscopía de un paciente con otorrea sug iere...

2.

TBC (es rara).

3.

La m astoiditis.

4.

Cuadros que cursan con otorrea fétida.

4. Colesteatoma, cuerpo extraño, carcinoma epiderm oide (serosanguinolenta, fé tid a , masa en lim ite CAE/OM, no atical), OMA sarampionosa o por escarlatina.

5.

¿Cómo influye el embarazo en la otosclerosis? y ¿qué es el signo de Schwartze?

5.

La empeora. El signo de Schwartze es un signo raro (pero típico) de mal pronóstico que consis­ te en visualizar hiperem ia en el prom ontorio en la otoscopía.

6.

¿Cuál es la sinusitis con mayor frecuencia de complicaciones?

6.

Etm oidal (orbitarias).

7.

Intenta localizar el origen de la parálisis: 1.

2.

3.

Parálisis que asocia disfonia, disfagia, y aspi­ raciones por anestesia laríngea, con cuer­ das espásticas.

a.

Parálisis supranuclear.

b.

Parálisis del recurrente.

Cuerda vocal derecha en posición paramediana...

c.

Parálisis flácida con disfonia y cuerda en po-

286

www.FreeLibros.me

Parálisis nuclear o proxim al a ram as fa rín ­ geas.

P e d ia t r ía CA PÍTULO 1. •

NEONATOLOGÍ A.

Evaluación del RN. Apgar: más puntos es m ejor.

REPASA:

[T

est de

A

pgar,

T a bla

I,

C

a p ít u l o

I

del

M

anual

CTO 8a E d . ]

• Evaluación del distrés respiratorio del RN. S ilve rm an : más puntos es peor.

REPASA: • •

[T

est d e

S

il v e r m a n ,

T a bla 6 , C

a p ít u l o

I

del

M

anual

CTO

8 a E

d

.]

Causas de edemas en el RN. Prem aturidad (la más frecu ente), Turner, hidrops, fibrosís quística, hijo de diabética etc. Causas de convulsiones en el RN. A n o xia (causa más frecu ente de co nvulsio nes en el RN ), h ipo glucem ia. hipocalce m ia , h ipom agnesem ia, hip ernatrem ia, etc.



Hallazgos cutáneo-mucosos fisiológicos en el RN. Cutis m arm orata, cutis arleq u ín , quistes de rnilium ihiperplasía sebácea), m ancha m ongólica (nada que ver con el síndro m e de D o w n ), perlas de Ebstein en el paladar.



Hallazgos dermatológicos que pueden asociar patologías: M an chas café con leche (neurofibrom atosis tipo I m anchas hipo cróm icas lanceoladas (esclerosis tuberosa), nevus flam m enu s en área deltrigém ino (Sturge W e b e r .

Eritema tóxito vsMelanosis pustulosa.

APARECE LO CALIZACIÓ N



MELANOSIS PUSTU LO SA

1-3 d ía s

N a c im ie n t o

• Variable.

• V a r ia b le .

• N o p a lm o p la n ta r.

• S í p a lm o p la n ta r .

FROTIS

E o si n ófi los

N e u tró filo s

C U LT IV O

E sté ril

E s té ril

Luxación congénita de cadera. M aniobras de d ete cció n : -



ERITEM A TÓXICO

A sim etría de los pliegues glúteos. Barlow: "BarlO W " = O U T (saca la cadera). Ortolani: "Ortho" = correcto (reduce la cad era, lo correcto).

Hemorragia Intracraneal. -

H em orragia ¡ntraventricular: diagnóstico por eco-transfontanelar.

Patrones de

respiradónen el R.N. RESPIRACIÓN PERIÓDICA

APNEA IDIOPÁTICA DEL PREMATURO

Definición

Apnea 5 - 10 seg + polipnea 5 -1 0 seg

Grupo de riesgo

Más frecuente en pretérmino (pero también en a término)

Inicio

1- semana

Fin

36 semanas postconcepción

• Apnea >10 seg. • Con cianosis y bradicardia. Más frecuente en pretérmino • 1- semana. • No al nacimiento. 36 semanas postconcepción

Bradicardia/cianosis

Raro

Sí aparecen

Pronóstico

Es normal

Depende de la gravedad de la apnea Cafeín a/Teofi Ii na/CPAP/N E EP/VM

Tratamiento

Respecto a las siguientes afirm aciones, indique si son verdaderas o falsas: 1.

La causa más frecuente de masa abdominal en el período neonatal es el nefroma mesoblástico, que es un tumor congénito, benigno, pro­ ductor de renina y de predominio epidem ioló­ gico en varones.

1.

Falsa. La masa abdom inal más fre c u e n te en el re cié n nacido es la h id ro n e fro sis.

287

www. F ree Li bros.me

ULTRA-RESÚMENES. Manual CTO de Medicina y Cirugía, § .a edición

2.

La adicción a heroína y cocaína aumentan la incidencia de malformaciones congénitas.

2.

Falsa. Aum entan la m u erte in tra ú te ro , p e ro no la incidencia de m alform aciones.

3.

El hiperparatiroidismo materno puede provocar hipocalcemia neonatal.

3.

Verdadera. Por paso d el calcio al f e t o , su p ri­ miendo las p a ra tiro id e s fe ta le s .

4.

Se considera oligoamnios un volumen de líquido amniótico menor de 1000 mi en el tercer tri­ mestre.

4.

Falsa. Es m enor de 500 m i.

5.

Los hijos de madres alcohólicas se caracterizan por un aumento del tamaño cefálico respecto a niños sanos.

5.

Falsa. Presentan re tra so en la ta lla , peso y p e ­ rím e tro cefá lico .

Isoinmunizaclón en el R.N. RH (madre -> hijo +)

A B O (madre 0)

Frecuencia

M enor

M ayor

Aparición

D e sp u é s del p rim e r e m b a ra z o

P rim e r e m b a ra z o

Severidad Diagnóstico Tratamiento

• M ás grave (ic te ric ia , a n e m ia ).

• Le ve (ic te ric ia , a n e m ia ).

• H id ro p s .

• N o h id ro p s.

C o o m b s d ire cto + , in d i recto +

C o o m b s d ire cto +/-, in d i recto +

• E x a n g u in o tra n s fu sió n + F o to te ra p ia .

F o to te ra p ia

• P re v e n c ió n con g a m m a g lo b u lin a a n ti-D .

y/o e x a n g u in o tra n sfu sió n

PATOLOGÍA DEL PRETÉRMINO. Son patologías típ ic a s de esta situació n: enferm edad de m em brana h ialina (E M H ), ductus arterioso persistente (D A P), hem orragia intraventricular, leu co m alacia periventricular, enterocolitis necrotizante y la hipoterm ia.

1.

¿Cuál es la clínica de la leucomalacia periventri­ cular ?

1.

Norm alm ente es asintom ática hasta que apare­ cen las secuelas en form a de d ip le jía espástica.

2.

¿Qué patología típica del recién nacido pretérmino aparece cuando mejora la enfermedad de la membrana hialina?

2.

El ductus a rte rio so p e rs is te n te , que su ele apa­ re c e r a los 3-5 días de vid a , y se debe sospechar en caso de un p reté rm in o q u e, estando m e jo r de su EMH, com ienza con apnea p e rs iste n te de causa desconocida o reten ció n de carbónico o aum ento de las n ecesidades de oxígeno.

3.

¿Qué sospecharías ante un prematuro que pre­ senta en su 2a semana de vida una brusca dis­ tensión abdominal y en la Rx simple de abdo­ men se objetiva nematosis intestinal y patrón "en miga de pan"?

3.

E n te ro c o litis n e cro tiza n te .

4.

¿Cuáles son los criterios de tratamiento quirúr­ gico en la enterocolitis necrotizante?

4.

P e rfo ra c ió n , p e r it o n it is , se p sis re fra c ta ria a trata m ien to m édico.

5.

Asocia cada trastorno con el tipo de neonato al que corresponde generalmente:

5. R esp uesta s:

1. 2. 3. 4. 5.

Neumotorax S. aspiración meconial (SAM). Fibroplasia retrolental. Ductus arterioso persistente. Enfermedad de la membrana hialina.

a. A térm ino o postérm in o. b. P retérm in o.

R e s p u e s ta s : 1) a ; 2) a ; 3) b ; 4 ) b ; 5) b.

288

www.FreeLibros.me

P

e d ia t r ía

7.

¿Cuáles son los principales problemas metabólicos en el recién nacido pretérm ino ?

7.

H ipoglucem ia, hipo o h ipern a trem ia, h ip e rp o ­ tasem ia e hipocalcem ia.

8.

¿Por qué es im portante que los prem aturos re­ ciban aportes calcio , fósforo y vitam ina D?

8.

Porque ios niños p rem atu ro s de m uy b ajo peso ai n acer con fre cu e n cia p resenta n osteopen ia (d e sa rro lla n d o fra c tu ra s y d esm in era iiza ción ósea).

9.

¿Cómo se define policitem ia y cuándo se trata?.

9.

Es e l h em a tocrito ce n tra l m ayor o igual al 65’:. Se tra ta cuando p rese n ta clínica y/o el hem a­ to crito es m ayor del 70%.

Distrés respiratorio. PCF

EMH

• Pretérm ino. • Hijo de DM .

Parto

SAM

• A térm ino. • Postérmino.

SEPSIS PRECOZ

TAQUPNEA TRANSITORIA

Bolsa rota, SF

35-38 semanas cesárea o parto rápido

1--------------------DBF Pretérmino que necesitó oxígeno sobre i do si hubo oarotrauma o EMH

• A telectasia, edema Fisiopatología

M antiene DAP y shunt D-l con foramen oval permeable

Clínica

Cianosis

Radiología

No es típica (cardiomegalia, oligohemia)

Laboratorio

• p 0 2 muy baja. • p C O , normal o alta. • Acidosis.



intersticial y alveolar. Distensibilidad baja por falta de surfactante.

Aspiración del m econio con

• Hiperinsuflación. • Clínica de sufrimiento fetal.

• Polipnea. • Cianosis. • Tiraje.

Patrón retículonodular (vidrio esmerilado) con broncograma aéreo

Respuesta al oxígeno 100%

-

Otros tratamientos

A lcalinizar (bicarbonato, hiperventilar, tolazolina, óxido nítrico)

Pronóstico

Variable

• A pnea. • Shock. • Ictericia.

• Enfisema. • Infiltrados

• p O , baja. • p C Ó 9 alta. • Acidosis.

Com o EMH

• Prevenible con corticoides a madre. • Surfactante. • Antibióticos (similar a sepsis).

Variable

absorción del líquido pulm onar.

• Polipnea. • Algo de tiraje.

trama broncovascular. • Líquido cisural.

• p O , baja. • p C Ó 2 alta. • Acidosis

• Toxicidad del oxígeno. • Barotraum a.

¡

D ependencia a los 28 días del oxígeno para m antener PaO 9> 60 mmHg

Patrón "en esponja"

Normal

H ipoxia si se desconecta el ventilador

+++

++

j • Neutropenia y desviación

++

-/+

• Adaptativo. • Dism inuye

• Aum ento de

parcheados algodonosos.

• Prenatal: cociente lecitina/ esfingom ielina < 2. • p O , muy baja. • p C Ó 2 alta. • Acidosis.

S. agalactiae, E. coli

• Prevenible aspirando antes del llanto (en paritorio). • M edidas de soporte.

+

• Restricción de A ntibioterapia (am picilina + gentamicina)

No precisa

M alo

M uy bueno

Depende de otras patologías añadidas y com plicaciones

• Teofilina. • Diuréticos. • Beta-2-agonista.

Variable

ENTEROCOLITIS NECROTIZANTE. Es una lesión ¡sq uém ico -n ecró tica intestinal que se sigue h abitualm ente de sepsis b acteriana desde foco digestivo. Se sospecha ante un prem aturo que desarrolla una repentina distensión abd om inal (p rim er signo), a m enudo con depo sicio nes sanguinolentas en la segunda sem ana de vid a. Es un cuadro de inicio insidioso que puede acab ar en shock séptico y m uerte. Es im portante que recuerdes que el diagnóstico se realiza con una radiografía de ab d o m e n , que m uestra edem a de asas, neum atosis intestinal (signo diagnóstico), patrón en miga de pan, asa fija ; si existe perfo ració n: neu m op eriton eo, y

289

www.FreeLibros.me

ULTRA-RESÚMENES. Manual CTO de í\/ledkíiaa-.v¿G¿ajoía..SLa edición

si el cuadro sigue e vo lu cio n a n d o , gas en la ven a porta.

ICTERICIA NEONATAL

Tipos de

kteritia.

TIPO

M OM ENTO DE APARICIÓN

DURACIÓ N

Patológica

P rim e ra s 2 4 h

> 1 0 d ía s

TIPO

TRATAMIENTO

D ire cta

• Según c a u sa .

o

• E xa n g u in o tra n s fu sió n según e d a d

in d ire c ta

Fisiológica

2 Q ó 3 er d ía

Lactancia materna (Sd. Arias)

Fin d e la 1 se m a n a

< 1 0 d ías

In d ire cta

2 -1 0 se m a n a s

In d ire cta

g e sta c io n a l, p eso y m o m e n to d e a p a ric ió n N o p recisa (fo to te ra p ia o c a s io n a l) N o p re c isa , p ero a vece s se s u s p e n d e la la cta n c ia m a te rn a tra n sito ria m e n te

La ictericia por lactancia m aterna o síndro m e de Arias ha de cu m p lir algún criterio patológico; si no, no se deno m ina síndro m e de A rias; por lo tanto , es una ictericia patológica en la cual la única causa de ictericia es la lactancia m aterna.

Algoritmo diagnóstíto frente a la kteritia neonatal. K 1i

'

—- j x

Coom bs (+) —

Incom patibilidad A B O , Rh, grupos m enores

H ay hem olisis (reticulocitos T)

_► Buscar hem orragia fetal o transfusión fetom aterna Coom bs

e £

Trastornos intrínsecos del eritrocito: esferocitosis, eliptocitosis, d éficit G luco sa-6P-D H

Ictericia directa

— Colestasis neonatal

No hay hem olisis i Ictericia in d ire c ta __________ ►Ictericia fisiológica, por lactancia m aterna, hipotiroidism o congénito, Crigler-N ajjar

Tipos de tolestasis neonatal. Hepatitis neonatal idiopática (la más frecuente) rf> Lobul i I lar —j—> Hepatitis neonatal infecciosa (e j.: C M V) Hepatitis neonatal m etabólica (déficit oc,- antitripsina) | Intrahepática

Sd. Alagille: ductopenia + alteraciones faciales + vértebras en mariposa + estenosis pulm onar

C o le s t a s is

-£> D uctal -

Sd. Zellw eger: alteraciones hepáticas y cerebrales

N e o n a ta l

— Sd. Byler: cirrosis congénita — ►Sd. Aagenaes: linfedem a de m iem bros inferiores B d rahepát¡ca j ;:= 0 Atresia de vías biliares Es la causa más frecuente de trasplante hepático en la infancia

1.

¿En qué se debe pensar cuando la icteria apa­ rece después del tercer día y dentro de la pri­ mera semana ?

1.

P o sib le se p sis (p e ro tam bién TORCH, o b stru c­ ción g a stro -in te stin a l, lactancia m aterna).

2.

¿Qué se entiende por kernicterus?

2.

Tin ció n a m a rilla de los n ú cle o s b á sa le s d e l c e re b ro (p o r acúm ulo de b ilirru b in a ), aunque tam bién aparecen teñidas o tra s zonas.

3.

¿Qué es más frecuente, la enfermedad hemolítica por incompatibilidad Rh o por incompatibi­ lidad ABO ?

3.

Es 4-5 veces más fre c u e n te ABO (que nunca p ro ­ duce hydrops).

4.

¿Cuándo sospecharías que la ictericia del neo­ nato no es fisiológica? 1. Inicio a las 12 horas de vida.

4.

2. Duración de 10-15 días. 3. Bilirrubina directa >1 mg/dl. 4. Bilirrubina en sangre de cordón de 2 mg/dl.

a : fisio ló g ica , b : no fisio ló g ica .

R e s p u e s ta s : 1) b ; 2) a ; 3) b ; 4 ) a.

290 www.FreeLibros.me

P e d ia t r ía

«■teoetcomediooo.org HIJO DE DIABÉTICA. C o m p lic a c io n e s del hijo de m adre d iab ética: -

H ipo glucem ia, hip o calcem ia, m acrosom ía con viscerom egalias, p olicitem ia e ictericia, distress respiratorio, card ió ­ p a ta s (la m alform ación más frecu ente en ellos), colon izq u ierd o hip o p lásico , agenesia lum bosacra (la m alfo rm a­ ción más característica).

ONFALOCELE Y GASTROSQUISIS.

ONFALOCELE

Recubierto de peritoneo

Central

Peor pronóstico Asocia B eckw ith W iedem m an

GASTROSQUISIS

No recubierto de peritoneo

Lateral

M ejor pronóstico

INFECCIONES CONNATALES. En el cuadro siguiente se consignan las características más típicas de estos cuadros; pero es preciso que los conozcas bien: MOMENTO DE ADQUISICIÓN

VÍA DE ADQUISICIÓN

ESTIGMAS CARACTERÍSTICOS

Rubéola

Sobre todo 1er trimestre

Placentaria

• Tríada de Gregg: catarata, sordera,cardiopatía (ductus sobre todo). • O tros: coriorretinitis, retinopatía sal-pim ienta, estenosis pulm onar periférica, púrpura trom bopénica.

CMV (ía más frecuente)

Más frecuente en 3er trimestre M ás grave en 1er trimestre: más clínica

Placentaria, can al, leche

• Calcificaciones periventriculares cerebrales, niicrocefalia, coriorretinitis. • M uchos ¿sintom áticos con o sin secuelas tardías: la más frecuente es la sordera .

VHS (75-85% VHS II)

Más frecuente en parto

Canal trasplacentario postparto i90°o)

• Precoz: G R , vesículas, alteraciones necrológicas v ocu ares queratooonjuntivitisc • Tardía en par. 10 g/dl. t (8,14) t (9 ,22 ).

a) Buen p ron ó stico. b) Mal p ron ó stico.

R e s p u e s t a s : 1) a ; 2) b ; 3) a ; 4) b ; 5) a 6) b ; 7) a ; 8) b ; 9) b.

5.

¿Cuál es la causa más frecu ente de invaginación en el niño mayor de 6 años?

5.

El linfom a abdominal (localización más fre c u e n ­ te en la región ile o ce ca l).

6.

¿Cuál es la variante de linfoma de Hodgkin más frecuente en los niños?

6.

La e sclero sis nodular.

295

www.FreeLibros.me

ULTRA-RESÚMENES. Manual CTO de M^OÍCina y Cirugía, 8.a edición

7.

¿Cuál es la localización más frecuente de los tumores cerebrales en los niños?

7.

In fra te n to ria le s (fosa p o ste rio r).

8.

¿Cuál es el tumor de fosa posterior más frecuen­ te en la infancia y de mejor pronóstico?

8.

A strocito m a cereb elo so q u ístico.

9.

¿Cuál es el tumor del SNC más prevalente en niños < 7 años?

9.

M eduloblastom a (es el 2 o tum or más fre c u e n te de fo sa p o ste rio r)

10. ¿Cuál es el signo más importante de hiperten­ sión intracraneal?

10. El edem a de papila.

11. ¿Cuál es el tumor supratentorial más frecuente de la infancia ?

11. E l cráneofaringiom a (bolsa de R a thke).

12. ¿Qué tumores pueden producir hidrocefalia y síndrome de Parinaud?

12. Tumores de la glándula p in e a l. P a rálisis de la mirada hacia a rrib a y re fle jo s p u p ila re s a te ­ nuados.

13. ¿Cuál es el tumor ocular más frecuente en el niño? ¿Y el tumor orbitario más frecuente en el niño?

13. O cular : retin ob la stom a . O rb ita rio : rabdomiosarcom a.

14. ¿Cuál es la manifestación clínica más frecuente del retinoblastoma?

14. Leu co co ria , estrabism o y d olor ocular.

15. ¿Qué tumor óseo benigno produce dolor sobre todo nocturno que mejora con aspirina?

15. O steo m a o steo ide.

16. ¿Cuál es la localización más frecuente del osteosarcoma?

16. M e tá fisis fé m u r d ista l > tib ia p roxim al.

17. ¿Qué tumor predispone a la aparición del osteosarcoma?

17. El retin ob la stom a .

18. ¿Qué sospecharías en un niño con dolor óseo, pérdida de peso, aumento de VSG, y en la Rx, imágenes líticas y en "capas de cebolla"?

18. Sarcom a de Ewing.

19. ¿En qué consiste el Síndrome de Hand-SchullerChristian?

19. Lesion es óseas m ú ltip le s, exo fta lm o s y d ia b e­ tes insípida.

CAPÍTULO 6.

HEMATO-ONCOLOGÍA PEDIÁTRICA

En este apartado es importante reco rd ar cuáles son los tum ores más frecu e n te s en la infancia y las d ife re n cia s entre neuroblastom a \ neíroblastom a o tum or de W ilm s. Las leucem ias constituyen la patología oncológica más frecu ente en la infancia, representando el 30% de la oncología infantil. La leucem ia más frecuente es la linfoblástica aguda. Los tum ores cerebrales son los segundos en frecu encia (20% ), siendo el astrocitom a el más frecu ente. Por últim o, los linfom as representan el tercer tum or más frecu ente en la infancia.

296 www.FreeLibros.me

P e d ia t r ía Neuroblastoma vs Nefroblastoma.

E P ID E M IO L O G ÍA

N e u ro b la sto m a

• Tum or sólido extracraneal más frecuente en niños. • Sobre todo < 2 años. • Delección crom osom a 1.

W ilm s (N e fro b la sto m a)

• Tum or ab dom inal más frecuente en niños. • M ayores que los niños del neurob lastom a ( > . 3 años ). • D elección crom osom a 1 1.

L O C A L IZ A C IÓ N

C L ÍN IC A

T R A T A M IE N T O

• 70% abdom en (suprarrenal o param edial). • Pasa línea m edia. • 2 0 % tó ra x.

• M asa. • Sd. paraneo plásico s (VIO opsoclonom ioclono).

• Cirugía en estadio I. • O tro s: Q X + Q T + RT.

• Bueno. A veces se diferencia o regresa esp ontáneam en te. • M etástasis a hígado, m .o., piel y hueso.

• R iñ ó n . • No pasa línea m edia. • A veces bilateral (fam iliares).

• A socia a veces hem ih ipertrofia, an irid ia (sd. de Beckw ith W ied em a n y m alform aciones g enitourin arias. • M asa ab d o m in al. • H TA .

• Q x. • RT. • Q T (según estadio).

• B u en o , sobre todo en m enores de 2 años. • M etástasis a pulm ó n.

1.

La causa más frecu ente de estridor congénito e s ...

1.

Larin$omalaciQ.

2.

¿Qué enfermedad cursa típicam ente con "gallo" inspiratorio y leucocitosis con linfo cito sis ab ­ soluta?

2.

La tos fe rin a .

CA PÍTULO 3-

P R O N Ó S T IC O

APARATO RESPIRATO RIO ,

CRUP INFECCIOSO.

LARINGITIS AGUDA (estridulosa) Etiología

LARINGOTRAQUEITIS

Alergia + psicológico

Virusparainfluenzae 1 (el más frecuente)

EPIGLOTITIS AGUDA

H. influenzae b \

Antecedentes

No hay

Catarro vías altas (paciente y / o familia)

-

Clínica

Espasmo laríngeo recortado (generalmente nocturno). No fiebre

Fiebre, tos, disnea alta, estridor inspiratorio

Fiebre alta + babeo - disfagia + disnea - cabeza extendida

Duración

1-2 noches

Días - semanas.

Fulminante

• Ambiente tranquilo, humidificar.

Tratamiento

• Corticoides. • Adrenalina racémica en aerosol.

\

• Intubar + oxígeno. • Cefalosporina de generación. • Tto. en UVI.

297 www.FreeLibros.me

ULTRA-RESÚMENES. Manual CTO de Medicina y Cirugía, 8.a edición

FIBROSIS QUÍSTICA.

1.

H e re n c ia ...

1.

AR (crom osom a 7, p ro te ín a C FTR).

2.

Epidem iología...

2.

En ferm ed ad h e re d ita ria le ta l más fre c u e n te en raza blanca.

3.

Órganos a fe c ta d o s...

3.

Pulm ón, páncreas (exo y endocrino), te stíc u lo , cé rv ix , hígado, tubo d ig estivo (pro la p so re c ta l, íleo m econial -15% de los RN con FQ-), glándulas sudoríparas (alcalosis h ipoclorém ica p o r aum en­ to d el su d o r), pólip o s n a sa le s...

4.

Tratam iento del íleo m e co n ia l...

4.

Enem a hiperosm olar o cirugía.

5.

Pruebas diagnósticas:

6.

¿Cómo era el lío ese de los bichos que colonizan al paciente con FQ?

6.

El que coloniza con más frecuencia es Pseudomonas aeruginosa, y el 2 o es Staphylococcus aureus, que es el más precoz . Si aparece Burkholderia cepacia, es signo de mal pronóstico.

7.

¿Cuál es el órgano que determ ina el pronóstico?

7.

E l pulm ón.

5. - Test de sudor (2 p o sitiv o s). - Estu d io g en ético . - Screening en el RN: la m e jo r p rueba , es la más e fic ie n te (co ste ): io n o te st.

BRONQUIOLITIS AGUDA. Es una enferm edad viral (VRS más frecuente) que cursa con obstrucción inflam atoria de pequeñas vías aéreas. Se padece típ icam en te durante los dos prim eros años de vida con una in cid e n cia m áxim a a los seis meses de edad. -

El cuadro clín ico co m ie n za con síntom as de infección respiratoria leve, para ap arecer posteriorm ente tos, dificultad

-

respiratoria e irritab ilidad . N orm alm ente cursa sin fieb re, con fe b rícu la. El paciente se encu en tra taq u ip n e ico , con signos de dificultad respiratoria, y en la auscultación se oirá una espiración alargada, estertores finos al final de la inspiración y principio de la espiración con sibilancias. LaVadiografía de tó rax m uestra h iperinsuflación pulm onar. El tratam iento es de soporte, hum edad am biental y broncodilatadores (adren alina en aerosol y salbutam ol) y en

-

El porcentaje de m orta lid ad es inferio r al 1 %.

algunos casos está ind icado el uso de ribavirina en n eb u lizació n . \

1.

¿Cuáles son los agentes productores de bronquiolitis aguda del lactante?

1.

Los virus. E l más fre c u e n te es el virus re sp ira ­ t o r io s i n c i t i a l (>50%), p a r a in flu e n z a e 3 , m ycoplasm a y algunos adenovirus.

2.

¿Cuál es la base del tratam iento de la bronquiolitis aguda del lactante?

2.

Oxígeno y adrenalina. Deben e v ita rse los sedan­ te s. No están indicados los a n tib ió tico s si no e x iste neum onía b a cteria n a secundaria. Los cor­ tico id e s no producen b e n eficio s y pueden se r nocivos.

3.

¿Cuál es el tratam iento de un niño de 6 meses con una cardiopatía congénita con bronquiolitis aguda ?

3.

Ribavirina ( 1,5

3

¿C ó m o se ajusta la dosis?

Mantenimiento

(4, ¿C ó m o se d ebe m an ejar el tem b lo r en alguien que tom a litio?

6.

1r IN TERACCIO NES

EFECTO S SECU N DA RIO S 1. Neurológicos: - Tem blor fin o distal (lo + fre c.)

IN TO XICACIO N

A U M E N T A N N IV ELES (To xicid ad ):

CAU SAS:

-

D iu ré tico s: tiazid a s, d iu rético s de asa

- Lo m ás fre cu e n te : pérdidas de N a y agua.

y ah o rrad o res K +

- O tra s: alteració n del

- E xtrap iram id ale s (raro)

2. Renales:

-

A IN E s (no el A A S ni el p a racetam o l).

- D iabetes insípida nefrógena (lo + frec.)

-

lE C A s, m etro n id azo l, te tra c ic lin a s...

- N ecrosis focal/fibrosis intersticial (dudoso)

3. Endocrinológicos:

flu jo plasm ático

renal por A IN E s, n e fro p a tía ... S IN T O M A S :

D IS M IN U Y E N N IV E L E S (su b terap eú tico ):

Tem blor grosero, nauseas y vóm itos con

- H ipotiroidism o su bclín ico (lo + frec.)

-

M e tilxa n tin as.

v is ió n

- H ip ertiro id ism o (raro). B ocio.

-

In h ib id o res de an h id rasa carb ó n ica.

h ip e rre fle x ia , con fu sión con alteración

4. Otros:

-

D iu ré tico s osm óticos (m an ito l).

del

nivel

de

n is ta g m o ,

co n cie n cia

y

a t a x ia , arritm ias

card ía cas.

Con fre cu e n cia (leucocitosis, acn é , in crem en to de peso). Trastornos de la

b o rro sa ,

¿C o n o ces alguna otra in teracció n ?

7,

T R A T A M IE N T O :

co n d u cció n y del ritm o, agravam iento de psoriasis y edem as.

-

M a n te n im ie n to

de

las

co n stan tes

vitales. ¿C on q ué m e can ism o fisiológico se relacio nan sus dos efectos secu nd ario s característicos? 5

-

La\ ado gástrico si sob redo sisc

-

D iálisis. ¿Q u é otras causas de p érd id a de sodio con oces? 8

Respuestas al esquema del litio: 1.

E l litio se ho usado como p oten cia do r en el trata m ien to del trastorno d epresivo re siste n te mayor asocia­ do a a n tid ep resivo s. Otras indicaciones son la p revención de recu rren cia s del trasto rn o unipolar y tra s­ tornos e sq u izo a fectivo s y ciclo tim ico s, el con trol ante conductas im pulsivas y la ce fa le a en racim os y el síndrom e de K lein e-Levin (hipersom nia re cu rre n te ).

307

ULTRA-RESÚMEIMES. Manual CTO de /\b©dítiim y Cirugía, 8.a edición

2.

En una manía aguda, se asocia al trata m ien to con litio un n e u ro lé p tico . La razón es el tiem po de laten cia d el litio (7-10 d ía s), que hace necesario apoyar e l trata m ien to in icia lm en te.

3.

O tros e sta b iliza d o res del hum or son: carbam acepina, ácido valproico y clonacepam .

4.

De acuerdo con la s litem ia s. E l m e jo r indicador de la concentración en el SNC es el litio in tra e ritro c ita rio . Las personas con dism inución de la fun ción ren a l, como los ancianos, necesitan una dism inución en sus d o sis p ara alcanzar los n ive le s te ra p é u tico s. Las dosis h a b itua les oscilan e n tre 600-1800 m g/día, siendo su fic ie n te s, a veces, 900 m g/día en los ancianos.

5.

La in te rfe re n c ia con la ADH y la TSH, responsables de la d ia b ete s in síp id a n efrogénica y del h ipotiroid ismo, es debida a la inhibición de la aden ila to ciclasa p o r el litio .

6.

Si e s fin o d ista l, trata n do e l síntom a (p ro p ra n o lo l); si es g ro sero e in ten so , in d ica la in to xicación p o r litio .

7.

E l litio p ued e p o te n cia r los e fe c to s sed a n tes d el alcohol, a n tih ip e rte n sivo s ce n tra le s y o tro s d ep reso res del SNC.

8.

Pérdidas de sodio pueden o cu rrir debido a la d ie ta , la desh id ra ta ció n , los d iu ré tic o s, la fie b r e y los vóm itos. F íja te que unos e fe c to s secundarios fre c u e n te s como la d ia rrea y los vóm itos fa vo re ce n una deshidratación y, p o r lo tan to, la in to xicación , p o r lo que es im prescin d ible m antener una buena hidratación e ingesta de sodio en las in toxicacion es por litio .

CA PÍTULO 4.

TRASTORNOS PO R SUSTAN CIAS.

Vam os a prestar especial atención a las intoxicaciones y síndromes de abstinencia, en cuanto a su clínica y su tratamiento. A LC O H O L

La definición clínica del alco ho lism o es "un consum o de alcohol en cualquier cantidad com o para p ro d ucir proble­ mas fam iliares, laborales, legales o físicos (in clu yend o síntom as de abstinencia)". El consum o excesivo se valora según los gramos de alcohol totales al día o con el po rcentaje de calorías que el alcohol aporta a la dieta. En el siguiente esquem a te sim p lificam os el estudio de los trastornos neuropsiquiátricos asociados al co nsum o de alcohol.

Efectos secundarios del consumo de alcohol. CONSUMO CRÓNICO

CONSUMO ACUDO Depresión SNC

In su ficie n cia hepática K \

\ C o n la \ ab stin en cia

Am nesia iacunar

Complicaciones agudas

B la c k -O u t (Palim psesto)

Am nesia Iacunar + violencia extrema

En cefalo p atía po rto -cava.

In to xica ció n aguda

/ C o n la / variació n / del co n su m o (au m ento o cese)

B o rra ch e ra patológica. (In to xica ció n ¡diosincrática)

- C risis de p á n ico . - A taq u es a e l "ron" \ (co n vu lsio n es). / - D e liriu m trem en s (co n vu lsio n es + agitación + V alu cin acio n e s)

A lu cin o sis (sensorio d esp ejad o )

1. E n cefalo p atía m inor.

CONSUMO ACUDO 0 CRONICO + PREDISPOSICIÓN GENÉTICA

\

2. E n ferm ed ad M a rch ia fa va -B ig n a n i. )





Encefalopatía de W e rn ic k e (cu ad ro agudo)

>

Com plicaciones / Ir crónicas / / f

Psicosis de Ko rsakoff (co m p licació n cró n ica)

_________________

' í

308

www.FreeLibros.me

3. S ín d ro m e d e p resivo . 4 . D e m e n c ia alco h ó lica . 5. D e lirio p aran o ico ce lo típ ico .

P s iq u ia t r ía 1.

¿Cuál es la tríada típ ica d é l a encefalopatía de Wernicke?

1.

O ftalm o pa resia , ataxia y síndrom e confusional.

2.

¿Cómo no tratarías la encefalopatía de Wernicke?

2.

Con glucosa, ya que esto prod uciría un aum en­ to del consumo de tiam ina.A n tes de dar la g lu ­ cosa debes dar siem pre tiam ina.

3.

¿Cuál es la causa más frecu ente de dem encia en el alcohólico?, ¿y la de m ejo r pronóstico?, ¿y la que tiene una alteración orgánica más ca ra c­ te rística?

3.

Demencia alcohólica ( irre v e rsib le ) ; p sicosis de K o rsa k o ff (pa rcia lm en te re v e rsib le ); e n fe rm e ­ dad de M archiafava-Bignam i (degeneración del cuerpo calloso y la com isura blanca a n te rio r).

4.

¿Qué p o rcentaje de pacientes se recupera de la Psicosis de Korsakov?

4.

Un 25% del todo,un 50% p arcia lm en te y un 25% no se recuperan.

5.

¿Cómo diferenciarías un delirium trem ens de una alucinosis alcohólica?

5.

Repasa la tabla sigu ien te.

DELIRIUM TREMENS

ALUCINOSIS ALCOHÓLICA

Desencadenante

A bstinencia brusca.

A um ento o dism inución del consum o (s.t. aum ento).

Alt. de la conciencia

Sí (delirium ).

No (conciencia clara).

Alucinaciones

- Visuales m icrozoopsias). - Escen oráticas.

Auditivas i insulto s1.

- Inducib es.

Delirio

’O cu p ad o n a P .

Raro secundario a las aludnadones.

Alt. somáticas

Frecuentes.

\0.

Mortalidad

A lta sin tratam iento.

Rara.

Tratamiento

- Asegurar ctes. vitales. - B D Z , clorm etiazol. - Evitar NL (sólo tiapride).

-C e se del consum o. - H aloperidol. - Profilaxis de la abstinencia.

- Si co nvulsiones: M g. -Su p lem en to s vitam ínicos.

1.

¿Cómo actúan los fárm acos interdictores?

1.

Actúan inhibiendo la aceta ld eh íd o d esh id ro ge­ nase, provocando una reacción , cuando se toma a lcohol, con ru b efa cción , su do res, taquicardia, h ip e r o h ip o te n sió n ... O tros fárm acos que p ro ­ vocan un e fe c to p a re cid o al disu lfira m so n : me-

tronidazot, cloranfenicol, isoniacicfa, chaman­ do/, cefop era zo na , clorpropam ida y otros. 2.

¿Qué propiedades tiene el clorm etiazol?

2.

Es un derivado de la vitam ina B 1 con p o d e r s e ­ dante y a n ticon vulsivante.

3.

¿Cómo se debe o rientar el diagnóstico de una crisis convulsiva en un alcohólico?

3.

Si es generalizada tónico-clónica (Gran Mal o "ataque de ron"), es un m arcador de gravedad y a n teced e al síndrom e de a bstin en cia , p ero no re q u ie re TC. Si es p arcial o fo c a l, hay que des­ ca rta r un hem atom a subdural y se debe hacer TC.

309

ULTRA-RESÚM ENES. Manual CTO de Medicina y Cirugía, 8.a edición

4.

¿Qué síndromes neuropsiquiátricos podemos en­ contrar en un síndrome de abstinencia grave?

4.

A taques de "ron" (convulsiones) y deliriu m tremens (con fusió n , agitación, convulsiones).

5.

¿Por qué el único neuroléptico indicado en el tratam iento del síndrome de abstinencia alco ­ hólica es el tiapride?

5.

Porque no tiene m etabolism o h ep ático.

D E S IN T O X IC A C IO N Y D E S H A B IT U A C IO N . El siguiente esquem a puede ayud arte a distinguir la desinto xicació n de la deshab ituació n :

Tratamiento del alcoholismo. IN T O X IC A C IO N A G U D A

A LC O H O LIS M O C R O N IC O

D E S IN T O X IC A C IO N (cesar el co nsu m o )

1. Tratamiento de los síntom as: D e p resió n resp irato ria H ip o g lu ce m ia H ip o te rm ia

2. Tratamiento de com plicaciones:

.a

• A gitació n —» NL sedantes, D iace p am

i

LEVE (ambulatorio)

2 mm o fragmento intraarticular

LUXACIONES DE CADERA Anterior

Posterior Lesión del n. ciático

Avulsiones (jóvenes)

Pérdida del anillo pélvico en más de un punto. En traumatismos de energía. Más frecuente complicaciones.

Fx. de ramas (ancianos)

Complicaciones Ffemorragias. Roturas vesicales.

Longitud MMII Rot. EXTERNA ABDucción

Rotura de uretra. Lesión de nervio ciático

- Longitud MMII - Rot. INTERNA - ADDucción ("bañista sorprendido")

Complicaciones FRACTURAS DE CADERA Necrosis avascular de cadera Lesión del nervio ciático (en la posterior» Afectación del paquete vasculonervioso femoral (en la anterior)

Dolor inguinal + Rot. EXT + acortamiento + impotencia funcional + adducción

Cefálica Subcapital

FRACTURAS 1/3 MEDIO DIÁFIS1S FEMORAL

joven: osteosíntesis Intracapsu lares

Transcervical

l-ll: osteosíntesis Anciano

Complicaciones: 4» - Importante sangrado i500-800 mi. - Embolia grasa, pseudoartrosis I - TTO: encla\ado endomedular - En niños se puede admitir tto ortopédico mediante tracciones blandas

lll-IV: prótesis

Basicervical Pertrocantérea

Extracapsulares -► Osteosíntesi: Subtrocantérea

FRACTURAS 1/3 DISTAL DE FÉMUR TRAUMATISMOS DE RÓTULA Supracondíleas Posible afectación A. Poplítea TTO: Osteosíntesis

Fracturas de rótula: - Transversales (+ frec.) - TTO: osteosíntesis Luxaciones de rótula: - Desviación a lateral (+ frec.)

FRACTURAS 1/3 PROXIMAL DE TIBIA Fx. meseta tibial - Lateral: Valgo forzado Lesión meniscal/ligamentos - Medial: Lesión N. CPE (varo forzado) - TTO: Osteosíntesis si desplazada

FRACTURAS 1/3 MEDIO DIÁFISIS TIBIAL Localización más frecuente de las Fx. abiertas Fx. cerradas: enclavado endomedular o tto. conservador Fx. abiertas: fijador externo Complicaciones

FRACTURAS DE TOBILLO Fx. MAISSONEUVE 1/3 proximal peroné o cuello del peroné

Suprasindesmal Transindesmal -

Tto. conservador

Infrasindesmal-

+ LESION DEL LIG. DELTOIDEO

Síndrome compartimental Riesgo de pseudoartrosis

Osteosíntesis

FRACTURAS DEL PIE

(salvo afectación del comf osteolígamentoso medial

Calcáneo: Complicaciones: síndrome compartimental

Lesión del lig. deltoideo

1/3 distal peroné Fx. DUPUYTREN

«e asocia a Fx. vertebral y calcáneo contralateral Astrágalo: Osteosíntesis si desplazamiento Signo Hawkins: buen pronóstico Necrosis avascular

FRACTURAS DE METATARSIANOS

Fx. metáfisis 5QMTT (de Jones) \

í\

LUXACIONES DEL MEDIOPIE Lisfran c (tarso m etatarsiana)

\

Fx. cuello

2-

MTT

("del recluta" o de Deutschlanden

Fx. base 5QM T T -

- T T O : osteosíntesis

C h o p a rt (calcáneo/astrag alina V s cuboid es/escafoid es) - Estables . TTO : conservador

,(M. peroneo lateral corto)

339 www.FreeLibros.me

ULTRA-RESÚMENES. Manual CTO de M^5Í!«^siyiéíttó@*i,(#^ edición Lesiones traum átitas del miembro superior LUXACION ES DE H O M B R O ----------------------------------------------A n t e r io r

1 LUXAC IÓ N A C R O M IO C LA VICU LA R - Caídas sobre el hombro - TTO : Ortopédico - Cirugía en grado III si gran demanda funcional o motivos estéticos.

R e c id iv a n t e

- La más frecuente. Lesión del nervio axilar - TTO : Ortopédico P o s t e r io r

- En descarga eléctrica o crisis epiléptica. - RX AP y transtorácica - TTO : Ortopédico

^ FRACTURA DE CLAV ÍCU LA Lesión de Hill-Sachs T T O : Ortopédico

In v e t e r a d a

- M ás fre c u e n te en p o ste rio r

Lesión de Bankart T T O : Quirúrgico

- Más frecuente: 1/3 medio. - TTO : Ortopédico. - Fx. más frec. del recién nacido.

- TTO : Quirúrgico (reducción abierta)

FRACTURA DE ESCAPULA

FRACTURAS 1/3 PROXIMAL DE H U M ER O

Cuerpo escápula. TTO: ortopédico o cirugía si desplazadas o causan inestabilidad del hombro. Fx costales y lesión neurológica

Tratamiento (según clasificación de Neer) Troquiter

O rto p é d ico t il J \ c u e llo - No desplazadas ¡Ájl í quirúrgico O ste o sín te sis - 2 fragmentos. - 3 fragmentos desplazados en joven. A rtro p la stia - 3 o más fragmentos desplazados en anciano.

Hematoma de Hennequin

FRACTURAS Y LUXACIONES DEL C O D O Luxación

FRACTURAS DIAFISARIAS DE H Ú M ER O Espiroideas / oblicu as

Transversas

TTO: Ortopédico TTO: Conservador (Yeso colgante de Caldwell) o clavo intramedular

- Más frecuente a poste rol ate ral. Fractura s u p ra c o n d ílea

Lesión nervio radial (Fx. de Holstein-Lewis)

- TTO: Ortopédico (niños). ¿ - Osteosíntesis en adultos. & - Sd. Compartimental, neurovasculares/^ consolidación viciosa y miositis osificante.

J

FRACTURAS Y LUXACIONES DEL ANTEBRAZO

F X -L U X A C IO N DE M O N T E G G IA

- Fx cúbito proximal + Lx cabeza del radio - Lesión nervio interóseo posterior - TTO: Osteosíntesis del cúbito

FRACTURAS BASE 1er MTC

FX DE O L E C R A N O N

Nervio interóseo posterior

- Transversales. - TTO: Osteosíntesis

Rolando

Bennett

T T O : O rto péd ico

FX DE C U B IT O

FRACTURAS DE LA M AN O

RADIAL

Flexor común profundo (IFD) Flexor común superficial (IFP)

Fx del boxeador (cuello 5Q M TC )

/ / r ) /

/J (J / ■ ]

Luxación perilunar — Lesión nervio m ediano

Fx de falanges S in d a c tilia y m ovilización precoz

Pulgar del guardabosques O A jí

FX DE C Ú B IT O Y R A D IO

Grado I: Ortopédico

Grado II: Osteosíntesis

Grado III: Resección

Grado IV: Fx + luxación RCD (Essex-Lo prestí). Osteosíntesis.

(ligam ento colateral cubital M C F)

- TTO: Osteosíntesis F X -L U X A C IÓ N DE G A L E A ZZ I

- Fx radio distal + Lx cabeza del cúbito. - TTO: Osteosíntesis del radio.

FRACTURAS DE 1/3 DISTAL RADIO C o lle s

R h ea -B a rto n

- Desviación supinación - dorsal - radial. -T T O : ortopédico o cirugía si inestable. - Lesión tendón extensor largo del pulgar.

- Fx dorsal o palmar + luxación carpo -T T O : osteosíntesis

G o y ra n d -S m ith

- Estiloides radial - Cirugía si desplazadas

- Desviación palmar (Colles invertido) -T T O : osteosíntesis

Lesión de Stener (interposición de fascia apro xim ad o r corto)

- "Del bastonazo" - TTO: ortopédico.

H u tc h in s o n o d e l " c h a u ffe u r "

Fx escafoides T T O : O rto péd ico (yeso durante 8-10 semanas)

Fx Colles (deform idad en dorso tenedor)

No d e s p la z a d a D e s p la z a d a .............

TTo. conservador 2 fragmentos (osteosíntesis) 3 fragmentos: jóvenes (osteosíntesis) ancianos (artroplastia) 4 fragmentos (artroplastia)

340 www.FreeLibros.me

Fx Smith (deformidad en pala de jardinera)

________________T r a u m a t o l o g í a Esquema de las tomplUadonesgenerales de las fraeturas.

P S E U D O A R T R O S IS

S Í N D R O M E D E D O L O R R E G IO N A L C O M P L E J O

Ausencia de consolidación con la formación de una "articulación" en el foco de fractura. Tratamiento: refrescar bordes + injerto + osteosíntesis.

)

• Etiología: inm ovilización prolongada • Clínica: - F. aguda: dolor, rubefacción, edem a - F. distrófica: edem a, rigidez y am oratamiento - F. atrófica • Rx: osteoporosis m oteada: posteriormente difusa • T ratamiento: F isioterapia + - Fármacos - Anestésico - Sim patectomía - Calcitonina

C O M P LIC A C IO N E S GEN ERALES DE LAS FRAC TU RA S

S . C O M P A R T IM E N T A L

S . D E E M B O L IA G R A S A

• Etiología: fracturas de huesos largos, enclavados intramedulares. • Cuadro clínico con intervalo lúcido, petequias en tórax superior y conjuntiva, hipoxemia. • Rx: "torbellinos de nieve"

• Etiología: aum ento de presión • Clínica: - Dolor desproporcionado - Se localiza principalm ente en la tibia,antebrazo y codo • Tratamiento: - Apertura de vendaje + elevación - F asciotomía • Secuelas:

Retracciones importantes • T ratamiento: - Oxigenoterapia - Glucocorticoides a dosis TT - Estabilización precoz de la fractura

1.

p o r fib ro s is y Contractura isquém ica de Volkmann

¿Cuáles son las principales complicaciones de las fracturas abiertas?

1.

m alposiciones articulares

La infección y las alteraciones del proceso de consolidación. Son más frecuentes conforme au­ menta el grado de la fractura de la clasifica­ ción de Gustilo.

341

www.FreeLibros.me

ULTRA-RESÚM ENES. Manual CTO de MdaMte^€cfa¡p'a,(S-! edición

2.

¿Cuál es el tratamiento de las fracturas abiertas?

2.

Desbridamiento quirúrgico deí tejido necrótico + ATB i.v. (Cefa 1a G + aminoglucósido + penici­ lina) + profilaxis antitetánica + estabilización rígida de la fractura (con un fijador externo).

3.

¿Cómo definirías el grado lile de la clasificación de Gustilo?

3.

Se define como una fractura abierta con lesión vascular asociada.

D e c a r a a l t r a t a m ie n t o g e n e r a l d e la s f r a c tu r a s , e s im p o r t a n t e q u e c o n o z c a s la s in d ic a c io n e s g e n e r a le s d e l t r a t a m ie n t o

quirúrgico.

REPASA:

[T r a ta m ie n to de la s f r a c t u r a s , T a b la

C A P IT U L O 6-

4,

C a p ítu lo

1 del

M anual

CTO 8a E d .]

O R T O P ED IA IN FA N TIL Y D E L A D O L E S C E N T E ._________________________

A c o n t in u a c ió n v e m o s e n la t a b la lo s c u a d r o s p a t o ló g ic o s m á s f r e c u e n t e s d e cad era e n la o r t o p e d i a in f a n t il y d e l a d o le s c e n t e .

Cadera infantil y del adolestente. Dispfasia congénita de cadera

Artritis séptica

Sinovitis transitoria

RN y lactante

F. riesgo

-

Sexo femenino Laxitud familiar Presentación nalgas M acrosomía O ligohidram nios

Distribución hematógena S. Aureus S. (3 grupo B

Barlow + Ortolani + Limitación de la abducción Asimetría de pliegues

Cuadro séptico Inmovilización del miembro afectado

Dx

Ecografía (elección) Rx a partir de los 3 meses

ECO Artrotomía

Tto

< 6m : doble pañal y ortesis 6m -24 m: tracción y tenotomías > 24 m: osteotomías femorales y/o acetabu lares

Clínica

REPASA:

Sexo m asculino Infección vía respiratoria

Dolor irradiado a la rodilla Marcha dolorosa Cojera

De exclusión

Artrotomía urgente

+

Sintomáticos

AB iv.

www.FreeLibros.me

Epifisiolisis femoral proximal

4-9 años

Adolescente

Sexo m asculino Alt. coagulabilidad sanguínea Factores mecánicos

Sexo masculino O besidad • Alt. maduración sexual Alt. endocrinas

Claudicación Limitación de la rotación interna y abducción M archa dolorosa

Dolor crónico Limitación de la rotación interna

Rx ECO RM (dx precoz)

Rx

• Buen pronóstico* - Observación y tto. sintomático • Mal pronóstico - Conservador -> ortesis de centraje - Q x -> osteotomía

[ F a c t o r e s d e m a l p r o n ó s t i c o en l a e n fe rm e d a d d e p e r th e s , T a b la

342

E. de Perthes

18 ,

C a p ítu lo

6 del

Epifisiodesis in situ 1/3 reducción previa O steotom ía si está muy avanzado

M anual

CTO 8a

E d .]

T r a u m a t o l o g ía PRONACIÓN DOLOROSA

O C O D O DE

NIÑERA.

C o n s is te e n la s u b lu x a c ió n d e la c a b e z a d e l r a d io f u e r a d e l lig a m e n t o a n u la r . C u r s a c o n d o lo r , c o d o e n s e m ie x t e n s ió n , a n t e b r a z o e n p r o n a c ió n y a u s e n c ia d e m o v i lid a d a c t iv a d e l m i e m b r o s u p e r io r .

1.

¿Cuál es la maniobra que se emplea como trata­ miento de la pronación dolorosa?

1.

Supinación forzada del antebrazo en extensión y, a continuación, flexión sin necesidad de in­ movilización posterior.

2.

¿Qué lesión estamos obligados a descartar en un niño que, tras una caída, refiere dolor en la región del codo?

3.

Une los siguientes conceptos en relación a la epifisiólisis: 1. 2. 3. 4. 5. 6. 7.

2.

Tipo 1. Tipo 2. Tipo 3. Tipo 4. Tipo 5. Fractura triplanar. Epifisiodesis.

Una fractura supracondilea. Una complicación severa es el síndrome compartimental.

a. Es el tipo más frecuente. b. Es típica del cóndilo humeral infantil. c. Compresión axial con destrucción de la zona proliferativa. d. Es típica de los niños. e. Es muy frecu en te en la fisis distal de la tibia (Chaput-Tillaux). f. Formación de puente óseo. g. Tipo 3 con tipo 4 en distintos planos. R e sp u esta s: 1-d, 2-a, 3-e, 4-b, 5-c, 6-g, 7-f.

^

REPASA:

[O

s t e o c o n d r o s is ,

T a bla

í9y

F ig u r a

104,

C

a p ít u l o

6 del

M

anual

CTO 8a

E d .]

El pie infantil. PIE Z A M B O

PIE P LA N O

PIE C A V O

Equino. Varo. Adducto. (EVA) Tratamiento: - Comenzar desde el 1er día. - Si es flexible, correción con yesos (sigue el orden inverso: AVE). - Si es rígido, tto. quirúrgico.

C A P IT U L O 5.

T U M O R ES T U M O R ES

• ^ del arco plantar longitudinal. ■Dos tipos: ■Flexible (corrige al levantar el 1er dedo* • Contracto doloroso (tto. Quirúrgico)

" del arco plantar. Dolor e hiperqueratasia plantar Tratamiento conservador; si no cede, quirúrgico.

Y L E S IO N E S O S E A S P S EU D O T U M O R A LES . DE P A R T E S BLA N D A S.

C a s i la t o t a l id a d d e la s p r e g u n ta s d e t u m o r e s ó s e o s s e p u e d e n c o n t e s t a r a t e n d ie n d o a tres datos típicos ( lo ca liza ció n ,

e d a d e im agen radiológica). F íja te e n e l s ig u ie n t e e s q u e m a d e la p á g in a s ig u ie n t e :

1.

¿Qué sospecharías en un varón de 15 años que presenta dolor de predominio nocturno y que calma con aspirina?

1.

Osteoma osteoide.

2.

¿Cuál es su imagen radiológica característica?

2.

Imagen "en nidus".

3.

¿Cómo definirías la enfermedad de Ollier?

3.

Encondromatosis múltiple.

4.

¿Y el síndrome de Mafucci?

4.

Encondromatosis múltiple con angiomas caver­ nosos.

343

www.FreeLibros.me

ULTRA-RESÚM ENES. Manual CTO de Megj§é)^>h 1000 cel sézary

C a p ít u l o 2 5 . O t r o s t u m o r e s c u t á n e o s 254

Forma más frecuente de presentación de las metástasis cutáneas:

Nodulo duro/pétreo asintomático

255

Nodulo de la hermana Maria José:

Nodulo periumbilical (posible origen un a de noca remoma gástrico)

256

¿Qué prueba debes hacer para confirmar una mastocitosis sistémica?

Biopsia de médula ósea

257

Signo de Darier consiste en:

Eritema, edema y prurito tras rascado de lesiones

3 7 4

www. FreeLibros. me

Dermatología

Preguntas y respuestas más frecuentes

258

Es patognomónico de:

Mastocitosis

259

Forma clínica más frecuente de mastocitosis:

Urticaria pigmentosa

260

Forma clínica más frecuente de mastocitosis en la primera infancia:

Mastocitoma

261

Célula de la histiocitosis X:

Células de Langerhans [granulos de Birbeck)

Enfermedad de Letterer-Siwe

Forma más grave de Histiocitosis

[: 262

C a p ít u l o 2 6 . D e r m a t o s is p a r a n e o p l á s ic a s 263

Manifestación cutánea típica del glucagonoma:

Eritema necrolitico migratorio

264

Piel aterciopelada-grisácea en pliegues:

Acantosis nigricans

265

Eritema violáceo en párpados y nudillos con debilidad proximal:

Dermatomiositis paraneoplásica

266

Sdr. de trousseau, ¿qué es?

Flebitis superficiales recurrentes

267

¿Con qué se relaciona?

Adenocarcinomas de páncreas

268

Clínica del sdr. de Sweet:

Placasdolorosaseritematoedematosasencarayextremidades con fiebre, artralgias y neutrofilia en sangre periférica

375 www.FreeLibros.me

Preguntas y respuestas más frecuentes

Inmunología C a p ít u l o 1 . E s t r u c t u r a d e l s is t e m a in m u n e 1

¿Qué células forman parte de la inmunidad natural?

Monocitos/macrófagos, polimorfonudeares y células NK

2

¿Y qué elementos humorales?

Lisozima, complemento, interferones, proteínas de fase aguda ycitocinas

3

¿Cuáles son los órganos linfoides primarios y qué células se forman en ellos?

Médula ósea-linfocitos B;Timo-linfocitosT

D

¿Cuáles son los órganos linfoides secundarios y cuál es su fun­ Ganglios linfáticos, bazo y MALT; su función es facilitar que ción? tomen contacto los linfocitos ya maduros y los antígenos

5

¿Cuáles son los linfocitos mayoritarios en la corteza de los gan­ Linfocitos 8 glios linfáticos?

6

¿Y en la paracorteza?

Linfocitos T

7

¿Y en la médula?

Linfocitos B y T activados

8

¿Y en los cordones medulares?

Células Plasmáticas

C a p ít u l o 2 . I n m u n o g l o b u l i n a s 9

¿Cuál es la palabra mnemotécnica que indica la mayor a menor concentración en suero de los tipos de inmunoglobulinas?

GAMDE

10

¿Cuántas cadenas forman una inmunoglobulina?

Dos pesadas iguales entre si, que definen la dase y subclase, y dos ligeras también iguales entre sí

11

¿Existen varios tipos de cadenas ligeras?

Dos; kappa y lambda, predominando las primeras (relación 2/1)

12

¿Cómo se llaman los dos fragmentos resultantes de la disección enzimática de las inmunoglobulinas por papaína?

Fab (fracción antibody, donde están las regiones variables, responsable de la unión al antígeno) y Fe (fracción constante, responsable de las funciones efectoras)

13

¿Qué inmunoglobulina atraviesa la placenta y cuánto dura en la sangre del niño postparto?

IgG, se mantiene hasta los 6 a 12 meses de vida

14

¿Qué inmunoglobulina es la de actividad antivírica por excelen­ IgA cia?

15

¿Y la de mayor potencia antibacteriana porfijarcon más potencia complemento?

IgM

16

¿Cuál es la función de la IgD?

Ser receptor del linfocito B junto a IgM

3 7 7

www.FreeLibros.me

Inmunología J

Preguntas y respuestas más frecuentes

17

¿Y la de IgE?

Es la implicada en la inmunidad frente a helmintos (a través de los eosinófilos) y en las reacciones alérgicas (a través de basófilos y mastocitos)

18

¿Cuál es la inmunoglobulina implicada en la respuesta primaria inmunológica?

IgM

19

¿Y en la secundaria?

IgG, IgA...

20

¿Qué IgG no fija complemento?

lgG4

21

¿Quién sintetiza el componente secretor de la IgA?

Las células epiteliales

22

¿Y la cadena J?

Las células plasmáticas

23

¿En qué se diferencia la cadena J de la IgM (que es pentamérica) de la de la IgA (que es dimérica)?

Son iguales

24

¿Qué es un epítopo?

La región concreta de un antígeno donde se une el anticuerpo

25

¿Y un hapteno?

Moléculas no inmunógenos por si solas, pero que pueden comportarse como tal si se unen a un portador o "carrier* proteico

26

¿En qué consiste el cambio de clase de inmunoglobulina?

En que el linfocito B pase de secretar sólo IgM (respuesta primaria) a poder secretar otras clases de inmunoglobulinas (respuesta secundaria)

27

¿Qué moléculas intervienen?

CD40 y CD40L

28

¿Qué síndrome se produce si hay una mutación en estas molé­ Síndrome de hiperlgM culas?

C a p it u l o 3 . C é l u l a s d e l s is t e m a in m u n e 29

¿Cuál es la proporción de linfocitosT, B y NKen sangre periférica?

75 a 85% de linfocitos T, 5 a 15% de linfocitos B y 5 a 15% de células NK

30

Dentrode los linfocitoT, ¿qué tipos se incluyen según sus moléculas de superficie, y cuál es su proporción respecto al resto?

Linfocitos gamma-delta (5%del total de linfocitosT) y principal­ mente CD4 y CD8, en relación 2/1 en condiciones normales.

31

¿En qué condiciones se invierte el cociente CD4/CD8?

En el VIH y en situaciones postinfecciosas principalmente.

32

¿Qué receptor no tiene forma soluble, el del linfocito T o el del linfocito B?

El del linfocito T(RCT).

33

¿Qué moléculas reconocen los linfocitos T a través de CD4 y CD8?

CD4-HLAII; CD8-HLA I

34

¿Cuál es la molécula transductora de la señal estimuladora en el linfocito T?

CD3

35

¿Y cuál es la molécula coestimuladora y su homologa en la célula presentadora de antígeno?

Linfocito T-CD28, y de la CPA-B7

378 www.FreeLibros.me

Inmunología /Jfl

Preguntas y respuestas más frecuentes

36

¿Qué sucede si se producen la señal estimuladora y la coestimuladora?

37

¿Y si se produce la señal estimuladora en ausencia de la «¡esti­ El linfocito T entra en anergia. muladora?

38

¿A qué punto del linfocito T se unen los superantígenos?

A la zona lateral de la cadena beta del RCT

39

¿Son éstos internalizados por el linfocito T?

No

40

¿Qué producen?

Activación policlonal del 20% de los linfocitos T periféricos.

41

¿Qué citoquinas producen los linfocitos Th 1 y qué tipo de inmu­ IF gamma e IL2: Inmunidad celular nidad estimulan?

42

¿Y losTh2?

IL 4 e IL6: Inmunidad humoral

43

¿Y losTh3?

IL 10 y TGF beta: función reguladora o supresora

44

¿Qué moléculas producen los linfocitos T citotóxicos cuando se activan?

Perforinas (inmunidad celular)

45

¿Cuáles son los marcadores característicos de los linfocitos B?

Inmunoglobulinas de superficie, CD19, CD20, CD21

46

¿Cómo reconoce el receptor de la célula B el antígeno, presentado por otra molécula o en su forma soluble?

En su forma soluble, a diferencia del linfocito T

47

¿Cuál es la principal función de las células NK?

Destruir a las células que carecen de HLA 1o lo tienen alterado (infectadas por virus y tumores)

48

¿Cuáles son sus moléculas características y cuál reconoce el HLA 1?

CD16, KAR y KIR, que es el que reconoce el HLA 1

49

¿Cuáles son las células presentadoras de antígeno?

Monocitos-macrófagos, células dendríticas y linfocitos B

50

¿Sólo ellas tienen HLA II?

No, también los linfocitos T, pero sólo cuando están activa­ dos

51

¿Qué tipos de células dendríticas conoces y a quién presentan los antígenos?

Células interdigitantes (por ejemplo, la de Langerhans) a los linfocitos Th; células foliculares a los linfocitos B

La activación del linfocito T.

C a p ít u l o 5 . L a r e s p u e s t a in m u n e

52

¿Porquéesmejorlarespuestainmuneporanticuerpos secundaria que la primaria?

Porque presenta mayor rapidez en instaurarse, la fase de meseta es más prolongada, el título de anticuerpos es más alto y la afinidad de los anticuerpos por el antígeno es mayor

53

¿En qué consiste la inmunidad por anticuerpos antígeno T-dependiente?

En la interacción entre el linfocito B, que internaliza el antigeno y lo presenta mediante su HLA II al linfocito Th, el cual segrega citoquinas que a su vez activan al linfocito B para que produzca inmunoglobulinas

54

¿Qué tipo de respuesta se produce entonces por parte del lin­ Respuesta secundaria focito B?

379 www.FreeLibros.me

Preguntas y respuestas más frecuentes

55

En que los linfocitos B reconocen el antígeno (polisacáridos por ¿Y en qué consiste la inmunidad por anticuerpos antígeno Tejemplo) pero no son capaces de internalizarlo y por tanto no ¡ndependiente? se lo presentan a los ünfocitos Th

56

¿Qué tipo de respuesta se produce entonces?

Respuesta primaria

57

¿Cuál de las dos (T-dependiente y T-independiente) presenta memoria?

Respuesta T-dependiente

58

¿En cuál no se produce cambio de clase?

Respuesta T-independiente

59

¿Cómo se puede incrementar la inmunogenicidad de losantígenos T-independientes?

Uniéndolos a un "carrier" proteico, de forma que puedan ser internalizados por el linfocito B. Éste es el fundamento de las vacunas conjugadas

60

¿Son CD8+ todos los Ünfocitos T citotóxicos?

La mayoría sí, pero hay linfocitos Te CD4+, al igual que hay unos pocos linfocitosTh CD8+

61

¿Quién lleva a cabo la citotoxiddad dependiente de anticuerpos?

Las células NK

62

¿Qué tipo de anticuerpo reconocen éstas?

IgG

63

¿Qué es la alorreactividad?

El procesoen el que los linfocitosTde un individuo reconocenel HLA de otro individuo como si fuera una molécula HLA propia presentando un antigeno extraño

64

¿Qué mecanismos de tolerancia son menos estrictos, los de los linfocitos T o 8? ¿De qué tipo es normal encontrar linfocitos autorreactivos en sangre periférica?

Linfocitos B

65

¿Cuál de estos tres procesos se da con menor frecuencia en el anciano que en el joven: Neoplasias, enfermedades autoinmunes o infecciones?

Enfermedades autoinmunes

C a p ít u lo

4. E l c o m p le jo

m a y o r de h is t o c o m p a t ib ilid a d

66

¿Cómo se designa el HLA I?

Con una letra (HLA A, B, C ...)

67

¿Y el HLA II?

Con dos letras (HLA DR, DQ, etc...)

68

¿Qué células expresan HLA I?

Todas las células nudeadas y las plaquetas

69

Por tanto, ¿Qué estirpes celulares debes recordar que no tienen HLA I?

Hematíes, sincitiotrofoblasto y algunos timocitos.

70

¿Qué cadenas la componen y cómo son éstas?

Una cadena pesada alfa con regiones variables y una cadena ligera constante, la beta-2-microglobulina

71

¿Qué células expresan HLA II?

Células presentadoras de antígeno y linfocitos T activados.

72

¿Qué estructura tiene el HLA II?

Una cadena alfa y otra beta, ambas con regiones variables

73

¿En qué cromosomas se encuentra el material genético del HLA?

La mayor parte en el cromosoma 6, y el de la beta-2-microglobulina en el 15

380 www. FreeLibros. me

Inmunología |

Preguntas y respuestas más frecuentes

74

Si un virus es reconocido y fagocitado por un macrófago, ¿en qué HLA expresará esos antígenos?

HLA II

75

¿Porqué?

Porque el HLA II expresa antígenos procedentes de fuera de la célula, tras el procesamiento de los mismos en el fagolisosoma de la célula presentadora de antigeno

76

Y si el virus infecta al macrófago, integrando su material genético con el de éste, ¿en qué HLA expresará los antígenos del virus?

HLA I

77

¿Por qué?

Porque el HLA I expresa antigenos procedentes del interior de la célula, y el macrófago aparte de expresar HLA II también expresa HLA I, al igual que el resto de células nucleadas

78

Porque el HLA I expresa antigenos procedentes del interior de la célula, y el macrófago aparte de expresar HLA II también expresa HLA I, al igual que el resto de células nucleadas

Codominante, al igual que los grupos sanguíneos.

79

¿Cómo se llama el genotipo del HLA que cada individuo hereda de cada progenitor?

Haplotipo

80

¿A qué HLA se asocia la narcolepsia?

DR2

81

¿Y la enfermedad celiaca?

DQ2

82

¿Y la espondilitis anquilosante?

B27

83

¿Qué tipo de predisposición genética tiene la artritis reumatoide?

Es una enfermedad poligénica

C a p ít u l o 6 . C o m p l e m e n t o 84

¿Qué produce el déficit de los factores iniciales del complemento (C1 aC3)?

Clínica autoinmune tipo lupuslike

85

¿Porqué?

Porque esos son los factores principalmente encargados de eliminar los inmunocomplejos circulantes

86

¿Y el déficit de los factores finales (C3 a C9)?

Susceptibilidad a infecciones por Neisserias

87

¿Cuál es el déficit de complemento más frecuente a nivel mundial y qué produce?

Déficit de C9; asintomático

88

¿Y cuál el déficit más frecuente en nuestro medio y qué produce?

Déficit de factor C1 inhibidor; edema angioneurótico familiar

C a p ít u l o 7 . I n m u n o l o g ía c l ín ic a

89

¿Qué tres tipos de compatibilidades inmunológicas hay que estudiar para hacer un trasplante en general?

Compatibilidad de grupo sanguíneo, de HLA, y que no existan anticuerpos preformados en el receptor del órgano contra HLA del donante

90

¿En qué tipode trasplante no importaque nohaya compatibilidad de grupo sanguíneo?

En el de progenitores hematopoyéticos

91

¿En cuál no importa la compatibilidad de HLA?

En el de córnea

www.FreeLibros.me

Inmunología

Preguntas y respuestas más frecuentes

92

¿En cuál tiene que haber compatibilidad casi total o total de HLA?

En el de progenitores hematopoyéticos

93

¿Cuál es la compatibilidad mínima de HLA requerida para un trasplante renal?

Tienen que coincidir tres de seis de los principales HLA

94

¿Cuáles son los HLA que más influyen en la superviviencia del injerto?

DR, B, A y C, por ese orden

95

¿Cómo se evidencia la existencia de anticuerpos preformados contra el HLA del donante?

Mediante prueba cruzada

96

¿En el trasplante de qué órgano es especialmente importante realizarla previamente al trasplante, por ser muy frecuente la existencia de anticuerpos preformados?

En el de riñón

97

¿Y en cuáles se puede hacer después de la operación, porque es muy poco frecuente la presencia de estos anticuerpos?

En el cardiaco y hepático

98

¿Por qué? ¿En qué 3 situaciones es más frecuente que se hayan formado estos anticuerpos?

Cuando ha habido contacto entre el receptor y HLA ajeno, principalmente cuando ha habido trasfusiones previas (lo cual es muy frecuente en la insuficiencia renal crónica y por tanto en el trasplante renal), trasplantes previos (por lo que en retrasplantes no vale usar un órgano con los mismos HLA que el primer trasplante) y embarazos previos, porque se crean anticuerpos contra el HLA paterno del niño.

99

¿En qué consiste el rechazo hiperagudo?

En la existencia de anticuerpos preformados contra el HLA del donante, que fijan complemento, destruyendo las células del injerto rápida y masivamente.

100

¿Qué tipo de reacción de hipersensibilidad es?

Hipersensibilidad tipo II

101

¿Tiene tratamiento y/o prevención?

Se previene realizando la prueba cruzada; si aparece no tiene tratamiento, hay que quitar el injerto

102

¿En qué consiste el rechazo agudo?

En la aparición de inmunidad celular que se produce frente al injerto a través de los linfocitos T CD4 del receptor (reaccionan contra el HLA II del donante) y los CD8 (contra el HLA 1)

103

¿En cuánto tiempo se instaura?

A las pocas semanas

104

¿Qué tipo de reacción de hipersensibilidad es?

Hipersensibilidad tipo IV

105

¿Tiene tratamiento y /o prevención?

Se puede prevenir realizando cultivo mixto de linfocitos, lo cual tarda una semana, por lo que sólo se hace para predecir la enfermedad de injerto contra huésped en el trasplante de progenitores hematopoyéticos. Se puede tratar con inmunosupresores.

106

¿En qué consiste el rechazo crónico?

En un envejecimiento acelerado del órgano, como una ateroesclerosis acelerada de causa aún poco clara

107

¿Cuánto tiempo tarda en instaurarse?

Años

108

¿Tiene tratamiento y /o prevención?

No

109

¿En qué consisten las reacciones de hipersensibilidad tipo 1?

Son reacciones mediadas por IgE

382 www.FreeLibros.me

Inmunología

Preguntas y respuestas más frecuentes

110

Pon algún ejemplo

Atopia

111

¿En qué consisten las tipo II?

Anticuerpos circulantes que se unen a antígenos presentes en la célula diana

112

Pon algún ejemplo

Enfermedad hemolitica del recién nacido, rechazo hiperagudo de trasplantes, miastenia gravis.

113

¿En qué consisten las tipo III?

Anticuerpos circulantes que se unen a antígenos solubles, formando inmunocomplejos

114

Pon algún ejemplo

Lupus

115

¿En qué consisten las tipo IV?

Son reacciones de hipersensibilidad retardada mediadas por células

116

Pon algún ejemplo

Rechazo agudo de trasplantes, granulomas, prueba del Mantoux

117

¿Son sinónimos alergia y atopia?

No; la alergia puede estar mediada por cualquiera de los cuatro tipos de hipersensibilidad, mientras que la atopia es la predisposición genética a desarrollar reacciones mediadas por IgE (tipo I)

118

¿En qué trasplante es importante por su frecuencia de aparición de enfermedad injerto contra huésped?

En el trasplante de progenitores hematopoyéticos

C a p ít u l o 8 . I n m u n o d e f i c ie n c ia s 119

¿Cuál es la clínica principal de las inmunodeficiencias humora­ Infecciones ORI. neumonías, diarreas (cuadros bacterianos) les?

120

¿Hay alguna infección especialmente característica de éstas?

La encefalitis por virus ECHO es infrecuente pero muy caracteristica, porque depende de la inmunidad humoral y no celular, como el resto de los virus

121

¿Cuál es la clínica principal de las inmunodeficiencias celulares o combinadas?

Infecciones por virus y hongos, similar a los cuadros que apa­ recen en el SIDA

122

¿Y en los defectos de la fagocitosis?

Infecciones por S.aureus, bacterias intracelulares y hongos

123

¿Qué inmunodeficiencias son más frecuentes, las primarias o las secundarias?

Las secundarias

124

¿A qué suelen ser debidas?

A fármacos (inmunosupresores, corticoides, quimioterápicos), malnutrición, uremia, diabetes y VIH

125

¿Cuál es en general el tratamiento de mantenimiento de las in­ La administración de gammablogulina intravenosa munodeficiencias humorales?

126

¿Se han de administrar también en la fase aguda de infección?

No, las infecciones se tratan con antibióticos.

127

¿Con qué otras patologías suelen cursar las inmunodeficiencias primarias, además de infecciones de repetición?

Con fenómenos autoinmunes, neoplasias y atopia.

128

¿Cuáles son dentro de éstas las neoplasias más frecuentes?

El linfoma no Hodgkin y el carcinoma gástrico.

www.FreeLibros.me

Inmunología

Preguntas y respuestas más frecuentes

129

¿Cuándo empiezan a dar clínica las inmunodeficiencias primarias humorales?

A partir del sexto mes/ primer año de vida

130

¿Por qué?

Porque la IgG materna que ha atravesado la placenta permanece en ¡a sangre del niño entre 6 y 12 meses

131

¿Cuándoempiezan a dar clínica las inmunodeficiencias primarias celulares o combinadas?

A partir del nacimiento

132

¿Cuál es la alteración que existe en el síndrome de Bruton?

Los linfocitos B están ausentes, por lo que los niveles de in­ munoglobulinas son casi indetectables, siendo los linfocitos T normales

133

¿Qué tipo de herencia tiene?

Ligada al cromosoma X

134

¿Cuál es su tratamiento?

Administración periódica de gammaglobulina intravenosa

135

¿Cuál es la etiología más probable de la inmunodeficiencia va­ Se achaca al virus de Epstein Barr, que infecta a los linfocitos B y altera la producción de inmunoglobulinas riable común?

136

¿Qué característica tiene peculiar, a diferencia del resto de inmunodeficiencias primarias?

137

¿Cómo es el número de linfocitos y los niveles de inmunoglo­ El número de linfocitos B es normal pero los niveles de inmu­ noglobulinas están descendidos bulinas?

138

¿Cuál es la clínica del déficit de IgA?

En la gran mayoría de pacientes; asintornática. En los sintomáti­ cos; infecciones respiratoriasy digestivas,clínicamente similares a la enfermedad celiaca, siendo característica la infección por Giardía lamblia, y trastornos autoinmunes.

139

¿Cuál es el tratamiento de mantenimiento?

Ninguno

140

¿Por qué no se puede administrar gammaglobulina intravenosa ni realizar trasfusiones sanguíneas?

Porque frecuentemente no sólo hay déficit de IgA sino que hay anticuerpos anti-lgA, por lo que al administrarla a través de la gammaglobulina (quees una mezcla de IgG, IgA, IgM...) o de la sangre del donante se han descrito reacciones graves

141

¿Quéanomaliasen númerodelinfocitosy producción deinmunoglobulinas existen en la inmunodeficiencia combinada severa?

Hipogammaglobulinemía intensa, linfopenia y ausencia de linfocitos circulantes

142

¿Qué diferencia hay entre la forma ligada al sexo y la autosómka recesiva?

Que en la ligada al sexo sólo hay un descenso en el número de linfocitos T, y en la autosómica recesiva hay déficit de lin­ focitos B y T

143

¿Qué arcos faríngeos se afecta en el síndrome de di George?

El tercero y cuarto

144

¿A qué anomalías se asocia?

A ausencia de timo, ausencia de paratíroides, anomalías faciales y cardiacas, especialmente en la salida de los grandes vasos

145

El número de linfocitosT está disminuido, siendo los que quedan ¿Qué anomalías en número de linfocitos y producción de inmu­ inmaduros, y la producción de inmunoglobulinas es normal o noglobulinas existen? discretamente inferior a lo normal

146

¿En qué patología debes pensar cuando en un caso clínico hablen d e foliculitis de repetición por 5. aureus ?

Que se manifiesta en adultos

Enfermedad granulomatosa crónica

384 www.FreeLibros.me

Preguntas y respuestas más frecuentes

147

¿Qué microorganismos son más frecuentes en ésta?

Los cata lasa positivos; estafilococo aureus, E. coli. Serrada y hongos

148

¿Qué tipo de inmunodeficiencia produce la ataxia telangiectasia?

Inmunodeficiencia combinada severa

149

¿Cuáles son las inmunodeficiencias que se heredan ligadas al sexo?

De las humorales; Síndrome de Bruton e hiperlgM. Celulares; inmunodeficiencia combinada severa ligada al X. Déficits dei complemento;déficitdeproperdina. Defecto en la fagocitosis; Enfermedad granulomatosa crónica en el 70% de los casos

O tr o s tem a s 150

¿Qué tipo de hipersensibilidad produce la enfermedad del suero?

Hipersensibilidad tipo III

151

¿Cuáles son sus principales manifestaciones clínicas?

Fiebre, vasculitis-urticaria, artritis y neuropatía por depósito de ¡nmunocomplejos

152

¿Qué patología se produce cuando hay un déficit de la molécula CD18?

Déficit de adhesión leucocitaria

153

¿Cuál es e! mejor método para diagnosticarla?

Citometria de flujo

154

¿Qué es la proteómica?

El estudio de las proteínas, utilizando el espectrómetro de masas

www.FreeLibros.me

385

Preguntas y respuestas más frecuentes

Miscelánea F a r m a c o l o g ía 1

Vías de administración que evitan el primer paso hepático

Sublingual y transdérmica, rectal lo evita parcialmente

2

Reacciones de fase I de la biotransformación

Oxidación, reducción, hidrólisis y descarboxilación (inactivan o destruyen)

3

Reacciones de fase II de la biotransformación

Síntesis o conjugación (lo hacen más polar)

4

Grupo deoxidasas más importante en ef metabolismo

Citocromo p450

Efecto sobre el metabolismo de fármacos de: fenitoina, fenobarbitai y rifampicina

Inductores enzimáticos

Efecto sobre el metabolismo de fármacos de: cimetidina, ketoconazol, macrólidos y valproato

Inhibidores enzimáticos

Principales medicamentos en los que hay que reducir la dosis en caso de insuficiencia renal

Aminoglucósidos, vancomicina. digoxina

8

Principales medicamentos en los que hay que reducir la dosis en caso de insuficiencia hepática

Teofilinas, Benzodiacepinas. eritromicina

9

Proceso farmacocinético más afectado en ancianos

Eliminación (disminuye)

10

Proceso farmacocinético menos afectado en ancianos

Absorción (disminuye biodisponibitidad)

11

Alteración en e! metabolismo de fármacos en ancianos

Disminuyen las reacciones de fase I; disminuye efecto de primer paso

12

Cambio composición corporal en ancianos

Aumenta grasa, disminuye masa magra y agua (aumenta volumen distribución fármacos lipofílicos, disminuye el de hidrofílicos)

13

La fracción libre de los fármacos en el anciano

Aumenta

14

Fármacos de uso frecuente contraindicados en embarazo

Anticoagulantes orales (dicumarínicos),retinoides, tetracidinas, quinolonas, ketoconazol, díetilestilbestrol

15

Cambios en la absorción en el embarazo

Retraso en la absorción; aumento biodisponibilidad intra­ muscular

16

Cambios en la distribución de fármacos en el embarazo

Aumento del volumen de distribución y de la fracción de fár­ maco no unido a proteínas

17

Cambios en la eliminación en el embarazo

Aumento de la depuración renal (disminuye vida media de eliminación)

D 6

B

387 www.FreeLibros.me

Miscelánea

Preguntas y respuestas más frecuentes



18

Biodisponibilidad

Fracción de dosis que alcanza, inalterada, la circulación sistémica

19

Biodisponibilidad de la vía intravenosa

100%

20

Vida media

Tiempo que tarda en reducirse a la mitad la concentración plasmática de un fármaco

21

Aclaramiento

Volumen plasmático que es depurado de una sustancia en unidad de tiempo

22

Nivel valle

Justo antes de la siguiente dosis

23

Nivel pico

Tras una dosis

24

Tipo de reacción adversa a medicamentos más frecuente

Tipo A: dosisdependiente

25

Tipo de reacción adversa a medicamentos más grave

Tipo B: idiosincrásica

26

Intoxicación más frecuente en el anciano

Digoxina

27

Dos sustancias son bioequivalentes si

Proporcionan la misma biodisponibilidad en magnitud y velocidad

28

Niveles terapéuticos de digoxina

0,5 - 2 ng/mt

29

Síntomas más frecuentes en la intoxicación digitálica

Alteraciones digestivas (náuseas, vómitos...)

30

Arritmia más frecuente en intoxicación digitálica

Extrasístoles vernaculares (asintomáticas generalmente)

31

Arritmia más frecuente sintomática

Bloqueo AV

32

Arritmia más específica de la intoxicación digitálica

Taquicardia supraventricular con bloqueo AV variable

33

Fármacos que aumentan la toxicidad de la digoxina

Quinina, amiodarona, verapamil, espíronotactona, erítromicina...

34

Situaciones que aumentan la toxicidad de la digoxina

Hipercalcemia, hipo... (hipopotasemia, hipotiroidismo...)

35

Volumen distribución digoxina

Elevado, no es útil la diálisis

36

Diurético que más hiponatremia produce

Tiazidas

37

Fármacos que producen hiperpotasemia

Diuréticos ahorradores de potasio, IECA, AINEs, betabloqueantes

38

Fármaco que más hipopotasemia produce

Furosemida

39

Antibiótico de elección en la sífilis

Penicilina G

40

Antibiótico de elección en infecciones estafilocócicas

Penicilinas resistentes a betalactamasa (cloxacilina)

41

Antibiótico de elección para Listeria

Ampicilina

388

www. FreeLibros.me

Miscelánea /i

Preguntas y respuestas más frecuentes

Piperacilina-tazobactam; ceftazidima / cefepime; aztreonam; imipenem / meropenem; quinolonas/ aminoglucósidos

Antibióticos activos frente a Pseudomonas

Todos los betalactámicos presentan reacciones de hipersensibi­ lidad cruzadas excepto Fármacos que producen exantema cutáneo en pacientes con mononudeosis infecciosa

Ampicilina y amoxicilina

Únicas cefalosporinas activas frente a anaerobios

Cefoxitina y cefotetan (2* generación)

I Síndrome de la bilis espesa

Efecto secundario de ceftriaxona

Bacterias que no cubre ¡mipenem y meropenem

Enterococcus faecium y Staphilococcus resistentes a meticilina

Efecto secundario más importante de imipenem

Convulsiones

Fármaco de elección en infecciones por Staphilococcus meticiín resistentes Reacción adversa más frecuente de los glucopéptidos

El 'hombre rojo"

Indicación de vancomicina vía oral

Colitis pseudomembranosa

Espectro de los glucopéptidos

Cocos gram positivos Bacterias gramnegatrvas aerobias. Sinergia con otros antibió-

I Espectro aminoglucósidos

Reacciones adversas aminoglucósidos

Nefrotoxicidad (reversible), ototoxicidad (irreversible)

Reacción adversa más grave del cloranfenicol

Anemia aplásica irreversible

Reacciones adversas tetraciclinas

Fototoxicidad, teratógenicas, manchas en los dientes (con­ traindicadas en embarazo y niños), hipertensión intracraneal benigna

Único antibiótico útil por vía oral para Pseudomonas

Ciprofloxacino

Interacción principal de la rifampicina

Inductor enzimático (p450)

Anaerobicida que mejor pasa la barrera hematoencefálica

Metronidazol

Fármaco de elección en crisis parciales

Carbamacepina

Fármaco de elección en crisis generalizadas

Valproico

Fármaco de elección en crisis de ausencia típicas

Etoxusimida (o valproico)

Fármaco de elección en estatus epiléptico

Diacepam más fenitoína

Fármacos del primer escalón de la escala analgésica de la OMS

AINEs, paracetamol, metamizol (+/- coadyuvantes)

389 www.FreeLibros.me

Miscelánea

Preguntas y respuestas más frecuentes

65

Fármacos del segundo escalón de la escala analgésica de la OMS

Un fármaco del primer escalón + Un opiáceo débil (codelna, tramado!)

66

Fármacos del tercer escalón de la escala analgésica de la OMS

Un opiáceo fuerte

67

Tratamiento de la intoxicación por benzodiacepinas

Flumacenilo

68

Tratamiento de arritmias ventriculares por antidepresivos tricicticos

Lidocaína o fenitoina, más bicarbonato sódico

69

Antídoto de la intoxicación por paracetamol

N-acetilcistelna

70

Alteraciones ácido-base en intoxicación por aspirina

Primero alcalosis respiratoria (por hiperventilación, luego acidosis metabólica (por acidosis láctica)

71

Antagonista opiáceo usado en intoxicación

Naloxona

72

Antídoto metano! y etilenglicol

Etanol

G e n é t ic a 1

Los segmentos de ADN codificantes se denominan

Exones

2

El paso de ADN a ARN se denomina

Transcripción

3

El paso de ARN a péptido se denomina

Traducción

La mayoría de las células del organismo adulto se encuentran en la fase del ciclo celular

G0

5

En la meiosis se divide una célula diploide (2n) para originar

4 células haploides (n)

6

El proceso de recombinación consiste en:

Las cromátides homólogas se entrecruzan e intercambian ADN entre ellas

7

La recombinación tiene iugaren

La primera división meiótica

8

La segregación meiótica de de dos genes distintos es menos probable:

Cuanto más próximos estén entre sí en un mismo cromosoma

9

Las enzimas que fragmentan el ADN reconociendo secuencias específicas son:

Enzimas de restricción

10

La técnica de western blotting se usa para el estudio y localiza­ proteínas ción de:

11

La técnica que usa una sonda marcada para localizar genes con­ hibridación in situ cretos se denomina:

12

Una variación en la secuencia del ADN detectable al menos en un 1% de la población se denomina:

Polimorfismo del ADN (RFLP)

13

¿En qué consiste la técnica arrays de DNA?

Se coloca sobre una superficie fragmentos de DNA concretos, se híbrida sobre ésta cDNAobtenidoapartirdeRNAdelacélula problema (para estudiar la expresión génica en dicha célula)

H

390 www. FreeLibros. me

Miscelánea

Preguntas y respuestas más frecuentes

14

¿Qué técnica permite la detección y cuantificación de caracte­ rísticas estructurales (como marcadores de superficie) de una Citometria de flujo célula?

15

Causa más frecuente de anomalías prenatales en el desarrollo humano

Desconocida

16

La composición genética de un Individuo es el

Genotipo

17

La expresión del genotipo es

Fenotipo

18

El lugar que ocupa un gen en un cromosoma es

Locus genético

19

Distintas formas de expresión de un gen polimorfo

Alelos

20

La transmisión vertical en una enfermedad monogénica corres­ Autosómico dominante (AD) ponde a un patrón de herencia

21

La transmisión horizontal en una enfermedad monogénica co­ Autoso mico recesivo (AR) rresponde a un patrón de herencia

22

Si un progenitor tiene una enfermedad AD, ¿que porcentaje de hijos afectos tendrá?

50%

23

Si un progenitor tiene una enfermedad AR, ¿que porcentaje de hijos afectos tendrá?

50% portadores; 50% sanos

24

Si ambos progenitores son portadores de un gen AR, ¿que por­ 25% enfermos, 25% sanos, 50% portadores centaje de hijos afectos tendrá?

25

La capacidad de expresión fenotíplca de un gen es su

26

La fuerza con la que se manifiesta un determinado gen pene­ Expresividad trante es

27

Patrón de herencia de: Poliquistosís renal del adulto

AD

28

Hemocromatosis

AR

29

Distrofia muscular de Duchenne

Ligado a X

30

Corea de Huntington

AD (también expansión de tripletes e imprinting)

31

Distrofia miotónica de Steinert

AD (también expansión de tripletes

32

Fibrosis quística

AR

33

Alteración estructural cromosómica en la que se produce una delección en dos cromosomas y en la reparación se intercambian los segmentos

Traslocación

34

División transversal del centrómero con formación de un cromo­ isocromosoma soma simétrico

35

Trisomias que se ven en la práctica (que llegan a nacer)

Penetranda

21 (Down), 18 (Edwards), 13 (Patau)

391 www.FreeLibros.me

Preguntas y respuestas más frecuentes

36

Mecanismo de enfermedad genética en enfermedades como: diabetes, epilepsia, enfermedad coronaria, esquizofrenia, etc

Herencia poligénica

37

Las alteraciones del ADN mitocondrial tienen una herencia

Materna

38

Los oncogenes suelen codificar

Formas anómalas de proteínas que inician el ciclo celular

39

Los factores supresores suelen codificar

Proteínas cuya misión es sacar a la célula del ciclo celular y pasarla a GPara que induzcan transformación es preciso que las dos copias del gen estén alteradas

40

Gen más frecuentemente alterado en la patología tumoral hu­ p53 mana

G eria tría 1

Anciano sano

No padece enfermedad crónica ni problemática funcional ni social

2

Anciano enfermo

Afectado de una patología crónica, si ser anciano de riesgo

3

Anciano en situación de riesgo

Mayores de 75 años, con varias enfermedades en evolución, que tienden a la cronicidad y la invalidez, y con deterioro mental y/o problemática social asociada

D

Síndromes geriátricos más importantes

Delirium; incontinencia; inestabilidad y caídas; inmovilidad y úlceras por presión; estreñimiento; malnutrición; depresión, y demencia

5

Áreas estudiadas en la "valoración geriátrica global"

Biológica, funcional, mental, psicoafectiva, sociofamiliar

6

Actividades de la vida diaria básicas

Aquellas que engloban las capacidades de autocuidado más elementales de la función física (comer, lavarse, vestirse, mic­ ción...)

1

Actividades de la vida diaria instrumentales

Actividades más complejas, se pierden más precozmente siendo marcador de los primeros gradosde deterioro (teléfono, compras, uso dinero...)

8

Etiología delirium

Orgánica (patología cerebral, procesos sistémicos, fármacos, factores ambientales)

9

Diagnóstico delirium

Clínico. Exploraciones complementarias para búsqueda causal

10

Curso clínico del delirium

Fluctuante, periodos de lucidez durante el día

11

Nivel de conciencia y atención en delirium

Alterado y cambiante

12

Tipo de incontinencia urinaria más frecuente

De urgencia

13

Mecanismo de incontinencia de urgencia

Contracción involuntaria del detrusor

14

Clínica de incontinencia de urgencia

Urgencia miccional, aumento de la frecuencia

392

www.FreeLibros.me

Miscelánea

Preguntas y respuestas más frecuentes

15

Tratamiento incontinencia de urgencia

Anticolinérgicos (oxibutinina), relajantes músculo liso, antide­ presivos triciclicos

16

Clínica incontinencia esfuerzo

Pequeñas pérdidas con el esfuerzo

17

Tratamiento incontinencia esfuerzo

Ejercicios pélvicos de Kegel, alfaadrenérgicos, cirugía

18

Etiología incontinencia por rebosamiento

Obstrucción infravesical, o hipocontractilidad vesical

19

Clínica incontinencia por rebosamiento

Dificultad inicio micción, sensación de micción incompleta, episodios de retención urinaria

20

Complicación característica de un traumatismo craneoencefálico (muchas veces mínimo) en un anciano

Hematoma subdural crónico

21

Lesión asociada a caídas que con más frecuencia requiere hospi­ Fractura de cadera talización en el anciano

22

Consecuencias de que un anciano permanezca caído durante un tiempo prolongado tras una caída

Hipotermia, deshidratación y rabdomiolisis

23

Causa principal del síndrome de inmovilidad

Encarnamiento por convalecencia tras enfermedades agudas

24

Medida preventiva más eficaz para las úlceras por presión

Uso sistemático de escalas de detección de riesgo para su desarrollo

25

Tratamiento postural para prevenir úlceras por presión

Cambios posturales cada 2 horas durante las 24 horas del dia

26

Método más eficaz de eliminación del tejido necrótico en las úlceras por presión

Desbridación quirúrgica

27

Tratamiento antibiótico de las úlceras por presión

Sistémico. Cuando exista infección: confirmación microbioló gica, sobre todo si se acompaña de signos y síntomas

28

Causa más frecuente de incontinencia anal en ancianos

Impactación fecal secundaria a estreñimiento crónico

29

Etiología del colon catártico

Abuso de laxantes

30

Tratamiento del colon catártico

Interrumpir laxantes, medidas higiéníco-dietéticas para el estreñimiento

31

Anomalía nutrlcional más frecuente en ancianos

Malnutrición proteicocalórica

32

Malnutrición más frecuente en enfermedades agudas

Malnutrición hipoalbuminémica

33

Parámetro bioquímico para valorar estado nutricional de vida media larga

Albúmina (20 días)

34

Parámetros bioquímicos nutricionales de vida media corta

Transferrina (8 días) y Prealbúmina (2 dias)

35

La depresión en anciano presenta más frecuentemente que en jóvenes

Sintomassomáticos,alteracionescognitivas(pseudodemencia), depresión psicótica

36

El anciano deprimido presenta mayor riesgo de

Suicidio

393

www.FreeLibros.me

Miscelánea

Preguntas y respuestas más frecuentes

37

Antidepresivos que no deben usarse en ancianos

Triciclicos. Por sus efectos anticolinérgicos

38

Forma más frecuente de demencia en países occidentales

Enfermedad de Alzheimer

39

Principal factor de riesgo para la enfermedad de Alzheimer

La edad

40

Tratamiento farmacológico de la enfermedad de Alzheimer

Inhibidores acetilcolinesterasa (fases leve y moderada), memantina (fases moderadas y avanzadas)

A n ato m ía

a

Entre los músculos escalenos anterior y medio (triángulo de los escalenos) discurren

Ramas anteriores del plexo braquial, y arteria subclavia

2

Las arterias oftálmica, cerebral anterior y cerebral media son ramas de

Arteria carótida interna

3

El nervio laríngeo recurrente derecho pasa por debajo de

Arteria subclavia derecha

4

El nervio laríngeo recurrente izquierdo pasa por debajo de

Arco aórtico

5

Lasfibras para simpáticos del nervio motorocularcomún terminan en el ganglio

Ciliar

6

La glándula lagrimal es inervada por

Nervio petroso mayor, rama de Vil

7

Las glándulas submandibular y sublingual son inervadas por

Nervio cuerda del tímpano, rama de VII

8

La glándula parótida es inervada por

Glosofaringeo (IX)

9

El líquido cefalorraquídeo circula por

Espacio subaracnoideo

10

El músculo motor ocular externo está inervado por

VI

11

El músculo oblicuo mayor está inervado por

IV

12

El manguito de los rotadores está formado por cuatro músculos:

Supraespinoso, infraespinoso, redondo menor, subescapular

13

El plexo braquial está formado por las raíces

CS -T I

14

El reflejo bicípital depende de la raíz

C5

15

El reflejo estiloradial depende de la raiz

C6

16

El reflejo tricipital depende de la raíz

a

17

La inervación sensitiva de la cara dorsal del antebrazo y 1er, 2oy 3er dedos depende de

Nervio radial

18

La inervación sensitiva del territorio más extenso de la mano depende de

Nervio Mediano

La inervación sensitiva del 5odedo y mitad cubital del 4odepende de

Nervio cubital

19

www.FreeLibros.me

Miscelánea

Preguntas y respuestas más frecuentes

20

El nervio afectado en el síndrome del túnel del carpo es

Nervio mediano

21

La musculatura de la eminencia tenar de la mano depende de

Nervio mediano

22

Ei suelo de la tabaquera anatómica está formado por el hueso

Escafoides

23

La inervación de los músculos queflexionan la cadera y extienden la rodilla depende de

Nervio femoral

24

La marcha y signo de Trendelenburg se produce por

Debilidad de los músculos separadores

25

El músculoqueseinserta como tendón rotulianoen la tuberosidad anterior tibial es

Recto anterior

26

El reflejo rotulíano depende de la raíz

L4

27

El reflejo aquíieo depende de la raíz

SI (a veces L5)

28

La descendente anterior es rama de

Coronaria izquierda

29

La artería circunfleja es rama de

Coronaria izquierda

30

La "dominancia" en la vascularización cardiaca depende del origen de

Arteria descendente o ¡nterventricular posterior

31

El 75% de las personas presentan dominancia

Derecha

32

El nodo AV está irrigado en la mayoría de los casos por

Coronaria derecha

33

La irrigación de la cara inferior del ventrículo izquierdo depende de

Coronaria derecha

34

La irrigación de la cara anterior y surco ¡nterventricular anterior

Descendente anterior

35

La irrigación de la pared libre del VI

Circunfleja

36

El conducto torácico desemboca en

Tronco braquiocefálico venoso izquierdo

37

El borde derecho de la silueta radiográfica del corazón corres­ Vena cava superior y Aurícula derecha, vena cava inferior ponde

38

El borde izquierdo de la silueta radiográfica del corazón corres­ Arco aórtico, tronco pulmonar, orejuela izquierda y ventrículo ponde izquierdo

39

Músculos de la pared anterior del abdomen (de superficial a profundo)

Oblicuo externo, oblicuo interno, transverso, recto anterior

40

El suelo del conducto inguinal

Ligamento inguinal

41

Relación del orificio inguinal profundo con respecto a los vasos epigástricos

Lateral

42

Relación del orificio inguinal profundo con respecto a los vasos epigástricos

Medial

395

www.FreeLibros.me

Miscelánea

Preguntas y respuestas más frecuentes

43

Las hernias inguinales indirectas se originan en

Orificio inguinal profundo (intrafuniculares)

44

Las hernias inguinales directas se forman

Protrusión en la parte posterior del conducto inguinal (extrafuniculares)

45

El tronco celíaco tiene como ramas principales

Arteria gástrica izquierda, arteria hepática común, arteria esplénica

46

Ramas principales de la arteria mesentérica inferior

Cólica izquierda (irriga ángulo esplénico y colon descendente), sigmoideas, hemorroidal superior

47

Origen más frecuente de la arteria cfstica

Arteria hepática derecha

48

El conducto biliar deriva del

Endodermo

P a c ien t e ter m in a l 1

Cuidados paliativos son

Cuidado total de paciente cuya enfermedad no responde a tratamiento con intención curativa

2

Objetivo principal de estos

Cuidar y no abandonar, ayudar en el proceso de duelo.

3

Informar de una enfermedad terminal

Respuestas aproximadasy nunca exactas sobre su pronóstico

4

Síntoma más frecuente del paciente terminal

Dolor

5

Segundo síntoma más frecuente del paciente terminal

Estreñimiento

6

En el tratamiento del dolor en situaciones paliativas se usa pre­ La vía oral ferentemente

7

Manejo de los análgesicos

Administración pautada y dosis de rescate si aparece dolor

8

En el tratamiento paliativo del dolor no se debe

Utilizar placebo

9

Tratamiento de elección de la disnea en la fase terminal de la Morfina enfermedad

10

Paciente terminal con dolor por metástasis óseas a pesarde AINEs a dosis plenas, actitud correcta

11

En una úlcera por presión estadio III con tejido necrótico y signos Hemocuitivos, desbridación quirúrgica y antibióticos intra­ venosos. de ifección, tratamiento inicial

12

Tratamiento dolor leve

AINE, paracetamol, f'ramaco adyuvante

13

Dolor moderado

AINE, opiáceo débil: codeina, tramadol..., fármaco adyuvante

14

Dolor grave

AINE, opiáceo potente: morfina..., fármaco adyuvante

15

Mecanismo de acción general de los AINES

Inhibición de la COX-1 yCOX-2

16

Efectos secundarios de los AINEs dosis-dependientes:

Lesiones gastrointestinales leves, úlceras gástricas

Utilizar un opioide y un AINE al mismo tiempo

396 www. FreeLibros.me

Preguntas y respuestas más frecuentes

17

Los efecto secundarios más frecuentes de los AINEs son

Gastrointestinales y depués los del SNC

18

Características de los receptores opiáceos mu

Analgesia supraespinal, depresión respiratoria, euforia, sedación moderada, miosis

19

La eliminación de la morfina es

Por vía renal

20

Qué opiáceo no es adecuado para el tratamiento del dolor crónico

Meperidina

21

Efecto secundario más frecuente de los opiáceos

Estreñimiento

22

Efecto secundario potencialmente más severo de los opiáceos

Depresión respiratoria

23

Principales fármacos adyuvantes cuidados paliativos

Antidepresivos, neurolépticos,ant¡comiciales,Benzodiacepinas, esteroides, antihistamínicos

24

EN el manejo de los estertores de agonía está indicado

Administración de anticolinérgicos

25

Tratamiento de la obstrucción intestinal no quirúrgica en fase terminal del cáncer

Morfina, buscapina, y haloperidol por vía subcutánea

26

Triada de la hipercalcemia maligna

Somnolencia, sed, poliuria

27

Tratamiento de la hipercalcemia maligna

Suero salino fisiológico! 2-3 litros/día) junto con furosemida tras corregir el déficit de volumen existente.

28

Clínica del síndrome de vena cava superior

Edema en esclavina, cianosisen cara y extremidades superiores, circulación colateral toracobraquial y disnea

29

Tratamiento antiemético más adecuado en la obstrucción intes­ Octreótido tinal maligna refractaria

30

Cuatro principios básicos de bioética

Autonomía, beneficencia, no maleficencia, justicia

31

Actitud ante pacientes terminales

Ofrecer garantías de continuidad de los cuidados

32

El consentimiento informado en un ensayo clínico debe solici­ A todos los participantes, independientemente del grupo al que puedan ser asignados tarse

33

Tratamiento en un paciente con dolor oncológico de intensidad leve moderada

Paracetamol más codeina

34

Un metabolito de la meperidina puede acumularse produciendo

HiperexcitabilidaddelSNCconalteraciones del humor, temblor, míodonías, convulsiones

35

Los efectos secundarios de los opiáceos a ios que se desarrolla tolerancia son

Naúseas, vómitos, sedación

36

Los efectos secundarios de los opiáceos a los que no se desarrolla tolerancia son

Estreñimiento, Síndrome confusional, alucinaciones

37

Los efectos secundarios de los opiáceos dosis-dependienes son

Estreñimiento, naúseas, vómitos, sedación

38

En la disnea terminal con ansidad además de morfina deben pautarse

Benzodiacepinas: diacepam, midazolam

3 9 7

www.Freel_ibros.me

Miscelánea

Preguntas y respuestas más frecuentes

39

Ante lamásmínimasospechadesindromedecompresión medular debe iniciarse tratamiento con

Cortico ¡des (dexameta sona)

40

Los efectos sobre la mucosa gástrica de los AINEs dependen de la inhibición de

La COX 1

M ed icin a p r ev en tiv a 1

¿Cuál es la vacuna frente a la polio en el calendario español actual?

Vacuna de la polio intramuscular tipo Salk

2

¿qué riesgo elimina esta vacuna frente a la polio oral?

Riesgo de parálisis fládda

3

Vacunas víricas inactivadas:

Antigripal, antipolio parenteral.antihepatitis A y B,

Vacunas víricas vivas atenuadas:

Antisarampión, antirrubeola, antiparotiditis, antivaricela, antipolio oral

5

Vacunas bacterianas inactivadas:

Antitetánica, antidiftérica, antimeningocócica A-C, anti H, Influenzae b...

6

Contraindicaciones de las vacunas vivas atenuadas:

Embarazo, inmunodeficiencias, tratamiento inmunosupresor intenso y prolongado, radioterapia

7

La inmunoglobuiina im antihepatitis B es:

Hiperinmune

8

La vacuna antineumocócica está indicada en:

Pacientes con insuficiencia cardíaca, cirrosis, infección por VIH avanzada, fístula crónica de LCR y alcohólicos

9

En pacientes esplenectomizados debe realizarse profilaxis contra:

Meningococo, neumococo, H.influenzae y frente a la gripe

10

El calendario vacunal entre los 15 -44años incluye:

Tétanos, difteria, sarampión, parotiditis, rubéola

11

El calendario vacunal en mayores de 65 años incluya:

Tétanos, difteria, gripe y neumocócica

12

Vacuna en al que hay que valorar el grado de inmunosupresión antes de administrarla:

Triple vírica

13

¿Es una contraindicación la lactancia materna para administrar vacunas de virus vivos atenuados?

No

14

El tratamiento continuado con dosis altas de corticoides se considera:

Una contraindicación general para la administración de va­ cunas

15

En los pacientes VIH está indicada la triple vírica excepto:

Cuando existe inmunosupresión grave, menos de 15% de CD4

16

Los niños con infección VIH deben recibir la vacuna de la varicela si:

Están asintomáticos y sin inmunodepresión,>25% de CD4

17

Situaciones que aumentan la probabilidad de encontrar cáncer colorrectal:

Eli de larga evolución, endocarditis por Streptococcus Boris, tabaquismo, ureterosigmoidostomía

18

Screening recomendado por encima de los 65 años:

Próstata, colon y mama

19

Neoplasia que se asocia en su carcinogénesis con el tabaco:

Cáncer de Páncreas

D

398

www.FreeLibros.me

Miscelánea

Preguntas y respuestas más frecuentes

20

El tabaco aumenta el riesgo de padecer:

Cáncer de pulmón, de vejiga...

21

El cáncer gástrico se ha asociado en estudios caso-control a:

Pescado ahumado

22

Cáncer en el que no existe un patrón de agregación familiar:

Laringe

23

El único procedimiento que se ha demostrado preventivo en el cáncer de pulmón es:

Dejar de fumar

24

El papiloma virus está relacionado con el cáncer de;

Cévix, vulva y ano

25

El VEB está relacionado entre otros con:

Linfoma de Burkitt, cáncer de cavum

26

Screening del cáncer de mama:

Mamografía anual/bianual en mujeres mayores de 50 años

27

Programas de cribaje que disminuyen la mortalidad:

Test de Papanicolau para el cáncer de cérvix, mamografía para el cáncer de mama, colonoscopia en el cáncer de colon

28

El principal predictor de cardiopatía isquémica en varones de edad media es:

El colesterol sérico

29

El tabaco no es factor de riesgo para el desarrollo de:

Hipertensión arterial

30

Objetivos de colesterol LDL:

En pacientes de riesgo elevado < 1OOmg/dl, riesgo intermedio 60 años, hipoacusia de percepción coclear simétrica y pro­ gresiva (peor en agudos al inicio)

73

Aminoglucósidos ototóxicos de predominio vestibular

Estreptomicina y gentamicina

74

Características del trauma acústico

Acúfeno e hipoacusia neurosensorial con escotoma a 4000Hz

75

Es imprescindible en niños hacer el diagnóstico de sordera antes de

Los 2 años de edad

76

Protocolo de screening neonatal

Otoemisiones en las I a 72h, si alteradas repetir al mes, si alte­ radas de nuevo: PEATC

77

Tríada típica de las crisis del síndrome de Méniére

Vértigo, acúfenos e hipoacusia neurosensorial coclear

78

Causa del síndrome de Méniére

Aumento de endolinfa, hidrops endolinfático

79

Tratamiento de mantenimiento

Dieta hiposódica, diuréticos, vasodilatadores. Si no, cirugía

80

Causa más frecuente de vértigo periférico

Vértigo posicional paroxistico benigno (VPPB)

81

Maniobra diagnóstica delVPPB

de Díx-Hallpike

82

Tto del VPPB

No fármacos: maniobras de reubicación canalicular

83

Clínica de la neuritis vestibular

Crisis única de vértigo que dura varios días, audición normal, antecedente de infección.

84

El neurinoma del acústico procede de

La porción vestibular del VIII par

85

Si es bilateral sospechar

Neurofibromatosis tipo 2 (AD;cr 22)

86

Screening y diagnóstico del neurinoma

PEATC (alargamiento l-V); RM con gadolinio

87

La afectación del facial es constante {100%) en un tipo de fractura temporal

La oblicua

88

En la fractura longitudinal la hipoacusia y la parálisis facial son

De transmisión y transitoria en la mayoría de los casos.

89

En la fractura transversal son

Perceptiva (cófosis) y permanente

90

Una parálisis facial con test de Schirmer normal se localiza

Distal al ganglio geniculado

91

La parálisis facial periférica más frecuente es

La idiopática, de Bell o "a frigore", y es de buen pronóstico

42 2

www. FreeLibros.me

Preguntas y respuestas más frecuentes

92

Vesículas en el pabellón y CAE -f parálisis facial de mal pronóstico + afectación del VIII par

Síndrome de Ramsay-Hunt o zóster ótico

93

Srindome de Heerfordt consiste en

Una forma de sarcoidosis: parálisis facial+fiebre+uveitis anterior+parotiditis

94

Síndrome de Melkersson-Rosenthal consiste en

Brotes de parálisis facial+edema hemifacial+lengua geográfica

C a p ít u lo 3 . R in o lo g ía 95

En el meato medio desembocan

Los senos frontal, maxilar y celdas etmoidales anteriores

96

La mayoría de las epistaxis tienen su origen en

El área de Kiesselbach

97

Rinorrea unilateral purulenta y fétida en un niño sospechar

Cuerpo extraño intranasal

98

Rinorrea unilateral purulenta y fétida en un adulto descartar

Cáncer nasosinusal

99

La fractura facial más frecuente es

La fractura nasal

100

Actitud ante hematoma septal tras fractura

Drenarlo siempre (riesgo de infección y de tromboflebitis del seno cavernoso)

101

Triada de la fractura del suelo de la órbita

Enoftalmos, diplopia, alteraciones sensitivas

102

Rinorrea clara persistente, unilateral, que aumenta con el Valsalva

Sospechar fístula de LCR

103

Mujer joven asiática + cacosmia con anosmia=

Ocena

104

Sinusitis más frecuente en el niño

La del seno etmoidal

105

Sinusitis más frecuente en el adulto

La del seno maxilar

106

Sinusitis en diabético + ptosis, oftalmoplejia, afectación de pares

Mucormicosis rinocerebral

107

Prueba de imagen de elección en las sinusitis

Tomografía computerizada

108

Tratamiento de las sinusitis

Amoxi-clavulánico10-14días, analgésicos, antiinflamatorios y aerosoles y vasoconstrictores tópicos

109

La tríada ASA es

Asma + intolerancia al AAS + poliposis bilateral

110

Niño con poliposis bilateral descartar

Fibrosis quistica

111

El pólipo antrocoanal de Killian es

Un pólipo unilateral en chicos jóvenes que sale por el meato medio desde el seno maxilar. Tratamiento quirúrgico.

112

Localización principal del osteoma

Seno frontal

113

Ebanista+rinorrea purulenta unilateral^

Adenocarcinoma de etmoides

114

Cáncer nasosinusal más frecuente

Carcinoma epidermoide

4 2 3

www.FreeLibros.me

Preguntas y respuestas más frecuentes

115

Tumores de cabeza y cuello (ex) en que no hay que tratar profi­ Nasosinusal. Labio inferior y glóticos T1, T2 lácticamente las cadenas ganglionares

116

Localización más frecuente del cáncer nasosinusal

Seno maxilar

C a p ít u lo 4 . E s t o m a t o l o g ía 117

La caries dental es debida sobre todo a infección por

Streptococcus mutans

118

La angina de Ludwig es

Un flemón del suelo de la boca, a partir de un molar infectado

119

La medida más eficiente para la profilaxis colectiva de la caries es:

La fluoración del agua

120

Fármacos con efecto 2°: hiperplasia gingival

Fenitoina y nifedlpina

121

Localización de las vesículas en la gingivoestomatitis herpética Toda la mucosa oral (VHS I)

122

Localización de las vesículas en la herpangina (coxsackie A4)

Pilares amigdalinos y paladar blando, respetando amígdalas y lengua

123

Estudio de elección de las fracturas mandibulares

Ortopantomografía

124

Mujer joven con bruxismo, otalgia y/o cefalea inespecífica

Síndrome de dolor-disfunción de la articulación temporomandibular

125

Localización más frecuente del cáncer de cavidad oral

1° labio, 2° suelo de la boca, 3° lengua móvil

126

Lesiones premalignas de la cavidad oral

Leucoplasia, liquen plano y eritroplasia

127

Factores de riesgo del cáncer (Ca) de la cavidad oral

Tabaco (90%), alcohol, déficit de vit A y hierro, higiene bucodentaria deficiente, radiación solar, RT, papiloma, herpes

C a p ít u lo 5 . P a t o lo g ía de la fa r in g e 128

Niño con rinolalia cerrada, ronquido, respiración oral, OMS de repetición...

Adenoides o vegetaciones

129

Ante una hiperplasia amigdalar unilateral descartar

Carcinoma epidermoíde y iinfoma

130

Diagnóstico definitivo de sd de apnea obstructiva del sueño

Polisomnografía: índice de apnea-hipopnea>10 en adultos, >5 en niños

131

La insuficiencia velopalatina es contraindicación de

Adenoidectomía

132

Etiología de las amigdalitis agudas

1° víricas, 2° bacterianas (St. betahemolítico del grupo A)

133

Tratamiento de elección de amigdalitis bacteriana

Penicilina

134

Complicación más frecuente de la amigdalitis y localización

Flemón o absceso periamigdalino; entre la amígdala y aponeurosis del músculo constrictor de la faringe

135

En el flemón periamigdalino la úvula...

Se desplaza hacía el lado sano

424 www. FreeUbros.me

Preguntas y respuestas más frecuentes

136

Diagnóstico del flemón periamigdalino

Punción y aspiración del espacio periamigdalino

137

En el absceso parafaringeo es obligado

Valorar su extensión con un TAC

138

Complicación que consiste en una tromboflebitis séptica de la yugular interna

Sepsis postanginosa o enfermedad de Lemiere

139

Tiene un riesgo elevado de

Tromboembolismo pulmonar séptico

140

El absceso retrofaringeo puede ser causado por

Enclavamiento de cuerpo extraño (espina, hueso)

141

Y tiene riesgo de

Mediastinitis

142

Varón adolescente + epistaxis repetidas

Angiofibroma nasofaríngeo juvenil

143

que nunca se debe

Biopsiar

144

y se trata

Con cirugía con embolización previa

145

y si recidiva o no es abordable por qx

Con Radioterapia

146

Características especiales del Cáncer de cavum

hombres =mujeres, no relación con tabaco-alcohol, sí con VEB, China; tratamiento con Radioterapia

147

Localización más frecuente del Cáncer de orofaringe

Amígdala

148

Localización más frecuente del Cáncer de hipofaringe

Seno piriforme

C a p it u lo 6 . P a t o lo g ía d e la la r in g e 149

Inervación motora

Todos por el nervio recurrente, menos el cricotiroideo por el nervio laríngeo superior

150

Inervación sensitiva

Glotis y supraglotis por nervio laríngeo superior; subglotis nervio recurrente

151

Drenaje linfático de la glotis

Carece de drenaje

152

Función del músculo cricotiroideo

Tensa la glotis

153

Único músculo que dilata la glotis

El crlcoaritenoideo posterior

154

El estridor en la laringomalacia es de tipo

Inspiratorio

155

Tratamiento de la laringomalacia

No precisa

156

Voz bitonal, cuerda en posición paramediana...

Parálisis unilateral del nervio recurrente

157

Parálisis unilateral del nervio laríngeo superior

Fatiga vocal, posición normal de las cuerdas con acortamiento

158

Parálisis bilateral del nervio laríngeo superior

Lo mismo, más riesgo mayor de aspiraciones

159

La disnea es característica de la parálisis...

Bilateral del nervio recurrente y precisa traqueotomia

4 2 5

www.FreeLibros.me

Preguntas y respuestas más frecuentes

Otorrinolaringología

160

Causas torácicas de lesión del nervio recurrente izquierdo

Tumores de pulmón y mediastino, cirugías torácicas, enferme­ dades cardíacas ( estenosis mitral, aneurisma aórtico)

161

Lesión del tronco del vago

Cuerda en posición intermedia, aspiración y disfonía

162

Tratamiento de la disfonía espasmódica

Infiltración en las cuerdas con toxina botulínica

163

Características de los nódulos vocales

Bilaterales, anteriores, por mal uso de la voz, tto foniátrico y si no, qx

164

Lesión benigna de la cuerda vocal más frecuente en la población general

Pólipo vocal: unilateral y anterior

165

Tumoración bilateral gelatinosa a lo largo de toda la cuerda es

Edema de Reinke

166

Y se favorece por

Irritantes, abuso vocal, hipotiroidismo

167

Y se trata por

Decorticación o cordotomia

168

Diferencia entre laringitis tuberculosa y cáncer de laringe

La tuberculosis no produce fijación de la cuerda

169

Tipos de VPH más frecuentes en la papilomatosis laríngea

6 y 11

170

Síntoma más frecuente y tratamiento de la laringitis por reflujo gastroesofágico

Disfonía y omeprazol

171

Cáncer más frecuente de cabeza-cuello

Cáncer de laringe

172

Características del cáncer supraglótico

Típico de países mediterráneos, síntomas tardíos (parestesias faríngeas), mayor riesgo de metástasis ex, peor pronóstico

173

Características del cáncer glótico

Países anglosajones, síntomas tempranos (disfonía), rara afec­ tación ganglionar, mejor pronóstico

174

Características del cáncer subglótico

Peor pronóstico, disnea y estridor

175

Varón con disfonía de duración >2 semanas...

Siempre descartar cáncer laríngeo

176

Tto de cáncer supraglótico TI, T2 NO

Laringuectomía parcial horizontal supraglótica + vaciamiento ganglionar profiláctico

177

Tto de cáncer glótico T I, T2 NO

Cordectomía (o Radioterapia en TI NOMO) sin necesidad de vaciamiento ganglionar

178

Localización de la traqueotomía

En 2oo 3oanillo traqueal

179

Localización de la coniotomía

En membrana cricotiroidea (sólo hacer en casos de extrema urgencia fuera de un hospital)

C a p it u lo 7 . P a t o lo g ía d e la s g l á n d u l a s s a l iv a l e s 180

Conducto y tipo de secreción de la glándula parótida

Conducto de Stenon, secreción serosa sobre todo durante deglución

426 www.FreeLibros.me

Preguntas y respuestas más frecuentes

Otorrinolaringología

181

Conducto y tipo de secreción de la glándula submaxilar

Conducto de Wharton, secreción seromucosa, sobre todo en reposo

182

Secreción de la sublingual

Secreción mucosa, sobre todo en reposo

183

Características de la sialolitiasis

Sobre todoen la submaxilar,dolorytumefaccióncon la ingesta, mejora por desobstrucción con aparición de sialorrea

184

Tratamiento de la sialolitiasis

Beber mucho líquido, sialogogos,espasmoliticosy antibióticos de eliminación salival (betalactámicos y macrólldos)

185

¿Qué es una ránula?

Quiste de retención por obliteración en el suelo de la boca, en conductos de glándula sublingual

186

La sialodenosis es característica de las glándulas

Parótidas y lagrimales

187

A más pequeña la gln más frecuentemente el tumor será

Maligno

188

Localización más frecuente de tumor en glándulas salivales

La parótida (mayoría benignos)

189

Tumor másfrecuenteen glándulas salivales, en mujer, en parótida y templado en gammagrafía es

Adenoma pleomorfo

190

Tumor maligno más frecuente en la parótida

Carcinoma epidermoide

191

Tumor en glándula submaxilar, con dolor intenso y metástasis en pulmón

Cilindroma o carcinoma adenoide quistico

192

Parálisis facial precoz + ulceración de la piel +edad avanzada

Carcinoma epidermoide de la parótida

193

Síndrome de Frey

Las 3*h": hiperhidrosis, hiperemia e hiperestesia en mejilla con masticación como secuela de parotiroidectomía

C

a p it u l o

8.

P

a t o l o g ía c e r v ic a l

194

Quiste o fístula del conducto tlrogloso, tratamiento quirúrgico Quiste medial más frecuente, que asciende con deglución y que consiste en extirpar todo el tracto del ducto tirogloso incluyendo el cuerpo del hioides (técnica de Slstrunk) por tratamiento cervlcotomia medial

195

Quiste lateral, enfermedad congénita cervical más frecuente

Quiste del segundo arco branquial o del seno cervical

196

Masa laterocx, elástica, pulsátil y con soplo

Glomus o paraganglloma cervical

197

El glomus se diagnostica por

Arteriografía o angloRM

198

Con un glomus se debe hacer un estudio de

Catecolaminas en orina, por si fuera funcionante

199

Signo de Hoster

Dolorenlasadenopatiasaltomaralcohol.tipicodeenfermedad de Hodgkin

200

Adenopatia submentoniana, tumor primario en

Zona anterior de cavidad oral (labio, suelo de la boca, lengua anterior)

427 www.FreeLibros.me

4 2 8

www.FreeLibros.me

Preguntas y respuestas más frecuentes

Traumatología C a p ít u lo 1 . F r a c tu r a s 1

Mejor prueba de imagen para ver una fractura

TAC

2

Características del callo duro

en la zona periférica, facilitado por ia alta tensión de oxígeno,osificación directa endomembranosa con colágeno tipo I y osteoblastos

3

Características del callo blando

zona central, hipoxia, osificación cartilaginosa con colágeno tipo II y condroblastos

4

¿Dónde podemos encontrar el hueso inmaduro o woven bone?

Sólo en los huesos fetales y en el callo de las fracturas

5

Proteínas oseas morfogenéticas más implicadas en el proceso de consolidación

BMP-2 y BMP-7

6

diabetes, tratamiento con corticoides, ¡ndometacina, y Factores más importantes que dificultan el proceso de consoli­ citostáticos,fuerzas de distracción o compresión excesiva, dación infección

H

Características de los autoinjertos

osteogénico,osteoinductor,osteoconductor,no¡nmunogén¡co y no transmite enfermedades

8

Principal ventaja de los aloinjertos

Su disponibilidad

9

Tratamiento de la ausencia de consolidación

Atrófica: quirúrgico, con autoinjerto o sustitutivos oseos. Hi­ pertrófica: en muchos casos no es necesario abordar el foco de fractura

10

5 indicaciones de fijadores externos

fracturas abiertas, necesidad de estabilización rápida, fracturas de pelvis, fracturas de pilón tibial, alargamientos de miembros y ausencias de consolidación infectadas

11

Principal contraindicación de fijación interna

Infección

12

Grado de Gustilo en una fractura con lesión vascular asociada

IIIC

13

Fracturas que suelen asociar síndrome compartimental

tibia, antebrazo (sobre todo en niños) y codo

14

Estado del pulso en el síndrome compartimental

suele estar conservado

15

Actitud ante sospecha de síndrome compartimental

retirar vendajes y yesos, elevar la extremidad, y si persiste o se agrava el cuadro, se recurre a la fasciotomía

16

¿Qué es la contractura isquémica de Volkman?

fase de secuelas del síndrome compartimental volar o anterior del antebrazo

17

Localización más frecuente de las fracturas de clavicula

tercio medio

429 www.FreeLibros.me

Traumatología

Preguntas y respuestas más frecuentes

18

Principal complicación de las fracturas de clavícula

consolidación en mala posición

19

Principal complicación a descartar en las fracturas del cuerpo de la escápula

lesiones pulmonares asociadas

20

Tratamiento resumido de las fracturas de húmero proximal

• Si no desplazada: tratamiento ortopédico • Si dos o más fragmentos: osteosíntesis salvo en ancianos y si la fractura pasa por el cuello anatómico (artroplastia) • Si paciente con mal estado general para cirugía: "skillful neglect”

21

Tratamiento de las fracturas de paleta de húmero distal

Osteosíntesis con placa y tornillos

22

Paciente con fractura de cabeza de radio que siente dolor crónico en la muñeca

pensar en migración proximal del radio por inestabilidad longitudinal

23

Técnica de elección de las fracturas de olecranon

"Obenque": cerdaje con alambre

24

Única fractura de diáfisis de antebrazo no quirúrgica

fractura aislada de la diáñsis cubital

25

Única fractura de radio distal no quirúrgica

fractura de Colles (reducción cerrada y contención con yeso)

26

Fractura de Hutchinson

fractura de la estiloides radial

27

Localización del dolor en las fracturas de escafoides

tabaquera anatómica

28

Principales complicaciones de la fractura de escafoides

ausencia de consolidación y necrosis isquémica del polo proximal

29

Tratamiento de la fractura de Bennet

reducción y osteosíntesis

30

Tratamiento de la fractura de Rolando

tratamiento ortopédico y movilización precoz

31

Elemento más importante de estabilidad del anillo pélvico

ligamentos sacroiliacos posteriores

32

Es prioritario descartar en las fracturas de pelvis

lesiones viscerales asociadas que pueden comprometer la vida del paciente

33

Principal problema de las fracturas intracapsulares de fémur proximal

• Biológico (vascularización precaria de la cabeza femoral interrumpida por la fractura) • Necrosis isquémica y ausencia de consolidación.

34

Principal problema de las fracturas extracapsulares de fémur proximal

• Mecánico (tendencia al desplazamiento de los fragmentos) • Consolidan en mala posición.

35

Significado de hematoma sobre la cara lateral de la región de la cadera sobre el trocánter mayor en fracturas de cadera

Indica que la fractura es extracapsular

36

Tratamiento de las fracturas intracapsulares de cadera según la clasificación de Garden

Graden I y II: osteosíntesis con tres tomillos canutados. Garden III y IV en jóvenes: osteosíntesis con tres tornillos canutados. Garden III y IV en ancianos: prótesis.

37

Tratamiento de las fracturas extracapsulares pertrocantereas de cadera:

Tornillo placa o clavo deslizante

38

Tratamiento de las fracturas extracapsulares subtrocantereas de cadera:

Tornillo clavo deslizante

4 3 0

www.FreeLibros.me

Preguntas y respuestas más frecuentes

c/

39

Tratamiento de elección en las fracturas de diáfisis femoral

enclavado intramedular

40

Tratamiento quirúrgico de elección en las fracturas de diáfisis tibial

enclavado intramedular (salvo las fracturas abiertas, en cuyo caso se realiza fijación externa)

41

Tratamiento resumido de las fracturas de tobillo

En general todas requieren tratamiento quirúrgico salvo algunas fracturas ¡nfrasindesmales no desplazadas

42

¿Qué es el signo de Hawkins?

Reabsorción subcondral en las fracturas de cuerpo del astrágalo que constituye un signo radiológico de buen pronóstico

43

Fracturas que ocurren en caídas de pie desde altura

fractura de calcáneo, meseta tibial y columna vertebral

C a p ít u lo 2 . L u x a c io n es

44

Tratamiento de la luxación acromiodavicular

• Grados I y II: Tratamiento conservador (cabestrillo, hielo y analgésicos) • Grado III: conservador o quirúrgico en el caso de jóvenes que vayan a hacer uso importante de la articulación (ej; cargar peso) • Grados IV, V y VI: Quirugía si el paciente lo desea (motiva­ ciones estéticas)

45

¿Qué es la lesión de Sankart?

Avulsión del labrum glenoideo y de los ligamentos glenohumerales Inferiores en la luxación recidivante de hombro

46

¿Qué es la lesión de HUI - Sachs?

Fractura por impactactón posterolateral de la cabeza humeral en la luxación recidivante de hombro

47

Lesión neurológica característica asociada a la luxación anterior de hombro

Nervio axilar

48

Signo radiológico típico de la luxación de codo

desestructuración del triángulo de Nelaton

49

Lesión neurológica característica de las luxaciones posterior y anterior de cadera

Nervio ciático y nervio femoral respectivamente

50

Actitud del miembro en las luxaciones posteriores de cadera

"Bañista sorprendido": miembro Inferior acortado, en rotación interna, aproximación y flexión

51

Prueba complementaria deobligada realización en las luxaciones de rodilla

arteriografia para descartar lesión asociada de la arteria po­ plítea

52

Tratamiento de las luxaciones de mediopié

Lisfranc: reducción anatómico +agujas de Kirschner o tornillos. Chopart: reducción cerrada + inmovilización con yeso

53

Grupo poblacional típico de aparición de luxación de rótula

chicas adolescentes

C a p it u lo 3 . les io n es t r a u m á t ic a s e in fla m a t o r ia s d e p a r t e s b la n d a s 54

Tratamiento general de los esguinces en ligamentos extraarticulares

Acrónimo"RICE,,=Rest, Ice, Compression,Elevation (reposo, frío local, vendaje compresivo y elevación del miembro)

55

¿Qué es el pulgar del guardabosques o del esquiador?

lesión del ligamento colateral cubital déla metacarpofalángica del primer dedo

431 www.FreeLibros.me

Traumatología

Preguntas y respuestas más frecuentes

56

¿Causa más frecuente de hemartros en la rodilla traumática aguda?

Lesión del ligamento cruzado anterior

57

Maniobras para explorar la lesión del ligamento cruzado anterior

Lachman (desplazamiento anterior excesivo de la tibia a 30° de flexión; más sensible) y maniobra del cajón anterior (a 90° de flexión)

Características del derrame en las lesiones meniscales:

Derrame articular escaso, seroso, no hemátlco, que tarda unas 18-24h en instaurarse

58 Características del derrame de la lesión del ligamento cruzado anterior:

Derrame articular abundante, hemático y de instauración rápida

59

Tratamiento de la lesión del ligamento cruzado anterior

conservadora inicialmente. Quirúrgico si el paciente es joven y desea continuar práctica deportiva y no puede por su ines­ tabilidad

60

Tratamiento de las lesiones de ligamento cruzado posterior

conservador. Potenciación del cuadríceps

61

Localización típica del dolor en las lesiones meniscales

zona posterior de la interlínea articular

62

Mecanismo de producción de las lesiones de ligamento colateral medial

traumatismos con valgo forzado

63

Mecanismo de producción de las lesiones del ligamento cruzado anterior

traumatismos con hiperextensión o valgo rotación de la rodilla

64

Mecanismo de producción de las lesiones del ligamento cruzado posterior

traumatismos directos sobre la tibia con la rodilla en flexión

65

Forma de instauración dolor en esguince de tobillo

dolor brusco quecedecasi completamente, periodo de latencia y reaparición progresiva hasta hacerse continuo

66

Clínica del síndrome de atrapamiento subacromial

dolor insidioso de predominio nocturno que aumenta cuando el paciente baja activamente el brazo

67

Dolor que aumenta con la elevación pasiva máxima de del hombro al chocar el manguito con la parte anteroinferior de! acromion en el síndrome de atrapamiento subacromial

¿Qué es el signo de Neer?

Desaparición del signo de Neer tras la administración de un anestésico local

68

¿Qué es el test de Neer?

69

Tratamiento quirúrgico de elección en el síndrome de atrapa­ acromioplastia(resecaruncentímetrodelmargenanteroinferior del acromion) miento subacromial

70

¿Qué es la tendinitis calcificante?

formación de un depósito de calcio en el espesor de los tendo­ nes del manguito de tos rotadores que produce un dolor muy intenso de predominio nocturno

71

Tratamientodeelecciónenloscasosgravesdecapsulitisadhesiva de hombro

capsulectomía artroscópica

72

¿Qué es la tenovitís estenosante de DeQuervain?

inflamación de la vaina común de los tendones del extensor corto y separador largo del pulgar a su paso sobre la estiloides radial

432 www.FreeLibros.me

Traumatología j ¡

Preguntas y respuestas más frecuentes

73

¿Qué es el test de Filkestein?

desviación cubital de la muñeca con el primer dedo sujeto en la palma de la mano que origina dolor en la tenosinovitis estenosante de DeQuervain

74

Localización del quiste de Baker

bolsa serosa asociada al semimembranoso en la rodilla

75

Enfermedad sistémica a la que hay que asociar el quiste de Baker

Artritis reumatoide

76

Alteraciones moleculares de la enfermedad de Dupuytren

aumento del colágeno tipo III y del FGF-2, PDGF y TGF- beta

77

Territorio de la mano afectado con mayor frecuencia en la enfer­ territorio cubital medad de Dupuytren

78

Tratamiento quirúrgico de la enfermedad de Dupuytren

C a p it u lo

4. E l sist em a

En general, fasciectomía (resección de la fascia) limitada o radical. En pacientes ancianos con mal pronóstico quirúrgico se prefiere fasciotomia percutánea. En pacientes con contracturas extremas: amputación

n er v io s o p er ifér ic o

79

Término utilizado para designar las lesiones más leves de un nervio periférico

neuroapraxia

80

Tratamiento de las neurotmesis

Sutura nerviosa, lo antes posible

81

Diagnóstico de la lesión del plexo braquial

Anamnesis y exploración física

82

¿Son reparables las lesiones preganglionares de los nervios periféricos?

NO

83

¿Cómo es la respuesta axonal en las lesiones preganglionares?

Normal

84

¿Qué es el signo de Tinel?

Aparición de parestesias en un territorio anatómico definido al percutir sobre un tronco. Ausente en lesiones nerviosas preganglionares

85

La actitud del miembro superior "en propina de maitre"es propia de

lesión proximal del plexo braquial (C5 y C6) o de Duchénne - Erb

86

¿Cómo es el reflejo de Moro en la lesión proximal del plexo bra­ Está ausente quial (C5 y C6) o de Duchénne - Erb?

87

La maniobra de Adson positiva sugiere

compresión neurovascular en la encrucijada cervicotorácica por anomalías de los escalenos

88

El test de hiperabducción de Wright positivo sugiere

compresión neurovascular en la encrucijada cervicotorácica a nivel costoclavicular

89

¿Qué nervio se puede ver comprimido a su paso por la apófisis y ligamento de Struthers?

Nervio mediano (síndrome del pronador)

90

Neuropatía por compresión más frecuente del organismo

síndrome del túnel carpiano

91

Enfermedad sistémica a la que se asocia típicamente el síndrome de túnel del carpo

Artritis reumatoide

433 www.FreeLibros.me

Traumatología

Preguntas y respuestas más frecuentes

92

Métodosdiagnósticoscomplementariosutilizadosparadiagnosticar un síndrome de túnel carpiano

Radiología simple y electromiografía

93

¿Qué es la maniobra de Phalen?

Flexión máxima de la muñeca durante un minuto que provoca parestesias en el síndrome del túnel carpiano

94

¿Qué es el síndrome del canal de Guyón?

Compresión distal del nervio cubital a nivel de la muñeca

95

¿Qué es la meralgia parestésica?

Compresión del nervio femorocutáneo

96

Clínica de la meralgia parestésica

Dolor y parestesias en cara anteroiateral dei muslo que gene­ ralmente aparecen cuando el paciente lleva un tiempo depíe, está sentado con flexión forzada de la cadera, o lleva un tiempo caminando

97

Localización más frecuente de los neuromas de Morton

Entre los metatarsianos tercero y cuarto

98

Tratamiento definitivo del neuroma de Morton

Resección quirúrgica

C a p itu lo 5 . T u m o r es y lesio n es ó sea s p s e u d o t u m o r a l es . tu m o r es de pa r tes bla n d a s 99

Tumor más frecuente en el hueso

Metástasis

100

Tumor más frecuente del hueso

Mieloma

101

Tumor oseo maligno más frecuente

Osteosarcoma

102

Tumor oseo benigno más frecuente

osteocondroma

103

Localización más frecuente de sarcoma de Ewing

diáfisis femoral proximal

104

Tumores que típicamente asientan en la metáfisis

quistes, osteosarcoma y osteocondroma

105

Tumores que típicamente asientan en la epífisis

con droblas toma y tumor de cálulas gigantes

106

Tumores que típicamente asientan en la diáfisis

"GEMMA"= Granuloma eosinófilo, sarcoma de Ewing, Mieloma, Metástasis, Adamantimoma

107

Tumor que asienta típicamente en las manos

encondroma

108

Osteoma osteoide...

Tumor en raquis posterior + imagen radiológica de "nidus" f dolor nocturno que cede con aspirina

109

Enfermedad de Ollier...

Encondromatosis múltiple

110

Traslocación del sarcoma de Ewing

t( 11,22)

111

Síndrome de Mafucci...

Encondromatosis múltiple + angiomas cutáneos.

112

Edad de presentación de los quistes oseos esenciales

menores de 10 años

113

Radiología del tumor de células gigantes

tumor multiloculado

4 3 4

www.FreeLibros.me

Preguntas y respuestas más frecuentes

114

Tratamiento del Osteocondroma

Generalmente no requiere tratamiento, a menos que produzca cuadros compresivos

115

Enfermedades asociadas al osteosarcoma

enfermedad de Paget y síndrome de Li-Fraumen¡

116

Localización más frecuente de metástasis de los osteosarcomas

a pulmón

117

imagen radiológica del osteosarcoma

lesión lítica con áreas blasticas, patrón permeativo y reacción perióstica. (sol naciente, triángulo de Codman)

118

edad de aparición del sarcoma de Ewing

10-20 años

119

Clínica del sarcoma de Ewing

Dolor, masa palpable y y con frecuencia fiebre y mal estado general

120

Tumor primario más frecuente que produce metástasis al hueso en mujeres

Cáncer de mama

121

Tumor primario más frecuente que produce metástasis al hueso en varones

próstata y luego pulmón

C a p ít u lo 6 . O r t o p e d ia in fa n til y d e l a d o l e s c e n t e 122

¿Cuál es la principal complicación de las epifisiolisis?

formación de una epifisiodesis que altere el crecimiento del miembro afecto

123

Tipo de epifisiolisis más frecuente y localización

Tipo II, en el radio distal

124

Localización más frecuente de las epifisiolisis tipo III

Tibia distal

125

Localización más frecuente de las epifisiolisis tipo IV

Cóndilo humeral

126

Tratamiento general de las epifisiolisis

* Tipos I y II: tratamiento conservador. * Tipos III y IV: Quirúrgico con agujas de Kirshner * Tipo V: Tratamiento de las secuelas

127

Localización más frecuente de las fracturas en rodete

metáfisis distal del radio

128

Localización más frecuente de las fracturas en tallo verde

diáfisis de cúbito y radio Supinación forzada del antebrazo en extensión con posterior flexión sin que sea necesaria la inmovilización posterior

129 Tratamiento de la pronación dolorosa

130

Fractura que con más frecuencia produce síndrome comparti­ fractura supracondílea de húmero mental en el niño

131

Lesión nerviosa asociada con mayor frecuencia a las fracturas supracondíleas en niños

nervio interóseo anterior

132

La tortícolis muscular congénita se asocia a ...

luxación congénita de cadera

133

¿Qué es la marcha de Trendelemburg?

En la luxación congénita de cadera, la marcha deTrendelemburg consiste en la desviación del tronco hacia el lado de la Iuxación cuando el paciente camina

4 3 5

www .FreeLibros.me

Traumatología

e : j r :as y respuestas más frecuentes

134

Principal complicación a largo plazo de la luxación congénita de cadera

artrosis secundaria

135

Tratamiento de la displasia congénita de cadera

En menores de 6 meses: Reducción cerrada + arnés de Pavlik. De 6 meses a 2 años: reducción cerrada + tracción blanda. En mayores de 2 años: Reducción abierta

136

Ante la sospecha de artritis séptica de cadera debe realizarse en todo caso...

artrotomia urgente de la cadera

137

Edad de aparición de la sinovitis transitoria de cadera

De 3 a 8 años

138

Clínica de la sinovitis transitoria de cadera

dolor en la cadera irradiado a la rodilla con limitación de la movilidad y ocasional mente actitud en flexo

139

Principal riesgo en la sinovitis transitoria de cadera

limitación de la extensión de la cadera con actitud permanente en flexo

140

Edad de aparición de la enfermedad de Perthes

entre 4 y 8 años

141

Factores de mal pronóstico más importantes en la enfermedad de Perthes

comienzo en edad avanzada, obesidad, sexo femenino, flexo, subluxación lateral mayor al 20%

142

Fases radiológicas de la enfermedad de Perthes

inicial, densificación, fragmentación, reosificación y remodelación

143

Edad de aparición de la epifisiolisis femoral proximal

11-16 años (típica de la adolescencia)

144

Fenotipo del paciente con epifisiolisis femoral proximal

obeso

145

Localización del dolor en la epifisiolisis femoral proximal

cara anterior del muslo o en la rodilla (hasta un tercio de los casos)

146

Actitud del miembro en la epifisolisis femoral proximal

miembro en rotación externa, y la limitación de la rotación interna aumenta a medida que se flexiona la cadera

147

Deformidad congénita más frecuente del pie

Pie zambo

148

Maniobra que pone de manifiesto un pie plano- valgo contracto doloroso

valgo del retropie que no se corrige al poner al paciente de puntillas

149

Definición de pie zambo

Combinación de deformidad en equino, varo y adducto

150

Exploración obligada ante la sospecha de pie cavo

Exploración neurológica completa

151

Localización de la enfermedad de Kónig

Cóndilo femoral

152

Localización de la enfermedad de Sever

Calcáneo

C a p it u lo 7 . P a t o lo g ía d e la co lu m n a v e r t e b r a l 153

¿Qué es una artrodesis?

fusión de una articulación. Se elimina el dolor procedente de la misma pero también su rango de movilidad

154

¿Que es la artroplastia de resección?

resecar los extremos articulares y dejar que el espacio entre ambos se rellene de tejido fibroso para reducir el dolor y man­ tener cierto grado de movilidad

4 3 6

www.FreeLibros.me

Traumatología

Preguntas y respuestas más frecuentes

155

Principal factor limitante de la longevidad de las artroplastias

desgaste de los componenetes

156

Principales contraindicaciones de la artroplastia de sustitución

infección activa y la atropatía neuropática de Charcot

157

Tratamiento de reducción de solicitaciones articulares en la artrosis

evitarsobrepeso, utilizar bastón, evitarimpactossobreel terreno (saltos, carrera...)

158

Tratamiento farmacológico sistémico de la artrosis:

analgésicos y antiinflamatorios no esteroideos. Preparados de glucosamina, condroitin sulfato, S-adenosílmetionina

159

Tratamiento de administración intraarticular de fármacos de la artrosis

infiltraciones intraarticulares de corticoides o de ácido hialurónico cuando la terapéutica oral no resultó eficaz

160

Tratamiento quirúrgico de la artrosis

artroplastia de sustitución cuando persiste un dolor intratable con medidas conservadoras durante al menos seis meses que interfieren con con la calidad de vida del paciente

161

Causa másfrecuente dedolor mecánico y cambios degenerativos en la cadera de mujeres jóvenes

displasia de cadera

162

Tratamiento de elección en la displasia de cadera

osteotomías acetabulares de reconstrucción

163

Indicaciones más frecuentes de la artroplastia total de rodilla

gonartrosis y artritis reumatoide

164

Principal indicación de osteotomía de la rodilla

pacientes con gonartrosis unicompartimental en varo jóvenes

165

Factores de riesgo más importantes para la necrosis avascular de cabeza femoral

alcohol y tratamiento con corticoides

166

Clínica de la necrosis avascular de cabeza femoral

limitación dolorosa de la movilidad de la cadera, especialmente en rotación medial

167

Cambios radiológicos patognomónicos de la necrosis avascular colapso de la cabeza femoral, radiolucencia subcondral - cresde cabeza femoral cent sign- o secuestro anterolateral

168

Imagen gammagráfica típica de la necrosis avascular de cabeza femoral

area de hipocaptación rodeada por un halo de hipercaptacíón

169

Imagen más precoz por RM de la necrosis avascular de cabeza femoral

linea de hiposeñal en TI

170

Principal diagnóstico diferencial de la necrosis avascular de cabeza femoral

osteoporosis transitoria de cadera

171

Grupo poblacional típico de la osteonecrosis de rodilla

mujeres mayores de 60 años

172

¿Qué es el Hallux valgus?

deformidad consistenteen la desviación lateral del primer dedo en relación con el eje del primer metatarsiano, con angulación metatarsofalángica superior a 15o

C a p ít u lo 8 . P a t o lo g ía d e la c o lu m n a v e r t e b r a l 173

Definición de inestabilidad de la columna vertebral

probabilidad de conduciren el momento de la lesión o en el futuro a dolor mecánico o alteraciones neurológicas

4 3 7

www.FreeLibros.me

Traumatología

Preguntas y respuestas más frecuentes

174

¿Qué condiciona la aparición de inestabilidad en las lesiones de columna?

La afectación de los elementos posteriores

175

Definición de shock medular

ausencia defunción medular debida no a lesiones morfológicas sino a disfunción

176

Única indicación de cirugía urgente en las fracturas vertebrales

existencia de una lesión neurológica incompleta y progresiva

177

Reflejo habitualmente explorado para el shock medular

Reflejo bulbocavernoso

178

¿Está justificado el uso de corticoides intravenosos en las primeras horas de una lesión medular aguda?

Si (estudios NASCIS)

179

¿Qué es la fractura de Jefferson?

Fractura en estallido del atlas

180

Tratamiento de las fracturas del cuerpo de la apófisis odontoide

tracción craneal seguida de halo - chaleco

181

¿Qués es la fractura del ahorcado?

fractura a través de la pars interarticularis de C2

182

¿Qué es la fractura del paleador o "day shoveller's fracture?

avulsión de la apófisis espinosa de C7

183

Tratamiento de las fracturas en estallido

inmovilización urgente con tracción craneal (frecuentes lesiones neurológicas asociadas)

184

Lugar más frecuente de asiento de las fracturas de columna toracolumbar

Transición TI 2 - L1

185

¿Cuál es el tipo de fractura de columna con una mayor incidencia de lesiones neurológicas asociadas?

Fracturas - luxaciones

186

¿En qué consiste el test de Adams?

cuando el paciente flexíona la columna existe asimetría de la parrilla costal o los flancos lumbares

187

¿Hacia dónde rotan las apófisis espinosas en las escoliosis es­ hacia la concavidad de la curva tructuradas?

188

¿Hacia dónde rotan las apófisis espinosas en las escoliosis posturales?

no rotan

189

Principales complicaciones a largo plazo de las escolisosis con curvas graves

insuficiencia cardiaca derecha y cor pulmona/e

190

Ante una curva escoliótica torácica izquierda debemos sospe­ siringomtelia o tumor subyacente char.,.

191

¿A qué edad suelen requerir con mayor frecuencia tratamiento quirúrgico las escoliosis idiopáticas?

Entre los 3 y 10 años (Escoliosis juvenil)

192

¿Qué es la cifosis de Scheüermann?

deformidad estructural (rígida) de la columna torácica y/o lumbar en el plano sagital con aumento de cifosis y cambios radiológicos característicos

193

¿Qué ocurre cuando el paciente con cifosis de Scheüermann intenta híperextender la columna?

que la cifosis no se corrige a diferencia de la cifosis postural

4 3 8

www. FreeUbros.me

Traumatología /i

Preguntas y respuestas más frecuentes

Fenotipo del síndrome de Klippel Feil

"Hombres sin cuello”(la cabeza parece incluida en los hombros), implantación baja del cabello, anomalías faciales y tortícolis irreductible

¿Cuál es la forma más frecuente de espondllolistesis?

la espondilolistesis ístmica

Localización más frecuente de espondilolistesis degenerativa

L4-L5

4 3 9

www .FreeLibros.me

4 4 0

www.FreeLibros.me

Preguntas y respuestas más frecuentes

Urología C a p ít u lo 1 . S em io lo g ía u r o ló g ic a y d e fin ic io n e s 1

La causa más frecuente de hematuria microscópica es:

La litiasis. Salvo en varones mayores de 50 años, que es la HBP

2

Debemos sospechar ante una hematuria macroscópica asintomática en un fumador

Ante un paciente con estas características, sospecha un cáncer de urotelio

3

¿Siempre se debe estudiar una enuresis?

No, sólo realizarás un estudio a aquellas enuresis en mayores de 6 años

C a p ít u lo 2 . I n fec c io n es d e l t r a c t o U r in a r io . C istitis in t er st ic ia l

B

Se llama ITU complicada a aquella que...

seacompañadecualquier circunstancia anómala (Transplante, reflujo, obstrucción,...) o es un varón

5

¿Cuántos tipos de ITU existen?

Existen 2 tipos de ITUS. Altas (infecciones renales) y Bajas (uretritis, prostatitis...)

6

De las distintas vías de acceso de los gérmenes al tracto genito urinario (TGU), la más importante es...

La vía ascendente

7

¿Cuál es el principal responsable de ITUs?

Es E.coli

8

¿Cuál es el grampositivo que presenta relevancia a la hora de producir ITUs?

El Staphylococcus saprophyticus

9

¿Cuál es el principal productor de ITUs en mujeres jóvenes?

10

El principal responsable de ITUs nosocomiales es...

Es el f. coli

11

¿Cuáles son las circunstancias en las que debemos sospechar una infección por Candida albicans?

Diabéticos, cateterizados, con tratamiento Antibiótico pro­ longado

12

La clínica de una ITU alta es...

Fiebre, dolor lumbar y escalofríos

13

Consideramos significativo un urocultivo cuando...

Cuando tiene >105 UFC/mL

14

¿Cualquier crecimiento en un urocultivo puede ser significativo?

Lo será si se ha recogido mediante punción suprapúbica

15

¿Piuria + síntomas urinarios = ITU?

Si en adultos, no en niños

16

Una bacteriuria asintomática es...

La presencia de 2 urocultivos positivos en una semana, sin clínica urinaria

Sigue siendo el E. coli, pero en este grupo es importante el 5. saprophyticus

441 www.FreeLibros.me

Urología

Preguntas y respuestas más frecuentes

17

Las circunstancias en las que hay que tratar una Bacteriruria asintomática son...

Embarazadas, varones menores de 5 años, Inmunodeprimidos, previo a cirugía urológica, infección por proteus, persistencia de cultivos positivos entre 3-5 días tras retirada de sonda

18

La pauta corta se pude dar...

En ITUs no complicadas en mujeres jóvenes

19

La pauta larga se da...

En ITUs complicadas o en embarazadas

20

Los antibióticos que pueden darse a una embarazadas son:

Penicilinas, Cefalosporinas, Nitrofurantoina, Fosfomicina

21

Los que están contraindicados en el embarazo:

Cotrimoxazol.Tetraciclinas, Fluoroquinolonas, Sulfamidas

22

Se entiende por ITU recurrente...

A la aparición de 4 ó más episodios al año

23

Cómo se trata:

Mediante profilaxis Antibiótico a días alternos duranteó meses. 0 postcoital sí tiene relación con el sexo

24

El tratamiento de una Pielonefritis no complicada se hará en función de:

La gravedad. Leve/moderada tratamiento vía oral. Severa, tratamiento i.v. SIEMPRE PAUTA LARGA

25

Ante una Pielonefritis con mala respuesta al tratamiento médi­ Realizaremos una ECO para descartar obstrucción o litiasis co

26

La prostatitis aguda se caracteriza por:

Una gran afectación general, mientras que la crónica es mucho más larvada

27

La prostatitis se trata con:

Fluoroquinolonasen pauta de larga evolución (es una infección en varón= complicada), durante 4 ó más semanas

28

La prostatodinia se trata con:

Alfabloqueantes, no con antibióticos

29

La principal causa de orquiepididimitis es:

En menores de 35, Clamidia; en mayores de 35, E. coli

30

Las vías de adquisición de un absceso renal son:

A partir de Pielonefritis contigua (E.coli), o por diseminación hematógena (S. aureus)

31

¿Cuál es la principal causa de infección nosocomial?

Es la ITU, y los catéteres son la primera causa hospitalaria

Los factores que aumentan el riesgo de ITU por catéter son:

Edad avanzada, mujer, mala técnica, uso drenajes abiertos, falta de higiene

33

¿Es frecuente la Tuberculosls(TBC) genitourinaria?

Es la localización más frecuente de TBC extrapulmonar. Se adquiere vía hematógena

34

Sospecharemos una TBC si la orina es...

Orina ácida con piuria estéril, (es estéril el urocultivo habitual, no el de Lówestein)

35

También sospecharemos TBC ante cuadros clínicos...

Cuadros crónicos (cistitis, orquiepididimitis,...) rebeldes al tratamiento

36

Una TBC genitourinaria se trata con:

Tratamiento antituberculostático con la misma pauta que la TBC pulmonar

37

¿Qué se hace ante un riñón con TBC con anulación funcional (riñón mástic)?

Se realiza una nefrectomía

4 4 2

www.FreeLibros.me

de sepsis

Urología

Preguntas y respuestas más frecuentes

De exclusión asociado a cistoscopia de distensión + biopsia vesical

El diagnóstico de Cistitis intersticial es:

C a p it u lo 3 . U ro lit ia sis 39

El tipo de litiasis más frecuente es:

Los cálculos de Oxalato cálcico

40

Las litiasis radiotransparentes son:

(SIUX) Sulfamidas, Indinavir, Úrico, Xantina

41

El cuadro clínico característico de una litiasis es:

Dolor cólico en fosa renal con irradiación a genitales, junto a cortejo vegetativo +/- síntomas irritativos

42

Sin embargo, las litiasis infectivas pueden producir:

Pueden producir infecciones de repetición e insuficiencia renal

43

La primera prueba de imagen a realizar ante una litiasis es:

Una Rx abdomen

44

La ECO no nos permite visualizar las litiasis que se encuentren en:

Uréter lumbar, salvo en las zonas cercanas a la vejiga o al riñón

45

Está contraindicado realizar una Urografía si el paciente presenta:

Alergia a! contraste, Creatinina >2, Mieloma Múltiple, deshi­ dratación

46

¿Qué se ve en el sedimento de un paciente con litiasis?

Hematuria, leucocituria (si hay mucha piuría, piensa en infección añadida), además de cristaluria

47

La causa más frecuente de producción de litiasis cálcica es:

La hipercalciuria idiopática

48

La causa más frecuente de hipercalciuria conocida es:

El hiperparatiroidismo primario

49

El tratamiento de la hipercalciuria idiopática es:

La administración deTiazidas

50

¿La hiperuricosuria favorece la aparición de litiasis cálcicas?

Sí. Facilita la producción de cálculos de calcio

51

Para sospechar litiasis por oxalato, un antecedente personal importante es:

La existencia de patología intestinal

52

Las litiasis infectivas (estruvita) están causadas por:

Gérmenes ureasa + (Proteus,...)

53

Las litiasis que aparecen en un pH urinario alcalino son:

Las cálcicas y las infectivas

54

Las litiasis que aparecen en un pH urinario ácido son:

Ácido Úrico y cistina

55

La litiasis cuyos cristales son radiolúcidos es:

Cistina

56

La hiperoxaluria se trata con:

Piridoxina o colestiramina

57

La litiasis por estruvita se trata mediante:

Acidificación de la orina (ác. propiónico,...) y ATB

58

Las litiasis cuyo tratamiento precisa de la alcalinización de la orina son:

Ácido Úrico y Cistina

59

La litiasis por uratos se trata con:

Alcalinización, alopurinol y disminución de proteínas

443 www.FreeLibros.me

Urología

-~ r:-T ta s , respuestas más frecuentes

La litiasis por cistina se trata con:

Alcalinización de orina, aumento de ingesta hídrica y D-penicilamina

61

Un cuadro agudo de litiasis no complicada se trata mediante:

Espasmolíticos, AINES, reposo e hidratación

62

Un cuadro agudo de litiasis se considera complicado cuando:

Presenta obstrucción severa, infección, fiebre, dolor incoercible, o riñón único

63

Ante una crisis litiásica complicada, hemos de solicitar:

Una ECO

64

El tratamiento de una crisis litiásica precisa de:

Ingreso y tratamiento agresivo. Con drenaje y medicación parenteral

65

Ante un cuadro de litiasis crónica, las actitudes terapéuticas son:

Tratamiento de la condición preexistente, Litotricia extracorpórea (LEOC) o cirugía

66

Las contraindicaciones absolutas de la LEOC son:

Embarazo, infección, obstrucción distal, riñón no funcionante ( 50 años es:

El linfoma

156

La metástasis más frecuente del Coriocarcinoma es:

El pulmón

157

La primera causa de metástasis sangrante cerebral es:

El coriocarcinoma

158

Los marcadores a solicitar en un cáncer testicular son:

Alfafetoproteina y B-HCG

159

En relación seminoma - alfafetoproteina:

¡¡¡Nunca eleva la alfafetoproteina!!!!

160

Masa en pulmón + masa testicular + B-HCG elevada es:

Coriocarcinoma

161

Para saber la localización de una masa escrotal, se realiza:

Una ECO

162

Ante una masa intratesticular, debemos:

Solicitar marcadores y orquiectomia

163

El tipo de orquiectomia a realizar es:

Orquiectomia inguinal

164

La primera causa de escroto agudo en menores de 12 años es:

La torsión del apéndice testicular

165

La primera causa de escroto agudo entre los 12 y 18 años es

La torsión testicular

166

Ante una torsión, se debe realizar:

Una orquidopexia BILATERAL

167

Los celes se caracterizan por:

Tener transiluminación positiva y ser extratesticulares en ECO

168

El tratamiento de un cáncer testicular incluye:

ECO+ Marcadores + cirugía + Marcadores postquirúrgicos + TAC

169

El seminoma se caracteriza pon

Ser muy radiosensible (además de proteína)

170

El tratamiento de un cáncer tipo seminoma en estadio I es:

Cirugía + observación

171

Ante la existencia de factores de riesgo en un seminoma en es­ Lo mismo, junto con radio o quimioterapia tadio I haremos:

172

EL tratamiento en estadios más avanzados incluye:

Radioterapia o quimioterapia

4 4 8

www.FreeLibros.me

por

no elevar la alfafeto-

Preguntas y respuestas más frecuentes

173

Los no seminomatosos se tratan asociando:

Quimioterapia

174

Ante una masa residual de un seminoma, haremos:

Observación si es pequeño, PET si es mayor

175

Ante una masa residual en un no seminomatoso, haremos:

Exéresis de la masa

176

Lo más probable es que una masa residual sea:

Necrosis

C a p ít u lo 8 . T r a sp la n t e ren a l 177

Las causas más frecuentes deTranplante renal son:

Glomerulonefritis y Diabetes mellitus

178

La complicación a tener siempre en cuenta es:

Et rechazo (hay varios tipos)

C a p ít u lo 9 . U r o p a tía o b st r u c t iv a 179

Una uropatía es realmente importante si asienta:

Sobre ambos riñones o sobre un único riñón funcionante

180

La obstrucción que dura más de un mes:

Produce alteración permanente

181

La obstrucción aguda se suele presentar:

En forma de cólico nefrítico

182

La obstrucción crónica se suele presentar:

De forma asintomática

183

El diagnóstico, la evaluación y el pronóstico se determinan me­ diante:

Una ECO

184

Ante obstrucciones bilaterales o agudas:

La desobstrucción es una urgencia

C a p ít u lo 1 0 . D isfu n ció n er éc t il 185

La causa más frecuente es:

Orgánica (dentro de ellas, las vasculares)

186

La enfermedad endocrina que más se asocia a la disfunción es:

La diabetes mellitus

187

El tratamiento de elección es:

Citrato de sildenafilo

188

Está formalmente contraindicado el siid en afilo en:

Personas que tomen nitratos, contraindicación de actividad sexual

I n co n tin en cia u r in a ria fem en in a 189

La incontinencia de urgencia consiste en:

Pérdida de orina precedida de deseo miecional

190

La incontinencia de esfuerzo consiste en:

Pérdidas desencadenadas con los esfuerzos

191

El tratamiento médico de la incotinencía de urgencia es:

Oxibutínina (anticolinérgico)

4 4 9

www. FreeLibros.me

OTROS TÍTULO S DE INTERÉS

Manual CTQ Manual CTO de Medicina y Cirugía, 8.a edición (3 volúmenes) Páginas: 2.500 aprox. ISBN: 9 7 8 -8 4 -1 5 0 6 2 -1 1 -0 PVP: 291 €

NOVEDADES PRÓXIMAS PUBLICACIONES

ISBN: 978-84-15062-60-8

9788415062608

9

788415 062608 www.FreeLibros.me

View more...

Comments

Copyright ©2017 KUPDF Inc.
SUPPORT KUPDF